ABSITE

Pataasin ang iyong marka sa homework at exams ngayon gamit ang Quizwiz!

CEA

1) Symptomatic patients with angiographic stenosis less than 50% and asymptomatic patients with stenosis less than 60% - No intervention and should be treated by medical therapy 2) Symptomatic patients with stenosis greater than 50% - Carotid Endarterectomy is preferred over stenting in unless there are contraindications such as uncompensated cardiac disease or local scarring, which increase the risk of cranial nerve complications. 3) Asymptomatic stenosis greater than 60%, - Carotid Endarterectomy may be recommended with medical therapy to reduce stroke risk as long as the combined stroke and death rate is less than 3%. 4)Symptomatic patients with stenosis greater than 50% and at high risk for Carotid Endarterectomy, Carotid stenting is preferred.

How long to scrub?

120 seconds

Splenic artery aneurysm

20-30% of these patients present with a "double rupture" phenomena where there is acute pain from rupture that is contained in the lesser sac. This allows for an important chance for diagnosis and treatment before free intraperitoneal rupture occurs. Free rupture typically occurs within 48 hours. Treatment requires ligation or embolization of both the artery proximal and distal to the aneurysm. Answer A: Females are more prone to these aneurysms as progesterone, estrogen, and relaxin may alter the arterial wall. Answer B: There is a high rupture rate of these aneurysms (20-50%), with rupture associated mortality as high as 25%. If ruptured in the third trimester, maternal and fetal mortality is 80-90%. Therefore, these aneurysms in pregnant patients should be repaired. Answer D: If only proximal ligation occurs, the aneurysm can still fill retrograde from the short gastric circulation. Answer E: Splenic artery aneurysms are the most common splanchnic aneurysm. Bottom Line: Splenic artery aneurysm should be repaired in women of child bearing-age.

How much weight loss following gastric sleeve?

5 year weight loss is approximately 60% of excess body weight

Warthin's tumor

A Warthin's tumor is also known as papillary cystadenoma lymphomatosum and is found almost exclusively in the parotid gland. It arises from the ectopic ductal epithelium. It represents approximately 5% of all salivary gland tumors and approximately 12% of benign tumors of the parotid gland. This tumor is more commonly seen in males in the fifth to seventh decades of life and there is an associated risk with smokers. There is approximately 5.0-7.5% bilaterality and 14% multicentricity in Warthin tumor. CT scanning may demonstrate a well-defined mass in the posteroinferior segment of the superficial lobe of the parotid. The diagnosis of a Warthin's tumor is easily made based on histologic findings, with rare confusion with other tumors. It is characterized histologically by papillary structures composed of double layers of granular eosinophilic cells or oncocytes, cystic changes, and mature lymphocytic infiltration. The treatment requires complete excision of the affected portion of the gland with uninvolved margins. This can usually be accomplished by a superficial parotidectomy. Most benign parotid tumors, such as pleomorphic adenomas (mixed tumors) or Warthin's tumors, can be treated by a superficial parotidectomy. Most malignant parotid tumors, such as mucoepidermoid or adenoid cystic carcinomas, require a total parotidectomy (with preservation of the facial nerve if possible), prophylactic modified radical neck dissection, and postop XRT if high-grade.

Hemorrhoidectomy

A bleeding complication within the first 24 hours after a hemorrhoidectomy is likely secondary to a surgical error that will need to be corrected. The patient should be taken back to the OR to identify and ligate the source of the bleed. Bleeding that starts after postoperative day 5 is likely secondary to sloughing of the eschar and should resolve. If late onset bleeding persists, then exam under anesthesia is also recommended.

Cholecystotomy tube

A cholecystotomy tube is an effective treatment option. Insertion is performed percutaneously and can be done at the bedside under ultrasound guidance and local anesthesia for patients too unstable for transport. This procedure provides immediate decompression of the inflamed gallbladder, thereby delaying definitive management until the patient is stabilized. Decompression is a mandatory component of the initial management. Once decompressed, cholecystography may be performed through the tube after resolution of the cholecystitis. If there is free passage of contrast into the duodenum, thereby denoting a lack of cystic duct obstruction, then the tube may be removed. If, however, gallstones appear to be the source of obstruction, then cholecystectomy should be performed once the patient is medically stable.

Volkmann's Ischemic contracture

A common pediatric fracture with potential for complications is the supracondylar fracture of the humerus. The mechanism of injury is usually a fall on the outstretched hand. The "extension type" is the most common. The injury is usually characterized by marked swelling around the elbow with associated deformity. Occasionally, neurologic and vascular deficits can occur. In a child, treatment of choice is closed reduction and percutaneous pinning first. Answers A & D: Volkmann's ischemic contracture is a complication related to closed reduction and splinting in an acute flexion that kinks off neurovascular structures (brachial artery) and leads to compartment syndrome, fibrosis, and a permanent withered hand. Answer B: Open reduction infrequently is necessary. Answer E: This is followed by splinting for 3 weeks, at which time the fracture is usually healed. Bottom Line: Volkmann's ischemic contracture is a potential complication of improper treatment of a supracondylar fracture of the humerus.

Cyanotic heart defect

A cyanotic heart defect is a group-type of congenital heart defects (CHDs). The patient appears cyanotic, due to deoxygenated blood bypassing the lungs and entering the systemic circulation. It can be caused by right-to-left or bidirectional shunting, or malposition of the great arteries. Cyanotic heart defects, which account for approximately 25% of all CHDs, include the following: Tetralogy of Fallot Total anomalous pulmonary venous connection Hypoplastic left heart syndrome Transposition of the great arteries Truncus arteriosus (persistent) Tricuspid atresia Interrupted aortic arch Pulmonary atresia Pulmonary stenosis Eisenmenger syndrome (reversal of shunt due to pulmonary hypertension) Patent ductus arteriosus (may cause cyanosis in late stage) Non-cyanotic heart defects are more common because of higher survival. In these, the shunt is initially from left (oxygenated) to right (non-oxygenated) and include the following: Atrial septal defect Ventricular septal defect Coarctation of the aorta Double outlet left/right ventricle

Kidney transplant complications

A decrease in urine output in the early postop period requires immediate intervention. A stepwise approach to diagnosing the cause of oliguria is recommended. Once mechanical obstruction of the indwelling Foley catheter has been excluded by irrigation, the volume status of the patient should be assessed, usually with a central venous pressure line or pulmonary artery catheter. Fluid replacement to restore intravascular volume should be initiated in the volume-depleted patient. Radiologic evaluation using various imaging techniques is required if urine output remains low in the face of adequate volume status. Ultrasonography is most helpful to diagnose ureteral obstruction caused by technical complications at the anastomosis or fluid collections that may be obstructing the ureter by extrinsic compression. With the addition of duplex and color Doppler imaging, blood flow to the kidney can be evaluated and technical problems with the vessels can be assessed. Answers A & B: Nuclear medicine techniques are also valuable for assessing renal function. Technetium 99m diethylenetriamine pentaacetic acid (DTPA) and iodine-131 iodohippurate (Hippuran) are the two major radionuclides used in the evaluation of renal allograft function. DTPA renal scans evaluate the vascular flow pattern of the kidney as well as the gross anatomy of the ureter and bladder. Scans are performed over 30 minutes, with uptake of contrast within 6 seconds indicating adequate flow to the graft. Peak activity in the parenchyma should be reached within 2 minutes, followed by a gradual decline in radioactivity in the renal parenchyma and an increase in radioactivity in the urinary collecting system and bladder. Iodine-131 iodohippurate scans are more sensitive in evaluating renal function. Peak activity is also reached in 2 to 4 minutes, with gradual decline over the next 30 minutes. However, ultrasonography is the diagnostic tool of choice and should be considered first. Answer C: The combination of delayed graft function and early rejection is clearly associated with worse outcome in the long-term, and an early biopsy of the kidney that is functioning poorly is helpful to diagnose rejection and guide therapy. Again, ultrasonography should be considered first to assess for vascular and ureteral assessment.

GCS

A focused neurologic examination should include the Glasgow Coma Scale (GCS) in all trauma patients. It provides useful information about necessity for intubation. it is an objective way to assess a patient over time and between different health care providers. The following values are used in calculating the GCS: Eye Opening: Spontaneous - 4 To Voice - 3 To Pain - 2 None - 1 Motor: Obeys verbal commands - 6 Localizes to pain - 5 Withdraws from pain - 4 Decorticate posturing - 3 Decerebrate posturing - 2 None - 1 Verbal: Oriented - 5 Disoriented/confused - 4 Inappropriate words - 3 Incomprehensible sounds - 2 None - 1 The total GCS is a sum of eye opening, motor and verbal scores. The minimum score is 3 and the maximum score is 15. This patient would receive a 2 for eye opening, a 3 for motor and 2 for verbal; for a total of 7.

VHL

A germline mutation in the gene VHL, with an incidence of 1 in 36,000 live births, causes von Hippel-Lindau syndrome (vHLs) and predisposes affected persons to hemangioblastomas of the central nervous system (CNS) and the retina, pheochromocytoma, clear cell renal cell cancer, endolymphatic sac tumors, cystadenomas and neuroendocrine tumors of the pancreas, and papillary cystadenomas of the epididymis and broad ligament. The best described function of the product of VHL, pVHL, is its regulation of hypoxia-induced factor 1 (HIF1). The protein pVHL complexes with other proteins to target HIF1 for ubiquitin-mediated proteasome degradation. HIF1 accumulates in the absence of pVHL and upregulates the expression of several genes, including vascular endothelial growth factor (VEGF), platelet-derived growth factor (PDGF), and transforming growth factor-alpha, among others. The upregulation of VEGF may explain the hypervascular tumors that are characteristic of vHLs. CNS hemangioblastomas occur in 60-80% of vHLs patients with a mean age at presentation of 33. The tumors are noninvasive, often multifocal, and enlarge over time, causing symptoms of mass effect such as headache, nausea, emesis, vertigo, and ataxia. Retinal hemangioblastomas occur in 60% of patients, with a mean age at presentation of 25, and are often bilateral and multiple. Retinal hemangioblastomas can cause blindness if untreated. Renal cell carcinoma occurs in up to 70% of patients, with a mean age at presentation of 35-40, and is often multifocal and bilateral. Renal cell carcinoma in vHLs is always clear cell type and is the leading cause of death in vHLs patients.

High dose dexamethasone test

A high dose dexamethasone suppression test can be used to determine whether Cushing's syndrome associated with elevated ACTH levels are caused by a pituitary adenoma or an ectopic source of ACTH production. The test is performed by administering 2 mg of dexamethasone every 6 hours for 48 hours. An elevated cortisol level after this study is diagnostic of an ectopic source of ACTH production. If there is a pituitary adenoma, the cortisol level should be suppressed after dexamethasone administration.

Gallbladder hydrops

A hydrops of the gallbladder is a distended gallbladder secondary to chronic obstruction of the cystic duct by impaction with a gallstone. Over time, the gallbladder becomes filled with watery mucoid material. The bile gets absorbed, but the gallbladder epithelium continues to secrete mucus. The gallbladder becomes massively dilated and enlarged without infection secondary to prolonged obstruction of the cystic duct. This condition can also occur secondary to fasting and dehydration. In patients with hydrops of the gallbladder, the gallbladder is tense and distended, making it difficult to grasp and easy to tear. In these patients, retraction of the fundus is difficult, and exposure of Calot's triangle is unsatisfactory. This problem is best-managed by aspirating the contents of the gallbladder either percutaneously with a 14 or 16-gauge needle inserted into the fundus of the gallbladder under laparoscopic vision.

Post operative antibiotics

A majority of the time preoperative antibiotics will be sufficient for prophylaxis. If infection is not present at the time of surgery, antibiotics should be continued postoperatively for 24 hours or less . If infection is present at the time of surgery, or more than 6 hours have passed between the time of injury and the time of treatment, then a therapeutic course of antibiotics is indicated

Whipple procedure

A pancreaticoduodenectomy (Whipple procedure) is generally indicated in patients with chronic pancreatitis with a nondilated duct. This procedure is reserved for patients with multiple cysts or calcifications in the head of the pancreas, obstructive symptoms, portal venous compression, or the possibility of malignancy.

Colon resection with simultaneous AAA

A patient with a symptomatic colon carcinoma and an incidental finding of an abdominal aortic aneurysm should have their malignancy addressed first with appropriate follow-up for the AAA once the neoplasm has been fully characterized. Contaminated case since working with GI tract, increases risk of graft infection. Clean cases can be done possibly with AAA such as renal or ovarian resection

ARDS

A patient with acute respiratory distress syndrome (ARDS) will have stiff, noncompliant lungs that are difficult to oxygenate. Many strategies have been employed to optimize the ability for oxygen exchange in patients with ARDS. Increasing the positive end expiratory pressure may be beneficial in treating patients with ARDS. This allows alveoli recruitment and improved surface area for oxygen exchange. Keep tidal volume at 6 mL/kg

PEG

A percutaneous endoscopic gastrostomy (PEG) tube is a preferred method for long-term feeding in patients who are unable to swallow or who require supplemental nutrition or gastric decompression. Indications include access for supplemental feeding, access for medications, decompression, etc. There are only a few absolute contraindications to endoscopic enteral access including esophageal obstruction and a limited life expectancy. Relative contraindications include prior abdominal surgery, prior gastric resection, gastric malignancy, ascites, severe malnutrition, and coagulopathy. Patients with gastroesophageal reflux disease will experience exacerbation of their symptoms with gastric enteral access as the gastroesophageal junction remains incompetent. Also, gastric residuals and postprandial pain or gastric distention will be exacerbated with PEG tube feeding in patients with gastroparesis.A percutaneous endoscopic gastrostomy (PEG) tube is a preferred method for long-term feeding in patients who are unable to swallow or who require supplemental nutrition or gastric decompression. Some contraindications to PEG placement for nutrition include gastric motility diseases and incompetent gastroesophageal junctions as seen in GERD and large hiatal hernias.

Phyllodes tumor

A phyllodes tumor is a rare malignant tumor of the breast that presents as a smooth, round, breast mass that may not be distinguishable from a fibroadenoma on physical exam, both of which may present in women of the same age group. The clinically distinguishing factor is the rapid growth of phyllodes tumors and the slow growth of fibroadenomas. A rapidly growing smooth, round, breast tumor should undergo core needle biopsy. Phyllodes tumors are graded as benign, borderline, and malignant based on mitotic count, pleomorphism, stromal overgrowth, and character of the tumor border (circumscribed vs. infiltrative). Phyllodes tumors should be resected with a border of 1-2 cm of normal breast tissue. About 20% of phyllodes tumors enucleated or resected with a few millimeters of normal breast tissue will recur.

Recto-vaginal fistula

A rectovaginal fistula is an abnormal communication between the anterior wall of the anal canal or rectum and the posterior wall of the vagina. A rectovaginal fistula is classified as low if a repair can be done perianally and high if repair can be accomplished only transabdominally. Simple or complex classification is based on location, size, and cause. There are many possible causes including congenital maldevelopment, trauma (obstetric, operative, blunt and penetrating injury), foreign body, infection of anal canal or vaginal septum, pelvic irradiation, neoplasm. However, obstetric trauma remains the most common cause of rectovaginal fistula.

Child-Turcotte-Pugh scoring system

A score of 5-6 is assigned to class A, 7-9 points is class B, and 10-15 points is class C. Overall surgical mortality rate is 10% for patients with class A cirrhosis, 30% for those with class B cirrhosis, and 75% to 80% for those with class C cirrhosis. Criticisms of the Child-Turcotte-Pugh include subjective variables (degree of ascites or encephalopathy), equal weighting of variables, and narrow score range. The score is composed of both qualitative and quantitative values and includes total bilirubin level, albumin level, international normalized ratio (INR) level, and degree of encephalopathy and ascites

Fluctuant mass following inguinal hernia repair

A scrotal hematoma is a common finding following inguinal herniorrhaphy. Generally, no specific intervention is indicated, and it is best managed with reassurance

Melanoma sentinel lymph node biopsy

A sentinel lymph node biopsy (SLNB) is indicated in the case of invasive malignant melanoma with the following characteristics: clinically negative nodes with tumor thickness greater than 1 mm or tumor less than 1 mm in thickness with either ulceration, regression, Clark levels IV or V, or mitotic rate > 1 per 10 high powered fields. A frozen section is not performed because special stains are needed including : S100, HMB 45, MART 1, Melan A, and Mitf. A positive sentinel lymph node biopsy will mandate a completion lymphadenectomy of the draining basin. Unlike breast zone 3 lymph nodes should be removed during lymphadenectomy Answers A & C & D: Radiation and chemotherapy are used as adjuvant treatments depending on the results of the completion lymphadenectomy. A completion lymphadenectomy is necessary at this time.

Small bowel mesenteric cyst

A smaller mesenteric cyst that is limited to a short segment of the mesentery can be excised laparoscopically without bowel resection. However, up to 60% of the time, resection of a wedge of mesentery with its associated intestine with primary end to end anastomosis is the most practical technique for removal. This is especially true for a large mesenteric cyst such as that presented in this stem. Unroofing the cyst or marsupialization of the cyst both have high rate of recurrence. If the above wedge and bowel resection can not be visualized well laparoscopically, then an open approach is indicated.

Neurohypophysis

ADH released by supraoptic and Oxytocin release from Paraventricular SAPO SAPO SAPO

Abdominal compartment syndrome

Abdominal compartment syndrome can be caused by many different things, including post-traumatic hemorrhage, intraperitoneal bleeding, retroperitoneal bleeding, and aggressive fluid resuscitation causing visceral edema. Abdominal compartment syndrome affects many different organ systems. It causes decreased cardiac output from decreased venous return, increased peak inspiratory pressures and pulmonary failure, decreased portal flow to the liver, decreased urine output from decreased flow to the kidneys, and decreased blood flow to the intestines. Patients usually present with a tense abdomen and ventilator insufficiency progressing to oliguria and cardiac collapse. The diagnosis is made by measuring bladder pressure and documenting new onset organ failure. The World Congress on Abdominal Compartment Syndrome states that abdominal compartment syndrome is persistent bladder pressures over 20 mmHg with the new onset of organ failure. Treatment is with a decompressive laparotomy.

PD catheter

Abdominal wall hernia repair using prosthetic tension-free repair is necessary in any patient who will have a PD catheter placed to minimize the risk of migration, catheter site infection, incarceration of catheter or bowel in the hernia defect, and recurrence of hernia. Similarly, adhesiolysis can be performed followed by placement of PD catheter so long as no enterotomies or other unexpected complications occur. Selective pelvic adhesiolysis is recommended to prevent catheter displacement, entrapment, or dysfunction. Adhesiolysis above the pelvis is unnecessary due to risk of visceral injury, hemorrhage, and re-formation of adhesions causing further damage to the peritoneal cavity.

Accessory spleen

Accessory spleens are small fragments of splenic tissue that are separate from the main body. They are present in approximately 20% of the population and up to 30% in patients with hematologic disorders. Accessory spleens can arise from the splenic vessels, in the distal pancreas, or along the GI tract. However, the most common location is at the splenic hilum. (Hilar region, 54%; pedicle, 25%; tail of pancreas, 6%; splenocolic ligament, 2%; greater omentum, 12%; mesentery, 0.5%; left ovary, 0.5%.

Zinc deficiency

Acrodermatitis Enteropathica is a disorder of children that causes inability to absorb sufficient zinc. It causes impaired wound healing and erythematous pustular dermatitis.

Active expiration

Active expiration is defined as a forced exhalation using the abdominal wall, internal intercostal muscles, and diaphragm. The internal intercostal muscles are involved in inhalation and chest wall expansion as well as forced expiration. The internal intercostals assist in expiration by pulling the ribcage down. They do not play a role in relaxed exhalation. During active expiration, the most important muscles are those of the abdominal wall (including the rectus abdominis, internal and external obliques, and transversus abdominis), which drive intra-abdominal pressure up when they contract, and thus push up the diaphragm, raising pleural pressure, which raises alveolar pressure, which in turn drives air out.

Addisonian crisis

Acute adrenal insufficiency (Addisonian crisis) usually manifests as shock in a patient with undiagnosed chronic adrenal insufficiency who has been subjected to physiologic stress. Patients with established adrenal insufficiency caused by exogenous steroid use may experience crisis if they do not increase glucocorticoid replacement during times of stress or illness (such as in the patient above undergoing surgery). Signs and symptoms of Addisonian crisis include fever, nausea, vomiting, refractory hypotension, and lethargy. Laboratory findings of adrenal insufficiency include hyponatremia, hyperkalemia, azotemia, and fasting or reactive hypoglycemia. Hypercalcemia may also be present. The rapid adrenocorticotropic hormone (ACTH) stimulation test is the best test for both acute and chronic adrenal insufficiency. Synthetic ACTH (250 µg) is administered by IV, and plasma cortisol levels are measured 30 and 60 minutes later. Normal peak cortisol response should exceed 20 µg/dL. Answer B: Measurement of ACTH by immunoradiometric assay is then used to distinguish primary from secondary and tertiary adrenal insufficiency. High plasma concentration of ACTH (>200 pg/dL) and low plasma cortisol (<10 mg/dL) are diagnostic of primary adrenal insufficiency. Low levels of plasma ACTH indicate secondary (pituitary) or tertiary (hypothalamic) adrenal insufficiency

Appendicits in pregnancy

Acute appendicitis is the most common nonobstetric indication for surgery in pregnancy. One in 1500 pregnancies will experience appendicitis. No one trimester is more affected than another. Prompt diagnosis and treatment is important, as non perforated appendicitis carries a 5% risk of fetal loss, while perforated appendicitis carries a 25% risk of fetal loss. Presentation varies compared with the classic presentation. The pain may be higher, due to cephalad displacement by the gravid uterus. Diagnosis is based upon imaging with ultrasound. If equivocal, MRI is the next best imaging study

Acute appendicitis

Acute appendicitis is usually initiated by a luminal obstruction caused by an appendicolith or a fecalith. This causes bacterial overgrowth and increased secretions that lead to luminal distension, which activates visceral pain fibers that cause the initial periumbilical pain. Once the inflammation irritates the parietal peritoneum adjacent to the appendix, somatic nerve fibers are activated that cause the right lower quadrant pain.

Aortic dissection progression

Acute dissection has traditionally been used to describe presentation within the first 2 weeks, whereas the term chronic is reserved for those patients presenting at more than 2 months after the initial event. Indications for operative intervention in patients with chronic aortic dissection include: Impending or actual rupture Symptoms related to dissection (congestive heart failure, angina, aortic regurgitation, stroke, pain) Malperfusion Aortic expansion >1 cm/year: type A, aneurysm size: ≥5.5 cm; type B, aneurysm size: ≥6.5 cm Answer A: Aneurysmal degeneration of an ascending aortic dissection >5.5 cm should be repaired. Answer C: Only if malperfusion is noted should the aortic dissection be repaired. Answer D: If a new acute kidney injury is noted it may signify end organ malperfusion; however, the creatinine is normal here. Answer E: Aortic expansion of 1 cm/year is an indication for repair. Bottom Line: Patients with chronic aortic dissection should undergo operative intervention for impending or actual rupture, symptoms related to dissection, malperfusion, aortic expansion >1 cm/year, or aneurysm size ≥5.5 cm in ascending aortic dissections and ≥6.5 cm in descending aortic dissections.

Acute vs chronic aortic dissection

Acute dissection has traditionally been used to describe presentation within the first 2 weeks, whereas the term chronic is reserved for those patients presenting at more than 2 months after the initial event. The acute form of aortic dissection is often rapidly lethal, whereas those surviving the initial event go on to develop a chronic dissection with more protean manifestations. The DeBakey system differentiates patients based on the location and extent of aortic dissection. The advantage of this system is that four different groups of patients with different forms of aortic dissection emerge. The Stanford system, a functional classification system, states that all dissections that involve the ascending aorta are grouped together as type A, regardless of the position of the primary tear or the distal extent of the dissection. Clinically, type A dissections occur with an overall greater frequency. Stanford type B include dissections that originate in the descending thoracic and thoracoabdominal aorta regardless of any retrograde involvement of the arch.

Acute ileitis

Acute ileitis can often mimic acute appendicitis. If discovered intraoperatively, the recommendation is to proceed with an appendectomy as long as there is no cecal involvement. This approach will rule out the appendix as a future source of abdominal pain. The ileitis is likely secondary to a bacterial infection that does not warrant any surgical resection. Answer A: Resection of the appendix is recommended to eliminate it as a source of abdominal pain in the future. Answer B: Resection of the appendix during the operation is recommended to eliminate it as a source of abdominal pain in the future. Answer D: Small bowel resection is reserved for recurrent disease as bowel preservation is very important in Crohn disease.

Acute limb ischemia

Acute limb ischemia can be secondary to an embolic or thrombotic event. Embolic events are most commonly due to a cardiac source. Common causes of cardiac emboli include atrial fibrillation, myocardial infarction, ventricular aneurysm, and atrial myxoma. Other causes of emboli include proximal atherosclerosis or a paradoxical emboli that is initiated from a deep vein thrombosis. Arterial thrombosis occurs in patients with longstanding peripheral vascular disease who progress to complete occlusion. These patients will usually have bilateral claudication or rest pain prior to the event. Previous bypass grafts are at higher risk for thrombosis than an embolic event. If the etiology is unclear in an otherwise healthy patient, a CT scan should be performed to rule out a malignancy that may be causing a hypercoagulable state and thrombosis.

Adenoid cystic carcinoma

Adenoid Cystic carcinoma is characterized by its unpredictable behavior and propensity to spread along nerves. It possesses a highly invasive quality but may remain quiescent for a long time. The tumor has an affinity for growth along perineural planes and may demonstrate skip lesions along involved nerves. Distant metastases are more common than to regional areas; occurring in 30-50% of all patients. Lung metastasis is the most common site of distant metastasis, but brain and liver are also reported. Three histologic types have been identified: cribrose, tubular, and solid. The solid form has the worst prognosis; the cribrose pattern possesses the most benign behavior and best prognosis. This tumor requires aggressive initial resection. Overall 5-year survival is 35%, and 10-year survival is approximately 20%.

ARDS

Acute respiratory distress syndrome (ARDS) is a process associated with an acute injury to the lung. The first phase is exudative and will have protein-rich fluid move into the alveolar spaces. Leukocytes will proliferate and cause intrinsic lung injury. The saturation of alveoli with protein-rich fluid results in poor oxygenation. The second phase is fibroproliferative and is associated with fibrosis and collagen formation. This creates a stiff, noncompliant lung, but the process is reversible over time. The third phase is resolution, where the lung tissue will begin remodeling and clearing the pulmonary edema. Answer A: Exudative phase involves protein-rich fluid moving into alveolar spaces followed by leukocytes, which proliferate and cause intrinsic lung injury. Collagen proliferation occurs in the second phase. Answer B: Type II pneumocytes are responsible for surfactant production. Poor oxygen exchange is due to saturation of alveoli with protein-rich fluid. Answer D: Lung injury is due to proliferation of leukocytes, not eosinophils. Answer E: Fibrosis of lung is improved in the third phase of ARDS, which involves remodeling. Bottom Line: The initial insult of ARDS is movement of protein rich edema into the alveolar spaces.

Duodenal malignancy

Adenocarcinoma is the most common malignant neoplasm in the duodenum. Depending on the location, symptoms may include obstruction, jaundice, or GI bleed. The treatment of localized adenocarcinoma of the small bowel is wide segmental resection. Therefore, given the location of this tumor in the fourth portion of the duodenum, a segmental duodenal resection is required. When a local extension or metastatic disease preclude curative resection, palliative procedures such as gastrojejunal anastomosis can be performed. While pancreaticoduodenectomy is required for tumors involving the first and second portions of the duodenum, segmental resection provides an equivalent survival benefit for lesions in the distal third and fourth portions of the duodenum as long as a margin-negative resection can be achieved. Duodenal segmentectomy should be preferred to pancreaticoduodenectomy, because it is associated with negligible rates of morbidity and mortality, while allowing for satisfactory margin clearance and adequate lymphadenectomy

Small bowel malignancy

Adenocarcinoma of the small bowel often presents late in its course due to vague and nonspecific symptoms that delay the diagnosis. As such, many patients present with already established metastatic or nodal disease. The majority of the small bowel adenocarcinomas are found in the duodenum; however, patients with longstanding Crohn disease are more likely to develop neoplasia in the distal ileum due to chronic inflammation in that area. The many risk factors for small bowel adenocarcinoma include familial adenomatous polyposis, hereditary nonpolyposis colorectal cancer syndrome, celiac disease, and Crohn disease. Secondary neoplastic involvement of the small intestine is more common than is primary small intestinal neoplasia, given the rarity of small bowel neoplasms. Extraluminal involvement of the small bowel is particularly common in the setting of widespread peritoneal carcinomatosis. Erosion through the bowel wall into the lumen can occur, particularly with tumors that have a predilection to involve the peritoneal cavity, such as ovarian, colon, and gastric cancer. Hematogenous spread to the bowel is also possible. In studies in which direct extension from peritoneal metastases was excluded, the most common tumors to hematogenously spread to the small bowel were melanoma, lung, breast, cervix, sarcoma, and colon. The small bowel is the most common site of gastrointestinal metastases in patients with metastatic melanoma. Answer A: Most small bowel adenocarcinomas present late in their course. Answer B: Crohn disease is a risk factor for distal ileum cancer. Answer C: The duodenum is the most common location. Answer E: In the case of primary small bowel neoplasia, neuroendocrine tumors recently surpassed adenocarcinomas as being the most common primary small bowel malignant tumor (incidence of 40% of primary small bowel tumors vs. 25-40% for adenocarcinoma), but the most common malignant small bowel tumor overall is secondary neoplasia. Bottom Line: Small bowel adenocarcinomas most commonly occur in the duodenum.

Trauma resuscitation

Adequate resuscitation in the trauma setting is actively undergoing revision in recent years. Previously, large volume crystalloid resuscitation was routinely administered. New evidence suggests that use of large volumes of crystalloid promotes acidosis and dilutional coagulopathy. Therefore, a more limited crystalloid volume is recommended. If fluids greater than 2 L are required, blood products should be administered. Systolic blood pressure goals are also changing, now allowing for an element of permissive hypotension. Goal blood pressure is now a SBP of 90 mmHg in the trauma setting along with definitive control of any hemorrhage. Use of vasopressors to achieve this goal is contraindicated, with the notable exception of patients with spinal cord injuries. In those requiring large resuscitative efforts, additional markers should also be used. Lactate levels should be normalized to < 2.5 mmol/L. A foley catheter should be placed for urine output monitoring with a goal urine output of 0.5 - 1 mL/kg/hr. Of note, in the acute setting with ongoing blood loss, hemoglobin measurement does not accurately reflect current hemoglobin levels or overall blood volume, and should not be used as the only tool for guiding transfusion. Answer A: Only 2 liters of crystalloid should be given in the acute trauma setting. Studies have shown that administration of high volume crystalloid resuscitation can promote acidosis and coagulopathy. If further resuscitation is needed, blood products should be administered. Answer B: Goal SBP should be > 90 mmHg. Permissive hypotension is preferable in the acute trauma setting to mitigate blood loss and reduce the risk of dislodging clots. Answer C: Goal urine output is 0.5 to 1ml/kg/hr. It is not necessary to target urine output above this range. Answer E: Hemoglobin levels often lag behind actual blood volume and blood loss. It is an unreliable indicator of resuscitation in the acute phase. Bottom Line: Goals for acute resuscitation are lactate < 2.5 mmol/L, urine output of 0.5-1mL/kg/hr and a goal SBP >90 mmHg.

Afferent Loop Syndrome

Afferent loop syndrome has many causes and is a mechanical problem that requires surgical correction. It is a chronic, partial obstruction of the proximal loop, consisting of the duodenum and jejunum, after gastrojejunostomy, resulting in duodenal distention, pain, and nausea following ingestion of food. The syndrome usually occurs when the afferent limb is greater than 30-40 cm in length and has been anastomosed to the gastric remnant in an antecolic fashion. The abdominal x-ray reveals a loop of markedly dilated small bowel located on the left side of the abdomen which is consistent with a proximal small bowel obstruction. A long afferent limb is usually the underlying problem, and treatment therefore involves the elimination of this loop. Some have advocated converting the Billroth II construction into a Billroth I anastomosis, whereas others have advocated an enteroenterostomy below the stoma, which is technically easier. Creation of a Roux-en-Y can also be done.Answer A: The syndrome usually occurs when the afferent limb is greater than 30-40 cm in length and has been anastomosed to the gastric remnant in an antecolic fashion. Answer B: Radionuclide studies imaging the hepatobiliary tree have also been used with some success in diagnosing this syndrome. Normally, the radionuclide should pass into the stomach or distal small bowel after being excreted into the afferent limb. If there is failure to do so, the possibility of an afferent loop obstruction should be considered. Answer D: If the obstruction has been present for a long period of time, it can also be aggravated by the development of the blind loop syndrome. In this situation, bacterial overgrowth occurs in the static loop, and the bacteria bind with vitamin B12 and deconjugated bile acids. This results in a systemic deficiency of vitamin B12 with the development of megaloblastic anemia. Answer E: Afferent loop syndrome can occur from a variety of causes. It can arise secondary to kinking and angulation of the afferent limb, internal herniation behind the efferent limb, stenosis of the gastrojejunal anastomosis, a redundant twisting of the afferent limb with a resultant volvulus, or adhesions involving the afferent limb. Bottom Line: Afferent loop syndrome has many causes and is a mechanical problem that requires surgical correction. The usual treatment options are either conversion to Billroth I or Roux-en-Y reconstruction.

Overwhelming post-splenectomy infection

After a splenectomy, the patient is at risk for Overwhelming Post Splenectomy Infection (OPSI). These patients are vulnerable mostly to encapsulated organisms, which include Streptococcus pneumoniae, Neisseria meningitidis, and Haemophilus influenza. The most common of these organisms is Streptococcus pneumoniae, causing up to 90% of overall cases. It is recommended to receive vaccines for these organisms within two weeks of splenectomy, or at least two weeks after if performed emergently

Splenic injury

After a splenic injury, patients that are hemodynamically unstable or fail nonoperative management require a splenectomy. Operation may also be warranted in those with multiple associated injuries or high grade injuries. Other relative indications are advanced age (>55 years) and head injury; however, this is not routine at many trauma centers. Exploration begins through a midline incision. The abdomen is packed and life threatening hemorrhage and bowel contamination are addressed first. Splenic exploration is performed by medializing spleen to help visual the injury. The splenocolic ligament may be ligated as this is often vascular. The splenorenal and splenophrenic ligaments are avascular and may be taken bluntly or sharply divided without ligation. The plane is continued posterior to the pancreas and the spleen and pancreas are brought medially. Grade I injuries may be repaired with topical hemostatic agents or cautery. Grade II and III injuries may be amenable to splenorrhaphy with suture or mesh repair. Sutures may be buttressed with omentum, teflon, absorbable mesh, and cellulose based hemostatic agents. Segmental resection may be considered for higher grade injuries; however, at least one third must be preserved for preserved immunologic function. Grade IV and V injuries usually require splenectomy. Drainage is considered only if there is concern for the tail of the pancreas. Routine drainage is associated with an increased risk for abscess. Subphrenic abscess occurs in 3-13% of patients. Other early complications include thrombocytosis which occurs in up to 50% of patients. The thrombocytosis abates in several weeks and usually requires no treatment. Overwhelming post-splenectomy infection occurs in less that 0.5% of patients.

Recto-vaginal fistula

After confirming recurrent cancer is excluded from the differential diagnosis, a rigid proctoscopy would be the next best test. Post-radiation rectovaginal fistulae are associated with radiation damage to the rectum. The quality of the rectal tissue (fibrosis, friability, etc.) and its associated compliance will help determine what type of operation should be performed. It also may help to localize the fistula. A CT scan, transvaginal ultrasound, transrectal ultrasound, and barium enema cannot reliably assess the condition of the rectal tissue like endoscopy can

cricothyroidotomy

After multiple failed intubation attempts, a surgical airway may be required. In those 12 ages or older, cricothyroidotomy is the procedure of choice in this scenario. It is superior to jet insufflation as it requires less specialized equipment and protects against aspiration. In children 10-12 years or younger, cricothyroidotomy is contraindicated as the cricothyroid membrane is very delicate in this age group. Needle techniques such as jet ventilation (also called translaryngeal catheter or needle cric) may be used as a quick method for improving oxygenation, however it provides no protection against aspiration. Tracheostomy is generally used only in non emergent conditions. To perform an open cricothyrotomy; First, the cricothyroid membrane is identified immediately caudad to the thyroid cartilage. Over this area a longitudinal skin incision is made, approximately 3 cm long. A transverse skin incision may be used in a thin patient with clear landmarks, however, a longitudinal incision has the benefit of easy extension if needed. Next, the tissues above the cricothyroid membrane are bluntly dissected by use of finger or skin retractor. This must be done by palpation. Then, a transverse incision is made in the cricothyroid membrane,and it is dilated with the blunt handle of the scalpel. It is important to preserve the cricoid cartilage by spreading transversely, not longitudinally. A number 5 or 6 tracheostomy tube or ET tube is inserted into the trachea and the cuff is inflated.

Necrotizing enterocolitis

After the diagnosis of necrotizing enterocolitis has been established, initial management consists of bowel rest with nasogastric tube decompression, fluid resuscitation, blood and platelet transfusion, and administration of broad-spectrum antibiotics. Medical management continues for 7-10 days and is successful in roughly half of cases. Answer B: The development of necrotizing enterocolitis is unusual in the first few days of life. About 80% of cases, however, occur within the first month of life. The clinical presentation of necrotizing enterocolitis is often nonspecific and unpredictable. Answer C: Prematurity is the single most important risk factor, although other factors such as ischemia, bacteria, cytokines, and enteral feeding are all likely significant. Answer D: The general principles of surgical management of necrotizing enterocolitis include resection of all nonviable segments of intestine with creation of a stoma. The risks for anastomotic leak and stricture formation have tempered widespread enthusiasm for anastomosis in this setting. Answer E: The absolute indication for operative management of necrotizing enterocolitis is the presence of intestinal perforation as revealed by the identification of free air on plain abdominal radiographs. Other relative indications for surgery include overall clinical deterioration, abdominal wall cellulitis, worsening acidosis, falling white blood cell or platelet count, palpable abdominal mass, or a persistent fixed loop on repeated abdominal radiographs. Bottom Line: The biggest risk factor for necrotizing enterocolitis is premature birth. Medical management is successful in 50% of cases. Surgical intervention should be undertaken when there are signs of perforation and resection of all necrotic bowel with stoma creation is the procedure of choice.

H. pylori

After treatment, the best test for eradication of the bacteria is with a urea breath test. Testing should be delayed for 4 weeks to ensure reliable results. If the patient needs to be evaluated with endoscopy for concern of brisk bleeding or perforation upon initial evaluation, then the rapid urease assay is recommended for diagnosis.The most appropriate diagnostic test for H. pylori is serology and for eradication is the urea breath test.

Caloric requirements

Age (years) Total Caloric Requirement (kcal/kg/day) 0-1 90-125 1-7 75-90 7-12 60-75 12-18 30-60 >18 25-30

Aldosterone

Aldosterone release is stimulated by hyperkalemia and angiotensin II. Hyperkalemia acts by stimulating renin secretion and acting directly on the zona glomerulosa. Hypokalemia has the opposite effect. When renin secretion is increased by either hyperkalemia, hypovolemia, or low sodium delivery to the distal convoluted tubule, renin causes cleavage of angiotensinogen to angiotensin I. Angiotensin converting enzyme then converts angiotensin I to angiotensin II which then stimulates release of aldosterone. Aldosterone then acts by promoting sodium and chloride retention in the distal tubule. In exchange for sodium, hydrogen and potassium are excreted in the urine.

Aldosteronoma

Aldosteronoma (unilateral aldosterone producing adenoma) or bilateral adrenal hyperplasia are two underdiagnosed causes of refractory hypertension. Classic signs include hypokalemia and hypertension. However, normokalemia may be present. A plasma aldosterone concentration to plasma renin activity ratio >20 is required for diagnosis of primary aldosteronism. Diagnosis also requires elevated plasma aldosterone concentration (usually greater than 15 ng/dL). The mean age at diagnosis is 50 years. Unilateral aldosteronoma or adrenal hyperplasia is potentially curable with surgical resection and responsiveness to spironolactone is a good predictor of responsiveness to surgery. There is an increased risk of MI, stroke, and atrial fibrillation with aldosteronoma or adrenal hyperplasia given the long-term hypertension. Answer A: Hypokalemia is present in only 9-37% of patients with primary hyperaldosteronism. Answer B: Spironolactone is used to treat patients with bilateral adrenal hyperplasia. Unilateral adrenalectomy is the gold standard of treatment in patients with unilateral adrenal hyperplasia. In patients who have unilateral disease, surgical intervention results in definitive treatment. Answer C: Plasma renin activity is low in patients with primary aldosteronism, usually less than 1 ng/mL/hr. Answer D: Spironolactone is used to treat patients with bilateral adrenal hyperplasia. Unilateral adrenalectomy is the gold standard of treatment in patients with unilateral adrenal hyperplasia. In patients who have unilateral disease, surgical intervention results in definitive treatment. However, in patients who are surgical candidates, spironolactone may be useful to predict the patient's response to surgery. When there is a good response to spironolactone, the response to surgery is usually better. Bottom Line: Primary hyperaldosteronism can be caused by an adrenal adenoma or bilateral adrenal hyperplasia. It manifests as refractory hypertension and hypokalemia. Surgery can potentially be curable. TrueLearn Insight : Primary hyperaldosteronism should be suspected in the setting of refractory hypertension and hypokalemia.

rectovaginal fistula

All of these tests can be used to assess for the presence of a rectovaginal fistula. Answer A: The methylene blue tampon test entails placing a tampon in the patient's vagina and then filling the rectum with methylene blue. Afterwards, the tampon is inspected for the presence of methylene blue. Answer B: The vagina "bubble" test involves filling the vagina with water while performing a rigid sigmoidoscopy on the patient. The vagina is inspected for the presence of air bubbles as the rectum is insufflated with air. Answers C & D: A CT with IV and rectal contrast as well as a barium enema can be performed to evaluate the passage of contrast into the vagina. These tests are not absolute however. A negative result does not automatically exclude a fistula, so clinical suspicion should guide further workup. Answer E: A CT with IV and rectal contrast as well as a barium or gastrograffin enema can be performed to evaluate the passage of contrast into the vagina. These tests are not absolute however. A negative result does not automatically exclude a fistula, so clinical suspicion should guide further workup. Bottom Line: Initial tests to assess for the presence of a rectovaginal fistula includes the methylene blue tampon test, vagina "bubble" test, CT with rectal contrast, and barium enema. The methylene blue tampon test seems to have the best sensitivity when done appropriately.

Thymoma

All thymomas, both well encapsulated and invasive, are treated with aggressive surgical management regardless of symptoms related to myasthenia gravis. Invasive thymomas have improved rates of survival if treated with a radical resection. Postoperative radiation can improve local recurrence rates if there is evidence of pericapsular growth, but treatment is still centered on surgical management. Anticholinesterase inhibitors should be stopped 72 hours prior to surgery to decrease pulmonary secretions.

AAA

Although all the answers increase the risk of abdominal aortic aneurysm (AAA) formation, smoking has the greatest impact. The most striking histopathological changes of aneurysmatic aorta are seen in tunica media and intima. These include accumulation of lipids in foam cells, extracellular free cholesterol crystals, calcifications, thrombosis, and ulcerations and ruptures of the layers. There is an adventitial inflammatory infiltrate. However, the degradation of tunica media by means of proteolytic process seems to be the basic pathophysiologic mechanism of the AAA development. Some researchers report increased expression and activity of matrix metalloproteinases (MMP) in individuals with AAA. This leads to elimination of elastin from the media, rendering the aortic wall more susceptible to the influence of the blood pressure. Hemodynamics affect the development of AAA. It has a predilection for the infrarenal aorta. The histological structure and mechanical characteristics of infrarenal aorta differ from those of the thoracic aorta. The diameter decreases from the root to the bifurcation, and the wall of the abdominal aorta also contains a lesser proportion of elastin. The mechanical tension in abdominal aortic wall is therefore higher than in the thoracic aortic wall. The elasticity and distensibility also decline with age, which can result in gradual dilatation of the segment.

GIST

Although gastrointestinal stromal tumors (GISTs) are the most common nonepithelial cell tumors of the small bowel, they are rare tumors of the gastrointestinal tract, representing only about 5,000 cases nationwide per year. Approximately 25% of GISTs arise in the small bowel, with 50% gastric, 15% rectal, and 10% colonic in origin. GISTs are diagnosed equally in men and women, with a median age at onset of 64 years. Pacemaker cells originate from the interstitial cells of Cajal and are situated between the intramural neurons and the smooth muscle cells. GISTs are defined by the presence of activating c-kit mutations, a transmembrane receptor tyrosine kinase involved in the regulation of cellular proliferation, apoptosis, and differentiation. Answer B: Enterochromaffin cells at the base of the crypts of Lieberkühn are part of the amine precursor uptake and decarboxylation system. Carcinoid tumors arise from this cell. Answer C: Neuroectodermal cells of the adrenal medulla and certain extra adrenal sites are the cell origin for pheochromocytomas. Answer D: Pancreatic islet cells are the cell origin of neuroendocrine tumors. Answer E: Kupffer cells in the hepatic sinusoids are macrophages with an important defensive role in the liver. Bottom Line: Gastrointestinal stromal tumors arise from interstitial cells of Cajal.

Pilonidal disease

Although one technique has not been conclusively shown to be superior to others, the preponderance of evidence suggests: A primary closure is associated with faster wound healing (complete epithelialization) and a sooner return to work, but a delayed (open) closure is associated with a lower likelihood of pilonidal disease recurrence. The choice should be individualized based on the extensiveness of the resection, presence/absence of infection, and surgeon experience/preference. Off-midline closures reduce complication rate, healing time, and recurrence rate compared with midline closure. Thus, for wounds undergoing a primary closure, an off-midline (lateral) closure rather than a midline closure should be used. Flap-based reconstructive options allow for excision of greater amounts of involved tissue and are associated with a decrease in tension in the healing wound. In addition, these techniques facilitate wound closure lateral to the natal cleft, an area characteristically moist, hypoxic, and bacteria laden. Rhomboid, V-Y, and other rotational flap reconstructions are typically reserved for patients with extensive disease or those who have failed simpler operations (eg, excision and sutured midline closure). The Karydaki's flap and Bascom's cleft-lift procedure can be used for initial surgical management or for recurrent disease presenting with anatomy favorable to those procedures. The rhomboid or Limberg flap is a rotational fasciocutaneous flap that permits primary off-midline closure of the wound and flattening of the gluteal cleft. The reported recurrence rate (0 to 6 percent) and surgical infection rate (0 to 6 percent) are both low and compare favorably to those of simple midline closure in some studies. Answer A: Incision and drainage is the procedure of choice for an initial presentation; however, it should not be used in a patient that has recurrent disease or extensive disease. In addition, the incision should be off-midline. Answer B: For patients undergoing a primary wound closure, off-midline (lateral) closure techniques have been associated with lower complication rates, healing time, and recurrence rates compared with simple midline closure techniques. This technique is most appropriate for non-recurrent disease, but the patient above has recurrent disease. Answer C: Pilonidal sinuses can be excised and the defect reconstructed using a standard Z-plasty. The primary reasons to perform a Z-plasty are to improve contour, release scar contracture, relieve skin tension, and mobilize tissue for reconstructive surgery. The 60º Z-plasty (ie, classic Z-plasty) is most commonly used because it provides the optimal balance between lengthening and ease of closure. A Z-plasty should be combined with an off-midline incision, not a midline incision. Answer E: A V-Y advancement flap is another technique of excising pilonidal disease and closing the wound defect. Healing rates of >90 percent and low recurrence rates have been reported in case series, with rhomboid flaps showing slightly lower recurrence rates. Bottom Line: Flaps are complex procedures which are reserved for extensive (stage IV) and recurrent (stage V) pilonidal disease. The rhomboid flap has low recurrence rates of 0-6%, but Z-plasty and V-Y advancement flaps are alternative techniques.

Ruptured umbilical hernia liver cirrhosis

Although umbilical hernias are seen in as many as 20% of cirrhotic patients with ascites, hernia rupture with loss of peritoneal fluid is an uncommon event. The strategy for addressing an umbilical hernia in this population is dependent upon the patient's level of symptoms. Hernias with thinned skin, eschars, rupture, or incarceration require emergent repair. Nonoperative management has been carried out in patients that have refused intervention and death was a nearly universal end point attributable to peritonitis largely resulting from gram positive cocci. Unlike an asymptomatic umbilical hernia associated with refractory ascites which can be managed by TIPS and an elective hernia repair, a ruptured umbilical hernia is a surgical emergency

Aminocaproic acid

Aminocaproic acid (EACA) inhibit fibrinolysis by competitive binding to plasminogen. The resulting complex does not cleave fibrin and acts as antifibrinolytic agent. EACA is used in liver transplantation during the perfusion of donor liver when a fibrinolytic state is encountered. This is due to the lack of production of fibrinolysis inhibitors during the anhepatic phase and the inability of the liver to metabolize profibrinolytic compounds.

Ampullary Mass

Ampullary masses arise from the ampulla of Vater. These must be distinguished from periampullary tumors or growths. Biopsy of an ampullary mass is typically required to determine if carcinoma is present. Endoscopic ultrasound can help identify any duodenal wall invasion. In the absence of invasion, benign adenomas, 2 cm or less, can be excised endoscopically or via transduodenal approach with 2-3 mm margins. A pre-operative diagnosis of carcinoma is a contraindication to local or endoscopic excision and the patient should undergo a pancreaticoduodenectomy. Lesions larger than 2 cm should be considered for pancreaticoduodenectomy. If the final pathology of a locally excised lesion reveals carcinoma, the patient should be returned to the operating room for pancreaticoduodenectomy.

Acute limb ischemia in a young healthy patient

An acute lower extremity ischemic event in a young patient with no real past medical history is usually from an embolus due to atrial fibrillation.

Fistula in ano

An anorectal fistula is an inflammatory tract or connection between the epithelialized surface of the anal canal and, most frequently, the perianal skin or perineum. It often evolves from a spontaneously draining anorectal abscess. Perianal fistulous disease has significant implications for patient quality of life as sequelae range from minor pain and social hygienic embarrassment to frank sepsis. The management of the anorectal fistula (also called fistula-in-ano) remains one of the most challenging and controversial topics in colorectal surgery. Surgery is the mainstay of therapy with the ultimate goal of draining local infection, eradicating the fistulous tract, and avoiding recurrence while preserving native sphincter function. The surgical approach depends on several factors, such as the etiology, location, type, and duration of the fistula, as well as previously performed procedures and preoperative sphincter function. The anatomy of most fistulas can be determined in the OR without preoperative imaging. If the internal openings and extent of sphincter involvement are difficult to identify, intraoperative radiographic studies can be helpful. Simple fistulas include superficial (perianal) fistulas, intersphincteric fistulas, and low-lying transsphincteric fistulas (< 30% sphincter involvement). Complex fistulas include high transsphincteric fistulas (>30% of sphincter involvement), suprasphincteric fistulas, extrasphincteric fistulas, horseshoe fistulas, and recurrent fistulas. For patients with no existing incontinence or risk factors for future incontinence, primary fistulotomy and curettage should be performed, rather than a sphincter-sparing procedure, for a well-drained, low-lying (< 30% of sphincter complex) transsphincteric, and intersphincteric simple fistula. In these patients, fistulotomy results in healing in over 90% of patients with minimal or no de novo fecal incontinence. In patients with preexisting incontinence, however, fistulotomy is contraindicated in all situations. Primary fistulotomy alone is inadequate treatment of complex fistulas because of the high risk of postoperative incontinence, so sphincter-sparing approaches are necessary. Sphincter-sparing procedures include endoanal advancement flaps, fibrin glue, fistula plugs, the modified Hanley procedure, ligation of the intersphincteric fistula tract (LIFT), or diversion. The choice of procedures is dictated by fistula anatomy as well as surgeon preference. Fistulotomy involves laying open the fistula tract in its entirety. It is an effective treatment for simple anal fistulas that results in healing in over 90% of patients. It is critical to assess all patients' incontinence score prior to fistulotomy. In patients with preexisting incontinence, fistulotomy is contraindicated in all situations. Fistulotomy can be performed by unroofing the fistula and performing a curettage to disrupt the tract. Alternatively, fistulotomy can be performed by placement of a snug ("cutting") seton and slowly tightening the seton over time. During fistulotomy and curettage, a probe is inserted into the internal opening and gently passed along the fistula tract to the external opening. An incision is made over the entire length of the fistula using the probe as a guide. The tract is gently curetted, and an absorbable stitch is used to marsupialize the tract to promote healing. The wound is gently packed with a petroleum-based gauze and covered with a sterile dressing. Answer B: A snug seton, also referred to as a "cutting" seton, is a reactive suture or elastic that is placed through the fistula tract and tightened at regular intervals. It slowly cuts through the tract, causing scarring, thus preventing the wide disruption of the anal sphincter associated with fistulotomy. By contrast, a noncutting or draining seton is a seton that is placed primarily for drainage. It does not cut through the sphincter. For complex fistulas, including those that traverse more than 30% of the sphincter, are proximal to the dentate line, or are high transsphincteric fistulas, draining setons are used at the time of the first operation to preserve the sphincter mechanism and help eradicate the septic focus. In general, snug seton placement has fallen out of favor owing to the high levels of pain that patients typically experience. Answer C: Endoanal and endorectal advancement flaps are the preferred approach for complex anorectal fistulas without coexisting incontinence. Because a small layer of circular muscle can be incorporated into the flap, this procedure is typically avoided in incontinent patients. Also, complex fistulas are typically treated first by draining (not cutting) seton placement to see whether the tract closes. Not all anorectal fistula patients are candidates for mucosal flap advancement. Very high fistulas, for example, are technically challenging to treat with this technique. Additionally, anal stenosis, active proctitis, and inflammatory bowel disease are relative contraindications due to high complication and failure rates. Answer D: LIFT is a sphincter-sparing procedure for complex transsphincteric fistulas. It is worth emphasizing that this technique uses ligation of the intersphincteric portion of the fistula tract as part of the treatment of a transsphincteric fistula. This approach is performed through the intersphincteric plane. The procedure involves two major steps: (1) fistulotomy, and (2) secure closure of the internal opening. While LIFT can be used to treat both simple and complex fistulas, it is typically used in simple fistula management only after primary fistulotomy has failed, particularly if there is minimal sphincter involvement. Fistula tract longer than 3 cm, previous procedures, and obesity have been associated with LIFT failure. Answer E: For patients with existing incontinence or concerns for future incontinence, and/or patients with a complex fistula, a draining seton is placed before a second sphincter-sparing procedure is performed after another 6 or more weeks. In one multicenter study, 200 consecutive patients with complex fistulas had loose setons placed and exchanged regularly; all fistulas eventually cleared with a recurrence rate of 6%; however, leaving draining setons indefinitely may result in low-grade incontinence of gas, mucus, or fluid discharge and have an adverse impact on quality of life. The choice of the secondary procedure is dictated by fistula anatomy and surgeon preference. Bottom Line: For patients with no existing incontinence or risk factors for future incontinence, primary fistulotomy and curettage should be performed, rather than a sphincter-sparing procedure, for a well-drained, low-lying (< 30% of sphincter complex) transsphincteric, and intersphincteric simple fistula. TrueLearn Insight : Rectal mucosal advancement flap is a technique that is used not only in the management of fistula in ano but also in rectovaginal fistula. It is the preferred repair technique for a low-lying rectovaginal fistula (over a vaginal mucosal advancement flap), owing to the relatively more robust muscular wall of the rectum that can partially be incorporated into the flap wall without sacrificing the overall integrity of the rectal wall.

Time after PD catheter placement to begin dialysis. Longer if abdominal wall hernia is fixed simultaneously

An approximate 2 week time delay is recommended by most experts to reduce the likelihood of catheter related infection and dialysate leak. However, small studies have shown no difference exists between waiting 2-3 days vs. 1-2 weeks in terms of catheter-related complications. Regardless, most experts would agree that waiting 1-2 weeks is optimal for most patients. Typically, peritoneal dialysis can start 1-2 weeks after a peritoneal dialysis catheter insertion; however, following an abdominal hernia repair, starting so soon should be avoided to allow time for healing. The dialysate in particular increases intraabdominal pressure and can lead to recurrence

Soft tissue mass

An extremity soft tissue mass that is hard, fixed to underlying structures, immobile, more than 5 cm in size, or deep in location (subfascial or intramuscular) requires tissue diagnosis to rule out malignancy after appropriate local imaging (MRI). CNB is the initial test of choice. It usually provides adequate tissue to make the diagnosis and it is less invasive than open biopsy. An incisional biopsy is an acceptable alternative. Principles include incision along a longitudinal access with minimal dissection and good hemostasis. Answer A: Clinically suspicious masses should be biopsied. Answer B: Fine needle aspiration will not provide adequate tissue and has low diagnostic accuracy for sarcoma. Answer D: Excisional biopsy can be done for lesions less than 3 cm, when CNB fails. Answer E: Radical excision would be acceptable following a diagnosis of soft tissue sarcoma. Bottom Line: Tissue diagnosis is important for clinically suspicious masses. Core needle biopsy is generally the diagnostic test of choice.

Anastomotic leak rates

Anastomotic leak is one of the biggest fears of the colorectal surgeon. Risk factors for increased leak rate are higher ASA, emergent surgery, prolonged operating time, and hand-sewn ileocolic anastomosis. The signs of leak are pain, fever, tachycardia, peritonitis, and feculent drainage from the wound. The anastomosis with the lowest leak rate is the ileocolic anastomosis with a 1-3% leak rate, and the highest is the coloanal anastomosis with a 10-20% leak rate. Answer B: Ileocolic anastomosis leak rate is lower at 1%. Answers C & D: Colocolonic leak rate is 1-10%. Answer E: The leak rate is much higher for coloanal anastomosis. Bottom Line: The anastomosis with the lowest leak rate is the ileocolic anastomosis with a 1-3 % leak rate.

Cancer associations

Angiosarcoma is a rare tumor of the liver that is caused by exposure to vinyl chlorides. It has also been linked to exposure to Thorotrast, a radiocontrast used in the 1930s and 1940s. Answer A: Renal cell carcinoma is associated with smoking. Answer C: Transitional cell cancer of the bladder is associated with smoking. Answer D: Lymphoma has a number of risk factors ranging from genetic causes to use of immunosuppressive drugs. Answer E: Gastric cancer is associated with increased nitrite consumption.

Arginine

Animal models have demonstrated enhanced immune function with supplemental arginine administration. It has been theorized that arginine increases T-cell stimulation. Studies have shown that patients with severe trauma, malnourishment, sepsis and ARDS benefit from receipt of enteral nutrition containing increased arginine levels. Answer A: Proline plays an important role in collagen formation. Answer C: Tyrosine plays an important role in signal transduction. Answer D: Glutamine plays important roles in protein metabolism and the citric acid cycle. Answer E: Leucine is involved in muscle protein synthesis. Bottom Line: Arginine is an amino acid that can enhance immune function.

Annular pancreas

Annular pancreas is a rare congenital condition in which a ring of pancreatic tissue from the head of the pancreas surrounds the descending duodenum. The abnormality usually presents in infancy as duodenal obstruction with postprandial vomiting. There is bile in the vomitus if the constriction is distal to the entrance of the common bile duct. X-rays show a dilated stomach and proximal duodenum and little or no air in the rest of the small bowel. Other causes of double bubble sign are duodenal atresia, duodenal web, duodenal stenosis, or external compression. Pancreatic divisum is a failure of the fusion of the dorsal and ventral pancreatic ducts resulting in the majority of the secretions exiting via the accessory pancreatic duct. Type 1 choledochal presents with the triad of abdominal pain, jaundice, and a palpable mass. In patients with an annular pancreas, fluid and electrolyte imbalances should be corrected first. The obstructed segment should then be bypassed by a duodenojejunostomy or other similar procedure. No attempt should be made to resect the obstructing pancreas because a pancreatic fistula or acute pancreatitis often develops postoperatively. Answer B: Endoscopic papillotomy of the minor papilla is the treatment of choice for pancreatic divisum, a process in which the dorsal and ventral pancreatic bud fail to fuse. It leads to an abnormally large duct of Santorini that drains separately into the minor papilla. It typically presents in the third to fourth decade of life with recurrent or chronic pancreatitis. Answer C: Pyloromyotomy is the treatment of choice for hypertrophic pyloric stenosis, which typically presents much earlier in life with nonbilious emesis. Answer D: Cholecystectomy is rarely indicated in the pediatric population, with the exception of sickle cell patients and other blood disorders. Answer E: Duodenojejunostomy is favored over gastrojejunostomy. Bottom Line: Annular pancreas requires surgical management with a duodenojejuonostomy with avoidance of transection of the pancreas.

Pneumocytes

Answer A: Alveoli production continues until approximately 10 years of age. Answer B: Type II (Great Alveolar) cells secrete pulmonary surfactant between 24 and 28 weeks. Answer C: Alveoli production begins at 7 months gestation and continues until approximately 10 years of age. Answer D: The pores of Kohn are found in alveolar walls and allow communication between individual alveoli but do not play a significant role in carbon dioxide delivery. Answer E: Alveoli are squamous-lined sacs at the terminal ends of the respiratory tract that are the sites for gas exchange in the lung.

Acute limb ischemia

Answer A: Bilateral symptoms suggest thrombotic disease. Answer B: The most common cardiac source of emboli is due to atrial fibrillation. Answer C: Previous bypass grafts are more likely to have a thrombotic event. Answer D: Paradoxical emboli embolize through right to left shunts, especially patent foramen ovale. Bottom Line: Acute limb ischemia with an unknown etiology should have CT scan performed to rule out a malignancy.

Pancreatic masses

Answer B: Adenosquamous carcinoma is a rare variant of ductal adenocarcinoma that shows both glandular and squamous differentiation. This variant appears to be more common in patients who have undergone previous chemoradiation therapy. The biologic behavior of adenosquamous carcinoma appears to be similar to that of ductal adenocarcinoma, with similar rates of perineural invasion, lymph node metastases, and dissemination. Answer C: Serous cysts are epithelial neoplasms composed of uniform cuboidal glycogen-rich cells that usually form numerous small cysts containing serous fluid. They can be located anywhere in the pancreas—head, body, or tail—and usually do not communicate with the pancreatic ducts. Answer D: Intraductal papillary-mucinous neoplasms (IPMNs) are intraductal mucin-producing neoplasms with tall, columnar, mucin-containing epithelium with or without papillary projections. These neoplasms extensively involve the main pancreatic ducts and/or major side branches. IPMNs lack the ovarian stroma characteristic of MCNs. Answer E: Mucinous cystic neoplasms (MCNs) are composed of mucin-producing epithelial cells associated with an ovarian type of stroma. These cysts usually do not communicate with the larger pancreatic ducts.

DIC

Anti-thrombin III and plasminogen levels will be decreased in DIC. Answer A: Both small and large vessels are at risk for thrombosis. Answers B & D: Laboratory values will show an elevated PT and aPTT, with thrombocytopenia and an increased bleeding time. Answer C: The patient is in disseminated intravascular coagulation (DIC) secondary to sepsis. DIC is a thrombohemorrhagic disease that can result from hemolysis, trauma, sepsis, obstetric emergencies, and malignancies. There is an increase in both thrombin and plasmin in the circulation which predisposes to both bleeding and clot formation. Bottom Line: DIC can cause bleeding as well as thrombosis of small and large vessels.

Side effects of topical burn treatments

Antimicrobial topical therapies are commonly used in the treatment of burn wounds. While individual treatments have specific advantages, they have associated complications as well. While silver sulfadiazine (silvadene) has broad spectrum coverage, including pseudomonas infection, it does not penetrate eschar well and is associated with transient neutropenia and thrombocytopenia. It can also lead to methemoglobinemia, and its use is contraindicated in patients with G6PD deficiency. Answer A: Mafenide acetate has broad spectrum coverage and penetrates eschar, making it effective in both treating and preventing burn wound infections. Mafenide acetate's mechanism of action is to inhibit carbonic anhydrase leading to a metabolic acidosis. This can ultimately lead to difficult ventilator management. Answer B: Bacitracin has good Gram-positive coverage and is optimal on shallow facial burns; however, it is oftentimes dosed with nephrotoxic agents. Answers D & E: Silver nitrate also has broad spectrum coverage but is associated with electrolyte disturbances, such as hyponatremia, hypochloremia, hypocalcemia, and hypokalemia. Bottom Line: More commonly used agents, such as silver sulfadiazine and mafenide acetate, are associated with transient neutropenia and thrombocytopenia and metabolic acidosis, respectively. TrueLearn Insight : Side effect profiles of topical burn therapies are commonly tested.

Anti-retrovirals

Antiretroviral medications should generally be continued through the perioperative period when feasible. However, if clinically necessary, stopping antiretroviral medications for a few days should not have a deleterious impact on their effectiveness. Viral resistance is more likely to occur when doses of some medications are intermittently missed over an extended period of time. When altered mental status or gastrointestinal tract dysfunction interfere with the ability of the patient to take or absorb oral medications, all antiretroviral drugs should be held. Parenteral alternatives should be sought for agents used for prophylaxis for opportunistic infections (e.g., intravenous trimethoprim-sulfamethoxazole) if it is anticipated that the patient will be unable to have oral intake for an extended duration of time. Liquid preparations are available for many antiretroviral agents for patients who may have difficulty swallowing.

Anti-thrombin III deficiency

Antithrombin deficiency is an autosomal dominant genetic disorder. Antithrombin III (ATIII) is a potent inhibitor of the coagulation cascade. It is a nonvitamin K-dependent protease that inhibits coagulation by lysing thrombin and factor Xa. ATIII activity is markedly potentiated by heparin; potentiation of its activity is the principle mechanism by which both heparin and low molecular weight heparin result in anticoagulation. Plasma is used to replace labile factors in patients with coagulopathy and documented factor deficiency, such as congenital deficiency of antithrombin III; prothrombin; factors V, VII, IX, X, and XI; protein C or S; plasminogen or antiplasmin. Answer A: Cryoprecipitate is useful in treating factor deficiency (hemophilia A), von Willebrand's disease (vWD), and hypofibrinogenemia and may help treat uremic bleeding. Answer B: Platelet transfusions are indicated for patients suffering from or at significant risk for bleeding as a result of thrombocytopenia or platelet dysfunction (as in renal failure or vWD), or both. Answer D: Factor VIIa is FDA approved for the treatment or prevention of bleeding episodes in hemophilia A or B patients with inhibitors to factor VIII or factor IX and for the treatment or prevention of bleeding episodes in patients with congenital factor VII deficiency. Answer E: Desmopressin (DDAVP) may temporarily raise factor VIII levels in a patient with mild hemophilia A.

Testicular mass

Any patient with a solid testicular mass, which has been confirmed on ultrasound, is considered to have testicular cancer until proven otherwise, and should undergo a radical orchiectomy to make a definitive diagnosis. When performing a radical orchiectomy, the surgery should be performed by an inguinal approach rather than a scrotal approach. If the scrotum is surgically violated by performing a scrotal orchiectomy, metastatic spread to both the retroperitoneal and the inguinal nodes becomes possible. Scrotal approaches should be avoided as they can expose the immune system to the haploid cells of the testicle.Inguinal exploration with early vascular control of the spermatic cord structures is the initial intervention to exclude testicular neoplasm. If cancer cannot be excluded by examination of the testis, then radical orchiectomy is warranted. Scrotal approaches and open testicular biopsies should be avoided.

SLNB for skin cancer complications

Approximately 10% of patients with lymphoscintigraphy and SLNB have complications. In the Multicenter Selective Lymphadenectomy Trial (MSLT-1), the rates of wound infection, wound separation, and seroma/hematoma were reported as 4.6%, 1.2%, and 5.5%, respectively. Anaphylaxis to lymphazurin for melanoma SNLB occurs in < 1% of patients. Answer B: Lymphedema occurs in 1% of patients undergoing SLNB for skin cancer. Answer C: Seroma/hematoma rate is 5.5% following SLNB. Answer D: Wound infection rate is less than 5%. Answer E: Frozen section may be sent if the sentinel node looks grossly metastatic. Otherwise, sentinel node is sent for permanent pathology most of the time. Bottom Line: Complication rate associated with SLNB is about 10%. Less than 1% of patients develop anaphylaxis with the lymphazurin dye.

CV response in geriatric population

As people age, a number of cardiovascular changes occur that can affect decision-making and patient presentation. The most significant change seen can be the lack of tachycardia or hypotension with hypovolemia or shock. Older persons have decreased cardiac function (up to 50% less) and a decreased sensitivity to catecholamines and thus, do not mount a robust sympathetic response. The heart is increasingly replaced by fatty and fibrous tissues resulting in a stiffer heart. Peripheral and central arteries are more atherosclerotic. The result is increased afterload. Since elderly patients are unable to compensate with tachycardia, systemic vascular resistance is increased in the face of hypovolemia. Many geriatric patients are also on beta blockers which further blunts the sympathetic response. Therefore, a high index of suspicion must be maintained for hemorrhage or hypovolemia.

Mechanism of Aspirin

Aspirin is an irreversible inhibitor of prostaglandin metabolism of platelets that acts by acetylating cyclooxygenase. This causes permanent dysfunction of the platelets over the span of their 7 day existence. These qualitative defects can be measured by abnormal bleeding timesA

Ranson's criteria

Assessment of severity of acute pancreatitis is critical because it dictates the potential for complications; thus, intensive monitoring and aggressive therapies can be instituted to attempt to prevent these complications. Routine clinical assessment at the time of admission is not reliable in detecting patients with severe acute pancreatitis. Therefore, several alternative methods have been designed for assessing disease severity based on scoring systems, CT scanning, and serum markers. The Ranson criteria is the most commonly used in clinical practice. It is easily calculated, and the scores are well correlated with morbidity and mortality rates. Parameters used to calculate the Ranson score 48 hours after admission are: hematocrit decrease, >10% blood urea nitrogen, increase >5 mg/dL calcium, <8 mg/dL PaO2, <60 mm Hg base deficit, >4 mEq/L fluid requirement, >6 L The total score is the initial score + the 48-hour score. This patient's score is 3 + 3 = 6. Three points are given for white blood count >16, age >55, and glucose >200 for the first 24 hours. Three additional points are given for hematocrit drop >10%, calcium <8, and fluid needed >6 L at the 48-hour mark. Three points are added at 48 hours to the Ranson score.

Atrial fibrillation s/p surgery

Atrial fibrillation is a relatively common complication after pulmonary resection. All patients undergoing major pulmonary resection should be admitted to units with telemetry capabilities to ensure rapid detection of any cardiac arrhythmia. Patients with new onset atrial fibrillation who are hemodynamically stable and have no or minimal symptoms should be treated with rate control first. The rate control agent of choice in patients with underlying COPD or bronchospasm is a calcium channel blocking agent.

atypical ductal hyperplasia

Atypical ductal hyperplasia is a benign, proliferative lesion with atypia. Although atypical ductal hyperplasia is a benign lesion, it is associated with a relative risk of developing breast cancer of 4.0-5.0. In 18-50% of cases of atypical ductal hyperplasia there is a concurrent associated cancer, which is invasive in about one-third of those cases. A finding of atypical ductal hyperplasia on core needle biopsy is an indication for excisional biopsy to evaluate whether or not cancer is associated with the lesion; this is done essentially to rule out sampling error of core needle biopsy. If the lesion is removed and there is not associated cancer then no further treatment is necessary. If cancer is identified, then treatment is based on the pathological diagnosis of the excisional biopsy. Other benign breast lesions that are indications for excisional biopsy include atypical lobular hyperplasia, radial scar, columnar cell hyperplasia with atypia, papillary lesion, lack of concordance between appearance of mammographic lesion and pathologic histology, and nondiagnostic specimen. Answer A: Atypical ductal hyperplasia is associated with cancer in 18-50% of cases; therefore, excisional biopsy is indicated. A 6-month followup puts the patient at risk of having an untreated cancer., Answer C: Atypical ductal hyperplasia is not an invasive cancer; therefore, sentinel lymph node biopsy is not indicated. Answer D: Repeat core needle biopsy is not indicated with a diagnosis of atypical ductal hyperplasia. Answer E: Although there is some benefit to tamoxifen chemoprevention in atypical breast disease, a concurrent cancer, which would require further treatment, has not been ruled out. Bottom Line: Excisional biopsy is indicated for a finding of atypical ductal hyperplasia on core needle biopsy. TrueLearn Insight : Benign and malignant breast diseases are heavily tested.

Autoimmune pancreatitis

Autoimmune pancreatitis (AIP) is a rare type of chronic of pancreatitis. Males are affected more often than females (2:1). AIP can affect the pancreas only or involve other organs including the bile ducts, small and large bowel, salivary and lacrimal glands, kidneys, and lungs, causing jaundice, symptoms of inflammatory bowel disease, xerostomia and xerophthalmia, mild renal insufficiency, and pulmonary infiltrates and lymphadenopathy, respectively. About a third of patients will present with disease limited to the pancreas. Patients with AIP most often present with jaundice, while abdominal pain is less common. Elevated serum IgG4 is specific, but not sensitive (a normal IgG4 does not rule out AIP). Computed tomography may reveal a diffusely enlarged pancreas with little to no peripancreatic fat stranding and intrapancreatic calcifications are not seen. Occasionally AIP focally involves the head of the pancreas and appears as a mass in the head of pancreas on computed tomography. Regional lymph nodes are often enlarged on imaging. The pancreatic duct may be focally, segmentally, or diffusely narrowed. A diagnosis of AIP can be made if clinical presentation is consistent with AIP in patients with a history of associated autoimmune disorders, elevated IgG4, and radiographic findings consistent with AIP. In some cases endoscopic retrograde cholangiopancreatography may provide useful information on the morphology of the pancreatic duct which is often strictured and the right angle ducts are lost. Answer A: This patient has xerostomia, xerophthalmia, mild renal insufficiency, and an elevated IgG4 making autoimmune pancreatitis more likely than alcoholic chronic pancreatitis. Answer B: Idiopathic chronic pancreatitis is a diagnosis of exclusion. This patient has enough clinical findings to make a specific diagnosis of autoimmune pancreatitis. Answer C: This patient's clinical presentation and laboratory findings are consistent with autoimmune pancreatitis. Answer E: It may be difficult to distinguish autoimmune pancreatitis from pancreatic adenocarcinoma. This patient presents with xerostomia, xerophthalmia, and mild renal insufficiency, along with an elevated IgG4 (which is specific for autoimmune pancreatitis), making autoimmune pancreatitis the likely diagnosis. Bottom Line: Extrapancreatic involvement and an elevated IgG4 can help make a diagnosis of autoimmune pancreatitis. TrueLearn Insight : An elevated IgG4 is specific, but not sensitive, for AIP. Therefore, a normal IgG4 level is also consistent with a diagnosis of autoimmune pancreatitis. It can be difficult to differentiate autoimmune pancreatitis from pancreatic adenocarcinoma.

Chronic appendicitis

Based on the gross appearance of the appendix, this patient most likely has chronic appendicitis. Chronic appendicitis is typically characterized by a 3-week (or longer) history of intermittent right lower quadrant pain and a normal white cell count. Gross examination and pathologic findings of the appendix will show chronic active inflammation of the appendiceal wall and/or fibrosis of the appendix. The majority of patients will typically have a complete resolution of pain after undergoing an appendectomy.

Beta blocker overdose

Beta-blocker overdose is characterized by hypotension and bradycardia. The initial management of beta-blocker overdose is IV fluid and atropine (1mg IV, which can be repeated up to 3 times) to treat rapidly the hypotension and bradycardia. If the desired response is not achieved, the next treatment of choice is glucagon. The next therapies, which can be given simultaneously if a patient is experiencing profound hypotension, include vasopressors, calcium chloride or calcium gluconate, lipid emulsion, and insulin with glucose. Glucagon antagonizes the cardiac depression which occurs in beta-blocker toxicity. It is important to remember specific antidotes to commonly used drugs in the perioperative period. Glucagon is also effective in reversing toxicity of calcium channel blockers

Antidotes

Beta-blocker overdose is characterized by hypotension and bradycardia. The initial management of beta-blocker overdose is IV fluid and atropine (1mg IV, which can be repeated up to 3 times) to treat rapidly the hypotension and bradycardia. If the desired response is not achieved, the next treatment of choice is glucagon. The next therapies, which can be given simultaneously if a patient is experiencing profound hypotension, include vasopressors, calcium chloride or calcium gluconate, lipid emulsion, and insulin with glucose. Glucagon antagonizes the cardiac depression which occurs in beta-blocker toxicity. It is important to remember specific antidotes to commonly used drugs in the perioperative period. Glucagon is also effective in reversing toxicity of calcium channel blockers. Sugammadex is a reversal agent for the neuromuscular blockers rocuronium and vecuronium.

Breast Screening

Biennial screening mammograms can begin starting at age 40, and annual screening mammograms should begin at age 50 or 10 years before the earliest diagnosed breast cancer in a relative. Screening should be yearly and continue as long as the patient is suitable for treatment if cancer is found

Bile leak

Bile leaks commonly present shortly after cholecystectomy (within 1 week) with right upper quadrant pain, fever, chills, and hyperbilirubinemia. Bile leak or bile peritonitis should be considered in any patient with persistent bloating or anorexia more than a few days after laparoscopic cholecystectomy. Leaks from the cystic duct stump or an unrecognized duct of Luschka may be problematic. If either complication is found, they are managed the same. The most common etiology for a cystic duct stump leak is an inflammation around the duct in the setting of acute cholecystitis, which dislodges placed clips. The port sites on the right side should be examined for bile staining. CT scan and ultrasound will confirm the presence of a complex fluid collection in the right upper quadrant. Percutaneous drainage of intra-abdominal fluid collections followed by an endoscopic biliary stenting resolves most leaks without the need for operative intervention. If bile leaks fail to resolve after 6 weeks, further imaging with magnetic resonance cholangiography and endoscopic imaging may be necessary to rule out a common bile duct injury. When the acute inflammation has resolved 6-8 weeks later, operative repair is performed. Immediate operative intervention with wide drainage is only indicated if the patient is in septic shock. Attempts at early repair are dangerous and doomed for failure because of the inflammatory response incited by the bile leak.

enterohepatic circulation

Bile salts are synthesized from cholesterol within the liver where they are conjugated to either glycine or taurine to form the primary bile salts: cholic acid and chenodeoxycholic acid. Once secreted into the bile, they pass into the duodenum and are absorbed throughout the small intestine with a predominance in the terminal ileum. The portal system returns the bile salts to the liver completing the enterohepatic circulation. This is an extremely conservative process with 95% of the circulating bile pool being recycled. Thus, although nearly 2 to 4 grams of bile salts are dumped into the duodenum daily, only approximately 600 mg are actually excreted into the colon. Colonic bacteria act upon the primary bile salts leading to the formation of the secondary bile salts, deoxycholate and lithocholate. A small amount of this is reabsorbed passively, however, the remainder is lost in the stool. Recirculation of these salts can be interrupted by resection of the terminal ileum as is often required in patients with complicated Crohn's disease. Answers B & D: The esophagus and gastric antrum are proximal to the inflow bile and thus unrelated to the enterohepatic circulation. Although, gastric antrectomy may lend to its own list of complications including iron, folate, and vitamin B12 deficiencies these do not lead to an increased loss of bile salts.

Apoptosis

Both intrinsic and extrinsic pathways activate the process of apoptosis. The extrinsic pathway is initiated by the binding of proapoptotic ligands, such as tumor necrosis factor (TNF), to death receptors on the cell surface which triggers an intracellular cascade. The intrinsic apoptotic pathway begins the cell death cascade by recognizing intracellular signals of cell demise such as genotoxic damage or growth factor and nutrient deprivation via the Bcl-2 protein family. These two activation systems merge to follow the same final cell death pathway. Caspase is the common facilitator of apoptosis for both the intrinsic and extrinsic pathways. The full name of this group of enzymes is cysteine aspartate proteinases. Once activated, caspases serve many functions in cell death. They destroy cells via the physical breakdown of cytoskeleton and other structural proteins. They affect DNA synthesis and repair by inactivating the repair enzymes and inducing DNA fragmentation enzymes. They also inhibit cell adhesion molecules thus isolating cells from other cells and their surroundings

Biliary injury

Biliary injury in hepatic trauma is relatively common. The majority of injuries are to small intrahepatic bile ducts. These are often treated intraoperatively with electrocautery and tying off large visualized vessels, debridement of devitalized tissues and placement of closed suction drains in high-grade liver injuries. Extrahepatic biliary tract injury is uncommon, with the gallbladder being the most common site. If the gallbladder is injured, a cholecystectomy is performed. If further injury is suspected, the porta hepatis should be inspected closely. Simple injury <50% in diameter may be primarily repaired. Complex injury may require Roux-en-Y reconstruction as stricture rate approaches 50% in this setting. Those that develop and abscess or biloma are usually treatable with percutaneous drainage alone. Further treatment is based upon drain output. Those that drain less than 300 mL/day will usually close spontaneously. The more than 300 mL drains daily, the injury should be localized with fistulogram, endoscopic retrograde cholangiopancreatography, radionucleotide scan, or transhepatic cholangiogram. Sphincterotomy may help close biliary leaks. Major ductal injuries may be stented or require operative repair. Persistence of drainage more than 50 mL/day beyond 2 weeks indicates development of a biliary fistula. Such cases often resolve without further intervention. Answer A: With gallbladder injury, cholecystectomy should be performed. Answer B: A cholecystectomy should be performed, but drains should also be placed in high-grade liver injuries. Answer D: A cholecystectomy is the treatment for gall bladder injury in trauma. Answer E: Closed suction drains should be placed in all high-grade liver injuries and biliary injuries. Bottom Line: Gall bladder injuries are treated with cholecystectomy. Drains should be placed in all high-grade liver injuries.

Bleeding Duodenal ulcer

Bleeding duodenal ulcers are usually a result of a posterior perforation into the gastroduodenal artery (GDA). This causes brisk bleeding that may present as bright red blood per rectum. Stable patients can have an endoscopy to attempt to clip or inject the vessel for hemostasis. If the patient rebleeds but remains stable, a second attempt at endoscopic therapy can be used. Any instance of hemodynamic instability or the need for over 4 units of blood is cause for operative intervention. For rapid control of bleeding, a longitudinal duodenotomy is made, and a figure of eight stitch applied above and below the bleeding vessel. A U stitch is then placed to ligate the transverse pancreatic branches that enter the GDA posteriorly. The duodenotomy is then usually closed transversely to complete the pyloroplasty. Oversewing of the vessel with a U stich to secure the GDA and transverse pancreatic artery is the procedure of choice for a bleeding duodenal ulcer in a hemodynamically unstable patient

Pancreatic vasculature

Both the celiac trunk and the superior mesenteric artery provide the arterial supply to the pancreas. Variations are common, but, for the most part, the body and tail are supplied by branches of the splenic artery, whereas the head and uncinate process receive their supply through arcades originating from the hepatic and gastroduodenal branch of the celiac artery and from the first branch of the superior mesenteric artery. The common hepatic, not the proper hepatic, branches into the gastroduodenal artery which does supply the pancreatic head. Answer A: The right gastroepiploic artery supplies the greater curve of the stomach. Answer B: The superior, not inferior, mesenteric artery supplies the head of the pancreas. Answer C: The common, not the proper, hepatic artery gives off the gastroduodenal artery which supplies the head of the pancreas. Answer E: The greater pancreatic artery is a branch off the splenic artery. The splenic artery gives off branches that supply the neck, body, and tail of the pancreas. Bottom Line: The arcades of the superior and inferior pancreaticoduodenal arteries which supply the head of the pancreas and the uncinate process originate from the celiac (common hepatic → gastroduodenal artery) and SMA.

Bloody diarrhea after endovascular AAA repair

Bloody diarrhea after any aortic case is concerning for bowel ischemia. The most likely inciting event for this process was coverage of the inferior mesenteric artery by the endograft. The first step in evaluation of bloody diarrhea in a stable patient is sigmoidoscopy as this watershed area is the most likely area for bowel ischemia. In patients who are hemodynamically unstable or have evidence of free air on x-ray signifying a perforated viscous, immediate exploration in the operating room is required. After aortic surgery bowel ischemia presents with bloody diarrhea and is typically in the sigmoid distribution. Sigmoidoscopy is necessary in stable patients, but in the presence of an acute abdomen or free air emergent operative intervention is required

Blunt cardiac injury

Blunt cardiac injury is a broad term used to describe a spectrum of cardiac injuries. The classic injury pattern is related to blunt precordial injury; however, sternal fracture does not directly correlate with cardiac injury. While the overall clinical significance is largely unknown, there are rare cases of hemodynamic instability and a wide range of electrocardiac abnormalities due to cardiac contusions. Sinus tachycardia is the most common, with premature atrial contractions and premature ventricular contractions also very prevalent. Occasionally, right bundle branch block and ST changes may occur. When blunt cardiac injury is suspected, an admission EKG is recommended as a screening test. ECHO may be performed if significant EKG abnormalities are present to assess motion and valve competency. Troponin levels are not routinely obtained, especially in younger individuals. Once a blunt cardiac injury is identified, the patient is monitored on telemetry. If hemodynamic changes or ECHO abnormalities are also present, ICU monitoring is recommended. Answer A: Serial troponins are not needed. ICU admission is only necessary if the ECHO is abnormal or hemodynamic instability is present. Answer C: Admission to the ICU is only necessary for hemodynamic instability or abnormal ECHO findings. This patient is hemodynamically normal. Answer D: Hemodynamic abnormality after a blunt cardiac injury is rare. There is no need for defibrillator implantation. Answer E: An ECHO should be obtained. At the least, the patient should be placed on telemetry. Bottom Line: Management of blunt cardiac injury is monitoring with telemetry and ECHO. If the ECHO is abnormal, ICU admission is recommended. Hemodynamic instability is uncommon.

Central pontine myelinolysis

Brain adaptations that reduce the risk of cerebral edema in chronic hyponatremia make the brain vulnerable to injury if the hyponatremia is too rapidly corrected. The neurologic manifestations associated with overly rapid correction have been called the osmotic demyelination syndrome (ODS, formerly called central pontine myelinolysis or CPM). This syndrome is caused by rapid correction of sodium deficit at greater than the appropriate rate of 0.5 mEq/L/hour. The risk is greatest for patients who have had a low sodium level for greater than 48 hours. Cerebral edema with brainstem herniation is seen with rapid correction of hypernatremia

brain death

Brain death relies primarily on clinical examination. Spinal reflexes, such as limb withdrawal, are not an indication of brain function and, therefore, do not rule out brain death. An apnea test is also performed. In order to perform this test there are prerequisites that must be met: core temperature of 36.5 ºC (97 ºF), systolic blood pressure greater than 90 mm Hg, diabetes insipidus corrected to a positive fluid balance, and partial pressure of carbon dioxide (PCO2) must be normal (35-45 mm Hg). The patient is preoxygenated with 100% O2 for 30 minutes. A pulse oximeter is connected, and the patient is disconnected from the ventilator. The examiner looks closely for respiratory movements. Partial pressure of oxygen, PCO2, and pH are measured after 10 minutes, and then the patient is reconnected to the ventilator. If respiratory movements are absent and arterial PCO2 is 60 mm Hg, the apnea test result is positive. Ancillary tests such as electroencephalography or cerebral blood flow analysis may also be employed as an adjunct to clinical findings. Most institutions require at least two examiners with an appropriate period of observation ranging from 6 to 24 hours.

Breast cancer during pregnancy

Breast cancer during pregnancy accounts for 7-14% of breast cancers when considering all women of childbearing age diagnosed with breast cancer. Breast cancer occurs in 0.022% of pregnancies. A palpable mass is the most common presentation. Patients often diagnosed with more advanced tumors and involved lymph nodes because of delayed diagnosis. The density of the breast tissue may limit the usefulness of mammography, but pregnancy is not a contraindication for mammography. Ultrasound often identifies a suspicious mass. Regardless of imaging results, core needle biopsy should be performed, and is preferred over fine-needle aspiration, for any suspicious mass found on physical exam. There is no benefit to termination of the pregnancy. Radiation is contraindicated throughout pregnancy. Partial mastectomy can be performed during any trimester if radiotherapy can delayed until after delivery. Mastectomy is an option during any trimester. Breast mound reconstruction is contraindicated during pregnancy because of the length of the procedure and the difficulty in achieving symmetry. Sentinel lymph node biopsy can be performed with radioactive tracers, but vital dyes should be avoided because they are teratogenic. Chemotherapy can be given during the second and third trimesters, but there is still a 2% risk of fetal malformation. Trastuzumab and hormone therapies are contraindicated during pregnancy. Women should not breastfeed if they are undergoing chemotherapy or hormone therapy.

Breast infections

Breast infections, or mastitis, can be classified as lactating or nonlactating. Lactating infections are usually caused by Staphylococcus aureus. They are most common during the first 4-6 weeks of breastfeeding or during weaning. They are caused by the proliferation of bacteria in poorly drained breast segments and present as cellulitis with fever, pain, redness, and swelling. First-line treatment is antibiotics, and breastfeeding should be continued to facilitate drainage of the engorged segment (if not epidemic puerperal mastitis). Tetracycline, chloramphenicol, and ciprofloxacin should be avoided because they enter breast milk. If the infection fails to resolve, an abscess should be considered. When the breast skin is intact, repeated aspiration of the abscess combined with antibiotics is the preferred treatment. When the skin is thinned, a small incision will facilitate drainage. Breastfeeding should cease in the case of epidemic puerperal mastitis (caused by methicillin-resistant S. aureus) with purulent nipple drainage. Answer B: Fevers are not a contraindication to continuation of breastfeeding. Answer C: Treatment with antibiotics is not a contraindication to continuation of breastfeeding. Answer D: While nipple fissuring may be painful, if there is no purulent drainage from the nipple, breastfeeding or pumping should continue. Answer E: Breast warmth and erythema are nonspecific signs of mastitis and are not contraindications to continuation of breastfeeding. Bottom Line: Mastitis is commonly seen in breastfeeding, postpartum women. Treatment includes antibiotics and continued breastfeeding unless the cause is epidemic puerperal mastitis, caused by methicillin-resistant Staphylococcus aureus.

Budd-Chiari syndrome

Budd-Chiari syndrome is caused by hepatic venous obstruction. The obstruction typically occurs at the level of the inferior vena cava, the hepatic veins, or the central veins within the liver. The etiology of this syndrome has a geographical variation. In the West, acute or chronic thrombosis and malignancy is the most common etiology. In Asia, membranous webs are the major cause of obstruction of the vena cava and hepatic veins. Answers A & B & C & D: The most common causes of Budd-Chiari syndrome in the WEST are hypercoagulable conditions associated with polycythemia vera, paroxysmal nocturnal hemoglobinuria, myeloproliferative disorders and conditions associated with high estrogen levels such as pregnancy and use of contraceptive pills.

Burns

Burn wounds are characterized based on depth of injury. First-degree burns are the most superficial, and fourth-degree burns are the most severe. Identifying the severity of a burn wound is important because treatments vary, depending on the depth of tissue involved. Answer B: Flame injuries also cause thermal destruction of tissue. When they penetrate into, but not through, the dermis, they are characterized as second-degree burns. These will require excisional debridement and grafting if the dermal appendages are lost. Answer C: Electrical injuries are often not classifiable based on first, second, or third degree. They follow the path of least resistance and often cause the most severe injuries deep in the muscles or bones. Answer D: Sunburn affects the epidermis or very superficial dermis only. It is classified as a first-degree burn. Answer E: Frostbite injuries are not classified based on first, second, or third degree. They often require a period of observation for the wounds to declare themselves prior to debridement.

Burn wounds

Burn wounds are characterized based on the depth of involvement. Deep second-degree wounds extend through most of the epidermis but not all the way into the dermis (third-degree burns). The wounds will characteristically appear blistered or weeping. Patients experience significant pain from these wounds. Large, deep second-degree wounds will require excision and grafting. Dermal appendages are destroyed by thermal injury and are not capable of epithelializing the wound. If left alone to heal, the wound would slowly heal from the edge of normal skin only. It would not heal the wound quickly enough, if at all.

HIV CMV infection

CMV infection of the GI tract may lead to submucosal vasculitis which can cause thrombosis of the vessels, ischemia, ulceration, gangrene, and perforation of the involved bowel.

Etiologies of SIADH

CNS disturbance (cerebrovascular accident, cerebral hemorrhage, CNS infection, head trauma, psychosis) Ectopic ADH production by tumor (small cell lung cancer, olfactory neuroblastoma) Drugs (chlorpropamide, carbamazepine, cyclophosphamide, vincristine, amitriptyline, MAOIs, SSRIs, methotrexate, imatinib, amiodarone, ciprofloxacin, vasopressin, desmopressin, oxytocin, and many others) Surgery (hypersecretion of ADH is commonly seen following surgery) Hypopituitarism Hypothyroidism Infections (pneumonia, HIV) Hereditary SIADH Idiopathic SIADH

Mesenteric venous thrombosis

CT scan is the best test to diagnose mesenteric venous thrombosis. It has been shown to detect nearly 100% of acute cases and over 90% of chronic cases. Treatment for mesenteric venous thrombosis consists of anticoagulation as first line therapy. Some patients may require exploratory surgery and resection of frankly necrotic bowel.

TPN calculations

Calculation of TPN composition requires the knowledge of the stored energy of the components. Protein provides 4 kcal/g of energy, dextrose provides 3.4 kcal/g of energy, and lipids provide 9 kcal/g of energy. Oral carbohydrates provide 4 kcal/g of energy.

Cameron ulcers

Cameron ulcers are associated with hiatal hernias. They are often found on the lesser curvature and are thought to be caused by mechanical trauma, ischemia, and peptic injury. They are found in about 5% of patients with hiatal hernias and more frequently with larger size hernias. Patients with bleeding Cameron ulcers may require repair of their diaphragmatic defect for healing to occur.

Cardiac tamponade

Cardiac tamponade is more often seen in penetrating injuries than blunt. Filling of the pericardium with blood can occur with injuries to the heart, great vessels, or pericardial vessels. Regardless of the mechanism, the collection of blood in the pericardium can result in tamponade. Only a small amount of blood in the pericardium is needed to restrict cardiac activity and impair cardiac filling. Rising intrapericardial pressure causes hemodynamic instability and decreased cardiac perfusion. This is described in three phases. In the first phase, rising pericardial pressure restricts ventricular diastolic filling and reduces subendocardial blood flow. In this phase cardiac output is maintained by compensatory tachycardia, increased systemic vascular resistance, and elevated ventricular filling pressure. In the second phase, rising pericardial pressure further compromises diastolic filling, stroke volume, and coronary perfusion, resulting in diminished cardiac output. Initial signs of shock such as anxiety, diaphoresis, and pallor become evident. In the third phase, compensatory mechanisms fail as the intrapericardial pressure approaches the ventricular filling pressure. Cardiac arrest results as profound coronary hypoperfusion occurs. Signs of cardiac tamponade may be difficult to detect. Beck's triad of muffled heart sounds, jugular venous distention, and pulsus paradoxus is present in only around 15% of patients who are later judged to have tamponade

Cardiac tamponade

Cardiac tamponade may occur after penetrating or, less commonly, blunt thoracic injuries. The pericardium is relatively resistant to distention. Consequently, as little as 100 mL of acute blood collecting in the pericardial space can cause symptoms of tamponade. Beck's triad of symptoms includes hypotension, dilated neck veins and muffled heart sounds. Hypotension is caused by pericardial pressure exceeding right atrial pressure leading to impaired atrial filling. Right ventricular preload decreases as a result and CVP increases. Myocardial blood flow is also reduced which can lead to cardiac ischemia and continued decrease in cardiac output.

Chronic rejection s/p cardiac transplant

Cardiovascular disease remains a significant cause of morbidity and mortality in transplant recipients. After the first year, the most common causes of death in transplant recipients are allograft loss from chronic rejection and death of the patient with a functioning graft secondary to cardiovascular death, disease, or infection. Among patients who survive beyond the first year, the primary limits to longterm survival are cardiac allograft vasculopathy and malignancy. Malignancy is a complication of a chronic immunosuppressive therapy. Answer B: In the setting of acute vascular rejection, the response is mediated by IgG antibodies that develop in response to the graft against the endothelial antigens. T cells contribute to the acute vascular rejection episode by responding to the foreign antigen. This response leads to direct lysis of the endothelial cells or the production of cytokines that further recruit and activate inflammatory cells. The end result is endothelial necrosis. This process occurs within the first week of allograft transplantation in the absence of immunosuppression. Answer C: Chronic rejection is manifested as accelerated atherosclerosis in heart recipients and as bronchiolitis obliterans in lung recipients. Answer D: Skin cancer is the most common malignancy following any transplant. Answer E: Lymphomas are the next most common tumor. These are usually non-Hodgkin's B-cell lymphomas and are often related to malignant transformation by Epstein-Barr virus (EBV). Clinical presentation is variable and includes fever, lymphadenopathy, gastrointestinal symptoms, infectious mononucleosis-like syndrome, pulmonary symptoms, CNS symptoms, and weight loss. Most cases are observed in the first post-transplant year. Bottom Line: The cause of mortality from chronic rejection after heart transplantation is from advanced diffuse coronary atherosclerosis.

Causes of pseudohyponatremia

Causes of pseudohyponatremia may lead to treatment that is not directed at the correction of sodium levels, such as in hyperglycemia, mannitol administration, or radiologic contrast medium.

GI surgical antibiotic prophylaxis

Cefoxitin, cefotetan, or cefazolin plus metronidazole are appropriate parenteral prophylactic antibiotics for surgeries of the colon.

Central Cord Injury

Central cord injury is characterized by extremity weakness which is more severe in the arms than the legs. Patients may also have patchy sensory loss and areflexia. This injury typically occurs in the mid to lower cervical spine. Older adults with cervical spondylosis who sustain neck hyperextension injuries are particularly susceptible to central cord injury.

CIN

Cervical intraepithelial neoplasia (CIN) is a premalignant lesion of the cervix that is identified by screening Pap smear exams. CIN with mild dysplasia (CIN-1) is often secondary to an infection and will regress in most patients, requiring only close followup if the patient is compliant. CIN with moderate or severe dysplasia (CIN-2 or -3) discovered on colposcopic examination is better treated by destruction of the cells with cryoablation or a loop electrosurgical excision procedure (LEEP).

Hypotonic solution for children

Children under the age of 2 have decreased ability to concentrate their urine. These children typically have a glomerular filtration rate that is approximately 25% that of an adult. Hence, a large sodium load to these immature nephrons leads to difficulty in sodium excretion, which is why a more hypotonic fluid such as 0.22% NaCl is preferable in younger children. Answer B: In children younger than age 2, their glomerular filtration rates are about 25% that of an adult.

Hepatoblastoma

Children with unresectable or metastatic hepatoblastomas should undergo a biopsy to confirm the histologic diagnosis, followed by chemotherapy to reduce the size of the primary tumor and control metastases. Chemotherapy can reduce tumor size, rendering the tumor resectable, and can even lead to the complete disappearance of lung metastases. Preoperative chemotherapy can also make the tumor less prone to bleeding, and better demarcated from the remaining healthy liver parenchyma, and thus makes surgical resection less challenging. Chemo regimens include cisplatin or vincristine, not vinblastine, is used in combination with cyclophosphamide, 5-FU, and doxorubicin in some regimens.

Cholangiocarcinoma

Cholangiocarcinomas occur more frequently in males and have an average age of 50 to 70 years. Known risk factors are primary sclerosing cholangitis, ulcerative colitis, biliary tract infection and choledochal cysts. Morphologically, cholangiocarcinoma are described as nodular, which is the most common, scirrhous, papillary, and diffusely infiltrating. Histologically, they are classified into acinar, ductular, trabecular, alveolar, and papillary. Papillary histology is associated with a better prognosis and improved outcome.

Reynald's pentad

Cholangitis is a clinical diagnosis described as Reynolds pentad. This constellation of symptoms is comprised of fever, jaundice, abdominal pain, mental obtundation, and hypotension. Patients may also demonstrate leukocytosis, hyperbilirubinemia, and elevated liver enyzymes. The most common causes are related to obstruction of the biliary tree including choledocholithiasis, benign biliary stricture, biliary enteric anastamotic strictures, cholangiocarcinoma, and periampullary tumors. The most common organisms causing cholangitis are Escherichia coli, Klebsiella pneumoniae, Enterococcus, and Bacteroides fragilis. Staphylococcus is not a common cause of infection in biliary sepsis

CCK

Cholecystokinin (CCK) is a gastrointestinal hormone produced by I cells of the duodenum in response to fatty acids and amino acids in the duodenum and inhibited by somatostatin and pancreatic polypeptide. It stimulates gallbladder contraction, sphincter of Oddi relaxation, and pancreatic enzyme secretion. It also decreases gastric emptying, producing the sensation of satiety (to a varying degree). Answer A: These actions aid in digestion and absorption of fatty acids and amino acids. Answer C: CCK is a gastrointestinal hormone produced by I cells of the duodenum. Answer D: CCK stimulates pancreatic enzyme secretion and may also stimulate bicarbonate production along with secretin. Answer E: CCK has an inhibitory effect on hunger by slowing gastric emptying and possibly direct effects on the vagus nerve. Bottom Line: The main function of cholecystokinin is to stimulate gallbladder contraction and relax the sphincter of Oddi. TrueLearn Insight : Gut hormones are high yield for the ABS exams.

CCK

Cholecystokinin is produced by the I-cells of the duodenum and jejunum. Release is stimulated by ingested fat, protein and amino acids. Answer A: CCK inhibits proximal gastric motility and increases antral and pyloric contraction. Answer B: CCK relaxes the sphincter of Oddi and stimulates gallbladder contraction. Answer C: CCK is produced by enteroendocrine cells, specifically the I-cells of the duodenum and jejunum. Release is stimulated by ingested fat and protein. Answer D: CCK stimulates pancreatic secretions. Answer E: CCK is produced by enteroendocrine cells, specifically the I-cells of the duodenum and jejunum. Actions of CCK include: Inhibits proximal gastric motility Increases antral and pyloric contraction Stimulates gallbladder contraction Relaxes the sphincter of Oddi mediates fat-stimulated GLP-1 secretion Stimulates pancreatic secretions Induces satiety Bottom Line: CCK's actions include relaxing the sphincter of Oddi, stimulating gallbladder contraction, stimulating secretion of pancreatic enzymes, and inhibiting gastric emptying.

Treatment of type 2 choledochal cyst?

Choledochal cysts are anatomically classified into type I-V. Type I is the most common type. Choledochal cysts present most commonly in infancy and early childhood with symptoms of obstructive jaundice. Current consensus for the operative treatment of type II choledochal cyst is excision of the cyst and primary closure of the choledochotomy.

Choledochal cysts

Choledochal cysts are anatomically classified into type I-V. Type I is the most common type. In infancy and early childhood, choledochal cysts present most commonly with symptoms of obstructive jaundice. In older children and adults, the classic clinical triad of abdominal pain, a palpable RUQ, and jaundice occurs in fewer than 50% of patients.The diagnosis of choledochal cysts in infants and most children can be confirmed by ultrasound examination and 99mTc-IDA imaging studies.

Cholesterol transporters

Cholesterol is insoluble in blood and must be transported via lipoproteins to enter the circulation. Lipoproteins are involved in transporting cholesterol, phospholipids, and triglycerides in varying concentrations. The lipoprotein with the highest concentration of cholesterol is low-density lipoprotein. Answer A: Chylomicrons is primarily made up of triglycerides. It only carries a small amount of cholesterol. Answer B: Very low-density lipoprotein carries cholesterol, but it is not the main carrier. Answer C: High-density lipoprotein carries a small amount of cholesterol in plasma. Answer D: Micelles are made up of phospholipids.

DNA replication

Chromosomal DNA replication followed by division into daughter cells are the two major events occurring during the cell cycle. These two major events are divided into four phases. G1 is the interval from the previous division to the beginning of DNA synthesis. G2 is the interval between DNA synthesis and nuclear division. S phase is when the DNA synthesis occurs. The M (mitosis) phase is when the chromosomes are separated into each daughter cell. The length of cell cycle and phase length is variable based on the cell type. When looking at a 24-hour cell cycle in a rapidly dividing cell, here are the likely times spent in each phase: G1 phase: 11 hours S phase: 8 hours G2 phase: 4 hours M phase: 1 hour

Most common cause for liver transplantation in the US?

Chronic hepatitis C is the most common indication for transplantation in the US, where there is an estimated 5 million individuals with hepatitis C and about 20% progress to the chronic state developing cirrhosis and liver failure. Hepatitis C infection recurs after transplantation because the infection resides in tissues other than the liver.

Chronic pancreatitis

Chronic pancreatitis is characterized by irreversible changes, including pancreatic fibrosis and the loss of functional pancreatic exocrine or endocrine tissue. Most patients develop chronic pancreatitis as a result of prolonged ethanol abuse. Answer A: This patient may have a history of alcohol use but there are other causes of chronic pancreatitis. Answer B: The pancreas has considerable functional reserve, and more than 90% of exocrine function must be lost before steatorrhea develops. Answer C: Radiographs or CT scans showing pancreatic calcifications are diagnostic of chronic pancreatitis. Those calcifications reflect the deposition of calcium carbonate in the intraductal protein plugs that frequently, but not invariably, occur in chronic pancreatitis. Thus, the absence of pancreatic calcifications does not rule out a diagnosis of chronic pancreatitis. Answer D: In its earliest stages, chronic pancreatitis is an acute inflammatory process, and repeated episodes of subclinical acute pancreatic injury and necrosis lead to the fibrosis of chronic pancreatitis. Bottom Line: Chronic pancreatitis is usually caused by prolonged alcohol abuse and patients present with chronic epigastric pain. Once 90% of the exocrine and endocrine function is lost, patients can develop steatorrhea and diabetes. CT scan can show calcifications and dilated pancreatic ducts.

Colovesicular fistulas

Colovesicular fistulas often present with pneumaturia, fecaluria, or recurrent urinary tract infections. They are usually the result of diverticulitis or perforated colonic cancers. The best diagnostic test is CT scan which may demonstrate air in the bladder. Barium enemas and IV pyelogram fail to show a fistula 50% of the time or more. Cystoscopy will often not show a fistula tract but may reveal bladder inflammation. It is important to assess for cancer as the underlying cause.

Chylous ascites

Chylous ascites is a milky-appearing peritoneal fluid that is rich in triglycerides (defined as a fluid triglyceride level greater than 110 mg/dL). It is due to the presence of thoracic or intestinal lymph in the abdominal cavity. The most common cause in the Western world is malignancy; lymphoma is the most prevalent. Other causes include cirrhosis and congenital lymphedema syndrome. In the developing world, the most common causes include infectious causes, e.g., tuberculosis and filiariasis. Answer B: Tuberculosis is a common cause of chylous ascites in the developing world. Answer C: Filiariasis is a common cause of chylous ascites in the developing world. Answer D: Kaposi sarcoma is a cause of chylous ascites in an HIV+ individual. Another cause of chylous ascites in an AIDS patient is Mycobacterium avium-intracellulare infection. Answer E: Ovarian carcinoma could be a cause of chylous ascites in a female patient with a long-standing history of nonspecific abdominal symptoms, including abdominal bloating.

Non-depolarizing agent of choice in kidney and liver disease?

Cisatracurium should be used in patients with renal and hepatic disease because it is cleared by Hoffman degradation and is not dependent on kidney and liver function. Cisatracurium is an isomer of atracurium, with fewer tendencies to induce release of histamine

Claudication

Claudication is a common complaint in the vascular surgery office. The complaint is often that they cannot ambulate without pain. The pain however is a dull crampy pain, not a sharp shooting pain. It is often confined to the calf or lower leg. The attached angiogram is essentially normal and with the patient's chief complaint she should be sent for MRI and follow-up with a spine specialist. No vascular abnormalities are found. It is key to notice three vessels branching from the below knee pop. This is known as the trifurcation.

Hemolytic transfusion reaction

Clerical error leading to ABO incompatibility is the most common transfusion reaction leading to death. Patients with an acute hemolytic reaction due to ABO incompatibility may describe warmth and pain along the site of transfusion, flushing in the face, pain in the lumbar region, and constricting pain in the chest. Anesthetized patients may have abnormal bleeding and continued hypotension despite adequate replacement. The first goal of an antibody (Ab) mediated hemolytic reaction is to stop the transfusion immediately. It is important to monitor renal function. Alkalinization of the urine prevents precipitation of hemoglobin (Hb). Patients with a hemolytic reaction need to be treated with aggressive fluid resuscitation with a goal urine output of 100 mL/hr. Diuretics can be considered if tolerated by the patient. Nonhemolytic febrile reactions are the most common transfusion reaction, occurring in 1-2% of transfusions. These reactions are due to white blood cells (WBC) in the donor blood. Urticaria and fever will occur within 60-90 minutes of start of transfusion. However, the allergic reaction may be severe enough to cause anaphylactic shock. The treatment of nonhemolytic febrile reactions is antipyretics, antihistamines, epinephrine, and steroids. One can prevent future febrile reactions with leukocyte filters. The filter size is large enough to allow red blood cells (RBC) through but small enough to trap WBC. Answer A: The treatment of nonhemolytic febrile reactions is antipyretics, antihistamines, epinephrine, and steroids. Answer C: The treatment of nonhemolytic febrile reactions is antipyretics and antihistamines. Answer D: HD is not first line treatment in acute hemolytic reaction. Answer E: One can prevent future febrile reactions with leukocyte filters. Bottom Line: The first line treatment for a hemolytic transfusion reaction is aggressive fluid resuscitation.

Hypernatremia

Clinical manifestations of hypernatremia can be subtle, consisting of lethargy, weakness, irritability, neuromuscular excitability, and edema. With more severe elevations of the sodium level, seizures and coma may occur.

Dual Antiplatelet therapy and elective surgery

Clopidogrel is an inhibitor of platelet aggregation mediated through selective inhibition of adenosine diphosphate. It is commonly prescribed for prophylaxis of thrombosis of drug coated stents placed for management of coronary artery disease. The effect of clopidogrel on platelets is immediate and irreversible, and should be withheld 5 to 7 days prior to elective surgery. However, there is increased risk of stent occlusion if clopidogrel is discontinued early. Previous literature had recommended continuing dual platelet therapy for at least 12 months, however more current guidelines would allow for holding clopidogrel after only 6 months as newer drug eluting stents have lower risk of thrombosis after discontinuation of antiplatelet therapy. There are still many practicing surgeons who would still recommend waiting the previously recommended 12 months. Given that surgery for an asymptomatic hernia is not urgent, postponing the surgery would be appropriate and the safest answer choice for this patient.

Colon Cancer survival rates

Colon cancer is the third most common cancer in the USA and is responsible for the third most cancer deaths. The survival is based on stage, so it is important to know the stage based on the pathologic findings. This tumor invades into the subserosa. This makes it a T3 lesion. The three positive lymph nodes make it an N1 lesion. There is no mention of metastatic disease so it is an M0. T3N1M0 lesions are classified as Stage IIIb. Paradoxically, 5-year survival rate is higher for certain stage III cancers than stage II ones. This data is based on 2004-2010 cancers and an older staging system which may account for the finding. For people with stage IIA colon cancer, the 5-year relative survival rate is about 87%. For stage IIB cancer, the survival rate is about 63%. The 5-year relative survival rate for stage IIIA colon cancers is about 89%. For stage IIIB cancers the survival rate is about 69%, and for stage IIIC cancers the survival rate is about 53%. Answer A: Metastatic, or stage IV colon, cancers have a 5-year relative survival rate of about 11%, but this number can change (dramatically) under certain conditions, such as metastasis to the liver, where combined resections have 5-year survival rates that approach 40-45%.

Colonic volvulus

Colonic volvulus occurs when an elongated, mobile segment of colon twists on a narrowed mesentery. Colonic volvuli can occur in the sigmoid colon, cecum, transverse colon, or splenic flexure in decreasing frequency. Risk factors include a high fiber diet, psychotropic medications, chronic constipation, and sedentary lifestyle. They are more common in older populations with the exception of cecal volvuli, which occur commonly in middle age. Spontaneous reduction is rare. Colonoscopic decompression is recommended only for sigmoid volvuli as the initial therapy, followed by sigmoid colon resection. Segmental resection is the gold standard in treatment for all colonic volvuli. A high fiber diet can cause elongation of the colon, predisposing to sigmoid volvulus. Colonic volvulus is responsible for up to 50% of large bowel obstructions in the region called the "volvulus belt," comprising parts of Africa and Asia where high fiber diets are common. Answer A: Neostigmine is indicated for relief of colonic pseudo-obstruction, not for colonic volvulus. Answer B: Colonic pexy is not an effective surgical strategy and is rarely performed due to a very high recurrence rate. Answer D: Transverse colon volvuli are very rare. Sigmoid volvulus accounts for about 2/3 of all cases of colonic volvulus, and cecal volvulus accounts for about 1/4 of cases. Answer E: Colonic volvuli are more common in adults with risk factors such as a longstanding history of constipation, neuropsychiatric disorders, or a high fiber diet. Bottom Line: Chronic constipation, older age, high fiber diet, and neuropsychiatric disorders predispose to colonic volvulus. The mainstay of treatment for colonic volvulus is resection. TrueLearn Insight : Colonic volvulus is a commonly tested topic on general surgery exams.

Enterocutaneous fistula

Commonly, enterocutaneous (EC) fistulas form after extensive lysis of adhesions or removal of infected or enteric anastomosis. Postoperative formation is responsible for 75-85% of all fistulas. Other causes include malignancy, radiation therapy, or inflammatory conditions (such as IBD) and are responsible for 15-25% of cases. EC fistulas present with fever and leukocytosis 3-5 days after surgery. What is presumed to be a wound infection is opened and progresses to efflux of enteric contents. Other possible presentations include abscess seen on CT, followed by percutaneous drainage, which results in fistula formation. After developing a fistula, fluid, electrolyte imbalance, malnutrition, and sepsis are common. The extent is related to the location and the volume of effluent. Fistulas are, therefore, divided into low output (< 200 mL/day), moderate (200-500 mL/day), and high output (>500 mL/day). Proximal fistulas tend to be high in bicarbonate and result in metabolic acidosis. The majority of fistulas will result in hypokalemia due to potassium efflux. Treatment of EC fistulas is a combination of nutritional, electrolyte, and wound management. With optimal care, approximately one-third will close spontaneously. However, mortality rate remains as high as 15-25%. The first step in control of a fistula is control of any septic source. Additional undrained collections should be identified and controlled, with liberal use of CT scanning and percutaneous drain placement. Operative control is usually unnecessary and may worsen the problem with creation of further enterotomies or spreading septic sources. The patient must then be stabilized, resuscitated, and electrolytes repleted. Nutritional support is then begun with either enteral or parenteral feeding. Enteral feeding may result in increased fistula output or simply not be tolerated due to obstructive symptoms. In these cases, total parenteral nutrition (TPN) may be necessary. If the fistula increases to high output levels, TPN should be used. Fistula output should also be controlled with a wound manager and agents such as psyllium, octreotide, and long acting somatostatin analogues used to decrease output, with the goal of decreased output levels from high to low or moderate output. One-third of fistulas will close within the first 4-6 weeks. If the fistula does not close spontaneously, the wound must be closed operatively. Aggressive nutritional support must be provided to achieve an albumin greater than 3 g/dL. After 6 weeks have passed after fistula formation, operative intervention should be considered. The operation consists of bowel resection, anastomosis, takedown of the fistula, and full thickness resection of the area of the abdominal wall. Complex abdominal reconstruction may be needed. If so, nonabsorbable mesh should be avoided. Answer A: Hypokalemia is more likely as fistula drainage tends to be high in potassium. Answer B: Proximal fistulas tend to be high in bicarbonate, and acidosis is common in these patients. Answer C: One-third of fistulas will close in the first 4-6 weeks. The remainder are less likely to close and may require surgical intervention. Answer E: Fistulas that do not close by 6 weeks should be evaluated for operative intervention. Bottom Line: Surgical intervention should be considered if the fistula persists after 6 weeks from formation.

Post-splenectomy complications

Complications after splenectomy include delayed hemorrhage, pancreatitis, pancreatic leak, and subphrenic abscess. A pancreatic leak can present 10-14 days after splenectomy with abdominal pain, fevers, nausea, vomiting, and leukocytosis. In these situations, patients need to be admitted, placed on intravenous antibiotics, CT-guided drainage, and serial abdominal exams to ensure the fluid collection is not worsening or the patient is not getting septic. Subphrenic abscess is primarily related to associated hollow viscus injuries and is uncommon. Routine "prophylactic" drainage of the subphrenic space should not be done; it is associated with an increased incidence of abscess.

Gastric adenocarcinoma

Considering the disappointing results of adjuvant therapy after surgical resection of gastric cancer and inability of those patient to complete the proposed treatment postoperatively, the notion of neoadjuvant therapy has been proposed and evaluated in several RCTs. Neoadjuvant therapy enhances resectability, and early administration of systemic therapy, which allows almost all patients to receive and complete the prescribed treatment. Based on the results from the Medical Research Council Adjuvant Gastric Infusional Chemotherapy (MAGIC) trial and several other trials that show improved survival among patients with locally advanced gastric cancer who received neoadjuvant chemotherapy, the NCCN now recommends neoadjuvant therapy for locally advanced (T3 or node positive based on EUS) gastric cancer.

Contained anastomotic leak

Contained anastomotic leaks without evidence of pelvic sepsis will heal spontaneously in 95% of patients. Intravenous antibiotics alone should be initiated. Further surgical intervention is not warranted unless there is abscess formation or progression to an uncontained anastomotic leak. Answer B: Repair should not be undertaken in most instances of an anastomotic leak. The choice is either diversion or watchful waiting. In this case the leak is contained, and the patient is not sick so she can be watched.

CIN

Contrast induced nephropathy (CIN) is defined as an acute decline in kidney function occurring after the administration of IV contrast in the absence of other causes of renal dysfunction. The pathophysiology of CIN is related to direct tubular epithelial cell toxicity, altered renal hemodynamics leading to ischemia, and concomitant embolic showers related to the procedure utilizing contrast media. The use of preprocedural hydration protocols and the use of non-ionic iso-osmolar IV contrast reduce the incidence and severity of CIN. CIN results in prolonged hospitalization, higher mortality, excessive health care costs, and possibly long-term kidney impairment. CIN may also delay important diagnostic or therapeutic procedures. CIN typically presents with an acute rise in serum Cr 24-48 hours after contrast exposure. Oliguria is not-typical, and FENa is of low utility. Serum Cr levels typically peak 3-5 days after IV contrast administration and return to baseline within 7-10 days. Permanent clinically relevant declines in renal function are exceedingly rare in patients with normal baseline renal function.

Contrast induced nephropathy

Contrast-induced nephropathy (CIN) is defined as an acute decline in renal function after exposure to intravenous radiocontrast media with creatinine levels rising to greater than 25% above baseline. Creatinine levels peak 3 to 5 days after exposure and usually return to baseline within 7-10 days. CIN is a parenchymal form of acute renal failure but uniquely presents with a fractional excretion of sodium (FeNa) of less than 1%. Urine studies will demonstrate granular brown casts and mild proteinuria (less than 300mg per day). CIN does not typically present with oliguria.FeNa < 1% is consistent with CIN, but unlike prerenal azotemia, urinalysis in CIN will show epithelial cell casts, debris, and urate and calcium oxalate crystals.

Veress

Correct placement of Veress needle is indicated by 1) Positive Hanging drop test 2) Irrigation of 10 cc saline with syringe without resistance followed by aspiration - If in abdominal cavity saline is not aspirated but if it is in preperitoneal space, water can be aspirated back 3) Proportionate increase in abdominal pressure on insufflation. If the pressure increases to 14mm Hg with flow of 400 cc of gas, it indicates incorrect placement. Answer A: Pre-peritoneal placement of the Veress needle is suspicious if the pressure goes above 14 mmHg. Answer B: Aspiration of yellowish or cloudy fluid or bile indicates injury to the bowel. If it occurs with Veress needle, it can be removed and reinserted. The viscera should be thoroughly examined after correct trocar placement. If bowel injury occurs with the trocar, then either repair of the bowel with open or laparoscopic approach is done. Answer C: Injury to the Inferior epigastric vessels occurs with incorrect choice of trocar site placement. It is usually seen as blood dripping from the trocar sites or blood on the viscera or late presentation of abdominal wall hematoma. It requires suturing for hemostasis. Answer D: Incorrect. Answer E: Gas embolism occurs usually with hepatic resection but can occur with any procedure requiring pneumoperitoneum. It occurs in any venous injury where central venous pressure is less than the pneumoperitoneum. Hence, central venous pressure should be maintained at optimal levels and pneumoperitoneum at the minimum required settings. It manifests as rise in End tidal Co2 and cardiopulmonary collapse. If it occurs, the patient should be placed in the left lateral decubitus position. If a central line is in place, a syringe is used to aspirate the gas embolism. The patient should be hyperventilated to blow off Co2. Bottom Line: The potential complications of intra-abdominal access during laparoscopy include 1) injury to hollow viscus like stomach, small and large bowel, bladder - occurs in patients with previous abdominal surgeries 2) Injury to solid organs like Liver or Spleen occurs when alternative sites of access like right or left upper quadrants of the abdomen are chosen, especially in patients with hepatosplenomegaly 3) Injuries to blood vessels like Inferior epigastric vessels,Mesenteric vessels, Aorta and IVC 4) Gas embolism 5) Surgical emphysema due to incorrect placement of Veress needle.

Cortisol producing adenomas

Cortisol-producing adrenal adenomas can be incidentally discovered during abdominal imaging or when the patient develops the signs and symptoms of Cushing syndrome. These signs are truncal obesity, round face, fragile skin, depression, and abdominal striae. These lesions secrete cortisol without the usual dependence on adrenocorticotropic hormone (ACTH). The first step is proving hypercortisolism via a low-dose dexamethasone suppression test and 24-hour urine cortisol. The next step is determining the source via a high-dose dexamethasone suppression test and plasma ACTH level. When the level is low and the cortisol is not suppressed, this points to an adrenal source. The best next step is a CT scan of the abdomen where you will see signs of possible malignancy.

Gastric adenocarcinoma

Current recommendations for lymph node dissections for gastric adenocarcinoma are a D1 or D2 resection of at least 15 nodes. D1 dissection involves a gastrectomy including the greater and lesser omental lymph nodes. The omental bursa along with the anterior leaf of the transverse mesocolon is removed and the corresponding arteries are cleared completely in a D2 dissection. A splenectomy is required for a D2 dissection for proximal gastric tumors. D2 resections are associated with higher morbidity and mortality, but some studies have shown increases survival. A D2 resection should be at a high volume center with an experienced surgeon. The bottom line is at LEAST 15 NODES must be sampled for an adequate determination of staging.

Breast cancer during pregnancy

Counseling of a pregnant woman with breast cancer should include a thorough review of treatment options as well as risks and benefits of each treatment option. Treatment options for breast cancer during pregnancy are similar to those for nonpregnant women. These options include breast-conserving surgery, mastectomy, and systemic chemotherapy. Chemotherapy is safe after the first trimester. The use of hormonal and immunotherapy such as tamoxifen and trastuzumab is contraindicated. Radiation therapy is contraindicated during pregnancy. There are limited data regarding sentinel lymph node biopsy during pregnancy. The use of blue dye, including methylene blue or isosulfan blue, is contraindicated during pregnancy. The use of radio-labeled sulfur colloid, however, appears to be safe.

Crohn's disease

Crohn disease complicated by partial bowel obstruction should first be managed nonoperatively with immunosuppressive medications. If there is no resolution of the obstruction, then surgical intervention is indicated. Strictures from Crohn disease can be managed with either resection and anastomosis or stricturoplasty. When there are multiple areas of short strictures, multiple stricturoplasties are the treatment of choice. If only the obstructing stricture is treated, the remaining strictures would be at high risk of causing future bowel obstructions. Long strictures are best treated with resection and primary anastomosis. Complete obstruction is treated surgically first, as medical management is very unlikely to be successful.

Cyclosporine

Cyclosporine is a hydrophobic peptide that binds to a cytoplasmic protein, cyclophilin. The cyclosporine-cyclophilin complex binds to and inhibits calcineurin, a cytoplasmic tyrosine phosphatase, which, in turn, blocks the activation of transcription-regulating factor NF-AT. This prevents the transcription of the IL-2 gene.

Cyclosporine

Cyclosporine was introduced in the early 1980s for immunosuppression following organ transplant. Cyclosporine binds with cyclophilin which leads to calcineurin inhibition which results in decreased Il-2 production and T-cell suppression. Cyclosporine has a vasoconstrictive effect on renal vasculature. Long-term use may result in interstitial fibrosis of renal graft parenchyema and renal failure may result.

Surgical ethics

Decision making for patients unable to make decisions for themselves and who lack a durable power of attorney, advanced directive, or living will is often a difficult and ethically treacherous area. All sincere efforts must be made to attempt to find a next of kin. If this fails and time permits, an ethics consultation or court appointed guardian can be attempted. However, both options can be lengthy. If a patient needs an urgent operation, it is recommended that the surgeon first seek the opinion of another surgeon and that this consultation is documented. If there is not enough time even to seek a second opinion then implied consent should be accepted and the patient should proceed to the operating room without haste. Ultimately, the decision must be made in the best interest of the patient and this burden will likely fall onto the shoulders of the surgeon.

ARDS

Decreased lung compliance in acute respiratory distress syndrome (ARDS) is due to alveolar flooding and collapse which leads to fewer alveoli to accommodate the tidal volume. Therefore, there is an increased stretch of the functional alveoli causing decreased compliance. Positive end-expiratory pressure (PEEP) is used in mechanically-ventilated patients with ARDS to improve oxygenation. In ARDS, three populations of alveoli can be distinguished. There are normal alveoli which are always inflated and engaging in gas exchange, flooded alveoli which can never, under any ventilatory regime, be used for gas exchange, and atelectatic or partially flooded alveoli that can be "recruited" to participate in gas exchange under certain ventilatory regimens. The recruitable alveoli represent a continuous population, some of which can be recruited with minimal PEEP, and others which can only be recruited with high levels of PEEP. Ventilation-perfusion (V/Q) mismatch is a cause of hypoxemia in pulmonary embolism among other pathologies but does not directly affect lung compliance and does not play a significant role in ARDS. While pulmonary edema can be manifest in ARDS and may contribute to hypoxemia, it does not decrease compliance. Increased peripheral oxygen utilization is the mechanism behind the decreased mixed venous oxygen saturation seen in sepsis

Congenital adrenal hyperplasia

Deficiency of 21-hydroxylase results in deficient glucocorticoids and mineralocorticoids resulting in negative feedback on the pituitary causing excessive production of adrenocorticotrophic hormone (ACTH). This results in preferential production of androgenic steroids from the reticular layer. 17-hydroxyprogesterone accumulates secondary to the P450c21 block, resulting in overproduction of pregnenolone and progesterone and their 17-hydroxylated derivatives, leading to the overproduction of testosterone and ultimately its reduced derivative, dihydrotestosterone. Elevations in 17-hydroxyprogesterone can be detected from a spot serum sample. Polymerase chain reaction (PCR), sequencing of known mutated genes, or fluorescent in situ hybridization (FISH) can now be used to make a prenatal diagnosis. Mutations in 21-hydroxylase (P450 c21) causes 90% of cases of congenital adrenal insufficiency (CAH). The mutated gene resides in chromosome 6p

Inguinal hernia repair contra-indications

Definitive treatment of most hernias is surgical repair. For inguinal hernias, open or laparoscopic approaches are available. Laparoscopic surgery is theoretically possible in most cases. However, there are several factors to consider when choosing an approach for surgery. Types of laparoscopic approaches include total extraperitoneal (TEP) and transabdominal approach (TAPP). Indications are the same for open and laparoscopic approaches. Relative contraindications to laparoscopic repair include: Previous or planned pelvic or lower abdominal operation Pelvic radiation Recurrent hernia following previous laparoscopic repair Incarcerated hernia Large scrotal hernia (greater than 3 cm) Ascites (TAPP should be avoided, however TEP is an option in some patients.) Absolute contraindications include: Inability to tolerate insufflation/pneumoperitoneum Active infection Strangulated hernia Answer A: Previous vasectomy is NOT a contraindication to laparoscopic inguinal hernia repair. Previous lower abdominal or pelvic operation, such as radical retropubic prostatectomy, is a relative contraindication because there is often difficulty accessing the pre-peritoneal space. Answer B: Incarcerated hernia is a RELATIVE contraindication to laparoscopic repair because traction on the intestines during the procedure risks injury and contamination of the sterile field. Incarcerated hernias should be repaired using an open approach in most cases. Answer C: Recurrent hernia following previous laparoscopic repair is a RELATIVE contraindication to laparoscopic repair. Recurrent hernias are generally repaired with an open approach. However, favorable results with laparoscopic transabdominal approach (TAPP) have been reported. Answer D: Pelvic radiation is a RELATIVE contraindication due to difficulty accessing the preperitoneal space. Open approach is generally used for these patients. Answer E: All strangulated hernias are repaired using an open approach due to high risk of bowel and surrounding structure injury. Further, if the surgical field is contaminated (i.e. perforation secondary to ischemia or necrosis, bowel injury), mesh placement is contraindicated therefore precluding a laparoscopic approach. In these instances, the repair is performed with primary sutured technique without mesh, using an open approach. Open or laparoscopic approaches are options for inguinal hernia repair in patients with no previous preperitoneal/pelvic/lower abdominal surgeries, ascites or complicated hernia(s). The choice of surgical approach then depends if the hernia is primary or recurrent, bilateral or unilateral, femoral or inguinal. Bottom Line: Strangulated hernias should be repaired using open approach.

dermatofibrosarcoma

Dermatofibrosarcoma is a tumor that arises from the fibroblasts in the dermis. It is associated with microscopic lateral extension; therefore, thorough margin assessment during excision is needed to prevent tumor recurrence. Multiple excisions may be required and reconstruction with tissue flap or skin graft is often needed. Answer B: Unlike myxofibrosarcoma, dermatofibrosarcoma rarely invades the underlying muscle, so en bloc excision of the tumor usually includes the underlying fascia, but not the muscle. Answer C: Adjuvant radiation for soft tissue sarcoma is indicated for tumors that are high-grade or more than 5cm in size. Answer D: Neoadjuvant chemo is used for high-grade, > 10 cm soft tissue sarcomas and for high-grade, > 5cm liposarcoma and synovial sarcoma. Answer E: Dermatofibrosarcoma is associated with high risk of local recurrence and positive margins need re-excision. Bottom Line: Wide local excision with thorough assessment of the excision margin is mandatory to prevent local recurrence of dermatofibrosarcoma.

dermatofibrosarcoma protuberans

Dermatofibrosarcoma protuberans most commonly affects adults in their third to fifth decades. These tumors typically presents as firm, flesh-colored to dull, red plaques that may be mistaken for keloids or hypertrophic scars. Histologically, these sarcomas are identified by their finger-like projections of spindle cells that likely account for tumor recurrence. Answer A: Merkel cell carcinoma clinically presents as a growing, dome-shaped or subcutaneous nodule that is red or purple in color. Although MCC can be mistaken for BCC, the "rolled edge" appearance is characteristic for BCC. Answer B: Squamous cell carcinoma most commonly presents as a non-healing skin lesion with ulceration or an erythematous papulonodule with overlying ulceration or keratotic crust. Answer C: Although DFSP can be mistaken for keloids or hypertrophic scars, the presence of finger-like projections on histological examination is characteristic. Answer E: Basal cell carcinoma most commonly presents as pearly nodules with telangiectasia with classic peripheral "rolled" borders. Bottom Line: DFSP presents clinically as firm, flesh-colored to dull, red plaques. Histologically, finger-like projections of spindle cells are typically identified and likely account for tumor recurrence.

Anal fissure

Digital and proctoscopy examination may trigger severe pain, interfering with the ability to visualize the ulcer. An endoscopic examination should be performed, but it can be delayed 4-6 weeks until the pain is resolved with medical management or until surgery is performed for those cases refractory to medical therapy. Answer A: Because it involves the highly sensitive squamous epithelium, fissure in ano is often a painful condition. With defecation, the ulcer is stretched, causing pain and mild bleeding. The diagnosis is secured by the typical history of pain and bleeding with defecation, especially if associated with prior constipation and confirmed by inspection after gently parting the posterior anus. Answer C: Characteristic associated findings include a sentinel pile or tag externally and an enlarged anal papilla internally. Answer D: An anal fissure is best seen by visually inspecting the anal verge with gentle separation of the gluteal cleft. If fissures occur laterally, then workup for another cause such as Crohn disease, hidradenitis, or sexually transmitted diseases should be sought. Answer E: An anal fissure is a linear ulcer of the lower half of the anal canal, usually located in the posterior commissure in the midline. Bottom Line: Anal fissures are caused by straining with hard stools. The usual location is in the posterior midline, and the presentation is pain and bleeding on defecation. History and physical can make the diagnosis without any need for further studies. TrueLearn Insight : Anal fissures usually occur at the posterior midline. If they occur laterally, then workup for another cause such as Crohn disease, hidradenitis, or sexually transmitted diseases should be sought.

Complications following ileal resection

Diseases and conditions affecting the ileal mucosa, such as Crohn disease or ileal resection, lead to a reduction of specific absorptive sites for the intrinsic factor-vitamin B12 complex. Ileal resections of more than 60 cm usually result in clinically significant vitamin B12 malabsorption. Fat malabsorption secondary to bile acid deficiency in patients with extensive ileal resection is associated with an increased risk of oxalate kidney stones if the colon is preserved. Interruption of the enterohepatic circulation of bile acids by ileal resection results in decreased hepatic bile acid secretion and altered composition of hepatic bile in terms of its organic components: bile acid, cholesterol, and phospholipids. Hepatic bile becomes supersaturated with cholesterol, with subsequent formation of cholesterol crystals and gallstones in gallbladder bile. Ileal resection of more than 100 cm usually results in severe bile acid malabsorption that cannot be compensated by increased hepatic synthesis; in such cases, steatorrhea results. Digestion and absorption of protein are usually 80-90% completed in the jejunum. Diseases causing malabsorption of dietary fat commonly cause malabsorption of fat-soluble vitamins (A, D, E, and K), because they require similar absorptive mechanisms. This point is especially important in diseases that result in impaired micelle formation from bile salt deficiency.

DIC

Disseminated intravascular coagulation (DIC) is associated with septicemia, polytrauma and malignancies. DIC associated with obstetrical catastrophes such as abruptio placentae or amniotic fluid embolism are severe but self-limiting. The diagnosis is based on increased consumption of coagulation factors and platelets resulting in increased PT and PTT, decreased platelets and increased in fibrin degradation products.

Crohn's disease

Distinguishing between Crohn's disease and ulcerative colitis can often be difficult. However, there are a number of specific findings that favor a diagnosis of Crohn's disease. These include granulomas, cobblestone appearance, transmural involvement, patchy areas of disease, rectal sparing, anal involvement, creeping fat, ulcerations, fissures, fistulas, and abscesses.

GI bleed

Diverticular bleeding is the most common type of severe GI bleeding. Even with a high index of suspicion, an upper GI bleed must be ruled out first with the use of nasogastric lavage. If the patient is stable, localization techniques such as colonoscopy, angiography, or tagged red blood cell scan can be used. If the patient is hemodynamically unstable or requires more than 4 units of blood, surgery is indicated. A subtotal colectomy without localization of the bleed has a high recurrence rate, so every attempt to find the location of the bleed should be made. The first step in management of a GI bleed is nasogastric lavage.

Colo-vesicular fistulas

Diverticulitis can cause fistulas to different organs including the skin, small bowel, bladder, and vagina. Colovesical fistulas can present with pneumaturia, fecaluria, or recurrent urinary tract infections. They are more common in men because the uterus creates a barrier which prevents a connection between the colon and the bladder. After a hysterectomy, the incidence increases in women as well. Colovesical fistulas should be treated, because patients will continue to have symptoms of recurrent urinary tract infections. Definitive treatment involves removing the diseased segment of colon, usually the sigmoid, and leaving the bladder in place. Repair of the bladder is usually not necessary since the sigmoid has the disease and caused the fistula. Prior to undergoing the sigmoidectomy, the patient should have a colonoscopy to confirm diverticular disease and not a tumor that caused the symptoms. If a tumor were present, an en bloc resection of the bladder would need to be performed in addition to the sigmoidectomy.

Dopamine

Dopamine is a broad-spectrum endogenous, sympathetic amine that functions as a central and peripheral neurotransmitter. At low doses (1-3 mg/kg/min), dopamine increases renal blood flow and maintains diuresis through DA1//DA2 receptor stimulation in the renal vasculature. At moderate doses (5-10 mg/kg/min), it increases contractility and cardiac output via stimulation of cardiac β-receptors. At higher doses (>10 mg/kg/min), peripheral vasoconstriction from increasing α-activity becomes more prominent, resulting in elevation of systemic vascular resistance and blood pressure.

Dopamine

Dopamine is an endogenous catecholamine that has several cardiovascular effects, including increased heart rate, increased contractility, and peripheral vasoconstriction. It is used primarily for inotropic support in order to maintain brain, heart, and kidney perfusion. Low-dose dopamine (1-3 µg/kg/min), also referred to as "renal-dose" dopamine, increases renal blood flow and maintains diuresis via the DA1 and DA2 receptors. At moderate doses (3-5 µg/kg/min), dopamine stimulates primarily cardiac β-adrenoreceptors, increasing contractility and thus cardiac output. Answer A: At higher doses of dopamine (10 µg/kg/min), peripheral vasoconstrictive effects from stimulation of α-adrenergic receptors predominate. This can result in significant coronary vasoconstriction, resulting in angina, vasospams, and increased pulmonary artery wedge pressure.

Soft tissue sarcoma

Drains are frequently placed after excision of soft tissue sarcomas to avoid hematoma/seroma. They should be placed close to the incision to be included in the radiation field. Answer A: It is agreed that 1-2 cm free margin is adequate for resection of soft tissue sarcoma. Answer C: Excision of the tumor with the adjacent vascular adventitia is appropriate if no gross invasion of the artery is identified. Answer D: Metallic clips should be placed at the surgical field periphery to guide future radiation. Answer E: Tumor excision should be carefully planned and complex closure is necessary if primary closure is not feasible. Bottom Line: If drains were placed after excision of sarcoma, they should be placed close to the incision.

Dysphagia following fundoplication

Dysphagia is the most common complication following fundoplication. It usually occurs during the first 2-3 weeks postoperatively. The most common cause for postfundoplication dysphagia is postoperative edema in the fundoplication or the perihiatal region. Swelling at the GE junction is common due to manipulation of the area. Dysphagia due to tissue edema is usually self-limiting within 6-8 weeks; however, if dysphagia persists beyond this time frame, further investigation is indicated.

Pancreatic cancer

Ductal adenocarcinomas account for more than 75% of all non-endocrine pancreatic cancers. They are white-yellow, poorly defined, hard masses that often obstruct the distal common bile duct or main pancreatic duct. They may cause fibrosis and chronic pancreatitis. Ductal adenocarcinomas tend to infiltrate into vascular, lymphatic, and perineural spaces. At the time of resection, most ductal carcinomas have already metastasized to regional lymph nodes. In addition to the lymph nodes,they frequently metastasize to the liver (80%).

Dumping syndrome. Early vs late

Dumping syndrome affects 5-10 percent of patients undergoing pyloroplasty, pyloromyotomy, or distal gastrectomy. It is characterized by diaphoresis, weakness, light-headedness, and tachycardia. It is generally broken down into two types: Early and Late. The early dumping syndrome occurs 30 minutes after eating and is due to a hyperosmotic load being delivered to the duodenum in the absence of a pylorus. This causes a large fluid shift and symptomatology. Late dumping syndrome happens 2-3 hours later and is due to a large insulin release due to the large bolus of food hitting the duodenum. This causes a hypoglycemia episode as manifested by diaphoresis, weakness, light-headedness, and tachycardia.

Nissen fundoplication

During a Nissen fundoplication, the dissection begins at the caudate lobe of the liver and moves toward the right crus of the diaphragm. After identifying the right and left crura, the lesser omentum is opened to gain access to the right crus. The gastrohepatic ligament is identified as it courses toward the diaphragm to the right crus. In a fundoplication, the gastric fundus is wrapped, or plicated, around the lower end of the esophagus and stitched in place, reinforcing the closing function of the lower esophageal sphincter. The esophageal hiatus is also narrowed down by sutures to prevent or treat concurrent hiatal hernia, in which the fundus slides up through the enlarged esophageal hiatus of the diaphragm. Retraction of the stomach to the left reveals the caudate lobe of the liver, the right crus of the diaphragm, and the posterior esophagus. The anterior (left) vagus nerve and its branches are often visible at this juncture. Care must be taken during this dissection to avoid inadvertent division of a replaced left hepatic artery. When this anomaly occurs, the primary arterial blood flow to the left lobe of the liver traverses this tissue.

Complications following renal transplant

During recipient surgery, lymphatics over the iliac vessels are divided that can cause lymphatic leak. Careful ligation can decrease the incidence but does not prevent the leak. These patients present with compression over the transplanted ureter or iliac vein. Ultrasound is diagnostic. Percutaneous drainage has poor results and has chances of infection. The treatment is surgical, either by laparoscopic or open approach to establish a peritoneal communication. Answer B: Arterial thrombosis is seen in immediate post operative period. Patient presents with sudden cessation of urine. Ultrasound is diagnostic. If there is a single normal renal artery and thrombosed, the salvage is minimal. If a segmental artery or upper pole branch is thrombosed, remaining part of kidney may be salvaged. If lower pole artery is involved, ureter becomes ischemic and urine leak may develop from necrosis. If the main artery is thrombosed, treatment is transplant nephrectomy. Answer C: Urine leak develops, usually, in the first week after transplantation. Patient presents with pain and swelling at the operative site. Urine leak is diagnosed by aspirating the perinephric fluid and checking the creatinine level. Delayed images in nuclear medicine scan will reveal the urine leak when the contrast material is seen outside of the bladder. Placement of a double-J stent at the time of transplantation may decrease the risk of this complication. Answer D: Polyomavarius also known as BK virus causes multiple late strictures in the post-transplant patient. They present with an asymptomatic rise in creatinine concentration. Ultrasound will demonstrate hydronephrosis. Acutely, percutaneous nephrostomy is placed. In distal stenosis, repaired by reimplanting the ureter. In proximal or long stricture, a ureteropyelostomy is done using patient's native ureter. Hence its important to confirm the presence of normal native ureter prior to reconstruction. Answer E: Clostridium difficile infection causes diarrhea. Bottom Line: In Immediate postoperative period, vascular thrombosis can cause cessation of urine output. Later in few weeks, Lymphocele can be the cause. In late case, Polyoma (BK) Virus can cause multiple strictures.

Sleeve gastrectomy

During the operation for sleeve gastrectomy, the greater curvature of the stomach is completely devascularized prior to being resected. Answer C: Blood supply to the gastric sleeve is received from the left and right gastric arteries as well as branches from the right gastroepiploic. Answer D: The posterior stomach does not require extensive dissection to mobilize in a patient who has not previously undergone a gastric procedure or had inflammation within the are of the lesser sac. Answer E: Larger staples (3.5mm) are needed to divide the stomach due to its thickness. The antrum is typically the thickest portion of the stomach so care must be taken when dividing this area. Smaller staple heights may strangulate the closure and increase risk for a leak.

Early post op skin infections

Early pos operative wound infections require prompt recognition and emergent treatment with gram positive flora antibiotic coverage. The most common single organism pathogens include Streptococcus pyogenes and Clostridium perfringens.

Renal physiological changes in the elderly

Elderly patients can be quite susceptible to acute renal injury due to a number of physiological changes. There is an overall decline in cardiac output, leading to subsequent decreased renal perfusion. There is also an overall decrease in renal size and volume with decreased number of glomeruli. Progressive glomerulosclerosis leads to decreased renal function and renal reserve. Given these factors, nephrotoxic agents can easily lead to renal injury in the elderly patient. The mortality risk of perioperative renal failure in all patients is approximately 50% and may be even higher in elderly patients.

Ulcerative colitis

Elective surgical treatment for ulcerative colitis is a total abdominal proctocolectomy with ileal J-pouch anal anastomosis. In the emergent setting, such as for toxic megacolon or fulminant colitis, a total colectomy is indicated. The completion proctectomy can be performed when the patient is more stable. Elective surgery is indicated if symptoms are refractory to medical management, there is severe GI bleeding, or if dysplasia is found on screening colonoscopy. Proctocolectomy can be helpful in decreasing the impact of some extra-intestinal manifestations such as erythema nodosum and arthritis but has no effect on primary sclerosing cholangitis. Anal incontinence is a contraindication for an ileo-anal anastomosis, and these patients should have an end ileostomy. Answer A: The treatment for fulminant colitis is surgical intervention, but an anastomosis is contraindicated due to the high risk of a leak. Answer C: Other medical options are available prior to surgery for the management of ulcerative colitis. Answer D: Total proctocolectomy does not affect the disease process of primary sclerosing cholangitis. Answer E: Patients with anal incontinence are not candidates for an ileal J-pouch anastomosis. Bottom Line: Dysplasia is an indication for elective proctocolectomy for ulcerative colitis.

Elemental formulas for nutrition

Elemental formulas use free amino acids or small chain peptides as protein sources and are useful in patients with maldigestive and malabsorptive disorders.Elemental formulas use free amino acids or small chain peptides as protein sources and are easy to digest. Elemental formulas are usually low in fat. They are typically prescribed for patients with maldigestion and malabsorption; e.g., pancreatitis , critical illness, short-gut syndrome, enterocutaneous fistulas, and diarrhea. In patients undergoing routine gastrointestinal operations, elemental formulas offer no benefit over standard polymeric formulas.

Foreign body removal

Endoscopic foreign body retrieval is a frequently performed and safe procedure. Age-appropriate equipment is selected before the patient is brought to the operating room. Rigid esophagoscopes were long considered the gold standard for removal of foreign bodies; however, with the advent of flexible scopes the majority of foreign bodies can be extracted with flexible scopes, without the need for general anesthesia and a lower risk of complications such as perforation and abrasion. Answer A: Direct laryngoscopy should be performed if the patient requires intubation or the foreign object is suspected to be in the airway. Answer B: Rigid esophagoscopes are more versatile than flexible scopes because they allow for passage of various different grasping forceps. However, they require general anesthesia and skilled operators and have a higher risk of associated complications. They are considered superior for visualization of the lower oropharynx and upper esophagus. For a foreign body in the lower esophagus, a flexible scope is used as the first line. Answer C: If the coin were out of the esophagus and into the stomach, it would be possible to watch and wait. However, this coin is lodged in the esophagus and, therefore, requires retrieval. Of note, if this were a battery, it would have to be retrieved even if it were in the stomach. Answer E: Bronchoscopy would be performed to retrieve a foreign object in the airway. Bottom Line: Esophagoscopy is the primary modality for removal of a foreign object in the esophagus.

perforated duodenum s/p ercp

Endoscopic retrograde cholangiopancreatography (ERCP) is an advanced endoscopic procedure that provides radiologic images of the pancreaticobiliary trees and can also provide access for therapy. Complications include perforation, bleeding, pancreatitis, and cholangitis. Perforation most commonly occurs during ERCP as a result of extension or tearing of the ampulla beyond the junction of the common bile duct with the duodenal wall. Retroperitoneal or free intraperitoneal air may be seen. As long as the patient is hemodynamically stable and has no frank peritonitis, intravenous antibiotics, hydration, and avoidance of oral intake are sufficient to manage such complications. If the patient's condition deteriorates, surgical exploration is indicated. Answers A & B: Surgical exploration is indicated in patients with hemodynamic instability, peritonitis, or if the patient's condition deteriorates despite conservative treatment. Answers C & D: There is no role for esophagogastroduodenoscopy or repeat endoscopic retrograde cholangiopancreatography (ERCP) in the management of duodenal perforation after ERCP. Bottom Line: Retroperitoneal doudenal perforation from endoscopic retrograde cholangiopancreatography can be managed non-operatively. If the patient's condition deteriorates, surgical exploration is indicated.

Enteral nutrition

Enteral nutrition should be considered in patients who have a functional gut; who cannot, should not, or will not eat adequately; and for whom there is a safe method of access. General indications for enteral nutrition include: protein-calorie malnutrition (> 10% loss of usual weight or serum albumin levels < 3.5 g/dL), oral intake inadequate or likely to be inadequate for 7 to 14 days, and a functional gastrointestinal tract (sufficient length and condition to allow adequate nutrient absorption). Indications for enteral nutrition include a head injury, cerebrovascular diseases, demyelinating diseases, neoplasms, inflammation, trauma, pancreatitis, inflammatory bowel disease, short bowel syndrome, intestinal fistulas, malabsorption, anorexia nervosa, severe depression, burns, chemotherapy, radiation therapy, kidney disease, liver disease, cystic fibrosis, or organ transplantation. Contraindications for enteral nutrition include a complete mechanical intestinal obstruction, diffuse peritonitis, intractable vomiting, paralytic ileus, intractable diarrhea, gastrointestinal ischemia, or hemodynamic instability

Epidermal cyst

Epidermal cyst can be acquired, congenital or hereditary. Gardner syndrome is an autosomal dominant condition that is associated with GI polyposis, osteomas and multiple epidermoid cysts. Answer A: Epidermal cysts are diagnosed clinically. Laboratory or imaging studies are not needed to confirm the diagnosis. Malignancy within the cyst is very rare. Imaging studies might be used if malignancy is suspected (Example-rapidly enlarging cyst). Answer C: The pathogenesis is related to implantation of epidermal component into the dermis. Answer D: The cyst is filled with keratin that is cheese-like and foul-smelling. Answer E: Epidermal cysts do not resolve unless the cyst wall is removed with its contents.

Benzodiazepine overdose

Flumazenil should be given in the event of benzodiazepine overdose. The initial dose is 0.2 mg IV over 30 seconds. Repeat doses can be given up to 3 mg. If no response is seen with that dose benzodiazepine overdose is not likely. Flumazenil inhibits the benzodiazepine receptor site. It will not reverse the effects of an opioid overdose.

Esophageal adenocarcinoma

Esophageal adenocarcinoma accounts for ~ 70% of esophageal cancer diagnoses in the United States. It is known to spread aggressively through the lymphatic channels. T1 tumors are divided into T1a and T1b, which signifies extent into the muscularis mucosae and submucosa respectively. T2 lesions spread into the muscularis propria, T3 lesions to the adventitia and T4 lesions invade surrounding structures. The likelihood of nodal invasion increases with the depth of the tumor. Upfront esophagectomy is indicated in patients with clinical T1N0M0 lesions and may be considered in those with clinical T2N0M0 lesions. Patients with T1a tumors or those who are poor surgical candidates may by considered for esophageal preservation using endoscopic resection techniques. Patients with nodal disease, T3 tumors, and select T4a tumors which are amenable to en bloc resection should receive neoadjuvant therapy prior to esophagectomy. There are no randomized trials which compare neoadjuvant chemotherapy to chemoradiotherapy, however chemoradiotherapy is often prefered due to improved rates of pathologic complete response. For patients who undergo upfront resection and are later found to have node positive, pT3, or pT4 tumors adjuvant chemoradiotherapy should be offered.

Esophageal pH monitoring

Esophageal pH monitoring may be done by two methods: wireless pH monitoring or by an intraluminal tube with nasopharyngeal catheter. With wireless monitoring, a single Bravo pH probe is placed 5 cm above the lower esophageal sphincter (LES), and 48 hours of measurements are obtained. Patients indicate when they experience symptoms as well as their change in position. Nasopharyngeal catheters measure pH for only 24 hours, however, and are considered to be more accurate due to the presence of multiple pH probes along the catheter, allowing for larger sampling of pH changes. Esophageal acid exposure may be physiologic; therefore, measurements include percent total time of pH <4 while upright, supine, and overall. The number and duration of each episode also are monitored. A composite score (DeMeester) is calculated based on these parameters. A score greater than 14.72 (95th percentile) is considered abnormal. Esophageal impedance is an adjunct to traditional pH testing. A low voltage current is applied to multiple electrodes within the probe which determine the presence of liquid, food bolus, or esophageal tissue via impedance. With multiple electrodes along a probe, direction of bolus transport may also be determined. This allows for detection of nonacid (bile) reflux and may be used to determine reflux even with the presence of proton pump inhibitors. Answer A: Esophageal impedance is measured by an electrical current, not pH, and is an adjunct to traditional pH monitoring to detect nonacid reflux. Answer B: Wireless pH probes are placed 5 cm above the LES. Placement at the lower level esophageal sphincter would result in a consistently low pH. Answer C: A DeMeester score is calculated by the amount of time below a pH of 4 with changes in position, duration, and number of episodes. Answer E: A normal DeMeester score does not rule out nonacid reflux. For this, additional testing, such as esophageal impedance, would be needed. Bottom Line: Esophageal pH monitoring measures the number of episodes and time with a pH below 4 with changes in position. A composite score (DeMeester) greater than 14.72 is considered abnormal.

Esophageal varices

Esophageal varices are a result of portal hypertension with numerous potential etiologies. Prehepatic causes include portal vein and splenic vein thrombosis as well as arteriovenous fistula. Intrahepatic causes include infiltrative liver diseases, cirrhosis and other fibrosing conditions, and polycystic liver disease. Post-hepatic causes include Budd-Chiari syndrome, IVC webs and thrombosis, and right heart failure. Varices occur through gastroesophageal collaterals (intercostal, diaphragmatic, and esophageal veins), hemorrhoids (middle and inferior hemorrhoidal veins), caput medusa (umbilical and abdominal wall veins), and retroperitoneal collaterals (left renal vein). Variceal hemorrhage occurs at a rate of 5-15% annually. Each bleed has a mortality approaching 20%. Initial management of esophageal variceal hemorrhage is focused on resuscitation and airway control. Two large-bore IVs are placed, rapid transfusion is begun, and the patient is intubated. Transfusion should target hematocrit levels of 25-30% as overtransfusion may worsen portal hypertension. Coagulopathy should be corrected. Infusion with vasopressin is begun, in conjunction with nitroglycerin to prevent ischemia. Additional infusion with octreotide may also be performed with proton pump inhibitors. After resuscitation is initiated, control is attempted with esophagogastroduodenoscopy using sclerotherapy and banding. If bleeding recurs after control, a surgical shunt or transjugular intrahepatic portosystemic shunt may be considered. If bleeding is not controlled on initial endoscopy, balloon tamponade with a Sengstaken-Blakemore tube or Minnesota tube is used. These consist of large round gastric balloons, which are placed to traction to compress the gastroesophageal junction. A distal port evacuates luminal contents. Minnesota tubes also have a proximal port to allow drainage of salivary secretions. Aspiration is the most common complication, and intubation is often necessary. With proper application, bleeding is controlled in 90% of cases with a 50% rebleed rate. Answer A: Minnesota tubes also have a proximal port to allow drainage of salivary secretions. The Sengstaken-Blakemore tube does not. Answer B: The tube is left inflated but cannot be left inflated for more than 36 hours. Answer D: With proper application, bleeding is controlled in 90% of cases, with a 50% rebleeding rate. Answer E: Insufflation on the large round gastric balloon is placed to traction to compress the gastroesophageal junction. Bottom Line: If bleeding is not controlled on initial endoscopy, balloon tamponade should be performed with a Sengstaken-Blakemore tube or Minnesota tube. These consist of large round gastric balloons, which are placed to traction to compress the gastroesophageal junction. With proper application, bleeding is controlled in 90% of cases with a 50% rebleed rate.

Esophageal varices

Esophageal varices are a result of portal hypertension with numerous potential etiologies. Prehepatic causes include portal vein and splenic vein thrombosis as well as arteriovenous fistula. Intrahepatic causes include infiltrative liver diseases, cirrhosis and other fibrosing conditions, and polycystic liver disease. Posthepatic causes include Budd-Chiari syndrome, IVC webs and thrombosis, and right heart failure. Varices occur through gastroesophageal collaterals (intercostal, diaphragmatic, and esophageal veins), hemorrhoids (middle and inferior hemorrhoidal veins), caput medusa (umbilical and abdominal wall veins), and retroperitoneal collaterals (left renal vein). Variceal hemorrhage occurs at a rate of 5-15% annually. Each bleed has a mortality approaching 20%. Initial management of esophageal variceal hemorrhage is focused on resuscitation and airway control. Two large-bore IVs are placed, rapid transfusion is begun, and the patient is intubated. Transfusion should target hematocrit levels of 25-30%, as overtransfusion may worsen portal hypertension. Coagulopathy should be corrected. Infusion with vasopressin is begun, in conjunction with nitroglycerin to prevent ischemia. Additional infusion with octreotide may also be performed with proton pump inhibitors. After resuscitation is initiated, control is attempted with esophagogastroduodenoscopy, using sclerotherapy and banding. If bleeding recurs after control, a surgical shunt or transjugular intrahepatic portosystemic shunt may be considered. If bleeding is not controlled on initial endoscopy, balloon tamponade with a Sengstaken-Blakemore tube or Minnesota tube is used. These consist of large, round gastric balloons that are placed to traction to compress the gastroesophageal junction. A distal port evacuates luminal contents. Minnesota tubes also have a proximal port to allow drainage of salivary secretions. Aspiration is the most common complication, and intubation is often necessary. With proper application, bleeding is controlled in 90% of cases, with a 50% rebleed rate.

Hydralazine

Hydralazine is used in the treatment of moderate-to-severe hypertension. Hydralazine causes direct vasodilation of arterioles with decreased systemic resistance.

LCIS

Lobular carcinoma in situ (LCIS) is a tumor marker for later development of breast cancer. The most common histology of the cancers that do occur is ductal. There is also an increased risk for bilateral disease when LCIS is found. DOES NOT BECOME LOBULAR CARCINOMA

Mattox maneuver

Exposure of the left retroperitoneum and the entire abdominal aorta involves the Mattox maneuver or the left-sided medial visceral rotation. The first step is mobilization of the most inferior portion of the descending colon. The white line of Toldt is divided cranially, typically bluntly. This allows for rotation of the left kidney spleen, pancreas, and stomach medially. The left crus of the diaphragm can also be divided for further suprarenal aorta access. Often, a large retroperitoneal hematoma facilitates the dissection by moving the retroperitoneal organs anteriorly.

external hemorrhoids

External hemorrhoids become symptomatic when thrombosed. Thrombosis may be the result of excessive straining, extreme physical activity, or random event. Patients present with acute onset of constant anal pain and often report a sensation of sitting on a tender marble. The physical examination identifies the external thrombosis as a purple mass at the anal verge. The treatment is dependent on the patient's symptoms and on timing. If a thrombosed external hemorrhoid is diagnosed within 3-4 days and is symptomatic, excision of the thrombus can be performed. This can be done in the office or emergency department setting using local anesthesia. The skin overlying the thrombus is then incised in an elliptical fashion. The entirety of the thrombus should be removed. If the patient presents in a delayed fashion (>4 days) and/or has minimal pain, then nonoperative management is appropriate. Typically, pain is significantly decreased by the 4th day. Answer B: Excision of the thrombus can be done under local anesthesia in an office or emergency department setting. Answer C: Asymptomatic external hemorrhoids can be safely watched. Answer D: After 4 days, the pain generally diminishes, and nonoperative treatment can be attempted. Failure may necessitate operative management. Answer E: External hemorrhoids arise from the anoderm below the dentate line, which is highly sensitive because of somatic enervation provided by the inferior hemorrhoidal nerve and lined with modified squamous mucosa. Therefore, rubber band ligation will result in profound pain and is contraindicated. Bottom Line: The treatment is dependent on the patient's symptoms. In the first 4 days following the onset of thrombosis, pain typically increases with a peak at 48 hours and excision is warranted. After 4 days, the pain generally diminishes, and nonoperative treatment can be attempted. Failure may necessitate operative management.

Low risk patients chemoprophylaxis before surgery

For general surgery patients, prophylaxis options of proven benefit in prospective trials include low-dose unfractionated heparin (LDUH), low molecular weight heparin (LMWH), intermittent pneumatic compression, and oral warfarin. Subcutaneous heparinoids must be administered 2h before induction to be maximally effective, making them somewhat inconvenient for use in ambulatory or same-day admission settings. For low risk outpatient procedures, there is no indication to administer preoperative heparin.

Factor V Leiden

Factor V mutation (Leiden ) mutation is associated with hypercoagulability. Factor V acts with Factor Xa to activate thrombin which in turn converts fibrinogen to fibrin resulting in clot formation. Factor V mutation results in resistance to degradation by activated protein C and hence hypercoagulability is encouraged. Factor V (Leiden) mutation usually manifests as venous thrombosis and diagnosed by activated protein C resistance assay or a genetic test to detect the mutation.

Fat necrosis

Fat necrosis presents as a firm, irregular, often nontender mass that may be associated with skin retraction and thickening and often is not distinguishable from cancer. Trauma, breast surgery, radiation, and infection are all causes of fat necrosis, but frequently a cause cannot be identified in the history. Mammography should be performed and compared to past images. A radiologic diagnosis can be made if an oil cyst, soft tissue and oil with a calcified rim, is identified. Biopsy should be performed in all other cases. Biopsies of fat necrosis reveal lymphocytes, histiocytes, fat necrosis, and saponification. Risk factors for fat necrosis include trauma, breast surgery, radiation therapy, infection, and obesity. The characteristic appearance of fat necrosis on mammography is one of soft tissue and fat with a calcified rim. Fat necrosis can have spiculated calcifications on mammography and can appear without a calcified rim. If there is doubt about the diagnosis of fat necrosis then a core needle biopsy should be performed.

Blood donation reactions

Febrile transfusion reactions are the most common type of transfusion reaction. Febrile transfusion reactions are mediated by circulating cytokines to donor blood as well as cytokines within the stored donor blood. Anaphylaxis after transfusion is caused by recipient antibodies to donor IgA. Viral contamination of the blood rarely causes an immediate reaction. Urticaria is caused by recipient antibodies to donor plasma proteins. Donor antibodies to recipients WBC's causes a transfusion related acute lung injury.

LCIS

Lobular carcinoma in situ increases the risk of breast cancer developing in either breast by > 7 times

Fibroadenoma

Fibroadenomas are common throughout the reproductive years and after menopause. The majority are small, very discrete, with smooth edges, mobile, and asymptomatic or minimally symptomatic. After menopause, fibroadenomas often present as a calcified density on mammogram. They can fluctuate with menses and grow with the hormonal stimulation of oral contraceptives and pregnancy. Their identity can usually be confirmed by ultrasound-guided core biopsy, after which most can be left in place and observed over time with serial exams or ultrasound if a mass is not palpable. Answer A: A fibroadenoma generally requires excision if it grows over time, is large (>2 or 3 cm) at diagnosis, or is painful. Answer B: Followup can include serial exams if the mass is palpable and ultrasound if it is not palpable.

Wound healing

Fibroblasts are stimulated by macrophages in wound healing and continue to proliferate due to a variety of growth factors. Fibroblasts are prevalent during the proliferative phase of healing, which starts approximately 5 days after injury. One of the main functions of fibroblasts is to produce collagen.

Finasteride

Finasteride's mechanism of action is competitive inhibition of the enzyme 5α-reductase. This leads to decreased serum and intraprostatic levels of dihydrotestosterone and decreased prostate volume. Finasteride can help relieve bladder outlet obstruction caused by an enlarged prostate

Paronychia and felon

Finger pulp infections account for 15-20% of all hand infections. Minor infections can be treated with warm soaks and oral antibiotics. However, felon is a severe, closed space infection of the fingertip pulp that is often a sequelae to paronychia. A paronychia infection is localized around the nail base and extends below the nail to cause severe pain. The volar pulp of the distal digital segment is a closed fascial space formed by multiple septae from the volar distal phalanx periosteum to the finger tip skin. The most common organism is Staphylococcus aureus. It can develop into an abscess and lead to 75% necrosis of the distal terminal phalanx, thus needing to be unroofed and drained for adequate treatment. Radiograph of the hand should be obtained in order to rule out any foreign body leading to the abscess. Antibiotics with anti-staphylococcal and/or anti-streptococcal coverage is an important adjunct but does not suffice alone. Cultures should be obtained after incision and drainage to tailor antimicrobial therapy.

Surgical antibiotics

First-generation cephalosporins (e.g., cefazolin) are ideal agents for prophylaxis. Third-generation cephalosporins are associated with an increased expense, do not improve efficacy, and promote emergence of resistant strains. The most important first-generation cephalosporin for surgical patients continues to be cefazolin. Administered IV in the operating room at the time of skin incision, it provides adequate tissue levels throughout most of the operation. A second dose administered in the operating room after 3 hours will be beneficial if the procedure lasts longer than that.

Gallstone Ileus

Fistulas can develop between the biliary tree and the small or large intestine as a result of intense inflammation surrounding a gallstone. The majority of fistulas develop in the area between the gallbladder fundus and duodenum. In gallstones greater than 2.5 cm in diameter there is an increased risk of it lodging itself in the narrowest portion of the terminal ileum, which is local to the ileocecal valve. This is considered a rare complication occurring in less than 6 in 1000 cases of cholelithiasis, and 3% or less of cases of intestinal obstruction. Patients with fistulas are typically elderly and present with intermittent symptoms over several days, due to the stone's descent toward the ileum. Imaging tests such as plain radiographs often indicate the presence of an intestinal obstruction, a stone lying outside the right upper quadrant, or air in the biliary tree. The recommended approach is exploratory laparotomy and enterotomy/removal of obstructing stone alone rather than additionally performing a cholecystectomy and closure of the cholecystoenteric fistula due to increased morbidity and mortality rates associated with the latter approach. Small bowel resection is typically not required. The gallstone can typically be retrieved via an enterotomy made about 30 cm proximal in healthier bowel. The enterotomy should be made in a longitudinal fashion along the antimesenteric side of the bowel. The stone is then milked from the ileocecal valve to the enterotomy and removed.

Neurogenic shock

Fluid resuscitation should be the initial management of neurogenic shock. In addition, Trendelenburg positioning and the use of vasopressors can be useful in the initial management of neurogenic shock.

Hepatic Lesion characteristics on imaging

Focal nodular hyperplasia (FNH) is the second most common benign neoplasm of the liver after hemangioma. The majority of FNHs are asymptomatic, found incidentally on imaging, or at laparotomy. Diagnosis of FNH can be established radiographically. On unenhanced CT, FNH may be well defined, with mass effect on adjacent vascular structures. The lesions are typically isoattenuated with a normal liver and may have the characteristic hypoattenuated central scar. With contrast in the arterial phase, FNHs enhance brightly and become isoattenuated to adjacent parenchyma in the portal phase. Small feeding vessels may be seen on the periphery of these lesions in the arterial phase. The central scar may enhance on delayed phases. Answer B: A well-defined mass with contrast enhancement during arterial phase and rapid washout during portal phase is consistent with hepatic adenoma. Answer C: A well-defined mass with peripheral to central enhancement on delayed imaging is consistent with hepatic hemangioma. Answer D: A well-defined mass with internal septae, papillary projections, intratumoral hemorrhage, and fine calcifications is consistent with benign hepatic cyst. Answer E: A poorly defined mass located peripherally with low attenuation, area of central necrosis, and cystic degeneration is consistent with a metastatic hepatic lesion. Bottom Line: Contrast enhancement during arterial phase along with the prominent central scar on imaging helps confirm the diagnosis of focal nodular hyperplasia.

Follicular cells of undetermined significance on FNA of thyroid nodule

Follicular cells of undetermined significance seen on fine needle aspiration (FNA) can not be used to make a definitive diagnosis of follicular cancer. In order to make the diagnosis, there must be capsular or vascular invasion seen on pathology slides which can only be obtained with surgical resection. When follicular cells are found on FNA, surgical resection should be performed. Lobectomy should be performed first since only 5-10% of nodules will contain cancer. If cancer is found, then a completion thyroidectomy should be performed at that time. Patients should be counseled on their risk of cancer and can be offered a total thyroidectomy on initial presentation. Core needle biopsy and radioactive iodine uptake scan would not add anything to the decision-making process and would not definitively make the diagnosis. Another option growing in popularity is molecular testing but this is still relatively new.

Stress response

Following surgery or trauma, the nervous system activates catecholamine release (epinephrine and norepinephrine release) and secretion of pituitary hormones (ACTH, GH, and ADH) via stimulation of the hypothalamus, creating a post-surgical stress response with increase levels of cortisol, insulin-like growth factors, and vasopressin. This "fight or flight" response increases glucose availability, blood volume expansion, perfusion of vital organs, and inflammation. Answer A: Protein breakdown exceeds protein synthesis in the post-surgical stress response Answer B: Thyroid stimulating hormone (TSH) does not play a major role in the stress response Answer C: Hypothalamic-pituitary feedback mechanisms fail during high stress, leaving high levels of ACTH and cortisol Answer D: Growth hormone (GH) and anti-diuretic hormone (ADH) are released from the pituitary in response to stress, however thyroid hormone has a limited role in the injured state. Answer E: ACTH secretion is increased, however it is secreted from the anterior pituitary Bottom Line: The hormonal stress-response to surgery or trauma include the involvement of ACTH, GH, ADH, catecholamines, and cytokines, increasing blood volume, glucose availability, perfusion, and inflammation. TSH does not play a significant role.

Breast Cancer screening

For women of average risk, screening includes clinical breast examination, counseling to raise awareness of breast symptoms, screening mammography. - For women under age 40, screening is not warranted since the incidence of breast cancer is low and the performance characteristics of mammography are poor. - For women age 40 to 49, biennial mammography can be offered, but the decision should be individualized based on patient preferences and values. A woman may opt for screening if she has substantial concerns about breast cancer risk and accepts the possibility of a false-positive result or overdiagnosis and the resulting evaluation and treatment. Another woman may find that the frequency of false positives and overdiagnosis provides a compelling reason to defer screening. Although screening in the 40s appears favorable when considering the number of years of life potentially saved, for an average-risk woman, the number of breast cancer deaths prevented is relatively low. - For women age 50 to 74, annual mammography should be offered - For women age 75 and older, annual mammography should be offered if life expectancy is at least 10 years

Fracture fixation

Fracture displacement is the primary determinant as to whether fixation is needed. If the fracture is undisplaced or minimally displaced (type I), then casting is appropriate. Type II and type III (displaced but with intact cortex) fractures would benefit from pin fixation after closed reduction. Open reduction is performed after failure of closed reduction. Even in the face of possible neurovascular compromise, an attempt at emergent closed reduction with pinning can be made, based on expert consensus. Evidence for open reduction is inconclusive and left to the discretion of the orthopedic surgeon. Answer A: If the fracture is minimally or nondisplaced, then casting is appropriate (type I fractures). Answer C: Open reduction is performed after failure of closed reduction. Answer D: Closed reduction of type II and III fractures should be accompanied by pin fixation. Answer E: A sling for comfort and analgesics is not the recommended initial treatment for this fracture. Bottom Line: Type II and type III (displaced but with intact cortex) fractures would benefit from pin fixation after closed reduction.

Frey syndrome

Frey syndrome may present after a parotidectomy or other surgical, traumatic, or inflammatory injuries of the parotid or submandibular glands. The pathology of this process involves aberrant connections between postganglionic secretomotor parasympathetic fibers to the parotid gland and postganglionic sympathetic fibers supplying the sweat glands of the facial skin on the ipsilateral side. Felty syndrome is the triad of rheumatoid arthritis, splenomegaly, and granulocytopenia.

GIST

GIST tumors are most commonly found in the stomach as a submucosal mass or lesion. They are c-Kit positive and spread hematogenously. Therefore, a lymph node dissection is not indicated. Grossly negative margins are all that is recommended for adequate resection. GIST originate from interstitial cells of Cajal. Imatinib is a tyrosine kinase inhibitor that has shown some benefit in treatment of GIST. Most common site of mets is LIVER

GIST tumors

GIST tumors have the potential to behave malignantly and metastasize to distant organs. The most prominent indicator of this potential is mitotic figures and size of the tumor. It is recommended that tumors with > 5 mitoses per high power field and size > 5 cm be treated with Imatinib postoperatively to marginalize the malignant potential.

Gallbladder cancer

Gallbladder cancer is considered a rare malignancy. Approximately 6,000-7,000 patients are newly diagnosed with gallbladder cancer each year. The 5-year survival rates for patients with gallbladder cancer are less than 5%. Due to the lesion's rarity and poor prognosis, the extent of surgical resection remains ill-defined. Attesting to the rarity of this lesion, after routine screening of abdominal ultrasounds in asymptomatic patients in Japan, only 0.01% were found to have gallbladder cancer. This tumor occurs more frequently in women (female-to-male ratio = 3:1), and peak incidence occurs in patients in their 70s. Choledochal cysts have an increased risk of developing carcinoma anywhere in the biliary tree; however, the incidence is highest in the gallbladder. This risk increases with age. Therefore, complete surgical resection is recommended for all patients with choledochal cysts at the time of diagnosis. Answer A: The increased risk of gallbladder cancer with cholelithiasis is well established; 70-90% of all patients with carcinoma also have gallstones. However, less than 0.5% of patients with gallstones are found to have gallbladder cancer. Answer B: The association of gallstones with carcinoma is probably related to chronic inflammation. Larger stones (>3 cm) are associated with a 10-fold increased risk of cancer. Answer C: In the past, the finding of a calcified gallbladder wall, or "porcelain gallbladder," was associated with a high risk of cancer, in some series ranging from 25% to 60%. Thus, the recommendation was for all patients with porcelain gallbladder to undergo open cholecystectomy, even if asymptomatic. Recent series evaluating this issue, however, suggest that the risk of gallbladder cancer in patients with porcelain gallbladder has likely been greatly overestimated. In fact, although patients with limited areas of calcification of the wall may have a higher incidence of gallbladder cancer (7%), patients with diffuse calcification of the gallbladder wall, the classic presentation for porcelain gallbladder, do not appear to have an increased risk of gallbladder cancer. Answer D: Large polyps, >10 mm, have the greatest malignant potential. Therefore, if large (>1 cm) polyps are present, even in asymptomatic patients without stones, cholecystectomy is warranted. Bottom Line: Cholelithiasis, polyps, choledochal cysts, and to a lesser extent, porcelain gallbladder increase risk of gallbladder cancer.

Gallbladder contraction

Gallbladder contraction is also stimulated by vagal fibers in response to eating a meal. Answer A: One of the main stimuli to gallbladder emptying is the hormone cholecystokinin, which is released endogenously from the duodenal mucosa in response to a meal. Answer B: Gallbladder emptying takes about 1-2 hours after the meal has been ingested. When stimulated by eating, the gallbladder empties 50-70% of its contents within 30-40 minutes. Over the following 60-90 minutes, the gallbladder gradually refills. Answer C: Hydrochloric acid, partly digested proteins, and fatty acids in the duodenum stimulate the release of secretin (not gastrin) from the duodenum that, in turn, increases bile production and bile flow. Answer E: The sphincter of Oddi actions include contraction of the gallbladder and relaxation of the sphincter of Oddi. Bottom Line: The vagus nerve stimulates contraction of the gallbladder, and splanchnic sympathetic stimulation is inhibitory to its motor activity

Gallbladder polyp

Gallbladder polyps have been noted in 3% to 6% of patients undergoing ultrasonography. Most are cholesterol polyps with no malignant potential. Additionally, most polyps are asymptomatic, and if no other cause is found to explain the abdominal symptoms, a cholecystectomy should be performed. The only polypoid lesions that have malignant potential and are associated with a significant rate of harboring malignancy are adenomatous polyps. Answer A: Routine cholecystectomy is recommended for polyps larger than 10 mm, polyps in patients with cholelithiasis, and in patients with primary sclerosis cholangitis. Answer B: Patients with polyps smaller than 1 cm who are asymptomatic should be followed up with serial ultrasound in 6 to 12-months intervals for 2 years to rule out the possibility of a growing adenomatous polyp. If the polyp size is stable, routine surveillance can be stopped. Additional studies should only be performed in the setting of new clinical findings or symptoms. Answer C: Surveillance with a CT scan is not recommended unless new symptoms emerge or there is a significant change in the lesion visualized on ultrasound. Answer D: Surveillance is standard for asymptomatic, gallbladder polyps that are assessed to be less than 1cm with no significant findings suggestive of malignancy. Answer E: Percutaneous cholecystostomy tube placement is not indicated for gallbladder polyps. Bottom Line: Patients with polyps smaller than 1 cm who are asymptomatic should be followed up with serial ultrasound in 6 to 12-month intervals for 2 years to rule out the possibility of a growing adenomatous polyp.

Ganglioneuromas

Ganglioneuromas are rare, benign, slow-growing tumors that arise from sympathetic ganglion cells. They are large encapsulated tumors, with an average size of 7 cm. They are more common in females younger than 20. They occur anywhere along the sympathetic chain, with common locations being the mediastinum, retroperitoneum, and adrenal glands. Histopathology reveals large, mature ganglion cells, axons, satellite cells, Schwann cells, and fibrous stroma. The Schwann cells are not neoplastic but associate with the neurons, though they do not elaborate any myelin. This separates them from schwannomas and neurofibromas, in which the Schwann cells are neoplastic. Immunohistochemistry shows strong staining of the ganglion cells for neurofilaments and strong staining of Schwann cells by S100.

ZES

Gastric acid hypersecretion can be effectively controlled with medications in all patients with ZES. Originally, total gastrectomy was the procedure of choice for the control of gastric acid hypersecretion, but it is no longer indicated. With successful control of gastric acid hypersecretion and the indolent growth pattern of the gastrinoma, distant metastatic disease is the most important determinant of survival. A histologic diagnosis of cancer is impossible to make pathologically, and malignancy is diagnosed by identifying lymph node and distant visceral metastases.

Gastric lymphoma

Gastric lymphoma is best diagnosed with endoscopy and biopsy. Low-grade lymphomas can be precipitated by a Helicobacter pylori infection and can be treated with antibiotics alone, which usually include clarithromycin and amoxicillin. High-grade lymphomas are better treated with a combination of chemotherapy and radiation. Surgery is rarely indicated with gastric lymphoma and is reserved only for instances of perforation or uncontrolled bleeding.

Gastric polyps

Gastric polyps are most commonly discovered incidentally during endoscopic evaluation. Polyps are more common in patients who have a history of Helicobacter pylori infection and atrophic gastritis. Polyps found in the fundus have no malignant potential. Similar to colonic polyps, gastric polyps are either hyperplastic or adenomatous. Adenomatous polyps can be tubular, tubulovillous, or villous and increase in their malignant potential, respectively. Gastric polyps should be excised endoscopically and sent to pathology to determine if any invasive cancer is involved. Answer A: Hyperplastic polyps are more common than adenomatous. Answer C: Gastric polyps are not hamartomas. In addition, Gardner syndrome is associated with polyps in the stomach, duodenum, spleen, kidneys, liver, mesentery, and small bowel. Answer D: Risk factors for gastric polyps include atrophic gastritis and Helicobacter pylori infection. Answer E: Polyps are usually discovered incidentally without symptoms. Bottom Line: Gastric polyps are most commonly benign hyperplastic polyps.

Gastrin

Gastrin is a gastrointestinal peptide that is produced by the G-cells of the gastric antrum. It is the main regulator of the gastric phase of acid secretion following food intake. Gastrin stimulates acid secretion via stimulation of the synthesis and release of histamine from enterochromaffin-like cells, which then induces acid secretion by binding to H2 receptors on parietal cells. Gastrin also stimulates acid secretion from parietal cells via cholecystokinin B receptor. Answer A: Gastrin release is inhibited by adenosine. Answer B: Gastrin release is stimulated by acetylcholine. Answer D: Gastrin is produced by the G-cells of the gastric antrum as well as, although in much smaller amounts, in the proximal small intestine, colon, and pancreas. Answer E: Gastrin released from G-cells interacts with receptors on enterochromaffin-like (ECL) cells and parietal cells to stimulate the release of histamine and hydrochloric acid. Acidification of the antral lumen results in the activation of D-cells. Somatostatin release from D-cells then causes a decrease of histamine and gastrin release from the ECL cells and G-cells, respectively. Somatostatin release also inhibits parietal cell secretion. Bottom Line: Gastrin stimulates the release of gastric acid and mucosal growth. Its release is stimulated by food, acetylcholine, gastrin-releasing peptide, secretin, beta-2/beta-3 agonists, 5HT, calcium, amino acids, amines, capsaicin, and alcoholic beverages.

Gastrinoma

Gastrinoma occurs in both sporadic (80% of cases) and familial or inherited (20% of cases) forms. The familial form is associated with multiple endocrine neoplasia type I (MEN-I) syndrome. Answer B: Although slow growing, more than 60% of gastrinomas are malignant. Answer C: There is a male preponderance of gastrinoma, with a male-to-female ratio of approximately 2:1. Answer D: Gastric acid hypersecretion also leads to secretory diarrhea, which occurs in up to 40% of patients with Zollinger-Ellison syndrome (ZES) and may be the sole initial complaint in 20% of individuals. Answer E: The majority of patients with ZES (80-90%) are found to have peptic ulceration, and the proximal duodenum is the most common site of ulcer. Bottom Line: Gastrinoma usually presents with symptoms from peptic ulcer disease, but it can present with diarrhea in 20% of cases. A total of 20% of cases are related to multiple endocrine neoplasia type I syndrome. TrueLearn Insight : Gastrinoma is the most common pancreatic tumor in patients with multiple endocrine neoplasia type I syndrome; however, insulinoma is the most common neuroendocrine pancreatic tumor overall.

Gastrointestinal lymphomas

Gastrointestinal lymphomas have a variety of etiologies, including both B-cell and T-cell origins. Small bowel lymphomas are more commonly from B-cell lines and often present with vague abdominal pain and weakness. They most commonly occur in the ileum, where the Peyer patches lymph node basins exist and can be diagnosed best on CT scan. Biopsies can be useful to confirm the type and grade of lymphoma, which will help to guide the type of treatment. Patients with a history of celiac disease are more likely to develop T-cell lymphoma of the small bowel. Immune-compromised patients are also at a higher risk. Answer A: Biopsy of small bowel lymphoma will help guide treatment. Answer C: B-cell lymphoma is more common in the small bowel. Answer D: B-cell mucosa-associated lymphoid tissue lymphomas are associated with H. pylori infections. Answer E: The ileum is the most common location for lymphoma. Bottom Line: Celiac disease is a risk factor for small bowel T-cell lymphoma.

Gastroparesis

Gastroparesis causes symptoms similar to that of a gastric outlet obstruction, but without any mechanical cause. These symptoms include nausea, inability to tolerate a diet, bloating, and vomiting. There are many causes of gastroparesis, the most common being idiopathic. Diabetes is the second most common cause followed by vagal injuries. The most common surgical procedure that leads to a vagal injury is a Nissen fundoplication. There are a host of medications and medical conditions that can also precipitate gastroparesis which include anything from narcotics and octreotide to scleroderma and electrolyte abnormalities. In most cases of mild gastroparesis, symptoms can be controlled with dietary modifications or with the use of certain medications. Answer A: It is more common in women. Answer B: Idiopathic gastroparesis is the most common cause. Answer D: There is no increased risk for gastric cancer with gastroparesis. Answer E: One test to determine an appropriate vagal response is a sham feeding test which looks at postprandial pancreatic polypeptide levels. Bottom Line: There are many causes of gastroparesis, the most common being idiopathic. Diabetes is the second most common cause followed by vagal injuries.

Gastroparesis

Gastroparesis is a disease of poor motility of the stomach leading to bloating, nausea, and inability to tolerate a diet. Gastroparesis can be due to previous gastric surgery with interruption of the vagus nerves or poorly controlled diabetes, but the most common cause is idiopathic. If gastroparesis is suspected, it is important to perform an upper endoscopy to ensure that there is no organic cause for the symptoms. These include gastritis, ulcers, gastric malignancy or gastric outlet obstruction. If the patient does have gastroparesis, upper endoscopy will often find retained food or gastric bezoars. Once other potential causes have been ruled out, then the gastric emptying study can be performed to confirm the diagnosis.

Gastroparesis

Gastroparesis is a disease that is mostly treated by dietary modifications and medical management. Patients who are diabetic should first be placed on strict glycemic control because uncontrolled hyperglycemia has a profound effect on the gastric motility. Non-diabetic patients, or those with good glycemic control should first be treated with changes in the diet. Fatty foods delay motility and should be avoided. With a poorly functioning stomach, high fiber foods can be difficult to digest and can lead to increased bloating and gas pains. Patients should also try eating frequent small meals. If dietary modifications fail, then promotility agents can be used. Reglan is a dopamine antagonist that has shown to be effective in increasing gastric motility. Erythromycin is also commonly used and acts upon the motilin receptors to aid in gastric function. A final treatment option is with surgery, and this includes gastric pacer implantations, pyloroplasty, botox injections, and partial gastrectomy. These have varying degrees of effectiveness and should be used as a last resort.

Gastroschisis vs. omphalocele

Gastroschisis results from an abdominal wall defect located just to the right of the umbilicus in the area where normal involution of the right umbilical vein occurs. The true cause of gastrochisis is unknown, but several theories include: failure of formation of abdominal wall mesoderm in this location, ruptured amnion around the umbilical ring, weakened body wall secondary to abnormal involution of the right umbilical vein, and abdominal wall damage secondary to disrupted right vitelline artery. Answer A: By the 11th week of gestation, following the physiologic midgut herniation, the midgut returns back into the abdominal cavity and undergoes normal rotation and fixation, along with closure of the umbilical ring. If the intestine fails to return to the abdomen, the infant is born with abdominal contents protruding directly through the umbilical ring. This defect is called an omphalocele. In omphalocele, a peritoneal sac is often present but can be absent if it is disrupted in utero. Answer B: Although several mechanisms have been proposed to explain the etiology of jejunoileal atresia, the most widely accepted theory is that of an intrauterine focal mesenteric vascular accident. Answer D: The persistence of the embryonic processus vaginalis can lead to a spectrum of abnormalities, including inguinal hernia, communicating hydrocele, hydrocele of the cord, or simple hydrocele. Answer E: An umbilical hernia occurs as a result of persistence of the umbilical ring. Bottom Line: Gastroschisis is an abdominal wall defect to the right of the umbilicus, containing bowel and/or other contents not covered by a peritoneal sac. Gastroschisis should be differentiated from omphalocele, which causes herniation of the abdominal contents through the umbilical ring, with frequent presence of a peritoneal sac.

Ureteral injuries

General principles for repair: 1. debridement; 2.tension free anastomosis and precise mucosal approximation and watertight closure (using interrupted 0-4 and 0-5 absorbable sutures); 3.internal ureteral stents, in most cases (Stenting is not mandatory, but typically used if the degree of transection approaches 50% of the ureteral circumference); 5. isolation of repair with omentum or fat in patients at high risk for infection; and 6. retroperitoneal drainage Options of repair: 1.Primary closure of partial transection of the ureter: If a partial transection of the ureter from a scalpel or scissors is seen during operation, often primary closure can be accomplished. 2. Primary uretrouretrostomy: Any complete transection can often be repaired by primary ureteroureterostomy. 3. Ureteral reimplantation and psoas hitch: Injuries of the pelvic ureter in which the distal ureteral segment is of poor quality or of insufficient length are best-managed by ureteral reimplantation 4.BOARI flap: The peritoneum is swept off the bladder dome, and a posteriorly based flap is created. The flap is folded back, a submucosal tunnel is developed in the flap and the ureteral reimplantation is completed. The flap is tubularized and closed in two layers. 5. Transureterouretrostomy: In cases of extensive pelvic ureteral loss where remaining proximal ureteral length is inadequate to allow reimplantation into the bladder, or concomitant significant ipsilateral pathology is present, a transureteroureterostomy may be considered. The recipient ureter is mobilized as little as possible to avoid any devascularization, and the injured ureter is brought under the mesentery above or below the inferior mesenteric artery, depending upon the degree of ureteral loss as well as the site selected for anastomosis Isolation and drainage: omental flaps the can be wrapped around the ureteral repair, providing physical isolation. Drains should be placed in the retro or extraperitoneal space.

Peak vs. trough levels

Gentamycin is a bactericidal antibiotic with a normal peak level of 6-10 mcg/ml and a normal trough of < 1 mcg/ml. Peak levels that are too high can be adjusted by decreasing the amount of the dose. Trough levels that are elevated are adjusted by decreasing the frequency of the dose. The converse is true: Peak levels that are too low can be adjusted by increasing the amount of the dose and trough levels that are decreased are adjusted by increasing the frequency of the dose.

CBD exploration

Glucagon can be used to relax the sphincter of Oddi. The typical dose is 1.0 mg Glucagon and can be repeated once if necessary or unsuccessful.

CBD stone

Glucagon causes sphincter of Oddi relaxation and allows CBD stones to be flushed more easily. Intravenous glucagon administration is the next step in stone removal when flushing alone fails

Energy source for enterocytes?

Glutamine

Primary fuel for enterocytes?

Glutamine

Gross contaminated wounds

Gross contaminated wounds include traumatic wounds that have a significant delay in attaining treatment, often including areas of necrosis or frank purulence. Had the bowel been frankly necrotic with purulent fluid in the hernia sac, this wound would have been a class 4 wound.

Health care associated infections

Healthcare associated infections (HAIs) are extremely common (>600,000/year) and associated with patients who are older, have longer lengths of stay, have invasive lines, catheters and ventilators or are in critical care units. The most common HAIs are pneumonia, mostly ventilator associated, gastrointestinal disorders, most commonly Clostridium difficile, catheter related blood stream infections, catheter associated urinary tract infection and surgical site infection in patients who have had surgery. Pneumonia is most frequently associated with Staphyloccocus aureus in the acute care setting. Answer A: Central line associated bacteremia is most frequently associated with candida species, followed by staphyloccocus aureus Answer B: Hospital acquired pneumonia is, according to some sources, the most common HAI in the United States. Staphylococcus aureus followed by pseudomonas aeruginosa are the most commonly causative organisms. The species associated with hospital acquired pneumonia and/or ventilator associated pneumonia are different than community acquired pneumonia (Streptococcus pneumoniae, Haemophilus influenzae). Pneumonia is currently the most common HAI. Answer C: Though surgical site infection causative species will differ by type of surgery, the most common overall is Staphylococcus aureus. Answer D: Urinary tract infections are common in hospital patients due to foley catheters. Escherichia coli is most often the culprit. Answer E: In hospitalized patients, clostridium difficile is the most common GI infection. C. difficile infection is frequently associated with antibiotic use. Bottom Line: Healthcare associated infections are common and species differ from community acquired infections.

Hemophilia A

Hemophilia A is a disorder of factor VIII deficiency. Dental procedures or the prevention of hemarthrosis may be repleted to 30% of normal levels to avoid bleeding. At least 50% of the levels are required if bleeding into a joint is already present. 100% repletion is required for life-threatening hemorrhage or major surgery. An initial dose of 25 units/kg will replenish the factor VIII level to at least 50%, assuming the factor level is at almost zero.

Hemophilia A

Hemophilia A is secondary to deficiency in Factor VIII. It is a sex-linked recessive genetic condition and the most common inherited coagulation disorder. It affects the intrinsic pathway resulting in prolonged PTT with normal PT and bleeding time. Clinical manifestations depend on the level of Factor VIII, varying from less than 1% in severely affected individuals to as high as 40% in those with mild hemophilia. Patients with Factor VIII levels greater than 5% rarely bleed spontaneously but will have bleeding problems after surgery or trauma. Most of the carriers have factor VIII levels that are greater than 50% and have no difficulty with spontaneous bleeding or with abnormal bleeding after surgery. Desmopressin (DDAVP), used for treating mild cases, causes release of Factor VIII from the liver sinusoids, endothelial cells and also release of vWf from endothelial cells resulting in transient increase in Factor VIII levels. Factor VIII concentrates are available and used in severe cases.

Hemorrhoidectomy

Hemorrhoidectomy is typically indicated for patients who fail nonoperative management. There are several techniques available to perform hemorrhoidectomy including open (Miligan-Morgan) technique, closed (Ferguson) technique, circumferential (Whitehead) technique, stapled hemorrhoidectomy, and transanal hemorrhoidal dearterialization. Transanal hemorrhoidal dearterialization is a nonexcisional technique that involves a Doppler-guided ligation of the arterial inflow to the hemorrhoids along with suture rectopexy. Answer A: Hemorrhoidopexy using a transanal circular stapler is a description of stapled hemorrhoidectomy. Answer C: Circumferential excision of the internal hemorrhoids just proximal to the dentate line is a description of circumferential (Whitehead) hemorrhoidectomy. Answer D: Excision of the hemorrhoids, then leaving the wound open, is a description of open (Miligan-Morgan) hemorrhoidectomy. Answer E: Excision of the hemorrhoids, followed by closing the wound, is a description of closed (Ferguson) hemorrhoidectomy.

Hemorrhoids

Hemorrhoids are considered a normal component of anorectal anatomy. The term "hemorrhoid" is used to describe the vascular cushions present in the anal canal. These cushions are a normal anatomic and physiological component of the anus. They serve to maintain closure of the anal canal, thus contributing toward fecal continence. Although gross inspection of hemorrhoids reveals a blue color, similar to veins, histologic analysis of hemorrhoids demonstrates absence of the muscular layer, suggesting that hemorrhoids are technically not veins or arteries. They are commonly located in the left lateral, right anterior, and right posterior configuration. Distal to the dentate line, external hemorrhoids are drained via the inferior hemorrhoidal veins. Internal hemorrhoids, located proximal to the dentate line, are drained via the middle hemorrhoidal veins. Answer D: Proximal to the dentate line, the anal canal is innervated by sympathetic and parasympathetic nerves. Distal to the dentate line, the anoderm is innervated by somatic nerves. Therefore, external hemorrhoids are sensitive to touch, pain, temperature, and stretch.

Chemoembolization

Hepatic metastases derive their blood supply from the hepatic artery preferentially. Hepatic artery infusion of chemotherapy (chemoembolization) of lesions provides a better response than systemic chemotherapy with a lower side effect profile. Regional infusion of chemotherapy has high extraction rates of 90%, causing high local concentration of chemotherapy agents. It should be used as an adjuvant therapy and will not result in cure.

Hepatoblastoma

Hepatoblastoma (HB) is the most common primary malignant tumor of the liver in children. HB is strongly associated with Beckwith-Wiedemann syndrome and hemihypertrophy. Hemihypertrophy is associated with an increased risk of other tumors such as Wilms tumor. In addition to Beckwith-Weidemann syndrome, HB is associated with a number of other genetic syndromes, including Li-Fraumeni syndrome, familial adenomatous polyposis, and trisomy 18. The strength of association between Beckwith-Weidemann syndrome has resulted in some experts to recommend that children with Beckwith-Weidemann syndrome be screened for HB with abdominal ultrasound and alpha-fetoprotein. Answer A: Hepatoblastoma affects young children between 6 months and 3 years old. Answer B: Elevated alpha-fetoprotein is observed in more than 90% of children with hepatoblastoma. Alpha-fetoprotein is used as serum tumor marker for postoperative surveillance. Answer C: Most children with hepatoblastoma present with unresectable disease. Answer E: Hepatoblastomas typically present as an asymptomatic abdominal mass or less commonly with symptoms of GI obstruction from the mass effect. Obstructive jaundice is unusual. Bottom Line: Hepatoblastoma is strongly associated with Beckwith-Wiedemann syndrome and hemihypertrophy.

Hepatocellular carcinoma

Hepatocellular carcinoma (HCC) has multiple variants, the most common being fibrolamellar HCC. Fibrolamellar is characteristically described as a well-circumscribed mass that has a central scar similar to that found in focal nodular hyperplasia (FNH). Histologically, fibrolamellar HCC has clusters of large polygonal cells interspersed with sheets of collagen. It is differentiated from HCC by its gross and histologic appearance as well as its normal alpha-fetoprotein levels. Fibrolamellar HCC often has an elevated neurotensin level. Answer A: The lesion may hyperdense with a central scar during the hepatic arterial phase. Answer B: Hepatoblastoma is a rare malignancy found most commonly in children. Answer C: Fibrolamellar HCC will be well circumscribed with an elevated neurotensin level. Answer E: These lesions can often be differentiated from FNH by the absence of punctate enhancement of the central scar during the arterial phase of a helical CT. Bottom Line: Fibrolamellar hepatocellular carcinoma is characterized as a well-circumscribed hepatic mass with an elevated neurotensin level.

Hepatocellular carcinoma transplantation

Hepatocellular carcinoma often develops in the cirrhotic liver. This precludes many from undergoing liver resection. Only normal liver or Child class A cirrhotic patients should be considered for limited resection. The initial data was quite poor for survival after transplantation because they included all comers. When strict criteria are applied the outcomes are markedly improved. When a single tumor is < 5 cm, or up to 3 tumors all < 3 cm, no vascular invasion, or no nodal or metastatic disease is detected, the 5-year survival is 65-90% after transplantation. In this case, 80% would be the most correct answer.

HNPCC

Hereditary nonpolyposis colorectal cancer (HNPCC) is an autosomal dominant disorder with a high incidence of colon cancer. The tumors are known to have high microsatellite instability and defects on the MLH1 and MSH2 genes, which are responsible for DNA mismatch repair. Lynch I syndrome is associated with colon cancer, whereas Lynch II syndrome is also associated with endometrial, ovarian, and gastric cancers. Even with the strong hereditary concerns, there still remains a 20% risk of sporadic mutation for HNPCC. Answers A & E: HNPCC is an autosomal dominant disorder with a high incidence of colon cancer. The tumors are known to have high microsatellite instability and defects on the MLH1 and MSH2 genes, which are responsible for DNA mismatch repair. Answer B: Lynch I syndrome is associated with colon cancer, whereas Lynch II syndrome is also associated with endometrial, ovarian, and gastric cancers. Answer C: Screening for these genetic associations is recommended for patients who fit the Amsterdam criteria. This includes three or more first-degree relatives with colon cancer, across two generations, with one member diagnosed prior to the age of 50. Bottom Line: Even with the strong hereditary concerns, there still remains a 20% risk of sporadic mutation for hereditary nonpolyposis colorectal cancer.

Hereditary pancreatitis

Hereditary pancreatitis is a rare form of recurrent pancreatitis with an estimated incidence of 0.13-0.57/100,000. Patients younger than 35 years old with a history of unexplained recurrent acute pancreatitis or a family history of unexplained pancreatitis should be referred for genetic counseling and, if they choose, undergo genetic testing for cationic trypsinogen gene (PRSS1) mutation. The diagnosis of hereditary pancreatitis can be made if other etiologies are excluded and the patient has a PRSS1 mutation or a family history of unexplained recurrent acute pancreatitis or unexplained chronic pancreatitis in 2 or more first degree relatives or in 3 or more second degree relatives from 2 or more generations. Hereditary pancreatitis is associated with several mutations, but the autosomal dominant mutation in the PRSS1 gene is the most common and is diagnostic for the disease. There is no specific treatment for hereditary pancreatitis. Episodes of acute pancreatitis and chronic pancreatitis among patients with hereditary pancreatitis should be managed as they are in patients with pancreatitis of other etiologies (the exception is autoimmune pancreatitis which may resolve with corticosteroids). All patients with chronic pancreatitis are at increased risk of developing pancreatic cancer, for which smoking is an independent risk factor, therefore patients should be advised about smoking cessation. Pancreatic exocrine enzyme should be replaced when symptoms of exocrine insufficiency develop. Exocrine and endocrine insufficiency occur in 5.5-34% and 12.5-26% of patients, respectively. Some have suggested exocrine enzyme replacement to mitigate the pain of pancreatitis, but the results of randomized studies are mixed and a meta-analysis showed no benefit of pancreatic exocrine enzymes for the treatment of pancreatitis pain.

Soft tissue sarcoma

High-grade histologic findings, deep location, and tumor >5 cm are independent prognostic factors for survival. Staging classifies lesions as T1 — < 5 cm T2 — 5 cm N1 — regional node involvement G1 — well-differentiated G2 — moderately differentiated G3 — poorly differentiated G4 — undifferentiated. Staging is based on T, N, M, and G classification. Stage IV disease is classified as any G or T with any N and evidence of metastatic disease. Primary therapy includes surgical resection with margin of normal tissue. Limb sparing is often possible. Local control preoperatively may be achieved with radiation therapy.

Vagotomy

Highly selective vagotomy removes only the innervation to the lesser curvature of the stomach and preserves the innervation to the pylorus. Consequently, a highly selective vagotomy does not require a concomitant drainage procedure. It also preserves pyloric function and therefore has a lower risk of dumping syndrome. Truncal vagotomy severs both vagi as they course along the esophagus near the diaphragmatic hiatus. This procedure comes with a higher risk of dumping syndrome because the pylorus loses its function and requires a drainage procedure such as pyloroplasty. Answer A: Since the function of the antrum remains intact, there is no need to perform a drainage procedure. Answer B: The blood supply runs with the nerve innervation, therefore the blood supply to the lesser curvature is disrupted. The stomach has a robust circulation and this is almost never a problem. Answer C: Truncal vagotomy requires a drainage procedure as the pylorus function is disrupted. Answer D: The pneumonic is: LARP-> Left Anterior, Right Posterior Bottom Line: Highly selective vagotomy has fewer postoperative complications and does not require a drainage procedure.

hyperacute rejection

Hyperacute rejection usually occurs within minutes of graft reperfusion. It is caused by preformed recipient antibodies toward the donor. The antibodies may be against the donor HLA or ABO antigens. They bind to the graft's vascular endothelium and activate the compliment cascade. This leads to a dark, swollen graft which undergoes ischemic necrosis. Treatment involves immediate removal of the graft. Hyperacute rejection should be prevented by conducting the final lymphocytotoxic cross match to detect IgG antibodies against the class I MHC receptors in the recipient.

IV Bisphosphonates

Hypercalcemia secondary to malignancies has shown good response to IV bisphosphonates

Cushing's Syndrome

Hypercortisolism is characterized by excessive circulating cortisol from either endogenous or exogenous sources. The collection of symptoms overall is called Cushing's syndrome. Cushing's disease is reserved specifically for cases of pituitary-dependent hypercortisolism. Centripetal obesity, weight gain, moon facies, amenorrhea, and hirsutism are common signs and symptoms. Workup involves first checking a 24 hour urine free cortisol to check for hypercortisolism. A single morning salivary cortisol is not useful as cortisol levels vary based on time of day. An MRI of the head is used to evaluate for a pituitary adenoma. ACTH level and the high dose dexamethasone suppression test can help differentiate between a pituitary adenoma or ectopic ACTH secreting source. CT chest/abdomen/pelvis, and inferior petrosal sinus sampling are used to look for potential ACTH secreting masses.

What electrolyte is falsely low in DKA?

Hyperglycemia causes hyperosmolar hyponatremia via plasma dilution of sodium concentration.

gynecomastia

Hypertrophy of breast tissue in men is a common clinical entity for which there is frequently no identifiable cause. Senescent hypertrophy is diagnosed in men older than 50 years. The enlargement is frequently unilateral, although the contralateral breast may enlarge with time. Both pubertal and senescent gynecomastia may be left untreated and do not require biopsy. Answer A: There is little confusion with carcinoma occurring in the male breast. Carcinoma is not usually tender, it is asymmetrical, and it may be fixed to the overlying dermis or to the deep fascia. This patient's lesion is not characteristic for carcinoma. Answer C: The mass in the description is most likely senescent gynecomastia and therefore most likely will not have any purulent discharge. Answer D: Senescent gynecomastia enlargement is frequently unilateral, although the contralateral breast may enlarge with time. Answer E: Gynecomastia may be a systemic manifestation of hepatic cirrhosis, renal failure, and malnutrition. Bottom Line: Senescent gynecomastia can be caused and exacerbated by certain drugs and medical conditions. No intervention is necessary unless there is suspicion for malignancy. TrueLearn Insight : Gynecomastia in children or adults should be observed unless there is concern for malignancy.

Hypocalcemia

Hypocalcemia is defined as a serum calcium level below 8.5 mEq/L or a decrease in the ionized calcium level below 4.2 mg/dL. The causes of hypocalcemia include pancreatitis, massive soft tissue infections such as necrotizing fasciitis, renal failure, pancreatic and small bowel fistulas, hypoparathyroidism, toxic shock syndrome, abnormalities in magnesium levels, and tumor lysis syndrome. In general, neuromuscular and cardiac symptoms do not occur until the ionized fraction falls below 2.5 mg/dL. Clinical findings may include paresthesias of the face and extremities, muscle cramps, carpopedal spasm, stridor, tetany, and seizures. Patients may exhibit a positive Chvostek's sign (spasm resulting from tapping over the facial nerve) and Trousseau's sign (spasm resulting from pressure applied to the nerves and vessels of the upper extremity with a blood pressure cuff). Hypocalcemia may lead to decreased cardiac contractility and heart failure. ECG changes of hypocalcemia include prolonged QT interval, T-wave inversion, heart block, and ventricular fibrillation.IV calcium is the treatment for severe hypocalcemia. The neuromuscular excitability, cardiac electrical instability, and associated symptoms are relieved by prompt administration of corrective doses of calcium. Associated deficits in magnesium, potassium, and pH must also be corrected. Hypocalcemia will be refractory to treatment if coexisting hypomagnesemia is not corrected first. Chronic hypoparathyroidism requires vitamin D, oral calcium supplements, and often aluminum hydroxide gels to bind dietary phosphate in the intestine.

Effects of magnesium on hypocalcemia

Hypocalcemia will be refractory to treatment if coexisting hypomagnesemia is not corrected first. Low levels of magnesium impair adequate PTH release, as well as end organ response to PTH. Associated deficits in magnesium, potassium, and pH must be corrected in addition to correcting the calcium levels. At high concentrations (>5 mg/dL), magnesium binds the calcium-sensing receptor on the parathyroid gland, imitating hypercalcemia, which will directly suppress PTH release. Both hypo- and hypermagnesemia result in suppression of PTH.

Electrolyte abnormalities

Hyponatremia can cause all of the symptoms the patient in this question is experiencing. An acute drop in serum sodium levels can cause symptoms just below the normal range for serum sodium. Chronic hyponatremia can remain asymptomatic until lower levels are reached. Symptoms include CNS (headache, confusion, delirium, seizure), musculoskeletal (fatigue, muscle cramps), and gastrointestinal (nausea and vomiting) findings. Runners may lose large volumes of sweat, which leads to vasopressin release. Volume loss may be replaced by beverages with lower than physiologic sodium concentrations. Answer A: Hypercalcemia often causes symptoms of nausea, vomiting, diarrhea, weakness, confusion, psychoses, and kidney stones. It is frequently caused by hyperparathyroidism, malignancy, thiazides, granulomatous disease, and prolonged immobilization. Answer B: Hypermagnesemia is characterized by loss of deep tendon reflexes, paralysis, coma, hypotension, and cardiac arrest. It may follow burn or crush injuries. It rarely occurs in the setting of normal renal function. Answer C: Hyperkalemia symptoms include paresthesia, weakness, and flaccid paralysis. It may progress to cause ventricular fibrillation and cardiac arrest. Causes are numerous and include decreased renal function, ACE inhibitor medications, succinylcholine, and ischemia-reperfusion injury. Answer E: Hypokalemia can cause muscle weakness, myalgia, tremor, muscle cramps, and constipation. In severe cases, flaccid paralysis and hyporeflexia can develop. Bottom Line: Hyponatremia symptoms include CNS: headache, confusion, delirium, seizure; musculoskeletal, fatigue, muscle cramps; and gastrointestinal: nausea, vomiting.

Hypothermia

Hypothermia is defined as core body temperature less than 35 ºC (95 ºF). It is divided into primary and secondary forms. Primary hypothermia occurs due to environmental exposure to cold temperatures and the natural heat-generating ability is overwhelmed. Secondary hypothermia occurs when illness or substance abuse undermines the body's ability to regulate temperature. Hypothermia is further divided into mild, moderate, severe, and profound. Mild hypothermia (32.2-34.4 ºC, 90-94 ºF) is associated with shivering and mild mental changes. Initial cardiac response at this stage is tachycardia to increase cardiac output. With moderate hypothermia (28.88-31.66 ºC, 84-89 ºF), patients become agitated or combative. There may also be muscle spasticity, dilated pupils, and slowing of respirations. Cardiac abnormalities such as atrial fibrillation and hypotension may also develop due to decreased peripheral vascular resistance. In severe hypothermia (21.1-28.88 ºC, 70-84 ºF), EKG changes such as prolonged QRS and Osborn waves may be present. Patients may become flaccid and comatose, then eventually develop ventricular fibrillation and death. Profound hypothermia (<21.1 ºC, <70 ºF) is associated with loss of vital signs, cardiac activity, and EEG tracings. Survival with core temperature of 13.72 ºC (56.7 ºF) has been reported. Diagnosis is made by measurement of core body temperature and history. The gold standard for continuous temperature monitoring is pulmonary artery catheter, as esophageal probes can be falsely elevated from warmed ventilation, and bladder catheters can be affected by peritoneal lavage. Treatment begins by removal of wet clothing. Vigorous rubbing and friction should be avoided. For mild hypothermia, active movement alone may be sufficient. For moderate hypothermia, passive warming with thermal packs, forced air, heated blankets, and warm IV fluids should be performed. For severe hypothermia, active rewarming should be instituted. This consists of advanced techniques such as extracorporeal membrane oxygenation, cardiopulmonary bypass, thoracic lavage, and peritoneal lavage. In the event of hypotension, pressors should be used cautiously due to increased risk of cardiac arrhythmia. If a hypothermic patient undergoes arrest, survival is still possible. CPR and ACLS should be continued, despite prolonged asystole, until the patient is warmed to 32 ºC (89.6 ºF) or serum potassium is greater than 12 mmol/L. Answer A: Ambulation and increasing activity is the treatment for mild hypothermia. Answer C: Active rewarming is the treatment for severe and profound hypothermia. Answer D: Cardiac abnormalities begin to become more prominent with severe hypothermia, with ventricular fibrillation and death being a final endpoint. Answer E: In the event of hypotension, pressors should be used cautiously due to increased risk of cardiac arrhythmia. Bottom Line: For moderate hypothermia, passive warming with thermal packs, forced air, heated blankets and warm IV fluids should be performed.d

ITP

ITP is largely a diagnosis of exclusion and results from increased platelet destruction mediated by autoantibodies directed at platelet membrane antigens. Medical management includes glucocorticoids. The spleen is believed to be the source of these antibodies and is also the site of platelet destruction. Patients with ITP do not have splenomegaly and therefore do not experience abdominal pain and early satiety as may be encountered in those with myeloid leukemias. The spleen also appears to be the major source of properdin and tuftsin. However, these proteins help to prevent overwhelming infection by encapsulated organisms through initiation of the alternative pathway of compliment activation and clearance of opsonized bacteria, respectively, and play no major role in the lifespan of platelets.

ITP

ITP is most commonly seen in young women between 10 and 40 years of age. It is largely a diagnosis of exclusion and results from increased platelet destruction mediated by autoantibodies directed at platelet membrane antigens. When platelet counts are greater than 50,000 /mm3 and patients are asymptomatic, no treatment is required other than careful observation. When patients do require therapy, glucocorticoids (prednisone 1 mg/kg) is usually the initial medical treatment. If a patient fails to respond or is unable to tolerate the therapeutic regimen then they are considered for surgical intervention. Splenectomy carries a 75% to 85% long-term response rate. Unlike preemptive vaccinations (given at least 2 weeks preoperatively) or prophylactic antibiotics (given within 1 hour of making skin incision), platelet transfusions are not a scheduled event for patients with ITP. Answer B: Transfusions that are required during splenectomy are ideally held until after arterial inflow to the spleen has been ligated. This practice prevents exposure of the administered platelets to antibody opsonization and clearance by the offending organ. Answer C: Unlike preemptive vaccinations (given at least 2 weeks preoperatively) or prophylactic antibiotics (given within 1 hour of making skin incision), platelet transfusions are not a scheduled event for patients with ITP. Platelets aren't given 24 hours prior to surgery. Answer D: Unlike preemptive vaccinations (given at least 2 weeks preoperatively) or prophylactic antibiotics (given within 1 hour of making skin incision), platelet transfusions are not a scheduled event for patients with ITP. Platelets aren't given 2 weeks prior to surgery. Answer E: Unlike preemptive vaccinations (given at least 2 weeks preoperatively) or prophylactic antibiotics (given within 1 hour of making skin incision), platelet transfusions are not a scheduled event for patients with ITP. Platelets aren't given prior to surgery. Bottom Line: Administration of platelets to patients with ITP undergoing splenectomy is indicated only for significant bleeding events.

Treatment of symptomatic hypercalcemia

IV hydration and loop diuretics

ITP

IVIG is indicated in patients that have an active acute bleed, all preoperative patients, and in female patients going through childbirth. Answer A: The patients can be observed if no active bleeding and platelets > 30,000. Answer B: Steroids are the initial course of treatment for ITP, but they are only indicated if the patient is symptomatically bleeding or if the platelet counts fall below 30,000. Answer D: Platelet transfusions are not indicated unless the patient is acutely bleeding. Answer E: Splenectomy is contraindicated if there is a response to steroids. Splenectomy is recommended if the symptoms of ITP persist, or the platelet counts remain < 30,000 after three months of maximal treatment. Bottom Line: Steroids are the first line treatment for ITP when symptomatic or platelet counts < 30,000.

Parotid mass

If the mass is adjacent to the facial nerve without invasion, a superficial parotidectomy can be performed, and adjuvant radiation therapy is added for treating any residual micrometastases. Radiation therapy is also utilized for high-grade malignancies, invasion of surrounding structures, neural invasion, or metastatic disease. Answer A: Adjuvant radiation therapy is added for treating any residual micrometastases. Answers B & D: Total parotidectomy is indicated for any tumor that lies in the deep lobe. Answer E: Acinic cell carcinoma of the parotid is a malignant lesion that needs en bloc surgical resection.

What happens in a pre-operative patient who FEV1 is not adequate for lung surgery?

If the preoperative testing for a lobectomy suggests that the patient will not tolerate the procedure, the next step is to perform a ventilation/perfusion (V/Q) scan. This will show the distribution of ventilation and blood flow to each lobe of the lungs. If the desired lobe has a minimal contribution to the FEV1, then the patient can still likely tolerate a resection

Zone 3 hematoma

If there is a clear indication for abdominal exploration and the retroperitoneum is intact, the hematoma should not be entered because of an increased risk for uncontrolled bleeding. In this circumstance, stabilization of the pelvis and angiographic embolization should be performed. If the retroperitoneum is ruptured and active bleeding is found during exploratory laparotomy, packing the pelvis with temporary closure of the abdomen, followed by external fixation and angiographic evaluation, is appropriate. Answer A: Blunt zone III injuries are best treated with angiography. Answer B: Angiography is critical to control a zone III hemorrhage in blunt trauma. Answer D: Orthopedic surgery should be consulted but the hemorrhage should be addressed first. Answer E: This would not address the bleeding. Bottom Line: Zone III retroperitoneal hematomas should not be explored for blunt trauma if they are found at the time of laparotomy. External stabilization and angiographic embolization is the preferred treatment in this setting

Cancer meds

Imatinib mesylate (formerly ST1 571, now Gleevec [Novartis]) is a competitive inhibitor of certain tyrosine kinases, including the kinases associated with the transmembrane receptor kit and platelet-derived growth factor receptors. Imatinib mesylate is approved for use in CD117-positive unresectable and metastatic gastrointestinal stromal tumors in a neoadjuvant manner and as adjuvant therapy for tumors with high risk features. Answer B: Angiogenesis inhibitors, typically vascular endothelial growth factor inhibitors, show promise in the treatment of certain metastatic cancers including gastric, colorectal, and renal carcinomas. Answer C: Purine synthesis inhibitors, such as azathioprine or mycophenolate mofetil, are important immunosuppressive agents. Allopurinol is also useful for gout treatment. Answer D: Induction of apoptosis is a promising area of research for a multitude of cancers but has not been proven yet for gastrointestinal stromal tumors. Answer E: Inhibitors of cyclic AMP-mediated transport have not been used for gastrointestinal stromal tumors. Bottom Line: Gleevec (imatinib) is used as a neoadjuvant and/or adjuvant therapy for gastrointestinal stromal tumors. Its mechanism of action is competitive inhibition of certain tyrosine kinases.

Fluid resuscitation in trauma patients

Resuscitation of a hypotensive patient starts with 1-2 liters of clear fluid resuscitation, which refers to crystalloid solutions. Whether to use normal saline (NS) versus Lactated Ringers (LR) has been long debated and extensively studied. A study published in the Journal of Trauma in 2007, entitled "Lactated Ringer's is superior to normal saline in the resuscitation of uncontrolled hemorrhagic shock" concluded that LR is superior to NS for the resuscitation of uncontrolled hemorrhagic shock in swine. Normal saline was found to be associated with hyperchloremic acidosis, greater urine output and dilutional coagulopathy. Further, resuscitation using NS in patients with uncontrolled hemorrhagic shock also required significantly greater volumes of fluid compared to LR (Todd, Malinoski, & Schreiber, 2007). Extensive research has been performed comparing the two crystalloids. The two fluids are even used interchangeably in certain institutions. However, the vast majority of trauma guidelines recommend the use of LR over NS for resuscitation, especially in patients with uncontrolled hemorrhagic shock, due to the reasons mentioned above. Packed red blood cells are then indicated if the patient remains hemodynamically unstable despite crystalloid administration. If the hypotension continues, initiation of a massive transfusion protocol may be indicated. Answer A: Half normal saline is a hypotonic fluid (77mEq/L). Hypotonic fluids are not the most efficient fluids available to expand the intravascular volume. Thus, may result in greater transmembrane loss to the extracellular space. Answer B: Dextrose half normal saline is a hypotonic fluid (77mEq/L). Hypotonic fluids are not good intravascular volume expanders, and result in greater trans-membrane loss to the extra-cellular space. Dextrose equilibrates rapidly between the ECF and ICF and therefore does not play a major role in fluid shifts. However, a rapid bolus of dextrose may result in glycosuria and subsequent osmotic diuresis, which is counterproductive during resuscitation. Answer C: Lactated ringers is the preferred crystalloid for resuscitation of trauma/acutely ill patients. ATLS protocol recommends administration of two liters of crystalloid followed by administration of blood products (colloids) if the patient remains hemodynamically unstable. Answer D: Normal saline has been found to be associated with hyperchloremic metabolic acidosis, hyperkalemia (which usually is seen at 6 hours following normal saline resuscitation; this has not been observed with LR), acute kidney injury associated with poor acid-base status, greater urinary output, dilutional coagulopathy, vasodilatory effects resulting in/related to a higher cardiac output and lower peripheral resistance. Further, resuscitation in patients with uncontrolled hemorrhagic shock with normal saline requires significantly greater fluid volume compared to LR. Normal saline is appropriate in the setting of brain injuries, hyponatremia and metabolic alkalosis. The use of albumin in the initial setting of trauma is not one of the basic tenets of ATLS. Answer E: Normal saline has been found to be associated with hyperchloremic metabolic acidosis, hyperkalemia (which usually is seen at 6 hours following normal saline resuscitation; this has not been observed with LR), acute kidney injury associated with poor acid-base status, greater urinary output, dilutional coagulopathy, vasodilatory effects resulting in/related to a higher cardiac output and lower peripheral resistance. Further, resuscitation in patients with uncontrolled hemorrhagic shock with normal saline requires significantly greater fluid volume compared to LR. Normal saline is appropriate in the setting of brain injuries, hyponatremia and metabolic alkalosis. Packed RBC transfusion (rather than FFP) is indicated in patients with hypovolemia secondary to blood loss. However, transfusion is initiated following initial crystalloid resuscitation. Extensive research has been performed comparing the two crystalloids. The two fluids are even used interchangeably in certain institutions. However, the vast majority of trauma guidelines recommend the use of LR over NS for resuscitation, especially in patients with uncontrolled hemorrhagic shock, due to the reasons mentioned above. Packed red blood cells are then indicated if the patient remains hemodynamically unstable despite crystalloid administration. If the hypotension continues, initiation of a massive transfusion protocol may be indicated. Bottom Line: Lactated Ringer's or normal saline followed by packed red blood cells are the IV fluids of choice for resuscitation. TrueLearn Insight : Fluid and electrolytes are always on the exam. You must know the indications, effects, etc. of the various fluid types and thoroughly understand electrolyte disturbances. The questions are usually an easy way to gain points if you take the time to study these topics.

barret's esophagus

In Barrett's esophagus, metaplastic columnar epithelium replaces the stratified squamous epithelium that normally lines the distal esophagus. This predisposes the patient to development of esophageal cancer. Once identified, Barrett's esophagus mandates routine surveillance and lifelong treatment with antireflux therapy, typically a proton pump inhibitor (PPI). The American College of Gastroenterology 2015 guidelines on Barrett's esophagus without evidence of dysplasia recommends a once daily PPI regimen with endoscopic surveillance every 3-5 years with four quadrant biopsies every 2 cm. If a patient continues to have reflux symptoms or persistent reflux esophagitis, twice daily PPI therapy can be employed. If symptoms persist, they can be considered for antireflux surgery (i.e., Nissen, Dor, or Toupet fundoplication). The routine use of antireflux surgery for prevention of progression of Barrett's esophagus is controversial. Answer A: If a patient continues to have reflux symptoms or has persistent reflux esophagitis, twice daily PPI therapy can be employed. Answer C: Observation is inappropriate as there is a risk of advancement to esophageal cancer. Answer D: Endoscopic mucosal resection is recommended if dysplasia is found. Answer E: Esophagectomy is unnecessary for Barrett's esophagus without dysplasia. With advancement of endoscopic resection, esophagectomy can often be avoided for high-grade dysplasia or intramucosal carcinoma. Bottom Line: The American College of Gastroenterology 2015 guidelines on Barrett's esophagus without evidence of dysplasia recommends a once daily proton pump inhibitor regimen with endoscopic surveillance every 3-5 years with four quadrant biopsies every 2 cm.

Laryngoscopy

In a patient with a previous neck surgery, a laryngoscopy should be performed to ensure the bilateral vocal cords are functional. The optimal position is the "sniffing" position, which entails having the patient sit upright, leaning slightly forward, and performing atlanto-occipital extension.

Damage control laparotomy

In a trauma patient who has developed the lethal triad of hypothermia, coagulopathy and acidosis, a damage-control operation is indicated. A damage control operation is performed to minimize hemorrhage and operative time in the setting of physiologic derangement. Some of the major indications for damage control are listed below. Core temperature 35 °C (95 °F) SBP <80 mmHg pH <7.2 Base deficit >14 mmol/L INR or PTT > 50% normal Blood loss >4L Blood transfusion >10 units Fluid replacement >10L Persistent non-surgical bleeding The goals of damage control are rapid evaluation and management of life-threatening injuries with definitive repair to be performed at a later time once physiologic capture has taken place. Hemorrhage from major vessels and solid organs should be controlled. Hollow viscus injuries are rapidly sutured to prevent contamination or resected without anastomosis or ostomy formation. Complex injuries to the duodenum may be treated with debridement and primary repair or pyloric exclusion. Pancreatic injuries are treated with drainage alone or with resection and drainage. Liver and splenic injuries are treated based on the severity of the injury with a primary goal of hemostasis. A second look laparotomy is then planned if bowel viability is questioned or further repair needed. After the initial operation, the resuscitative phase is begun. In addition to using active rewarming to bring core temperature to 37 °C (98.6 °F), coagulopathy must be corrected. Packed red blood cells, fresh frozen plasma and platelets should be administered in a 1:1:1 ratio for maximum benefit. Goals of resuscitation should be as shown below. PT <15 or INR <1.2 using fresh frozen plasma, vitamin K, calcium, and limited use of crystalloids Fibrinogen >100 mg/dL using cryoprecipitate or fibrinogen concentrate Platelets >100,000/mm^3 using packed platelets Heating to temperature of 37 °C (98.6 °F)

Omphalocele vs gastroschisis

In contrast to patients with an omphalocele, the risk for associated anomalies with gastroschisis is low. One major exception to this general rule is the association of gastroschisis with intestinal atresia, which may be present in up to 15% of cases. Answer A: The immediate treatment of an omphalocele and gastroschisis consists of nasogastric tube decompression for prevention of visceral distention due to swallowed air. An IV line is secured for administration of fluids and broad-spectrum antibiotics. The bowel is covered with a sterile, moist dressing and the infant transported to a tertiary care pediatric surgery facility. Answer C: In contrast to patients with an omphalocele, the risk for associated anomalies with gastroschisis is low. One major exception to this general rule is the association of gastroschisis with intestinal atresia, which may be present in up to 15% of cases. Answers D & E: In contrast with omphalocele, the defect seen with gastroschisis is always on the right side of the umbilical ring with an intact umbilical cord, and there is never a sac covering the abdominal contents. Bottom Line: Gastroschisis is less frequently associated with other anomalies than omphalocele, but they are managed the same initially. Gastroschisis has a defect to the right of the umbilical ring while the defect in omphalocele is through the umbilical ring.

Simple breast cyst

In women under 30 years of age, breast masses are usually first evaluated with ultrasound. Simple cysts are well circumscribed, mobile, and nontender. Fine-needle aspiration, when performed, reveals varying color and disappearance of mass on repeat ultrasound. If blood is not aspirated, no further intervention is required except for repeat ultrasound in 1-2 months. If the cyst recurs during this followup, then mammogram and excision of the cyst should be considered.

AV fistulas

In general, autologous arteriovenous fistulas are preferred over prosthetic grafts for hemodialysis access. The Kidney Disease Outcome Quality Initiative recommends the following order of access preference for end-stage renal disease patients and patients expected to require hemodialysis: 1. Autologous radiocephalic (forearm) arteriovenous fistula 2. Autologous brachiocephalic arteriovenous fistula 3. Transposed brachial-basilic arteriovenous fistula 4. Upper arm brachial-cephalic prosthetic graft The forearm is preferred because it preserves proximal vasculature and is the easiest site to cannulate. Relative contraindications to autologous forearm fistulas include thrombosed cephalic vein and small radial artery. Relative contraindications to brachial-cephalic fistulas include deep cephalic vein and poor quality of the cephalic vein. If radiocephalic or brachiocephalic arteriovenous fistulas cannot be established, the 2006 guidelines recommend the use of transposed brachial-basilic fistula over prosthetic arteriovenous grafts. If the basilic vein is used it should be transposed due to the close proximity of the median antebrachial cutaneous nerve. Care must be taken during basilic transposition. The basilic vein is thin-walled and thus is more prone to injury and devascularization during mobilization. Polytetrafluoroethylene (PTFE) is the most common form of hemodialysis access in the U.S. However, thrombosis rates are 10 times higher for PTFE access compared to autologous fistulas and thus PTFE grafts are not preferred. Indications to prefer a prosthetic graft include small basilic vein, patient with history of several long-term central venous catheter infections and obese patient.

Aortic valve repair

In individuals with normal aortic valves, the valve area is 3.0 to 4.0 cm². As aortic stenosis (AS) develops, a minimal transvalvular pressure gradient is present until the orifice area falls by more than 50%. In general, symptoms in patients with AS and normal left ventricular systolic function rarely occur until aortic stenosis is severe (generally defined as a valve area <1.0 cm², usually accompanied by an aortic jet velocity >4.0 m/sec and/or mean transvalvular gradient >40 mm Hg). Many patients do not develop symptoms until obstruction is even more severe, while some become symptomatic with lesser obstruction, particularly if there is coexisting aortic regurgitation. AS should be treated with valve replacement on an elective basis once symptoms of angina or syncope occur. Patients who present with symptoms of congestive heart failure need an emergent valve replacement. Asymptomatic patients with a valve area less than or equal to 1.0 cm² are also candidates for surgical replacement. Balloon valvuloplasty has some effectiveness in children but has no role in the treatment of adults with aortic valve disease.

Inflammatory process

In leukocyte recruitment, circulating leukocytes express integrins in a low-affinity conformation. It is not until the leukocyte comes into contact with cytokines released from macrophages (that have come into contact with a pathogen and have started their immune response) that these low-affinity conformations change to high affinity. Answer A: Circulating leukocytes express integrins in a low-affinity conformation. Answer B: Exposure to activated endothelium leads to rolling, which is mediated by L-selectin and P-selectin on the neutrophil and E-selectin on endothelium. Tight leukocyte-endothelial adhesion involves integrin engagement with counterligand expressed on the endothelium. Subsequent exposure to chemokines leads to diapedesis, which is further mediated by the family of β1 and β2 integrins. Answers C & D: Integrins are receptors that mediate attachment between a cell and the tissues surrounding it, which may be other cells or the extracellular matrix (ECM) components such as fibronectin, vitronectin, collagen, and laminin. They also play a role in cell signaling and thereby regulate cellular shape, motility, and the cell cycle. Answer E: Not only do integrins perform this outside-in signalling, but they also operate an inside-out mode. Thus, they transduce information from the ECM to the cell as well as reveal the status of the cell to the outside, allowing rapid and flexible responses to changes in the environment. Bottom Line: Integrins are receptors that mediate attachment between a cell and the tissues surrounding it. They also play a role in cell signaling and thereby regulate cellular shape, motility, and the cell cycle.

Prophylaxis surgical with penicillin allergy

In patients with documented allergic reaction to penicillin, vancomycin or clindamycin should be used for skin operations. The combination of clindamycin or metronidazole with either an aminoglycoside or a fluoroquinolone or the combination of clindamycin with aztreonam should be effective for colon operations.

Coagulation factor deficiency

In patients with factor XI deficiency, there is a risk of bleeding during surgical procedures and in patients with trauma. Factor XI has a long half-life of 60-80 hours; therefore, fresh frozen plasma (FFP) can be used for replacement. FFP, or solvent detergent-treated plasma, is the mainstay of treatment for patients with factor XI deficiency who must undergo surgery. Factor XI concentrates are also available, but there have been reports of thromboembolic complications, so FFP should be used preferentially. Factor XI should be replaced to a level of 30-45 U/dL. Answer A: DDAVP is used in renal failure patients. Answer B: Factor XI has a long half-life of 60-80 hours; therefore, FFP can be used for replacement. Answer C: Cryoprecipitate can be used in von Willebrand disease. Answer E: Prothrombin complex concentrate can be use in patients on coumadin who need emergent intervention. Bottom Line: Factor XI deficiency places patients at risk for significant bleeding during surgical procedures. Fresh frozen plasma should be given to replace factor XI levels before any surgical procedure.

Hereditary breast cancer

In patients with hereditary breast cancer syndromes, annual screening with imaging should start at 25 years old. MRI is preferred, but mammogram is acceptable if MRI is not available. It is recommended that mammogram be added to the annual screening at age 30 years. Screenings start before 25 if a first-degree relative was diagnosed at age <30, but the National Comprehensive Cancer Network Guidelines do not explicitly state how much earlier the screening should start.

Perforated appendicitis

In patients with non-perforated appendicitis, either open or laparoscopic appendectomy should be performed. With perforated appendicitis, the decision must be made to perform immediate appendectomy or treat conservatively with antibiotics and possibly percutaneous drainage. Some evidence suggests that early appendectomy carries an increased risk of bowel obstruction, wound infection and reoperation. Other studies have demonstrated an earlier return to activity, fewer hospital visits and CT scans with early appendectomy. If treated non-operatively, many recommend interval appendectomy but this is ongoing source of debate. Antibiotic choice and course length are often institution dependent, but the historic gold standard is ampicillin, gentamicin and clindamycin or metronidazole, though, ceftriaxone and metronidazole have been shown to be equally effective. If interval appendectomy is performed, the patient can be brought back for surgery after 6 to 8 weeks.

Perforated appendicitis

In patients with nonperforated appendicitis, either open or laparoscopic appendectomy should be performed. With perforated appendicitis, the decision must be made to perform immediate appendectomy or treat with antibiotics and possibly percutaneous drainage. Some evidence suggests that early appendectomy carries an increased risk of bowel obstruction, wound infection, and reoperation. Other studies have demonstrated an earlier return to activity, fewer hospital visits, and CT scans with early surgery. If treated nonoperatively, many recommend interval appendectomy. Antibiotic choice and course length are often institution dependent, but the historic gold standard is ampicillin, gentamicin, and clindamycin or metronidazole, though ceftriaxone and metronidaole have been shown to be equally effective. After a course of antibiotics, the patient returns for appendectomy after 6-8 weeks.

CXR in penetrating trauma

In penetrating trauma, chest radiograph is indicated in wounds above the umbilicus or suspected thoracoabdominal injury. CT scans are only used if the patient is stable.

Traumatic bladder injury

In severely injured patients, pelvic injuries are relatively common. While these patients usually suffer from the significant mortality of associated head injuries, genitourinary complications are a source of lifelong morbidity. About 5% of those with a pelvic fracture will suffer from a bladder injury as well, with a higher preponderance in males. Pelvic fractures with pubic diastasis and obturator ring fractures are more likely to have associated bladder injuries. Identification of bladder injuries begins with appropriate history and physical exam. While no definitive algorithm is established to determine who should receive cystography, 95% of patients with a bladder injury will have gross hematuria, while the remainder will likely have at least microscopic hematuria. Therefore blunt injuries with a pelvic fracture and gross hematuria or penetrating injuries with microscopic hematuria deserve further work-up. Imaging for bladder injuries includes plain film cystography with scout and post-evacuation films in order to identify anterior and posterior leaks. However, due to its sensitivity and expedience, CT cystography is now the standard in most trauma centers. Sensitivity and specificity are 95% and 100%, respectively. This is performed by back-filling the bladder with 350 mL of contrast. Intraperitoneal bladder rupture will show contrast extravasation with contrast outlining bowel loops. Extraperitoneal rupture will demonstrate contrast within the space of Retzius, lateral colic regions, groin or thigh. Answer A: 95% of bladder injuries are associated with gross hematuria. Microscopic hematuria alone may be due to nephric injury. Answer B: About 5% of pelvic injuries will have a bladder injury. Answer C: Obturator ring fractures and pubic diastasis are the most common fracture patterns associated with bladder injuries. Answer E: CT cystography is the gold standard for bladder injuries. Bottom Line: In a patient that presents with a blunt mechanism, pelvic fracture and gross hematuria, a bladder injury should be suspected.

Nitrogen balance

In the absence of postoperative complications, muscle protein synthesis begins approximately 3-6 days following an abdominal operation. This generally coincides with the initiation of enteral nutrition upon return of bowel function. Patients who fall outside of this category, when they experience an ileus or early postoperative small bowel obstruction, are at risk of developing a negative nitrogen balance with subsequent loss of lean body mass. The nitrogen balance is determined by the difference between the amount of nitrogen provided and the amount excreted over a 24-hour period of time. Malnourished patients can achieve nitrogen equilibrium when ~15% of their total caloric needs are provided as protein. An easier ratio for quick calculation is to estimate 1 gram of nitrogen for every 150 kilocalories required. Thus, for a patient with a calculated need of 1500 kcal/day, daily nitrogen requirement would be ~10 grams.

Incidental adrenalectomy

Incidental adrenal lesions on CT are a common occurrence. The majority of these lesions are benign nonfunctioning adrenal cortical adenomas. Other possibilities include cortisol secreting adenomas, aldosteronomas, pheochromocytomas, adrenal cortical carcinomas, and metastatic disease. Management of adrenal incidentalomas depends primarily on functional status and imaging characteristics. Evaluation of the functional status of an adrenal lesion includes serum electrolytes (to evaluate for hypokalemia), a dexamethasone suppression test, and 24-hour urine collection for catecholamines, metanephrines, and vanillylmandelic acid. Imaging characteristics associated with a benign lesion include low CT attenuation (<10 Hounsfield units) and rapid washout (>50% at 10 minutes). Size of the lesion is also an important characteristic. Lesions greater than 4 cm in diameter should be surgically excised in an otherwise healthy patient. Additional imaging with MRI may help evaluate nonfunctioning tumors with borderline CT characteristics. Fine-needle aspiration should only be performed in those with a history of malignancy and suspected metastasis. It is important to rule out pheochromocytoma prior to biopsy to prevent hypertensive crisis. All functioning tumors and those with suspicious radiographic appearance should be excised regardless of size. Those that are nonfunctioning and have a benign appearance should be followed with repeat imaging at 6, 12, and 24 months. Annual hormonal testing should also be performed for 4 years. Growth greater than 1 cm or development of autonomous hormonal secretion is an indication for adrenalectomy. Answer A: Adrenalectomy is indicated for lesions greater than 4 cm or functional lesions. Answer B: Fine-needle aspiration is indicated only when metastasis is suspected. Answer D: Lesions that are nonfunctioning and have a benign appearance should be followed up with repeat imaging at 6, 12, and 24 months. Annual hormonal testing should also be performed for 4 years. Answer E: Additional imaging with MRI may help evaluate nonfunctioning tumors with borderline CT characteristics. This lesion has benign characteristics. Bottom Line: All functioning adrenal tumors and those with suspicious radiographic appearance should be excised regardless of size. Those that are nonfunctioning and have a benign appearance should be followed up with repeat imaging at 6, 12, and 24 months. Annual hormonal testing should also be performed for 4 years. Growth greater than 1 cm or development of autonomous hormonal secretion is an indication for adrenalectomy.

Bleeding duodenal ulcer

In those who have recurrent upper GI bleeding, it has been shown that a second endoscopic attempt at controlling bleeding will fail in 25% of patients, who will then require emergency surgery. Randomized prospective studies have, however, shown no increase in mortality rate in patients who undergo a second therapeutic endoscopic procedure versus surgery after the first failed endoscopy. Therefore, most clinicians would encourage a second attempt at endoscopic control. If the patient continues to fail endoscopic attempts, surgical intervention is needed. A longitudinal incision is made, spanning 3 cm on each side of the pylorus, to give adequate access to the duodenal bulb for suture ligation of the bleeding duodenal ulcer. If active bleeding is encountered, it is controlled by digital pressure, which in addition to controlling the bleeding, gives time for fluid resuscitation of the patient. The bleeding vessel is then ligated. This vessel is often the gastroduodenal artery (GDA), which at the level of the posterior duodenal wall has a three-vessel junction. It is important to suture-ligate the GDA superiorly and inferiorly, followed by ligation of the medial transverse pancreatic branches with a U-stitch.

Increasing oxygenation

Increasing the inspired fraction of oxygen (FiO2) is the quickest and most reliable way of increasing oxygen delivery to the alveoli. Filling the alveoli with 100% oxygen prior to insertion of the bronchoscope increases the duration of time before which hypoxemia develops. Answer A: Increasing PEEP improves oxygenation by keeping otherwise collapsed alveoli open during the end of expiration; however, the effects of PEEP take several hours to manifest themselves. In addition, increasing PEEP has the disadvantage of reducing venous return and cardiac output. Answers B & D: Increasing the tidal volume and the respiratory rate are both ways in which ventilation is increased. An increase in ventilation does not imply an increase in oxygenation as the two are separate processes. Answer E: Changing from volume to pressure control does not play a significant role during bronchoscopy. Bottom Line: Increasing the inspired concentration of oxygen is the fastest way of increasing oxygenation.

Diagnostic laparoscopy

Indications for a diagnostic laparoscopy include investigation of gynecologic conditions, including endometriosis, primary amenorrhea, and pelvic pain; accurate diagnosis or identification of malignancy stage of the stomach, esophagus, or pancreas; biopsy of an enlarged lymph node when intra-abdominal lymphadenopathy is seen on imaging; assessment of cause of chronic abdominal pain with intermittent partial small bowel obstructions; or evaluation of a patient with high suspicion for inguinal hernia, which is not apparent on physical examination. A diagnostic laparoscopy is recommended prior to an open pancreatic resection for large tumors (>3 cm) and tumors of the body and tail of the pancreas as these tumors tend to present later and are more likely to harbor occult metastatic disease. Staging laparoscopy has been advocated for pancreatic cancer patients prior to definitive resection as it can predict tumor resectability, identifies occult metastases that computed topography could not identify, and most impressively, with laparoscopic ultrasound, eliminates the need for laparotomy in up to 20% of patients. Despite this, there is no consensus for its use in pancreatic head tumors. There is, however, consensus for its use in other situations: large tumors (>3 cm), significantly elevated CA 19-9 (>100 U/ml), uncertain findings on CT, and pancreatic body or tail cancers (which tend to present later and have higher risk for occult metastatic disease). Answer A: The patient has bilateral inguinal hernias that are evident on physical examination. A diagnostic laparoscopy is indicated if the patient has symptoms suggestive of an inguinal hernia without evidence of physical examination. With diagnostic laparoscopy, a small incision can be used to diagnose bilateral inguinal hernias. Answer B: Constipation is not an indication for diagnostic laparoscopy. If the patient has intermittent partial small bowel obstructions without a clear source then a diagnostic laparoscopy is indicated. Answer C: Given the age of this patient, she should undergo a workup for endometrial carcinoma. This includes a pelvic exam, endometrial sampling, and uterine ultrasound. Treatment for endometrial cancer is hysterectomy with bilateral salpingo-oophorectomy. A diagnostic laparoscopy prior to surgical resection is typically not indicated. Answer E: Projectile emesis is highly suggestive of pyloric stenosis, which would benefit from a laparoscopy pyloromyotomy.

Indications for bariatric surgery

Indications for bariatric surgery include: BMI greater than or equal to 40 or BMI 35-40 with comorbidities such as hypertension, GERD, urinary incontinence and obstructive sleep apnea. There are several relative contraindications for these operations. Some bariatric surgeons maintain an upper limit of age. Drug and alcohol addictions are absolute contraindications. Patients who do not appreciate or plan to make the necessary lifestyle modifications after surgery should also not be offered these operations. Psychological stability and adequate home support are also very important for these weight loss procedures to be effective. Preoperative psychological evaluation is very important. Patients who smoke may be denied surgery especially if they exhibit an unwillingness to quit or cut back. Finally, insurance coverage is a major hurdle for many patients. Compliance with preoperative schedules and previous weight loss attempts should be documented.

C Diff colitis

Infection by Clostridium (also known as Clostridioides) difficile infection (CDI) is the most common cause of nosocomial diarrhea. Symptoms can range from watery diarrhea to fulminant colitis. Enteral vancomycin is the first line treatment for non-severe or severe CDI, in addition to discontinuing the inciting antibiotics. Enteral fidaxomicin is an alternative first line antibiotic, however it is more expensive than enteral vancomycin and is less frequently utilized. The initial duration of antibiotic treatment is 10 days. Oral vancomycin is not absorbed systemically, and reaches high concentration in the gastrointestinal tract. Enteral metronidazole was used as a first line therapy for CDI, however it has been shown to be less effective than enteral vancomycin for this purpose, and is no longer recommended as first line therapy according to the 2017 update of the Infectious Diseases Society of America (IDSA) guidelines. For CDI with fulminant colitis, enteral vancomycin plus IV metronidazole is the initial treatment of choice. Rectal vancomycin can be used in patients with ileus who are unable to tolerate enteral medications.

Inflammatory breast cancer

Inflammatory breast cancer is a clinical syndrome in women with invasive breast cancer. Diagnostic criteria include rapid onset of signs and symptoms: erythema and edema (peau d'orange) involving at least one-third of the skin of the breast; enlargement of the breast; and tenderness, induration, and warmth on palpation. Tumor emboli in the dermal lymphatics may or may not be seen on punch biopsy of the skin and are not necessary, or sufficient, to make the diagnosis of inflammatory breast cancer.Inflammatory breast cancer is a clinical syndrome associated with invasive breast cancer. Though the clinical presentation may result from tumor emboli in dermal lymphatics, demonstration of these emboli is not necessary to make the diagnosis.

Infliximab for fistula closure

Infliximab can be used to facilitate closure of fistulas in Crohn disease of the small bowel.

Inguinal hernia repair

Inguinal hernia operations have a long and complex history. There are over 30 described repairs, each with a new improvement over the last. Open tissue repairs historically had a high recurrence rate, up to 30%. These have been largely abandoned. Today, the most common repair is the Lichtenstein repair with mesh because it has the lowest recurrence rate, 1.2-4% depending on the study

Benefits of inhaled nitric oxide

Inhaled nitric oxide (NO) causes vasodilation of pulmonary capillaries. Its beneficial action is shunting of blood toward better ventilated alveoli and away from damaged alveoli that are less ventilated. NO is rapidly inactivated by binding to hemoglobin, and it requires continuous administration. Any interruption in delivery can cause both fatal hypoxemia and right-sided heart failure. NO costs thousands of dollars a day to administer. Answer A: Inhaled nitric oxide causes vasodilation of pulmonary capillaries. Answer B: Nitric oxide is rapidly inactivated by binding to hemoglobin and requires continuous administration. Answer C: Nitric oxide costs thousands of dollars a day to administer. Answer E: Nitric oxide's beneficial action is shunting of blood toward better ventilated alveoli and away from damaged alveoli that are less ventilated. Bottom Line: Nitric oxide improves oxygenation by pulmonary capillary vasodilation. It shunts blood toward healthy, well-ventilated alveoli.

Nitrous oxide

Inhaled nitrous oxide is a common anesthetic agent that has the advantage of a fast onset of action. Nitrous oxide has a high minimum alveolar concentration (MAC) and, therefore, low potency due to its low lipid solubility. Nitrous oxide can diffuse into closed spaces and is contraindicated for conditions such as pneumothorax or small bowel obstruction. The side effects of nitrous oxide are minimal compared to other inhaled anesthetics, and have little effect on hemodynamics. Answer A: Nitrous oxide has a high minimum alveolar concentration (MAC). Answer B: NO has little effect on hemodynamics. Answer D: Nitrous oxide has a high minimum alveolar concentration (MAC) and, therefore, low potency due to its low lipid solubility. Answer E: NO does not cause bronchodilation. Bottom Line: Nitrous oxide is fast acting with low potency.

Ovarian tumor markers

Inhibin is a nonsteroidal polypeptide hormone that is secreted by granulosa cells of the ovary. This hormone is produced during the menstrual cycle or pregnancy but not in a postmenopausal woman. Immunohistochemical staining using antibodies against markers of granulosa cell tumors (like inhibin) appears to be the most sensitive and specific for granulosa cell tumors. The preoperative evaluation of a woman with suspected ovarian cancer includes measurement of the serum glycoprotein CA 125 concentration. The serum CA 125 (normal 65 units/mL) in over 80% of women with advanced epithelial ovarian cancer. Such a finding in a postmenopausal woman with a pelvic or abdominal mass should prompt consultation with a gynecologic oncologist. CA 125 is not specific for ovarian cancer. It is also increased in patients with other malignancies, including endometrial cancer and certain pancreatic cancers; in a variety of benign conditions, such as endometriosis, uterine leiomyoma, and pelvic inflammatory disease; and in approximately 1% of healthy women. Answer A: Choriocarcinomas produce human chorionic gonadotropin. Answer B: The presence of alpha fetoprotein with an ovarian neoplasm strongly suggests a diagnosis of endodermal sinus tumor. Answer C: Elevated lactate dehydrogenase is associated with dysgerminoma. Answer E: CA 19-9 is a tumor marker seen in pancreatic malignancy. Bottom Line: Inhibin is used to detect granulosa cell tumors, which are sex cord-stromal tumors of the ovary. CA 125 can be used to detect epithelial ovarian cancer.

Parkland Formula

Initial evaluation and resuscitation of a patient who sustains a burn is of paramount importance in the management of these individuals. The most commonly utilized fluid resuscitation formula, the Parkland or Baxter formula, allows for quick calculation of fluid requirements based on percent of total body surface area (TBSA) burned and weight in kilograms. The total fluid to be infused in the first 24 hours for patients with >20% TBSA burns are calculated by the formula 4 cc x kg x % TBSA. Half that amount is infused over the first 8 hours and the other half over the subsequent 16 hours. In adults, urinary output is an appropriate measure of adequate resuscitation. An output of 0.5-1 cc/kg/hr in adults is appropriate, and in pediatric patients urinary output of 1-1.5 cc/kg/hr is necessary. The rationale behind continuous administration of fluids is based on the principle that a burn injury or thermal injury directly causes an inflammatory response. This response results in leakage from surrounding capillaries, which leads to plasma leaking into the extravascular space. Infusing crystalloid replenishes the intravascular volume and maintains appropriate pressures in this patient population.

Continuous NG tube to suction electrolyte abnormalities

Initial nonoperative management of small bowel obstruction consists of bowel rest, IV hydration, and nasogastric decompression. Continuous nasogastric suction can cause external loss of potassium-rich fluid and hydrochloric acid that results in net gain of bicarbonate (hypokalemic, hypochloremic, metabolic alkalosis). Renal excretion of bicarbonate increases in response to the metabolic alkalosis accompanied by sodium excretion. In the dehydrated patient sodium is conserved in exchange for hydrogen ions that create an acidic urine. Paradoxical aciduria will occur in the presence of an existing metabolic alkalosis. Hypokalemia also results in increased renal bicarbonate absorption and acid secretion. Cellular potassium loss is accompanied by exchange of sodium or hydrogen to preserve membrane potential across the cell, elevating bicarbonate levels in the serum.

Proper pressure and flow for pneumoperitoneum?

Initial pressure of less than 8 mm Hg with flow of 1 L/min on placement of Veress needle demonstrates peritoneal entry. Pressure > 8 mm Hg with low flow is associated with interstitial placement.

Left thoracotomy

Injuries to the left ventricle, left subclavian artery, descending aorta, left pulmonary artery, left lung, left hilum, left internal mammary artery, and distal esophagus are best approached through a left thoracotomy

Ureteral injuries

Injuries to the ureter can be approached by dividing the ureter into thirds. Lower ureter injuries are commonly managed with reimplantation of the proximal ureter into the bladder. If the ureter has been ligated without transection, repair can be attempted with primary ureteroureterostomy over a stent. Answer A: Foley catheter placement and observation is not indicated due to morbidity from urinoma development. Answer B: Proximal ureteral injuries are typically managed with primary ureteroureterostomy. If there is significant ureteral loss, a segment of ileum may be used for ureteral replacement. Answer D: Autotransplantation is rarely performed for ureteral injury and is not advisable in trauma situations secondary to increased operative times. Answer E: Injuries to the midureter can be managed by primary ureteroureterostomy or transureteroureterostomy. Distal mobilization of the ureter provides additional length but it is important to avoid devascularizing the ureter. Bottom Line: Traumatic distal ureteral transaction is best managed with reimplantation of the proximal ureter into the bladder.

Median sternotomy

Injury to the right atrium left atrium, right ventricle, SVC, IVC, aortic root, aortic arch, innominate artery, proximal right carotid artery, main pulmonary artery, and pericardium are best approached through median sternotomy

Intersphincteric abscesses of the anal canal

Intersphincteric abscesses are often difficult to diagnose by external exam alone and require an exam under anesthesia. When the diagnosis is made, the abscess is typically drained internally by dividing the mucosa and internal sphincter along the length of the abscess cavity, allowing free drainage. Exam under anesthesia, followed by internal incision and drainage, is the treatment of choice for intersphincteric abscesses. Antibiotics are only necessary in special patient populations, such as the immunocompromised or those with prosthetic heart valves.

What cells are GISTs associated with?

Interstitial cells of Cajal

Bile acids

Intestinal bacteria further conjugate the primary bile acids into deoxycholic and lithocholic acids. Answer A: The bile acids are actively secreted into bile (choice E), and enter into the small intestine. Answer C: The bile acids are then actively reabsorbed in the terminal ileum and repeat the process via the enterohepatic circulation. Glucuronyl transferase is involved in the conjugation of bilirubin. Answer D: Their function is primarily for the digestion of fats. Within the hepatocyte, bile acids are conjugated with taurine and glycine (choice D). Answer E: Primary bile acids are formed in the liver from cholesterol (choice A). Bottom Line: Deoxycholic and lithocholic acids are secondary bile acids.

AV graft acute stop in working

Intimal hyperplasia is the most likely cause but is more likely to occur at the graft-venous anastomosis. Late access thrombosis occurs in these patients after about 1-2 years due to this intimal hyperplasia

Chylous ascites

Intra-abdominal malignancies can impinge on lymphatic drainage if the base of the mesentery or cisterna chyli is involved.

Pulmonary sequestrations

Intralobar sequestrations are found within the medial or posterior segments of the lower lobes, with about two thirds occurring on the left side. Extralobar sequestrations are found three times more frequently on the left side. Answer A: Because of the risk for infection and bleeding, intralobar sequestrations are usually removed, either by segmentectomy or lobectomy. CT and MRI have replaced the need for angiography and provide excellent mapping of the blood supply. Answer C: In contrast with those that are intralobar, extralobar sequestrations occur with multiple other anomalies in roughly 40% of cases. These include posterolateral diaphragmatic hernia, eventration of the diaphragm, pectus excavatum and carinatum, enteric duplication cysts, and congenital heart disease. Answer D: There is frequently an aberrant systemic blood supply in both intralobar and extralobar sequestrations. In about 85% of cases, the intralobar sequestration is supplied by an anomalous systemic vessel arising from the infradiaphragmatic aorta and located within the inferior pulmonary ligament. The venous drainage is usually through the inferior pulmonary vein but may also occur by way of systemic veins. Extralobar sequestration is almost exclusively discovered in infancy, due to early onset of symptoms. The sequestered lung has its own visceral pleura, and may even occur outside the thorax. The ELS blood supply is usually systemic, from branches of the aorta (80% of cases); however, the venous drainage is mainly via the azygos-hemiazygos system (80% of cases). Answer E: Sequestrations represent malformations of the lung in which there is usually no bronchial communication, but sometimes they have fistulous connections to the bronchial tree. Bottom Line: Pulmonary sequestration usually results from no communication with the bronchial tree and an anomalous arterial supply from the systemic circulation. Extralobar sequestration is more commonly associated with other anomalies.

Ischemia-reperfusion injury s/p kidney transplant

Ischemia-reperfusion injury results in acute tubular necrosis (ATN) or delayed graft function requiring dialysis in less than 5% of living donor transplants, but in cadaver transplants, the incidence is higher than 20%. In the absence of vascular or ureteral problems, initially nonfunctional cadaver kidneys may be assumed to suffer from ATN, especially if nuclear scans demonstrate good blood flow and poor tubular function. Answer B: Hyperacute rejection results from preformed antibodies against the donor organ. Within minutes of revascularization, the kidney turns blue and soon undergoes vascular thrombosis. Answer C: Extensive mobilization of the iliac vessels during the transplant operation or failure to ligate the lymphatics crossing them can result in lymphoceles. Symptoms may not occur until weeks later and consist of swelling of the wound; edema of the scrotum or labia, and lower extremity; or urinary obstruction from pressure on the collecting system or ureter. Ultrasound easily identifies a perinephric fluid collection. Answer D: Arterial or venous obstruction in the early postoperative period, though uncommon, is a surgical emergency and needs to be excluded if an established diuresis suddenly ceases. Radioisotopic scanning and ultrasound will confirm vascular occlusion. Answer E: Sirolimus (rapamycin/Rapamune) is a macrocyclic antibiotic produced by Streptomyces hygroscopicus that also binds to FKBP but does not exert its immunosuppressive activity via the calcineurin pathway. Sirolimus has much less nephrotoxicity but is associated with hyperlipidemia, thrombocytopenia, pneumonitis, rash, and profound delays in wound healing by interfering with fibroblast activity. The patient is postoperative day 1.5 and unlikely to have toxicity from a single dose. Bottom Line: In the absence of vascular or ureteral problems, initially nonfunctional cadaver kidneys may be assumed to suffer from acute tubular necrosis. Oliguria in the early transplant period is treated with aliquots of fluid and colloid to exclude hypovolemia

Ischemic orchitis

Ischemic orchitis can occur from venous congestion due to pampiniform plexus thrombosis or ligation of the testicular artery during hernia repair. The ipsilateral testicle becomes painful, tender, and swollen. Testicular atrophy may result. Ischemic orchitis usually manifests on post-operative day (POD) #2-5 and may last for up to 12 weeks. Minimizing the risk of ischemic orchitis involves avoidance of excessive dissection of the spermatic cord. The risk of ischemic orchitis increases with repair of recurrent hernias. Treatment initially involves anti-inflammatory medications and analgesics. Orchiectomy is seldom indicated.

Cause of isotonic hyponatremia

Isotonic hyponatremia can occur due to lab testing and is most commonly due to high blood triglyceride levels

Gallbladder cancer

It is clear that the only curative option in patients with gallbladder cancer is complete surgical resection. It is essential for optimal patient care that patients with gallbladder cancer be recognized before laparoscopic cholecystectomy is performed, because of the risk of port site seeding and bile spillage, with its potential for subsequent carcinomatosis. Tumor location may be important in determining the extent of resection. If the tumor arises in the gallbladder infundibulum, the common bile duct is often involved with tumor, either by direct extension or external invasion of the hepatoduodenal ligament. In this case, an extended liver resection and removal of a portion of the common bile duct (CBD) should be performed. Reconstruction is then performed by Roux-en-Y hepaticojejunostomy. Tumors arising in the fundus of the gallbladder, however, can be treated with limited hepatic resection without excision of the CBD. To clear the lymph nodes in the porta hepatis, complete lymphadenectomy should be performed, skeletonizing the CBD, hepatic artery, and portal vein.

Hernia

It is estimated that 10-15% of all incisions will develop into incisional hernias. Wound infection after surgery doubles the risk of a hernia development. Additional factors include poor technique, diabetes, immunosuppression, and advanced age. Midline abdominal incisions account for 90% of all incisional hernias and are the most likely to develop into hernias. In contrast to historic practice, not all hernias require repair. Indications include symptomatic hernias, expansion, loss of domain thinning of soft tissue, nonhealing wound development, and cosmesis. Additional considerations are the likelihood of developing hernia complications, such as a small defect with a large sac (as opposed to a wide mouth hernia that reduces easily).

Cancer metastasis

It is well recognized that colorectal cancer does not frequently metastasize to bone. Other kinds of cancer, such as breast, prostate, lung, thyroid, and kidney, are more likely to spread to bone than others. However, in certain circumstances, isolated bone metastases to the sacrum or vertebral bodies can arise from the rectum when tumor emboli travel through portal-vertebral venous communications known as Batson plexus. Answers A & B: Breast and prostate cancers are the 2 most common sources of cancer metastasis to the bone. Answers C & E: Lung and kidney cancers metastasize frequently to the bone. Bottom Line: Colon cancer does not generally metastasize to the bone.

Keloids

Keloids are characterized by wound healing rising above the original level of the skin and also extending beyond the original border of the wound. Steroids are first line therapy for keloids. Intralesional corticosteroid injection works by decreasing fibroblast proliferation, collagen glycosaminoglycan synthesis, and inflammation. These intralesional injections are more effective on early scars but will not cause the lesions to fully disappear. Complications may include atrophy of the skin as well as hypopigmentation, necrosis, and ulceration at injection sites.

Fuel sources

Know the fuel sources for the small bowel enterocyte (glutamine) and colon (short chain fatty acids).

lactational mastitis

Lactational infections are thought to arise from entry of bacteria through the nipple into the duct system. They are characterized by fever, leukocytosis, erythema, and tenderness. Infections are most often due to Staphylococcus aureus and may manifest as cellulitis, termed mastitis, or as abscesses. The highest incidence of lactational breast infection occurs in the first 12 weeks of the postpartum period. Breastfeeding from the infected side is thought to be safe for a healthy infant, although infants should be monitored for the development of infection.

LAGB

Laparoscopic adjustable gastric bands (LAGB) have a slower and overall lower weight loss results when compared to the sleeve gastrectomy (SG) and Roux-en-Y gastric bypass (RYGB). Weight loss after RYGB and SG tends to be rapid for the first few months postoperatively then reaches a peak around 1.5 years postoperatively, while LAGB reaches peak weight loss around 2 years postoperatively. Average excess body weight (EBW) loss with the LAGB is approximately 50% while the SG averages approximately 60% EBW loss and the RYGB may be upwards of 75% EBW loss.

Laparoscopic tacking for hernia repair

Laparoscopic inguinal hernia repair involves placement of a large mesh that covers a direct, an indirect, or a femoral hernia defect. Fixation of the mesh can be done with a laparoscopic tacking device to secure it in place. Tacks should not be placed inferior to the iliopubic tract and lateral to the epigastric vessels due to the high risk for injury to the femoral branch of the genitofemoral nerve, lateral femoral cutaneous nerve, spermatic vessels, and vas deferens. A safeguard to verify safe tack placement is to palpate the tip of the laparoscopic tacker during placement. Answer A: Tacks can be placed in Cooper's ligament without risk of neurovascular injury. Answer B: Tacks can be placed in the pubic tubercle without risk of neurovascular injury, but it is associated with increased incidence of chronic pain. Answer C: Tacks can be placed in the posterior rectus musculature, but this must be done carefully, avoiding inferior epigastric vessels. Answer D: Tacks can be placed superior to the iliopubic tract and lateral to the epigastric vessel without risk of neurovascular injury. Bottom Line: Laparoscopic tacks should not be placed inferior to the iliopubic tract and lateral to the epigastric vessels during laparoscopic inguinal hernia repair due to the risk of neurovascular injury.

Parastomal hernia

Laparoscopic parastomal hernia repair with mesh is now considered the treatment of choice by most surgeons. Stoma relocation is another option that is for patients who have other stoma problems, such as skin excoriation or suboptimal stoma construction. However, stoma relocation is a significantly more invasive procedure and subjects patients to a triple threat of hernia recurrence: (1) at the old stoma site, (2) at the new stoma site, and (3) at the laparotomy incision used to move the stoma. In this scenario, laparoscopic repair is avoided because stoma relocation would address the narrowing stoma, skin excoriation along with the hernia repair. Given that stoma relocation can lead to contaminated field, BIOLOGIC mesh is preferred

Splenectomy

Laparoscopic splenectomy is an accepted operative approach to elective splenectomies. Absolute contraindications are becoming fewer in number. Experts still agree that cirrhosis and portal hypertension are absolute contraindications. Laparoscopic splenectomy has safely been performed in pregnants patients well into third trimester although the ideal time is early 2nd trimester. Splenomegaly is a relative contraindication. Even massive spleens have been removed utilizing morselization or hand assist techniques.

Extracorporeal suturing

Laparoscopic suturing is an advanced skill with a steep learning curve. There are two well described techniques. Extracorporeal suturing is easier to learn and involves creating a knot outside the body then pushing it onto the tissues. Some physicians believe the traction applied to the maneuver creates a sawing motion on the tissue. This can be dangerous on friable bowel that has been obstructed and dilated. The safer technique may be intracorporeal suturing. This does require a large amount of skill but will cause less damage to friable tissue. Converting to an open surgery is never the wrong answer and is the safest techniqueIleostomy is warranted in the unstable patient on pressors in whom an anastomosis cannot be created. This is a stable patient who requires simple resection.

Graft infections

Late prosthetic graft infections are most commonly due to Staphylococcus epidermidis bacteria which forms biofilm over the implanted foreign body. The ability to form biofilms on plastic devices is a major virulence factor for S. epidermidis. It allows other bacteria to bind to the already existing biofilm, creating a multilayer biofilm. Such biofilms decrease the metabolic activity of bacteria within them. This decreased metabolism, in combination with impaired diffusion of antibiotics, makes it difficult for antibiotics to effectively clear this type of infection. S. epidermidis strains are often resistant to antibiotics, including penicillin, amoxicillin, and methicillin. These infections usually require initiation of intravenous (iv) antibiotics and removal of the infected graft. Early graft infections are caused by staph aureus.

Lateral suture patch angioplasty

Lateral suture patch angioplasty is the appropriate choice. Repair of the artery is indicated after evaluation of the lumen of the artery. Patch angioplasty is utilized to decrease the chance of stenosis with primary repair. Answer A: Angioplasty and stent are not recommended at this time as further wire manipulation may worsen the damage, and the injured artery is exposed. Answer B: The patient has most likely suffered either a flow limiting dissection or formation of thrombus due to trauma. Exploration has yielded what appears to be a small injury to the vessel. Appropriate management would be exploration of the artery and repair. Embolectomy alone will not repair the damage to the vessel. Answer C: Placing foreign material in a traumatic wound is not indicated. Answer D: It is recommended to utilize contralateral saphenous vein for trauma. Bottom Line: Surgical repair is warranted in flow limiting injuries to the artery.

Left anterolateral thoracic incision

Left anterolateral thoracic incision is primarily used during emergency department resuscitative thoracotomy (EDRT). EDRT is generally reserved for patients who deteriorate rapidly after arrival to ED and those who have undergone cardiac arrest just before arrival

Varicose Veins

Ligation of the GSV at the saphenofemoral junction can control gravitational reflux while preserving the vein for subsequent arterial bypass. The GSV is largely preserved after proximal ligation, but reflux usually continues and hydrostatic forces are not controlled. Recurrent varicose veins are more frequent after saphenous ligation than after stripping. Varicosities happen more frequently after ligation and sclerotherapy than after stripping and sclerotherapy. Routine GSV stripping reduces the rate of recurrent varicosities and the need for re-operation for recurrent saphenofemoral incompetence

Lithotomy position

Lithotomy position is commonly used in gynecology and general surgery to have access to the abdomen, pelvis, and perineum. Attention to detail prevents the development of injury related to peroneal nerve, vascular, or muscular compression. When the lower extremities are properly positioned using stirrups, all the weight will rest on the patient's heel. The stirrup underneath the leg will be angled below the calf to leave a space and avoid contact or compression. Iatrogenic injury to the common peroneal nerve can occur after incorrect leg placement in the lithotomy position. Other commonly tested reasons for common peroneal nerve injury include fibula head fractures and crossing legs for long periods.

LCIS

Lobular carcinoma in situ is not a premalignant lesion and can therefore be treated in various ways. It does require excisional biopsy, but it does not require margins. One option is conservative management and close observation since the patient is at higher risk for breast cancer. Another option is to start chemoprophylaxis with tamoxifen. Tamoxifen has been shown to reduce the risk of developing cancer. A third option is to have a prophylactic bilateral mastectomy. Since the risk for cancer is increased in both breasts, a unilateral mastectomy would not be recommended. There is no need for lymph node investigation without evidence of invasive disease. Cancer risk is increased in both breasts; therefore, prophylactic bilateral mastectomy is a potential option. Prophylactic unilateral mastectomy, however, is not an option.

long chains lipolysis

Long-chain triglycerides, which constitute the majority of dietary fats, must undergo lipolysis of constituent long-chain fatty acids and monoglycerides before being absorbed by the intestinal epithelium. These reactions are catalyzed by gastric and pancreatic lipases. The products of lipolysis are transported in the form of mixed micelles to enterocytes, where they are resynthesized into triglycerides, which are then packaged in the form of chylomicrons that are secreted into the intestinal lymph (chyle). Triglycerides composed of short- and medium-chain fatty acids are absorbed by the intestinal epithelium directly, without undergoing lipolysis, and are secreted into the portal venous circulation.

Puestow procedure

Longitudinal pancreaticojejunostomy (Puestow procedure) is indicated for patients with severe persistent pain from chronic pancreatitis which is refractory to medical therapy, repeated hospital admissions, and a dilated pancreatic duct > 8 mm in diameter. Contraindications include absence of pain, pancreatic cancer, cirrhosis, and cardiopulmonary comorbidities. The Puestow procedure involves creating a Roux-en-Y pancreaticojejunostomy. The pancreas is filleted to expose both the Ducts of Santorini and Wirsung, from neck to tail, and any stones encountered are removed. The Roux limb is then anastomosed to the pancreas. entails opening the pancreatic duct anteriorly along its length, medially to the level of the gastroduodenal artery, and laterally into the tail beyond all appreciable pancreatic duct strictures. The opened pancreatic duct is cleared entirely of stones and then sewn to a Roux-en-Y jejunal limb for drainage

Rectal cancer

Low rectal cancers can be palpated on digital rectal exam. These can be better evaluated on proctoscopy to visualize the extent of involvement. A full workup includes a carcinoembryonic antigen level test, liver function tests, full colonoscopy to rule out a synchronous lesion, and a CT scan for metastatic disease. PET scans are expensive studies and are not used routinely in the workup of a rectal mass. Either MRI or endoscopic ultrasound can be used to stage the patient's depth of tumor and nodal involvement. Answer A: Carcinoembryonic antigen level is a useful test in rectal cancer and should be ordered routinely. Answer C: Nigro protocol is used for squamous cell cancers of the anal canal. Answer D: A full colonoscopy is warranted in patients presenting with a new colorectal mass. Answer E: PET scans are not part of the routine workup for rectal mass. Bottom Line: Endoscopic ultrasound and MRI can both be used for rectal cancer staging.

Lymphedema

Lymphedema complicates about 20% of axillary clearance procedures. Radiation, obesity, and the extent of axillary dissection are all factors that increase the risk of breast cancer-related lymphedema. Answer A: Secondary lymphedema can occur with cancer treatment (e.g., axillary dissection for breast cancer or groin dissection form melanoma), cancer presentation (penile, endometrial, cervical, vulvar, or melanoma metastasis to groin lymph nodes), or trauma. Primary lymphedema is a congenital condition that could present up to age 30. Secondary lymphedema is more common than primary lymphedema. Answer C: The risk of lymphedema following axillary dissection is lifelong. Most cases occur in the first 3 years following the surgery, and then the rate becomes constant for the remaining survival period. Answer D: Lymphangiosarcoma (Stewart-Treves syndrome) can arise from areas affected by lymphedema. Answer E: The disease is stage I, where lymphedema is reduced by elevation and pitting may occur. In stage II, the edema is rarely reduced with elevation alone, and pitting is present. Stage III disease is associated with absence of pitting, skin atrophy, hardening of the underlying tissue, and warty overgrowth. Bottom Line: Lymphedema occurs in approximately 20% of axillary clearance procedures. Lymphedema increases the risk of skin infection, poor wound healing, loss of limb function, and lymphangiosarcoma of the affected limb.

Lymphedema

Lymphedema is the clinical term applied to the various disease states characterized by the interstitial accumulation of protein-enriched fluid. Isotopic lymphoscintigraphy is a reliable and reproducible method to confirm the diagnosis of lymphedema. A radiolabeled macromolecular tracer is injected intradermally or subdermally within one of the interdigital spaces of the affected limb and then observed. Answers A & C: CT and MRI, like ultrasound, can see the effects of swelling and edema. However, their greatest utility is to determine if there is actually an obstructing mass that has caused significant lymph outflow obstruction.

Lymph node dissection complications

Lymphedema is the most common long-term complication following superficial or deep lymph node dissection. Age and obesity are the most important risk factors for this complication. Intraoperative factors are less consistent risk factors such as thin skin flaps, transverse incision, and sacrifice of the saphenous vein or muscle fascia. Limb paralysis is extremely rare.

Liver hemangiomas

MASS WITH PERIPHERAL ENHANCEMENT The natural history of liver hemangioma is generally benign, and it appears that most of these tumors remain stable over long periods of time, with a low risk for rupture or hemorrhage. There has never been a report of malignant degeneration of a liver hemangioma. Answer E: Growth and development of symptoms do occur, however, occasionally requiring resection. Symptomatic patients undergo a thorough evaluation, looking for alternative explanations for the symptoms, but they are candidates for resection if no other cause is found. Rupture, change in size, and development of Kasabach-Merritt syndrome are indications for resection. In rare cases of diagnostic uncertainty, resection may be necessary to make a definitive diagnosis. Resection of liver hemangiomas should be performed with minimal morbidity and mortality. The preferred approach to resection is enucleation with inflow control, but anatomic resections may be necessary in some cases. Surgery on large central hemangiomas can be associated with significant morbidity. Bottom Line: Hemangiomas are benign tumors of the liver that should be managed conservatively unless they are symptomatic. Indications for surgery are rupture, increasing size, and development of Kasabach-Merritt syndrome.

MRSA gene that encodes altered PBP?

MRSA is becoming more and more prevalent in the community and hospital setting. It can affect almost any body fluid or cavity. Methicillin resistance results from the production of an altered penicillin-binding protein known as PBP2a, which has decreased affinity for most β-lactam antibiotics. PBP2a is encoded by the gene mecA.

Protective measures during trochar placement

Macrobracing is the most consistent technique to decrease the trocar-associated injuries. As the trocar enters the abdominal wall, it separates the abdominal wall layers and eventually enters the peritoneal cavity with resultant loss of resistance. Without adequate bracing by the non-dominant hand, the trocar can damage the intestines or other retroperitoneal structures. The non-dominant hand bracing prevents the distribution of the force to unwanted structures.

Which burn therapy causes metabolic acidosis?

Mafenide acetate is absorbed systemically. It has been shown to cause metabolic acidosis through the inhibition of carbonic anhydrase.

Malignant lesions of the salivary glands

Malignant lesions of the salivary glands are staged according to size and extent of local tissue invasion. The most common malignant tumor of the parotid gland is mucoepidermoid carcinoma, which histologically consists of epidermoid and mucous cells. These tumors are characterized as low-, intermediate-, or high-grade, directly related to the proportion of epidermoid to mucoid cells found on histologic examination. High-grade mucoepidermoid carcinomas are highly aggressive, with a local recurrence rate of 60%, regional metastatic rate of 50%, and distant metastatic rate of 30%. These tumors are treated with total parotidectomy, radical neck dissection, and postoperative radiotherapy, regardless of nodal status, with a 5-year survival of 50%. Few patients currently present with far-advanced parotid gland carcinoma. With such presentations, however, the surgeon and the patient must be prepared for an extended, radical parotidectomy, which may involve resection of overlying skin, adjacent mandible and soft tissue, temporal bone, and a portion of the adjacent external ear. With advanced presentations, the facial nerve is invariably sacrificed in these patients; free tissue transfer may be necessary for repair. The most common malignancy of the submandibular gland, and the second most common of the parotid gland is adenoid cystic carcinoma. This tumor has a propensity for perineural invasion and spread, as well as distant metastases. Surgical management of these tumors includes radical resection, sacrificing nerves only for direct tumor extension and postoperative radiation therapy. Despite this aggressive therapy, these tumors, which follow an indolent course, develop regional and distant metastases 40% of the time over a 10- to 20-year course. Benign or low-grade tumors can be treated with resection alone. The most common benign lesion of the major salivary glands is pleomorphic adenoma or benign mixed tumor. Grossly, these lesions appear smooth and lobular, with a well-defined capsule. Histologically, however, they have epithelial and mesenchymal components but incomplete encapsulation with pseudopod extension beyond the apparent borders of the mass. These features account for the high recurrence rate when tumors are removed by enucleation alone. Appropriate surgical therapy involves resection of the tumor with a margin of normal gland surrounding it. The intimate relationship between the parotid gland and the facial nerve necessitates facial nerve identification and dissection to ensure its preservation and complete tumor extirpation. Warthin's tumors are typically benign, cystic lesions in the tail of the parotid gland. They are often multicentric, and approximately 10% are bilateral. Treatment includes superficial parotidectomy, similar to pleomorphic adenoma.

Enlarged lymph node with unknown source

Management of the patient with a squamous cell carcinoma metastasis to the neck with an unknown primary depends on the N status. This patient is an N1 for which surgery is preferred according to the NCCN guidelines. A primary source can ultimately be found in approximately 40% of patients. Eighty percent of these will be from the oropharynx with the ipsilateral tonsil being the most common site followed by the base of the tongue. For tumors N < 2 straight to surgery is preferred but for tumor with N > 2 chemo or radiation is the first step.

Subclavian line insertion landmarks

Many landmarks have been described to facilitate the appropriate insertion of the needle into the subclavian vein including the deltopectoral groove, the sternal notch, and the medial third of the clavicle. With the introducer needle angled toward the sternal notch, horizontal to the chest wall, and about 1 cm inferior to the "bend" of the clavicle or medial third of the clavicle the subclavian vein may be aspirated. Once attained, the Seldinger technique is employed with the assistance of fluoroscopy. In a study analyzing common errors in technique in resident physicians, inadequate landmark identifications constituted 14.7% of cannulation failures.

Pancreatic cancer

Many of the difficulties associated with the management of pancreatic cancer result from our inability to make the diagnosis at an early stage. The early symptoms of pancreatic cancer include anorexia, weight loss, abdominal discomfort, and nausea. Specific symptoms usually develop only after invasion or obstruction of nearby structures has occurred. Most pancreatic cancers arise in the head of the pancreas, and obstruction of the intrapancreatic portion of the common bile duct leads to progressive jaundice, acholic stools, darkening of the urine, and pruritus. The onset of diabetes may be the first clinical feature in 10% to 15% of patients. It is important to consider pancreatic cancer in patients presenting with acute pancreatitis, especially those without an obvious cause for their pancreatitis (alcohol or gallstones). Answer A: Pain is a common symptom of pancreatic cancer. The pain usually starts as vague upper abdominal or back pain that is often ignored by the patient or attributed to some other cause. It is usually worse in the supine position and is often relieved by leaning forward. Answer C: The most common physical finding at the initial presentation is jaundice. Hepatomegaly and a palpable gallbladder may be present in some patients. Answer D: Blumer shelf sign is used to describe pelvic drop metastases from pancreatic cancer. Periumbilical adenopathy is known as Sister Mary Joseph nodules. Answer E: Ascites can be present in 15% of patients and usually indicate advanced disease. Bottom Line: The clinical presentation of pancreatic cancer is non-specific. New onset diabetes or an episode of acute pancreatitis may be the first feature of pancreatic cancer.

Pilonidal disease

Many surgical techniques are available for the treatment of pilonidal disease. Midline excision can be followed by leaving the wound open, marsupialization of the dermis, primary closure or wound vacuum. Midline primary closure is associated with higher failure rate and higher recurrence rate compared to leaving the wound open or marsupialization of the dermis. Answer A: Postoperative antibiotics are not routinely indicated following pilonidal disease surgery. Answer B: Asymptomatic or a minimally draining sinus can be reasonably treated with local care and shaving the hair around the area at least weekly; however, recurrent abscess, painful disease or a chronically draining sinus are indications of surgery. Answer C: Many surgical techniques are available for pilonidal disease. Flaps are complex procedures that should be reserved for recurrent (stage V) or extensive disease that is extending beyond the navicular area (stage IV). Answer D: Following midline excision, most primarily closed wounds will dehisce secondary to lateral pressure created by sitting. Bottom Line: Midline excision of pilonidal disease with primary closure is associated with shorter healing time, but a higher complication rate than midline excision with marsupialization of the dermis or leaving the wound open.

Marfan

Marfan syndrome is characterized by tall stature, arachnodactyly, ligamentous laxity, myopia, scoliosis, pectus excavatum and ascending aortic aneurysms. The underlying defect is in the protein fibrillin.

Marginal ulcers

Marginal ulcers may be present in as many as 20% of roux-en-y gastric bypass patients. These ulcerations usually form in the first few months after the bypass and are located at the gastrojejunal anastomosis. The formation of the ulcers is not fully understood; however, factors involved include ischemia, suture erosion, smoking, gastric acid, aspirin or NSAID use, H. pylori infection and gastric pouch size. Patients will often complain of food intolerance with nonspecific abdominal, nausea, vomiting. They may also complain of weight regain. Initial evaluation consists of upper endoscopy for diagnosis and to evaluate for possible foreign body and gastro-gastric fistula. The majority of uncomplicated ulcers will resolve with smoking cessation and proton pump inhibitors. Refractory cases may require anastomotic revision. Answer A: Gastro-gastric fistula may be common in refractory cases of marginal ulcer, but the majority of cases are due to other factors such as ischemia, NSAID use, and H pylori. Answers B & E: Most cases of marginal ulcer will resolve with conservative management with smoking cessation and proton pump inhibitors. Answer D: Most cases of marginal ulcer present within the first few months after surgery. For more information, see: Cameron, John. Current Surgical Therapy. 11th Edition, 2014. Bottom Line: Marginal ulcers are common after roux-en-y gastric bypass and usually present within the first few months after surgery. Conservative management with proton pump inhibitors and smoking cessation is usually successful.

Marginal ulcers

Marginal ulcers may occur at the gastrojejunostomy in patients who undergo gastric bypass surgery.

GI bleeding without obvious source

Massive GI bleeding with hemodynamic stability can be managed with angiography and embolization when possible. If the patient requires more than 6 units of blood or has ongoing hemodynamic instability, then surgical management is indicated. Without localization, the operation of choice is a total abdominal colectomy. Providing a stoma is preferable to a primary anastomosis in case the patient rebleeds. An anastomosis is not ideal in the setting of hypotension and hemodynamic instability. Localization techniques should be exhausted first, when feasible, to avoid unnecessary bowel resection. On-table colonoscopies are useful but only if the patient is more stable.

Massive hemoptysis more than 600 cc in 24 hours. Control?

Massive hemoptysis is considered 600 cc in 24 hours and is best treated initially with rigid bronchoscopy and airway packing to control the hemorrhage. Different modalities can be used for control including epinephrine soaked gauze or foley balloon insertion for tamponade. Once the source of bleeding has been identified and controlled, other measures can be taken to definitively control the bleeding. These include surgical resection, angiographic embolization, and laser ablation. In an acutely decompensating patient with massive pulmonary hemorrhage, hilar clamping or a 180 degree hilar twist have been described. These are methods of last resort which can occlude the vessels at the hilum in an attempt to identify the area of bleeding lung parenchyma. When a pneumonectomy is performed for trauma, it is usually not well tolerated by patients.

LeFort Fractures

Maxillary fractures may be described using the Le Fort classification. Separation of the maxilla from the skull base results in mid-face mobility with bimanual exam where the upper alveolar ridge is pulled and pushed carefully while stabilizing the patient's forehead. In each of these fractures the posterior vertical maxillary buttress (which is at the junction of the posterior maxillary sinus and pterygoid plate of the sphenoid bone) is disrupted. Le Fort I fractures involve the inferior portions of the lateral and medial maxillary buttresses. Le Fort II fractures involve the inferior lateral maxillary buttress and superior medial maxillary buttress. Le Fort III fractures involve the upper transverse, superior lateral, and superior medial buttresses and result in complete craniofacial separation. For each type of Le Fort fracture the maxillary dental arch is mobile and patients must be placed in maxillomandibular fixation before plating the fractures.

Contrast Nephropathy

Measures aimed at preventing contrast induced nephropathy (CIN) have been studied in several randomized trials. The most prudent method of preventing CIN is to identify patients with risk factors before radiocontrast media are administered, then to consider either delaying the diagnostic or interventional procedure until kidney function can be optimized or switching to an alternative imaging modality. At the same time, every effort should be made to identify and correct underlying volume depletion and discontinue potential nephrotoxins. If the procedure is performed, the volume of radiocontrast media employed should be kept to a minimum. The adoption of peri-procedural hydration protocols is the most effective way to prevent CIN. Adequate intravenous volume expansion with isotonic crystalloids 3-12 hours before the procedure and continued for 6-24 hours afterward is the most effective measure to prevent CIN

Mediastinal tumors in children

Mediastinal tumors in children are either benign or malignant. Mediastinal tumors are generally categorized based on their location: anterior, middle, or posterior mediastinum. Lymphoma is the most common anterior and middle mediastinal tumor. Neurogenic tumors are the most common in the posterior mediastinum. They originate from elements of the sympathetic chain. They may grow very large before detection and may be found incidentally. Symptoms of Horner's syndrome or tracheal displacement may be apparent.

ITP

Medical therapy options for idiopathic thrombocytopenic purpura include corticosteroid therapy, platelet transfusion, gamma-immunoglobulin, rituximab, and the Rho(d) immunoglobulin. High-dose corticosteroid therapy is the first-line treatment. Steroid therapy produces an initial response in most patients; however, this response is usually not sustained. Answer A: Platelet transfusion is indicated only in patients with active bleeding because platelets become rapidly coated with immunoglobulin G and then are sequestered and destroyed in the spleen. Answer C: Intravenous immunoglobulin is a second-line therapy. It takes 3-5 days to show an effect. Answer D: Rituximab and romiplostim are usually used as third-line agents after splenectomy failure. Answer E: Splenectomy is indicated in patients who fail to achieve a sustained remission with medical therapy. Bottom Line: High-dose corticosteroid therapy is the first-line treatment for idiopathic thrombocytopenic purpura.

Digestive disorders can be treated with what length fatty acid chain?

Medium chain triglycerides, not long chain triglycerides, are useful for those who cannot digest long chain fats, such as patients with malabsorption associated with pancreatic insufficiency, bile salt deficiency, short bowel syndrome, and bacterial overgrowth of the small bowel. Patients with pancreatic insufficiency cannot digest long chain triglycerides and may have steatorrhea.

MERKEL CELL CARCINOMA

Merkel cell carcinoma is a rare, aggressive tumor occurring mainly in the sun exposed areas, such as the head and neck, in patients over 65 years old. Metastases to regional lymph nodes are noted on initial diagnosis in 12 to 15% of patients. Regional metastasis eventually occurs in one half to two-thirds of patients. Distant metastases occur ultimately in one-third of patients. The mean time from diagnosis to systemic involvement is 18 months, with death occurring 6 months later. The 5-year survival rate has been reported as 30 to 64%. Surgical excision is the treatment of choice for primary tumors with margins at least similar to those for melanoma. Sentinel lymph node biopsy is used in all patients with clinically node-negative patients. Local recurrence following primary excision develops in 24 to 44% of patients. RADIATION should be considered for all patients with Merkel cell carcinoma, as it is radiosensitive.

Chronic lung allograft dysfunction

More than 1 year after transplant, chronic lung allograft dysfunction (CLAD) is the leading cause of death. Overall mortality rate is highest in the first year, but modern advances and improvements in surgical technique and perioperative care have produced improvements in short-term survival. However, long-term survival remains poor with a 27% survival rate at 10 years. CLAD, which consists of bronchiolitis obliterans syndrome and restrictive allograft syndrome, is the leading cause of mortality in those patients surviving more than 1 year. Answer A: Between 30 days and 1 year, non-cytomegalovirus infections are the leading cause of death. Answer B: The most common cause of 30-day mortality after lung transplantation is graft failure. Answer D: Long term risk for malignancy is increased in all patients taking immunosuppression, however this is not the leading cause of mortality after 1 year. Answer E: Acute rejection is not a major cause of mortality > 1 year after transplantation. Bottom Line: Chronic lung allograft dysfunction is the leading cause of death in patients > 1 year after lung transplant.

Merkel cell carcinoma

Merkel cell carcinoma is highly radiosensitive. Adjuvant radiation is indicated for the tumor bed in stage II (>2 cm without lymph node metastasis) and stage III (any size tumor with lymph node metastasis) tumors. Lymph node involvement (positive sentinel lymph node or clinically positive node) should be managed by lymph node dissection and/or radiation therapy of the nodal basin. Answer A: Sentinel lymph node biopsy is indicated in all patients with Merkel cell carcinoma, except those who have clinically palpable lymph nodes or distant metastasis. Answer B: About 70% of tumors are localized (stage I, <2 cm without lymph nodes, or Stage II, ≥2 cm without lymph nodes), 25% are Stage III (lymph nodes are positive), and 5% are Stage IV (distant metastasis) at the time of the diagnosis. Answer C: Wide, local excision (1-2 cm margin) is the appropriate surgical management. If sentinel lymph node is also indicated, it should be performed before the wide local excision. Answer D: The role of chemotherapy in the management of Merkel cell carcinoma is not well established and should mainly be reserved for stage IV disease. Patients with earlier stages may receive chemotherapy but should be determined on a case-by-case basis. Bottom Line: Merkel cell carcinoma is highly radiosensetive. Adjuvant radiation therapy should be used for tumors ≥2 cm in size. Radiation of the lymph node basin is also indicated if lymph node metastasis was identified.

Mesenteric cysts

Mesenteric cysts are rare, benign lesions. Their incidence is less than 1 in 100,000 people. Mesenteric cysts can be visualized on ultrasound, CT, and MRI. Cysts are typically unilocular without a solid component. Presentation varies and may range from an incidental finding in an asymptomatic patient to acute abdominal pain. Nonspecific complaints of nausea, vomiting, anorexia, or weight loss are also common. Answer B: Mesenteric cysts are approximately 4.5 times more common than omental cysts. Answer C: The most common location for mesenteric cysts is the small bowel mesentery (the terminal ileum being the most common). Answer D: The preferred treatment for mesenteric cysts is enucleation. This may require resection of associated bowel if resection of the cyst results in compromise of blood supply to adjacent bowel. Simply unroofing the cysts results in a high rate of recurrence. Answer E: Hematochezia is unlikely as mesenteric cysts are typically not in communication with the bowel lumen. Bottom Line: Mesenteric cysts may present with nonspecific abdominal symptoms or as an incidental finding on routine imaging. Cysts should be resected when identified.

Milrinone

Milrinone is a phosphodiesterase type III inhibitor that is used as a contractility agent in patients with cardiogenic shock. Milrinone also acts to increase pulmonary vasculature dilation, which can lower pulmonary hypertension. Milrinone works by blocking the degradation of cyclic adenosine monophosphate. Milrinone improves pulmonary hemodynamics and right ventricular function in chronic pulmonary hypertension.

Paronychia

Minor skin break down is the most common predisposing factor for acute paronychia; therefore, people with occupations that are associated with increased risk of such minor traumas have higher risk of developing acute paronychia. Staph aureus is the most common pathogen. Other possible organisms are streptococcus, enterococci and pseudomonas. Paronychia is the most common hand soft tissue infection. It represents 35% of all hand infections

Living donor Kidney transplant

Most kidneys begin to function right after implantation, especially when it comes from a living donor. When the initial function results in only a slow drop in serum creatinine, the diagnosis of slow graft function is made. When dialysis is needed in the first week because of poor function, the diagnosis of early graft dysfunction is made. Early graft dysfunction typically occurs in 15-30% of deceased donor transplants. The diagnosis of slow graft function or delayed graft function is important to establish because other problems, such as rejection, vascular occlusion, or ureteral obstruction, can also result in low urine output and need for dialysis. These other causes must be eliminated before the diagnosis of slow graft function or delayed graft function can be made. Answer A: Renal artery stenosis occurs in up to 15% of transplanted kidneys. Answer B: Renal artery thrombosis is a rare complication occurring in about 1% of transplants. Answer C: Ureteral obstruction occurs in 1-9% of transplants. Answer D: Venous complications are less frequent, with thrombosis occurring in 1-4% of transplants. Bottom Line: Graft dysfunction is the most common complication after kidney transplantation.

Alkaline reflux gastritis

Most patients suffering from alkaline reflux gastritis have had gastric resection performed with a Billroth II anastomosis. Symptomatology may occur at any time after the operation. After gastrectomy, reflux of bile is fairly common. In a small percentage of patients, this reflux is associated with severe epigastric abdominal pain accompanied by bilious vomiting and weight loss. It is usually not relieved by food or antacids. The vomiting may occur at any time during the day or night and can even awaken patients from sleep. Iron deficiency anemia and weight loss are also common. Although the diagnosis can be made by taking a careful history, bile scintigraphy or HIDA scans are usually diagnostic, demonstrating biliary secretion into the stomach and even into the esophagus in severe cases. Upper endoscopy can also be performed with multiple biopsy samples taken away from the stoma, and the gastric fluid can be analyzed for bile acid concentrations. On endoscopy, the mucosa is frequently friable and beefy-red, and superficial mucosal ulcerations may be apparent on microscopy. UGI can help distinguish between afferent loop syndrome and bile reflux gastritis. In a patient who does not worsen with food ingestion are less likely to have afferent loop syndrome

ESRD

Most patients with end stage renal disease are on hemodialysis. The main indications for hemodialysis are GFR 10-15 mL/min/1.73 m2 in a symptomatic patient or GFR < 5 mL/min/1.73 m2 in an asymptomatic patient. GFR > 15 mL/min/1.73 m2 rarely requires HD. Classically the symptoms are described by the pneumonic: AEIOU-Acidosis, Electrolyte abnormalities, Intoxication, Overload, Uremia.The main indications for hemodialysis access are creatinine clearance less than 25 ml/min, serum creatinine greater than 4 mg/dl, and a patient within 1 year of anticipated need for dialysis

Hypophosphatemia

Muscle dysfunction and weakness are the main presenting signs of hypophosphatemia, but i can also lead to mental status changes and poor function of leukocytes.

MAC

Mycobacterium avium complex (MAC) is made up of Mycobacterium avium and Mycobacterium intracellulare. They are characterized by being aerobic, non-spore forming, and non-motile bacilli. These organisms produce pulmonary disease in patients with normal immune function and may lead to disseminated disease in patients with immunodeficiency. Disseminated MAC is diagnosed with positive cultures from blood or other normally sterile sites including bone marrow, liver, and spleen. Blood cultures are the best test for disseminated infection. Sending two separate blood samples will achieve a diagnosis in 99% of cases of disseminated MAC. In cases of early disease, bone marrow biopsy with culture may be the most sensitive test.

Necrolytic migratory erythema

Necrolytic migratory erythema, or glucagonoma syndrome, is often the external manifestation of an underlying neoplasm, most commonly an alpha cell pancreatic tumor. Presenting findings include diabetes mellitus and dermatitis (painful pruritic erythematous plaques and patches). Biopsy will reveal superficial necrolysis with perivascular infiltrate. Diagnosis is confirmed with elevated glucagon, elevated glucose, and hypoaminoacidemia. Tumor resection will result in resolution of the rash. Answer A: Necrolytic migratory erythema is often the external manifestation of an underlying neoplasm, usually a tumor of the pancreatic alpha cells. Answer B: The diagnosis is confirmed with the demonstration of elevated serum glucagon levels (usually >500 pg/mL), hyperglycemia, and hypoaminoacidemia. Answer D: Histologic findings include superficial necrolysis with perivascular infiltrate. Answer E: Necrolytic migratory erythema is a term used to describe polycyclic, weeping, erythematous patches and plaques that are found on the face, extremities, and groin. Associated findings include glossitis and cheilitis. Bottom Line: Necrolytic migratory erythema or glucagonoma syndrome is characterized by diabetes and dermatitis. It is often the external manifestation of an underlying pancreatic alpha cell tumor. Tumor resection results in resolution of the rash. TrueLearn Insight : Necrolytic migratory erythema and other cutaneous manifestations of systemic diseases are often tested with regard to their associated underlying malignancies/disorders. Review these details prior to test day.

Necrotizing fasciitis

Necrotizing fasciitis is divided into 2 different types. Type I includes one anaerobic species (bacteroides or peptostreptococcus) isolated with an additional facultative anaerobic species (streptococci other than group A, enterobacteriaceae). Obligate aerobes such as Pseudomonas aeruginosa are rarely involved in these mixed infections. Type II includes infection with group A streptococci alone or in combination with other species such as Staphylococcal aureus. Initial symptoms normally include erythema without sharp margins and edema. The area will commonly be warm, shiny, and tender. Rapid progression of symptoms includes skin changes from red to blue-gray. Skin breakdown with bullae formation and cutaneous gangrene can occur

Which burn therapies can cause nephrotoxicity when used on large areas?

Neomycin, bacitracin, and polymyxin B are topical ointments that are useful for superficial partial-thickness facial burns. The most common associated side effect of these agents is nephrotoxicity, when used on large burns.

Nipple discharge workup

Nipple discharge is a common and usually benign complaint. Some causes of nipple discharge include duct ectasia, hyperprolactinemia, hypothyroidism, medications, and malignancy; however, bloody nipple discharge is commonly the result of an intraductal papilloma. Once a full evaluation has ruled out malignancy, pathologic nipple discharge in nonlactating women should be managed by a terminal duct excision with or without image guidance. If a core needle biopsy has demonstrated ductal carcinoma in situ (DCIS) or invasive breast cancer, appropriate cancer surgery should be planned. This will provide unilateral, spontaneous, clear, or bloody discharge that arises from a single duct. For an accurate diagnosis, the workup begins with mammography or ultrasound. If a lesion is identified, a tissue biopsy should be performed. The best imaging for papilloma is ductography. The duct is cannulated, contrast injected, and mammography performed to identify filling defects. If further workup is warranted, the duct should be excised. This is performed by placing a lacrimal probe into the draining and excising the draining duct. If a lesion is not identified, terminal duct excision of the affected quadrant in indicated. The patient should be warned that breast-feeding may not be possible after the procedure.

Nitroglycerin

Nitroglycerin, a potent arteriolar and venous smooth muscle dilator, is a useful agent when both preload and afterload are elevated. The cardiovascular effects are dose-dependent, with low doses (5-20 µg/min) primarily increasing venous capacitance and higher doses (>20 g/min) relaxing arterial tone.

Aortic infections

Non-aneurysmal aortic infections are rare, but the majority of them are found in the suprarenal aortic area. Salmonella infections are now the most prevalent infection in non-aneurysmal aortic infections. Although staphylococcal infections are the most prevalent aortic infections, they are associated with aneurysmal degeneration. Streptococcal used to be the most prevalent aortic infection due to bacterial endocarditis, but these are now less common with the use of antibiotics.

Nonparametric data

Nonparametric data, is generally skewed and therefore does not fit a bell curve. Discrete data, such as that measured on a nominal or ordinal scale, consists of only a few possible values or categories and will not follow the normal distribution. Assessment of nonparametric data generally involves the use of contingency tables and significance testing uses methods such as the Mann-Whitney U (also known as Wilcoxon rank-sum) or Chi Square tests. Nonparametric data are generally discrete in nature and measured on nominal or ordinal scales. They are often skewed, do not fit the bell-shaped curve of normal data, and thus must be assessed using nonparametric methods such as the Mann-Whitney U or Chi Square tests.

Secretin stimulation test

Normal values for the standard secretin test are: 2.0 mL of pancreatic fluid per kilogram per hour, bicarbonate output of greater than 10 mmol/L in 1 hour, and amylase secretion of 6 to 18 IU/kg. In patients with chronic pancreatitis, secretin test results are: normal total volume and enzyme secretion but decreased bicarbonate secretion. Answer A: In patients with pancreatic cancer, secretin test results are: decreased total volume with normal bicarbonate and enzyme secretion. Answer C: In patients with end-stage pancreatitis, secretin stimulation test classically demonstrates reduction in total volume, bicarbonate, and enzyme secretion. Answer D: In patients with malnutrition, secretin test results are: normal total volume and bicarbonate secretion but decreased enzyme secretion. Answer E: In patients with Zollinger-Ellison syndrome, secretin test results are: increased total volume with normal bicarbonate and enzyme secretion.

NNT

Number needed to treat (NNT) represents the number of patients that need to be treated with a medication, procedure, or intervention in order to prevent 1 adverse outcome or event. In order to calculate the NNT, the absolute risk reduction is needed. NNT= 1/ARR ARR = CER (Control Event Rate) - EER (Experimental Event Rate). In this case ARR = 5% - 2.5% = 2.5% or 0.025, NNT = 1/0.025 = 40

Obesity hypoventilation syndrome

Obesity hypoventilation syndrome is characterized by hypoxemia and hypercapnea in an obese patient. The pathophysiology is due to chest wall compression and alveolar collapse leading to decreased ventilation in the terminal airways. No direct correlation exists between degree of obesity and severity of this syndrome. Answer B: Spirometry alone is not diagnostic; however, when coupled with arterial blood gas values, a diagnosis can be made. Answer C: No direct correlation exists between degree of obesity and severity of obesity hypoventilation syndrome. Answer D: Patients present with similar symptoms to sleep apnea, including hypercapnea and excessive daytime sleepiness. Answer E: Over time, this condition can lead to pulmonary hypertension and right-sided heart failure. Bottom Line: Obesity hypoventilation syndrome is caused by alveolar collapse and may lead to long-term sequelae, including pulmonary hypertension and right-sided heart failure.

Post renal acute renal failure

Obstruction to urine outflow gives rise to postrenal ARF, which is the most common cause of functional renal impairment in the community (i.e., in nonhospitalized patients) and is usually secondary to prostatic hypertrophy. Obstructive ARF is also seen in hospitalized patients, albeit less commonly; typical causes in this population include prostatic hypertrophy, pelvic tumors, retroperitoneal fibrosis, papillary necrosis, and large calculi. If obstruction is suspected, ultrasonography can be easily performed at the bedside to identify any obstruction. Ureteric stones are a common cause of ureteric obstruction. However, it is associated with pain and CT scan is highly sensitive in identifying kidney or ureteric stones. Answer A: Acute presentation of papillary necrosis that is related to sloughing of the renal papillae and obstruction of the ureter is characterized by hematuria, acute onset of pain and predisposing factors such as the use of NSAIDs in diabetic patients. The obstruction is more likely to be unilateral than bilateral. Answer B: Intraperitoneal fibrosis is not a true defined entity. RP fibrosis is the most likely cause. Answer D: Ureteric compression by tumor recurrence or new tumor can offer an explanation for the patient presentation. However, the lack of masses on CT scan makes this diagnosis less likely. Answer E: Neurogenic bladder is a possible consequence of diabetic neuropathy. Back pressure from the bladder could cause bilateral hydronephrosis. However, one should expect dilatation of the urinary bladder too and no entrapment of the ureters bilaterally.

Omphalocele vs Gastroschisis

Omphalocele is a congenital abdominal wall defect characterized by the presence of herniated visceral contents into a translucent sac. The sac is composed of amniotic membrane, mesenchymal tissue known as Wharton jelly, and peritoneum. Unlike in gastroschisis, the bowel is typically normal in appearance because it has not been directly exposed to the amniotic fluid. In gastroschisis, the bowel is thickened and the mesentery may be foreshortened secondary to the inflammatory response induced by direct exposure to amniotic fluid. Answer A: In omphalocele, the umbilical cord attaches to the sac and may be eccentric in origin, whereas in gastroschisis the defect is almost always located to the right of the umbilicus. Answer B: Associated congenital anomalies are more common with omphalocele than with gastroschisis. Approximately 50-60% of infants with omphalocele have at least one associated congenital anomaly versus 10% in gastroschisis. Answer C: Malrotation occurs in both gastroschisis and omphalocele. Answer D: Given the typical small size of the abdominal wall defect, herniation of the liver in gastroschisis is extremely rare. Giant omphaloceles greater than 8 cm in diameter may result in extensive herniation of the stomach, bowel, liver, and spleen with subsequent underdevelopment of the abdominal cavity. Bottom Line: Unlike in gastroschisis, the bowel is typically normal in appearance with omphalocele because it has not been directly exposed to the amniotic fluid.

Lymphocele

Once the diagnosis of a lymphocele has been made, treatment can proceed. Simple aspiration will only temporarily drain the collection. Percutaneous drainage can be successful but may require a sclerosing agent to obliterate the cavity. Laparoscopic fenestration of the cavity to facilitate drainage into the abdomen is also another treatment option which will lead to eventual cavity obliteration. Answer B: It is potentially caused by failure to ligate lymphatics. Answer C: Lymphoceles are not a rare occurrence after renal transplant. They occur anywhere from 0.6% up to 18% of patients. Answers D & E: Incorrect Bottom Line: Lymphoceles are not uncommon and are caused by extensive iliac vessel mobilization or failure to ligate lymphatics. Perinephric fluid collection. Seen with doppler

Gallstone pancreatitis

One common cause of pancreatitis is gallstones, which can pass into the common bile duct and cause obstruction of both biliary and pancreatic drainage. Treatment of gallstone pancreatitis initially involves both supportive care with IV fluid resuscitation and antibiotics to help prevent or treat cholangitis. Endoscopic retrograde cholangiopancreatography (ERCP) should be performed early with patients with high or moderate suspicion of choledocholithiasis and common bile duct dilatation. The American Society of Gastrointestinal Endoscopy (ASGE) recommends performing ERCP early for patients with CBD stone on transabdominal US, clinical ascending cholangitis, bilirubin >4 mg/dL or both dilated CBD on US (>6 mm with gallbladder in situ) and bilirubin level 1.8-4 mg/dL. Moderate factors that generally warrant investigation with either MRCP, endoscopic ultrasound, intraoperative cholangiogram, or intraoperative ultrasound include abnormal liver biochemical test other than bilirubin, age older than 55, and clinical gallstone pancreatitis. Absence of any of these factors suggests a low likelihood of choledocholithiasis and proceeding with cholecystectomy is safe. In this patient, the direct bilirubin is 3 times the normal and the CBD is dilated. This is suggestive of choledocholithiasis and resulting gallstone pancreatitis. The next best step in this case would be ERCP. Supportive care be used but a study needs to be performed to confirm that the duct is cleared. ERCP provides both diagnostic and therapeutic capabilities. Answers B & D: Moderate factors that generally warrant investigation with either MRCP, endoscopic ultrasound, intraoperative cholangiogram, or intraoperative ultrasound include abnormal liver biochemical test other than bilirubin, age older than 55, clinical gallstone pancreatitis or one of the following: dilated CBD on US (>6 mm with gallbladder in situ) or bilirubin level 1.8-4 mg/dL. Answer C: Proceeding with laparoscopic cholecystectomy is not advised until the suspected choledocholithiasis is addressed. Answer E: This patient has multiple factors suggestive of common bile duct obstruction and likely requires both intervention for removal of stones and cholecystectomy for prevention of future episodes. Bottom Line: Gallstone pancreatitis with high suspicion of choledocholithiasis should undergo ERCP prior to cholecystectomy.

IgA

One of the major protective immune mechanisms for the intestinal tract is the synthesis and secretion of IgA. The intestine contains more than 70% of the IgA-producing cells in the body. IgA is produced by plasma cells in the lamina propria and is secreted into the intestine, where it binds antigens at the mucosal surface. The IgA antibody traverses the epithelial cell to the lumen by means of a protein carrier. This secretory component not only transports the IgA but also protects it against the intracellular lysosomes. IgA does not activate complement and does not enhance cell-mediated opsonization. IgG, however, is associated with complement fixation, opsonization, fixation to macrophages, and membrane transport. IgG is the most abundant immunoglobulin class in the serum. IgA also does not cause the destruction of infectious organisms or antigens, which sharply contrasts to the role of other immunoglobulins. Answer A: Secretory IgA inhibits the adherence of bacteria to epithelial cells, and does not function to aid in destruction in contrast to the other immunoglobulins. Answer B: Unlike IgG, IgA does not active the complement cascade. Answer D: IgA is not involved in the opsonization of organisms; that function is characteristic of IgG. Answer E: Secretory IgA prevents the colonization and multiplication of bacteria in the gut. Bottom Line: Secretory IgA in the intestinal lumen prevents adherence of bacteria to the intestinal wall and neutralizes bacterial toxins and viruses.

Complications after hemorrhoidectomy

One of the more common complications of a hemorrhoidectomy is urinary retention. This is most often due to spasms of the pelvic floor musculature. The treatment for urinary retention is with a temporary foley catheter placement and subsequent voiding trials.

Chronic inguinodynia

One of the most popular inguinal hernia repair options is the Lichtenstein tension-free hernia repair with mesh. With the open approach, multiple nerves are at risk of injury or chronic irritation from mesh placement. Chronic inguinodynia is postoperative pain from nerve irritation. The definitive treatment is a triple neurectomy in which the genitofemoral, the iliohypogastric, and ilioinguinal nerves are all identified and divided. This procedure is very successful.

Ovarian tumors

Ovarian tumors are associated with the production of specific tumor markers. These biological tumor markers are useful in identifying, assessing response to therapy, and monitoring for recurrence. Tumor markers, including alpha fetoprotein (AFP), beta-human chorionic gonadotropin (beta-hCG), and lactate dehydrogenase (LDH), are the most commonly used. CA 125 is the best available marker for epithelial ovarian cancer. CA 125 lacks sensitivity for and specificity for early ovarian cancer. Levels can also occasionally be elevated in some benign conditions, including endometriosis, uterine myomas, acute and chronic salpingitis, and pelvic inflammatory disease. Ovarian germ cell tumors (OGCTs) can be broadly divided into those that differentiate toward embryo-like neoplasms (teratomas and their subtypes and dysgerminomas) and those that differentiate primarily toward extraembryonic fetal-derived (placenta-like) cell populations or a mixture of both. Categories include teratomas, dysgerminomas, yolk sac tumors, mixed germ cell tumors, and "rare" OGCTs (eg, pure embryonal carcinomas, nongestational choriocarcinomas, and pure polyembryoma). OGCTs are often associated with hormonal or enzymatic activity. Some of these proteins can be measured in the serum, providing a highly sensitive and variably specific marker for the presence of certain histologic components. Some tumor markers are present in some, but not all, tumors of a specific histology. Tumor markers produced by tumor types are as follows: 1. hCG - Embryonal cell carcinomas and ovarian choriocarcinomas, mixed germ cell tumors, and some dysgerminomas. 2. AFP - Yolk sac tumors, embryonal cell carcinomas and polyembryoma carcinomas, mixed germ cell tumors, and some immature teratomas; most dysgerminomas are associated with a normal AFP. 3. Lactate dehydrogenase (LDH) - Dysgerminomas. Yolk sac tumors make up 14 to 20 percent of all malignant OGCTs. The name was chosen because the tumor structure is similar to that of the endodermal sinuses of the rat yolk sac and is derived from the primitive yolk sac. These neoplasms usually occur in young girls and women; the median age at presentation is 23 years; one-third of patients are premenarchal. Answer B: HCG is typically elevated in a patient with different germ cell tumors including seminoma, dysgerminoma, choriocarcinoma, and embryonal carcinoma. Answer C: Inhibin is used as a tumor marker for epithelial stroma tumors such as mucinous and endometrioid carcinoma and sex cord stromal tumors like granulosa cell tumor and Sertoli-Leydig cell tumor. Answer D: Serum CA 125 is the most commonly used laboratory test for the evaluation of adnexal masses for epithelial ovarian cancer (EOC), but the test has several shortcomings. CA 125 testing alone has a low sensitivity, particularly for early-stage ovarian cancer, and a low overall specificity. The specificity is particularly low in premenopausal women. CA-125 may be elevated in all OGCTs, but given its low sensitivity and specificity, especially in premenopausal patients, it is not the most likely elevated tumor in this yolk sac tumor, which is an example of an OGCT. Answer E: Serum LDH is a nonspecific marker that is widely distributed in human tissues. It is seen in dysgerminomas. Bottom Line: CA 125 is the most widely utilized tumor marker for epithelial ovarian cancer (EOC), despite its lack of sensitivity and specificity, whereas alpha fetoprotein is elevated in yolk sac tumors.

ovarian tumor markers

Ovarian tumors are associated with the production of specific tumor markers. These biological tumor markers are useful in identifying, assessing response to therapy, and monitoring for recurrence. Tumor markers, including alpha fetoprotein (AFP), beta-human chorionic gonadotropin (beta-hCG), and lactate dehydrogenase (LDH), are the most commonly used. Fetal yolk sac is the source of AFP early in human embryogenesis, thus elevations of this marker occur with yolk sac tumors. Answer B: HCG is typically elevated in a patient with different germ cell tumors including seminoma, dysgerminoma, choriocarcinoma, and embryonal carcinoma. Answer C: Inhibin is used as a tumor marker for epithelial stroma tumors such as mucinous and endometrioid carcinoma and sex cord stromal tumors such as granulosa cell tumor and Sertoli-Leydig cell tumor. Answer D: CA 125 is the marker for epithelial ovarian cancer. Answer E: Serum LDH is a nonspecific marker that is widely distributed in human tissues. Bottom Line: AFP is the tumor marker for yolk sac ovarian tumors.

Paget's disease of anus

Paget disease of the anus occurs predominantly in women in the seventh or eighth decade. The hallmark of the clinical presentation is severe intractable pruritus. Physical examination typically reveals an erythematous, eczematoid rash. Biopsy should be considered for any nonhealing lesion to rule out this diagnosis. Once the diagnosis of Paget disease is made, a thorough workup for an occult malignancy is indicated as half of patients have underlying internal carcinoma. Wide local excision with multiple perianal biopsies is the treatment of choice when Paget disease of the anus is suspected; however, a thorough workup should be completed first to rule out any internal malignancy.

Paget-Schroetter

Paget-Schroetter syndrome is exercise-induced thrombosis of the subclavian and axillary veins. It happens most often in young competitive athletes who undergo repetitive upper extremity exercise. It often presents with a short history of pain and swelling in the arm. The arm is often blue and if chronic can have superficial dilation of collateral veins. Quick catheter directed thrombolysis can help reduce the long-term complications of post thrombotic syndrome. Once the vein is free of thrombus, further anticoagulation and imaging can be performed to identify a possible causative lesion.

Pancreatic divisum

Pancreas divisum usually remains asymptomatic, but can present with pancreatitis from obstruction at the lesser papilla. It results from failure of fusion of the ventral and dorsal pancreatic ducts. This is treated with endoscopic retrograde cholangiopancreatography and sphincteroplasty with stent placement. Open sphincteroplasty is considered for refractory cases. Answer A: Most patients with pancreatic divisum will remain asymptomatic. Answer C: The Duct of Wirsung is drained via the major papilla and drains the uncinate process and part of the head of the pancreas. The Duct of Santorini is drained via the lesser papilla and drains the head, body, and tail of the pancreas. Answer D: Annular pancreas and duodenal atresia usually present with double bubble sign. Answer E: Diagnosis is usually made during endoscopic retrograde cholangiopancreatography when two papilla are seen. Bottom Line: Pancreatic divisum results from failure of fusion of the ventral and dorsal pancreatic ducts and is treated with endoscopic retrograde cholangiopancreatography, sphincteroplasty, and stent placement.

Unresectable pancreatic cancer

Pancreatic cancer is considered unresectable with any metastases, extension into the hepatoduodenal ligament, involvement of major arterial structures, or involvement of nodal or neural structures around the celiac or superior mesenteric arteries. Isolated involvement of the SMV or portal vein can be resected in selected patients.

Pancreatic polypeptide

Pancreatic polypeptide is known to suppress bile secretion, gallbladder contraction, and exocrine pancreatic function.

Pancreatic pseudocyst

Pancreatic pseudocysts are encapsulated collections of fluid with high enzyme concentrations that arise from the pancreas. Pancreatic pseudocysts are associated with an epigastric mass and pain, mild fever, persistent serum amylase elevation. The walls of a pseudocyst are formed by inflammatory fibrosis of the peritoneal, mesenteric, and serosal membranes, which limits spread of the pancreatic juice as the lesion develops. The term pseudocyst denotes absence of an epithelial lining, whereas true cysts are lined by epithelium

Pancreatic secretions

Pancreatic secretions contain a variety of enzymes, including trypsinogen, chymotrypsinogen, elastase, carboxypeptidase, pancreatic lipase, and amylase. These secretions are alkaline in nature due to the high concentration of bicarbonate ions. This is useful in neutralizing the acidic gastric acid, allowing for effective enzymic action. The centroacinar cells and ductular epithelium secrete fluid containing 20 mmol of bicarbonate per liter in the basal state and up to 150 mmol of bicarbonate per liter under maximal stimulation. The fluid, with a pH that varies from 7.6 to 9.0, acts as a vehicle to carry inactive proteolytic enzymes to the duodenal lumen. Sodium and potassium concentrations are constant and equal those of plasma. Chloride secretion varies inversely with bicarbonate secretion, and the sum of these two cations remains constant and equal to that of plasma. Pancreatic juice secretion is regulated by the hormones secretin and cholecystokinin (CCK), which is produced by the walls of the duodenum upon detection of acid food, proteins and fats. Pancreatic secretion consists of an aqueous bicarbonate component from the duct cells and enzymatic component from the acinar cells.

Parastomal hernia repair

Parastomal hernias have the lowest rate of recurrence when repaired with mesh. This can be done laparoscopically as an intraperitoneal onlay to minimize infection risk and bowel manipulation.

Gallstone ileus

Passage of a stone through a spontaneous biliary-enteric fistula leading to a mechanical bowel obstruction is known as gallstone ileus. Most (75%) of these fistulas develop between the gallbladder and duodenum, occur in elderly people, and account for 1% of all small bowel obstructions. Gallstone ileus may account for as many as 25% of cases of intestinal obstruction in patients older than 70 years who have no previous surgery or hernias on physical exam. Biliary-enteric fistulas usually follow an episode of acute cholecystitis with gangrene and perforation of the gallbladder wall into the adjacent viscus or from pressure necrosis from an impacted gallstone. Nausea, vomiting, and abdominal pain, signs and symptoms of intestinal obstruction, are the common presentation of patients with gallstone ileus. A history of gallbladder-related symptoms is present in only half of the patients. The pain may be episodic and recurrent as the impacted stone temporarily obstructs the bowel lumen and then dislodges and moves distally, known as tumbling obstruction. Abdominal films will demonstrate evidence of an intestinal obstruction with pneumobilia or a calcified stone distant from the gallbladder. The most common site of obstruction is the terminal ileum because of its narrow lumen. The diagnosis is suspected when signs of bowel obstruction are present, and pneumobilia is discovered on imaging.

Biliary dyskinesia

Patients who present with typical symptoms of biliary colic but have no evidence of gallstones on ultrasound exam should be investigated for biliary dyskinesia. Once other possible diagnoses have been excluded with CT scan, upper endoscopy, or endoscopic retrograde cholangiopancreatography, a CCK-Tc-HIDA scan should be performed to rule out biliary dyskinesia. CCK is infused intravenously after the gallbladder has been filled with the 99mTc-labeled radionuclide. Twenty minutes after the injection, an ejection fraction of less than 35% is considered abnormal. Patients with symptoms of biliary colic and an abnormal gallbladder ejection fraction should be managed with a laparoscopic cholecystectomy. The majority of patients with a low gallbladder ejection fraction and symptoms of biliary colic will be improved by cholecystectomy. Commonly, there is histopathologic evidence of chronic cholecystitis.

Crohn's disease anal fissures

Patients with Crohn disease may present with an anal fissure as part of the anorectal manifestations of the disease. The pathophysiology of anal fissure in patients with Crohn disease is not related to the concepts of trauma, ischemia, and high anal sphincter pressure. Therefore, anorectal surgery is contraindicated in patients with Crohn disease. Postoperative incontinence, nonhealing wounds, and the need for eventual proctectomy are the main concerns. Thus, the mainstay of therapy is medical management of Crohn fissures with a focus on bulking the stool, reducing diarrhea, and comfort measures. If medical management fails then examination under anesthesia is indicated to rule out the presence of other pathology. Answer B: Manometric studies have demonstrated increased resting internal anal sphincter tone in patients with anal fissures. Increased anal sphincter pressure results in decreased perfusion of the overlying anoderm, ultimately making the anoderm in the posterior and anterior midline the most tenuous and susceptible to fissure formation. In a patient without Crohn disease, anal dilation would be a viable option to aid in relaxation of the internal anal sphincter; however, it would not be the first choice. In a patient with Crohn disease, it is of even greater importance to promote noninvasive methods of fissure management first. Answer C: Lateral sphincterotomy is the mainstay of treatment for patients with anal fissures who fail conservative management; however, in patients with Crohn disease, it is relatively contraindicated because of the risk of creating a chronic, nonhealing wound and the risk of incontinence in a patient with diarrhea from underlying colitis. Answer D: Fissurectomy is not a surgical procedure that is indicated in a patient with Crohn disease. Treatment of fissures in this patient should be focused on control of infection, symptom control, and sphincter preservation. Fissurectomy would place the patient at increased risk of fecal incontinence. Answer E: An advancement flap is relatively contraindicated in patients with Crohn disease due to the underlying pathophysiology of Crohn disease and the high risk of poor wound healing and symptom recurrence. Bottom Line: The mainstay of therapy of anal fissure in patients with Crohn disease is medical management focusing on bulking the stool, reducing diarrhea, and comfort measures. TrueLearn Insight : The most common perianal lesions in Crohn disease are skin tags and fissures. Typically, fissures in this patient population are deep or broad and can be described as anal ulcers. They are often multiple and located in a lateral position rather than at the anterior or posterior midline as seen in idiopathic anal fissures. Fistulas tend to be complex and have multiple tracts.

Dialysis access

Patients with end stage renal disease require surgical access for long term dialysis treatments. The conduits for dialysis access can either be with native vein or prosthetic graft. The superficial veins, namely the cephalic and basilic veins, are examined preoperatively with an ultrasound to determine the diameter and to identify any structural abnormalities, such as a previous thrombosis. An ideal vein is at least 3mm in diameter, compliant, and with no signs of stenosis or thrombus. Veins that are of insufficient diameter are at risk for failure of maturation and inability to use for dialysis. A suitable vein is preferable to use over a prosthetic graft due to improved rates of patency and a lower risk of infection. If acceptable vein is available, the patient's non-dominant arm should be used and the most distal location is optimal to provide opportunities for future fistulas at a higher level in the future, if necessary. However, if there are limitations in the vein, the dominant arm can be used and is still preferable over a graft for the aforementioned reasons. Ideally, a 3mm vein will dilate and mature to 5-6mm after arterialization.

Infected aortic aneurysm

Patients with infected aortic aneurysms generally present with fever, abdominal, back, or chest pain, and a pulsatile tender mass. They frequently have elevated white blood cell counts and elevated erythrocyte sedimentation rate. Infected aneurysms are usually located in the infrarenal aorta. The most common microorganisms associated with infected aortic aneurysms is Staphylococcus aureus and Salmonella species.Infected aortic aneurysms will expand and eventually rupture. The timeframe can be short with high mortality. Salmonella and gram negative species are associated with rapid progression of dilation and rupture. Mortality exceeds 50% with medical treatment alone. Surgical treatment should involve excision or bypass of the infected segment with debridement of surrounding tissue infection and a prolonged antibiotic course.

Forrest classification

Patients with known ulcer disease can be stratified into their risk for bleeding based on endoscopic findings which is known as the Forrest classification model. Highest risk findings include active bleeding and a visible vessel. All high risk patients should undergo some endoscopic therapy to reduce the chance of rebleeding. An adherent clot is classified as an intermediate risk. The low risk characteristics include ulceration with a black spot or with a clean base.

AAA and renal artery stenosis

Patients with renal stenosis and coinciding abdominal aortic aneurysm (AAA) should only be treated with AAA repair and medical management of the renovascular hypertension

Pelvic fractures in the elderly

Pelvic fractures in the elderly are fairly common and pose significant risk for morbidity and mortality. Compared to younger patients, aged 55 years or less, mortality is four times higher after pelvic fractures, upward of 20.5%. Mortality remains high even after adjustment of for Injury Severity Score. Elderly patients are also far more likely (nearly 2.8 times) to receive blood transfusions during treatment. All elderly patients with hip fractures should be treated as hemodynamically unstable until proven otherwise. Another crucial element toward good outcome is timing of surgery. The risk of death within a year is more than doubled if a fracture is not fixed within 2 days. Early mobilization from bed and aggressive physical and occupational rehabilitation are necessary.

Treatment of acute lymphangitis

Penicillin therapy is first line treatment for acute lymphangitis. Prompt therapy is important due to rapid disease progression. Mild to moderate disease may be managed in an outpatient setting for nontoxic, reliable patients. In moderate cases, an initial dose of intramuscular ceftriaxone (1 g IM) may be administered, followed by high-dose oral therapy with penicillin V or amoxicillin (500 mg every 6 hours) if close monitoring is possible. More acutely ill patients warrant hospitalized and parenteral aqueous penicillin G (e.g., 2 million U every 4 to 6 hours). If a staphylococcal etiology is suspected, vancomycin (1 g every 12 hours) should be initially administered to hospitalized patients. Answer A: If a staphylococcal etiology is suspected, vancomycin should be initially administered to hospitalized patients.

Long gap esophageal atresia

Performing esophageal replacement is increasingly rare for several reasons. There are improved methods of repairing the native esophagus in infants with esophageal atresia. In addition, surgeons have adopted more aggressive approaches in antireflux surgery. The introduction of childproof containers has also decreased lye and caustic injuries to the esophagus. However, esophageal replacement is still performed worldwide for a variety of indications. Long gap esophageal atresia remains the most common indication for esophageal replacement.

esophageal replacement

Performing esophageal replacement is increasingly rare. First, there are improved methods of repairing the native esophagus in infants with esophageal atresia. Second, surgeons are adopting more aggressive approaches in antireflux surgery. Third, the introduction of childproof containers has resulted in decreased lye and caustic injuries to the esophagus. However, esophageal replacement is still being performed worldwide for a variety of indications. Overall, the posterior mediastinum provides the best route for esophageal substitutes during esophageal replacement

Phenylephrine

Phenylephrine is a potent, direct-acting α-adrenergic agonist with virtually no β-adrenergic activity. It works to produces systemic arterial vasoconstriction. Such increases in systemic vascular resistance result in dose-dependent increases in systolic and diastolic blood pressure and reductions in heart rate and cardiac output, especially in patients with heart failure. Phenylephrine is used in the treatment of hypotension and vascular failure in shock. It is also given as a vasoconstrictor in patients receiving regional analgesia.

I-131 MIBG scan

Pheochromocytomas can be localized using an iodine-131 methyl iodobenzylguanidine (I-131 MIBG) scan. I-131 MIBG is preferentially taken up by the adrenal medulla and chromaffin tissue. Pheochromocytomas are localized to the site of activity. Answer A: Adrenal vein sampling is used to localize aldosteronomas. Answer C: Somatostatin receptor scintigraphy is used to localize gastrinomas. Answer D: Endoscopic ultrasound is used to localize insulinomas. Answer E: Sestamibi scan is used to localize parathyroid adenomas. Bottom Line: Iodine-131 methyl iodobenzylguanidine is used to localize a pheochromocytoma. The tracer is preferentially taken up by the adrenal medulla.

Phyllodes tumor

Phyllodes tumor is a rare malignant tumor of the breast that presents as a smooth, round, breast mass that may not be distinguishable from a fibroadenoma on physical exam; both may present in women of the same age group. The clinically distinguishing factor is the rapid growth of phyllodes tumors and the slow growth of fibroadenomas. A rapidly growing smooth, round, breast tumor should undergo core needle biopsy. Phyllodes tumors are graded as benign, borderline, and malignant based on mitotic count, pleomorphism, stromal overgrowth, and character of the tumor border (circumscribed vs. infiltrative). Phyllodes tumors should be resected with a border of 1-2 cm of normal breast tissue. About 20% of phyllodes tumors enucleated or resected with a few millimeters of normal breast tissue will recur. Answers A & C & E: Malignant phyllodes tumors most commonly metastasize to the lungs. Answer B: Malignant phyllodes tumors rarely metastasize to the axillary lymph nodes, but if axillary lymph node metastases are found, they are a poor prognostic indicator.

Phyllodes tumor

Phyllodes tumors are classified as benign, borderline, or malignant, based upon the histological characteristics. These include number of mitoses, cellularity, margin behavior, and sarcomatous elements. Radiologic findings are nonspecific. Cytology by fine-needle aspiration is inaccurate, and core needle biopsies have been shown to have false negative rates up to 50%. Indeterminate or borderline tumors are difficult to pinpoint as malignant or benign and the recommended course of action is complete excision with 1-cm negative margins and a close followup schedule.Lesions that behave closer to malignant sarcoma are treated as such with en bloc surgical excision. Like sarcoma, metastasis is via hematogenous spread; therefore, lymph node dissection is unnecessary, and chemotherapy is ineffective.

Hernia imaging

Physical exam is often sufficient to diagnose most inguinal hernias. When uncertainty exists, multiple other imaging modalities are available for evaluating for a hernia. Ultrasound has the advantage of not exposing the patient to ionizing radiation and is often cheaper than other imaging studies; however, it has a sensitivity of 86% and a specificity of 77%. While CT scan utilizes radiation, it is widely available and quick; however, sensitivity is only 80% and specificity is 65%. MRI is expensive but does not use ionizing radiation. MRI is often utilized when ultrasound is inconclusive. It has a remarkable sensitivity of 95% and a specificity of 96%. X-ray and endoscopy are not useful methods for detecting inguinal hernia.

Pilonidal disease

Pilonidal disease is a clinical diagnosis. Hidradenitis, perianal fistula, and syphilis represent the differential diagnosis. Unlike the scenario above, when a sinus opens close to the anus or tracks caudally, perianal fistula should be ruled out. Answer A: The patient has stage III pilonidal disease. Answer B: Diagnosis of pilonidal disease is mainly clinical. Imaging is not needed to identify the extent of the disease. Answer C: While pilonidal disease can be identified clinically, ruling out perianal fistula is imperative if the opening is close to the anus and the sinus tracks caudally. Answer E: Biopsy of the sinus tract is not necessary, unless there is a fungating, enlarging mass in chronic pilonidal sinus. Bottom Line: Pilonidal disease is a clinical diagnosis. Acute presentation (abscess) is classified as class II and should be incised and drained. Forty percent come back with a pilonidal sinus. Perianal fistula is suspected if the sinus opening is close to the anus or tracking caudally.

Plasmin

Plasmin is the active clotting inhibitor that acts to degrade fibrin and fibrinogen and disrupt the coagulation cascade. It circulates in an inactive form of plasminogen and is cleaved and activated by tissue plasminogen activators, namely urokinase and streptokinase. Plasminogen activator inhibitor acts to counteract the effects of plasmin.

platelets

Platelets respond to endothelial injury by binding to exposed collagen at the site of an injury. Von Willebrand factor binds to the glycoprotein (GP) Ib receptor on the platelet. Platelets release multiple vasoactive factors, such as thromboxane A2, which contribute to platelet aggregation and sealing at the site of injury. GPIIb/IIIa receptors on the platelet surface bind to fibrinogen, which links platelets together to form the initial platelet plug. Thrombin catalyzes the formation of fibrin from fibrinogen, which stabilizes the platelet plug. Abciximab, eptifibatide, and tirofiban are drugs that inhibit GPIIb/IIIa receptors. Answer B: Thromboxane A2, released by platelets, is a potent vasoconstrictor and stimulator of platelet activation. Prostacyclin is an endothelial prostaglandin that counteracts the effects of thromboxane, causing vasodilation. Answer C: Thrombospondin, released from degranulation if alpha granules, stabilizes fibrinogen and platelet-to-platelet interactions. Answer D: Platelet factor 4 is a potent heparin antagonist. Answer E: Von Willenbrand factor is a protein present in the subendothelium. With intimal injury it is exposed to blood and binds GPIb on platelet surfaces to stimulate formation of the initial platelet plug.

Most common lung tumor in children?

Pleuro-pulmonary blastomas are the most common primary lung tumors in children according to the largest published review in literature. They are malignant embryonal mesenchymal neoplasms that may be solid or cystic. Treatment is primarily surgical, with adjuvant or neo-adjuvant chemotherapy and radiation used as adjuncts. Carcinoid tumors are the other common primary malignancy, with some reports suggesting that they are more common than blastomas.

Popliteal artery entrapment

Popliteal entrapment is a cause of intermittent claudication in young patients. These patients are typically male and can have palpable pedal pulses but on movement have complaints of claudication. Symptoms may progress to parasthesias as they develop fibrosis of the popliteal artery. Popliteal artery entrapment occurs most often due to a congenital anomaly that causes the medial or lateral gastrocnemius head to shift toward the popliteal artery during knee flexion, thereby compressing the artery. Progressive fibrosis typically occurs as a result of arterial compression. The fibrosis can ultimately lead to complete occlusion or thrombosis. Answer A: Gastrocnemius injuries consist primarily of sudden muscle tears that are sustained during ballistic movements involving eccentric loading of the muscle. This typically occurs during a sudden push-off when an athlete initiates a sprint or jump, at which point the ankle rapidly changes from a plantarflexed position to a dorsiflexed position, while the knee is fully extended or in some situations hyperextended. Patients with a gastrocnemius strain typically describe feeling a sudden tearing sensation or pop, accompanied by acute pain in the proximal posterior calf, that developed while they were running or jumping. With gastrocnemius tears, ultrasound shows disruption of the normal fiber alignment, typically at the musculotendinous junction, and a hematoma and fluid collection that is typically anechoic and visible between the aponeurosis of the medial head of the gastrocnemius and the soleus muscle. Popliteal artery velocities are normal. Answer B: Soleus strains develop when the ankle is passively dorsiflexed while the knee is flexed, as occurs during landing when running uphill. Soleus strains are usually chronic injuries that develop from overuse, primarily in distance runners. While the presentation may mimic popliteal artery entrapment, ultrasound usually shows normal popliteal artery velocities and disorganized muscle fibers of the soleus muscle. Answer C: An arterial embolus is typically a cause for an acute ischemic event and not consistent with this patient's presentation with intermittent claudication. Answer E: Popliteal aneurysm is extremely rare in young individuals, although pseudo-aneurysms have been associated with trauma to the popliteal fossa and can be delayed in presentation. Popliteal aneurysm is relatively common in older men and in an older athlete with a history and symptoms suggesting popliteal artery entrapment, a careful assessment to exclude aneurysm, possibly including ultrasound, is needed. Bottom Line: Claudication in young, healthy patients may be due to popliteal artery entrapment caused by entrapment by the medial head of the gastrocnemius muscle.

Budd Chiari syndrome

Post-partum females with a pre-existing hypercoaguable disorder are at risk for fulminant Budd-Chiari syndrome, and treatment may only be achieved with liver transplant. The best diagnostic study is Doppler ultrasound . Characteristic findings include absent flow in the hepatic veins and inability to visualize connections between hepatic veins and the IVC. CT or MRI can be used as second-line studies if ultrasound does not confirm the diagnosis. Previously, venography was the gold-standard. However, due to its invasive nature, it has been surpassed by advances in Doppler ultrasound technology. Liver biopsy is not necessary for the diagnosis

Posterior Hip dislocation

Posterior hip dislocations commonly occur from an axial load along a flexed femur, such as a motor vehicle crash with knee impact along the dashboard. The femoral head sits in the acetabulum in a slightly anteverted position (about 15 degrees). When the femoral head dislocates posteriorly, the femoral head internally rotates beyond the acetabular labrum and pushes the femur anteriorly, creating hip flexion and internal rotation. Adduction is the result of the distal femur being locked into internal rotation coupled with the fact that the natural position of the femur is that the proximal end is more lateral and distal end more medial; thus, with internal rotation, the medial distal end is locked into an adducted position whereas external rotation seen in an anterior dislocation "swings" the distal femur into a more lateral, abducted position. A posterior hip dislocation, therefore, presents with hip flexion, hip adduction, and internal rotation of the femur. Reduction can be attempted under appropriate sedation in the emergency department but often requires anesthesia in the operating room. Almost all pure dislocations reduce quite easily under general anesthesia and muscle relaxation. Internal fixation with intramedullary nailing, for example, is unnecessary unless there is an unstable fracture pattern associated with the fracture. The presence of a significant posterior wall acetabular fracture may render the hip irreducible or unstable, necessitating open reduction and internal fixation of the acetabulum and hip dislocation. A high incidence of avascular necrosis of the femoral head has been reported with dislocations that are reduced late. After successful reduction, a post-reduction CT must be performed for all traumatic hip dislocations to look for femoral head fractures, loose bodies, and acetabular fractures. The sciatic nerve runs posteriorly and can be injured with posterior hip dislocations. Also, the femoral head is typically injured and, if not addressed, can lead to avascular necrosis. Answer B: While vascular injury is not typically associated with pure hip dislocations, injuries to the ipsilateral knee are seen in up to 25% of cases. Answer C: Shenton's line is an imaginary arc along inferior femoral neck and superior obturator foramen. It is typically disrupted in hip dislocations. Answer D: Closed reduction should be performed promptly with appropriate sedation. Traction should be placed in line with the extremity. After reduction of a posterior hip dislocation, the hip should be checked for stability at 90 degrees of flexion in neutral rotation. Answer E: In the absence of a diminished injured extremity index (IEI), angiography is not required. Bottom Line: Posterior hip dislocations are more common. They can typically be treated with closed reduction but will require prompt ORIF in the presence of an irreducible hip or an unstable posterior wall acetabular fracture. If delayed, avascular necrosis of the femoral head may result. TrueLearn Insight : Hip dislocations are frequently tested. Remember: Posterior hip dislocations are more common (85-95% of hip dislocations). They present with an internally rotated and adducted leg. There is an associated risk of sciatic nerve injury.

post operative parotitis

Postoperative parotitis is a rare and serious staphylococcal infection of the parotid gland. It most commonly occurs in elderly, debilitated and malnourished patients. Dehydration and poor oral hygiene increases risk. It often occurs in the second week after surgery with prolonged intubation. Decreased secretion from the parotid glands leads to infected secretions within the gland. Staphylococci and gram negative bacteria from the oral cavity are the typical culprits. Inflammation leads to duct obstruction and abscess formation. Signs and symptoms begin with pain at the angle of the jaw that progresses to fever, elevated leukocytosis, edema and erythema overlying the parotid. Adequate fluid intake and good oral hygiene are important for prophylaxis. When signs and symptoms appear fluid from the Stensen duct can be cultured. Treatment should be started with vancomycin, warm packs and mouth irrigation. Progression despite these measures warrants surgical drainage

Gastric banding complications

Potential complications of gastric banding includes band slippage, gastric erosion, esophageal dilation, and port access difficulty.

Acute renal failure

Prerenal causes of acute renal failure stem from any pathologic process that steals renal perfusion. Dehydration, hemorrhagic shock, septic shock, hepatorenal syndrome, and abdominal compartment syndrome are all perceived by the kidney as an inadequate blood volume. The end result is a stimulation of renin release by the juxtaglomerular cells. This cascade ultimately produces aldosterone, which acts on the principal cells of the collecting ducts to salvage sodium in exchange for either potassium or hydrogen. Water reabsorption is always passive and secondary to solute transport; thus blood volume is restored as sodium is reclaimed from the renal filtrate. Understanding this physiologic mechanism allows for a more logical differentiation of prerenal and intrinsic causes of azotemia. As sodium is resorbed in the collecting ducts, the concentration remaining in the filtrate, and thus, urine (UNa) will decline to a level <20 mOsm. Answers A & D: This is consistent with intrinsic sources of renal failure. Answer C: The fraction of filtered sodium that is actually excreted in the urine will be 1% or less. Answer E: The effort to preserve blood volume leads to a concentration of the urine with a rise in osmolality to >500 mOsm/L. Bottom Line: Prerenal acute renal failure stimulates release of aldosterone, which leads to resorption of sodium from the collecting ducts. This will cause a fall in urine sodium to <20 mOsm/L and fractional excretion of sodium to 1% or less. The urine becomes more concentrated than plasma.

Bite wounds

Preventive antibiotics are needed for patients with high risk bites such as puncture wounds, involving hand, wrist or foot; immunocompromised individuals; bites by humans, domestic and large cats (deep puncture wounds), pigs and primates. Low risk bite are long clean lacerations that can be irrigated thoroughly and those involving face, scalp or mouth (except in infants due to high risk of cranial perforation). The initial antibiotic choice should be based on animal, location of the wound, and the severity. Patients seen after 24 hours without signs of infection do not usually need antibiotics. Infections developing within 24 hours of the bite are usually due to Pasteurella spp and are covered by appropriate antibiotic coverage. Initial antibiotic selection should cover Staphylococcus spp, Streptococcus spp, anaerobes, Pasteurella spp, for dog and cat bites and Eikenella corrodens for human bites. Amoxicillin clavulanate is an acceptable first-line antibiotic for most bites. Alternatives include second generation cephalosporin such as cefoxitin or a combination of penicillin and first-generation cephalosporin. Moxifloxacin has been suggested as monotherapy. Patients with serious infection require inpatient admission and parenteral antibiotics such as ampicillin-sulbactam, piperacillin-tazobactam, cefoxitin, ticarcillin-clavulanate or clindamycin combined with a fluoroquinolone, or trimethoprim-sulfamethoxazole.

Primary bile salts

Primary bile salts (cholate and chenodeoxycholate) are produced in the liver and subsequently conjugated in hepatocytes with glycine and taurine. These conjugated bile salts are then secreted into bile and ultimately into the intestines to aid in digestion/absorption of fat. The majority (80%) of conjugated bile acids are absorbed via active transport in the terminal ileum. The remainder are deconjugated by bacteria within the colon to deoxycholate and lithocholate. These deconjugated bile salts are absorbed by passive transport in the colon and returned to the liver. Approximately 5% of bile acids escape absorption in the intestine and are ultimately excreted in the stool. Answer A: 80% of conjugated bile acids are absorbed within the terminal ileum. Those that remain are deconjugated by gut bacteria prior to absorption in the colon. Answer B: Enterohepatic circulation is highly efficient, with only 5% of bile acids being excreted in the stool. Answer C: The majority of bile acids are absorbed by active transport in the terminal ileum. Answer D: Bile acids play a major role in the absorption of lipids and fat-soluble vitamins (vitamins A, D, E, K). Bottom Line: The majority of conjugated bile salts are absorbed by active transport in the terminal ileum while the remaining are deconjugated by gut bacteria and absorbed in the colon or excreted in the stool.

Treatment of type 1 choledochal cyst?

Primary cyst excision with Roux-en-Y hepaticojejunostomy reconstruction

Perforated esophagus secondary to malignancy

Primary repair may be attempted in early esophageal perforations, excluding special circumstances such as cancer, severe peptic strictures, caustic injury, or refractory achalasia regardless of the degree of inflammation or time interval. The patient is presenting with an esophageal perforation secondary to his tumor. Esophageal perforations require immediate adequate drainage to avoid life-threatening mediastinitis. In stable patients without known malignancy, the extent of the perforation can be evaluated with a swallow study and managed non-operatively if the leak is contained. However, if there is a free perforation, the patient will need operative repair. The same principle applies if there is malignancy given that the esophageal tear will not seal itself with non-operative management. For repair, primary repair is not recommended in the setting of cancer. If patients present within 24 hours and the tissue remains viable without extensive inflammation, esophagectomy with primary anastomosis may be attempted. If the presentation is delayed or if there is too much inflammation to proceed with a primary anastomosis, resection and diversion is necessary. In the setting of hemodynamic instability, damage control with wide drainage and diversion may be the only option. In this patient, primary anastomosis versus resection with diversion are both acceptable options. Answer A: Perforated esophageal cancer requires local control with a surgical intervention. Answer B: The mid esophagus is best accessed through a right thoracotomy. Answer C: Primary repair will only place the patient at risk for subsequent perforation and is contraindicated in the presence of cancer. Answer D: If there is too much inflammation, usually after 24 hours, or the patient is unstable, then either a diversion or resection is necessary. Bottom Line: Esophageal perforations secondary to cancer require an esophagectomy. The patient may be left diverted or primary anastomosis can be attempted based on patient stability.

Ulcerative colitis

Primary sclerosing cholangitis and ankylosing spondylitis often will not improve after colectomy, while arthritis, erythema nodosum, and pyoderma gangrenosum will improve or completely resolve.

Protein digestion

Protein digestion is initiated in the stomach, where gastric acid denatures proteins. Digestion is continued in the small intestine, where the protein comes in contact with pancreatic proteases. Pancreatic trypsinogen is secreted in the intestine by the pancreas in an inactive form but becomes activated by the enzyme enterokinase, a brush border enzyme in the duodenum. Activated trypsin then activates the other pancreatic proteolytic enzyme precursors. In normal humans, absorption of protein is usually 80-90% completed in the jejunum.

Pancreatic pseudocysts

Pseudocysts account for about 75% of cystic lesions of the pancreas. They are distinguished from other peripancreatic fluid collections (cystic neoplasms and congenital, parasitic, and extrapancreatic cysts) by their lack of an epithelial lining, a high concentration of pancreatic enzymes within the pseudocyst, and formation at least 4 weeks after an episode of pancreatitis or pancreatic trauma. Pseudocysts are formed by the inflammatory response that occurs after extravasated pancreatic secretions are walled off by the surrounding structures. The capsule of the pseudocyst can be thin fibrous tissue, which can progressively thicken as the pseudocyst matures. Pseudocysts present for fewer than 6 weeks were found to resolve spontaneously in 40% of cases, although they had a 20% risk for complications. However, pseudocysts documented to be present for longer than 12 weeks did not resolve and were associated with a complication rate of 67%. Pseudocyst size correlated with the eventual need for surgery: 67% of patients with pseudocysts larger than 6 cm required operative intervention, whereas 40% of patients with pseudocysts 6 cm or smaller required surgical treatment. Answer A: Pancreatic pseudocysts smaller than 6 cm and present for fewer than 6 weeks should be managed conservatively with serial CT scans. There is no role for magnetic resonance cholangiopancreatography in this patient. Answers B & C & D: Pancreatic pseudocysts smaller than 6 cm and present for fewer than 6 weeks should be managed conservatively with serial CT scans. Bottom Line: Pancreatic pseudocysts smaller than 6 cm and present for fewer than 6 weeks should be managed conservatively with serial CT scans. Invasive treatment should be performed only for symptomatic or enlarging pseudocysts.

Pancreatic pseudocysts

Pseudocysts may become secondarily infected, in which case they become abscesses. They can compress or obstruct adjacent organs or structures, leading to superior mesenteric-portal vein thrombosis or splenic vein thrombosis. They can erode into visceral arteries and cause intracystic hemorrhage or pseudoaneurysms. They have the potential to perforate and cause peritonitis or intraperitoneal bleeding. Answer B: Internal drainage may be performed with either percutaneous catheter-based methods (transgastric puncture and stent placement to create a cystogastrostomy), endoscopic methods (transgastric or transduodenal puncture and multiple stent placements, with or without a nasocystic irrigation catheter), or surgical methods (a true cystoenterostomy, biopsy of cyst wall, and evacuation of all debris and contents). Answer C: If infection is suspected, the pseudocyst should be aspirated by CT- or EUS-guided FNA, and the contents examined for organisms by Gram's stain and culture. If infection is present, and the contents resemble pus, external drainage is employed, using either surgical or percutaneous techniques. Alternatively, sampling and internal drainage can be performed, even in the same setting via endoscopy. Answer D: If the pseudocyst has failed to resolve with conservative therapy, and symptoms persist, internal drainage is usually preferred to external drainage, to avoid the complication of a pancreaticocutaneous fistula. Pseudocysts communicate with the pancreatic ductal system in up to 80% of cases, so external drainage creates a pathway for pancreatic duct leakage to and through the catheter exit site. Answer E: Because pseudocysts often communicate with the pancreatic ductal system, newer approaches to pseudocyst management are based on main duct drainage, rather than pseudocyst drainage. Transpapillary stents inserted at the time of ERCP may be directed into a pseudocyst through the ductal communication itself, or can be left across the area of suspected duct leakage to facilitate decompression and cyst drainage, analogous to the use of common bile duct stents in the setting of a cystic duct leak.

Pseudomyxoma peritonei

Pseudomyxoma peritonei (PMP) is the most common epithelial tumor of the appendix. PMP is more common in women than in men. Increasing abdominal girth is the most common presenting symptom in both men and women. Inguinal hernia is the second most common presenting symptom in men. When the hernia sac is noted to have mucoid fluid, it is critical that a sample of fluid is sent for histological analysis. Answer E: An ovarian mass found on pelvic examination is the second most common presenting symptom in women.

Pseudomyxoma peritonei

Pseudomyxoma peritonei is classically associated with low- or high-grade appendiceal mucinous neoplasms. Rupture of an appendiceal mucocele significantly increases the risk of developing peritoneal metastases from this uncommon and rare cancer. It seems unlikely that the diagnosis of appendiceal mucinous neoplasm was missed in this patient's appendectomy; regardless, clinical suspicion must remain high in this context. Typical presentation of pseudomyxoma peritonei is increasing abdominal girth followed by inguinal hernia in men. Radiographic appearance of this disease process is typical in that CT scans will demonstrate ascites with "scalloping" of solid organs and calcifications of the peritoneum. Peripheral location of the tumor and thickening of the undersurface of the diaphragm due to large cystic mucinous masses are also typical.

Pseudomyxoma peritonei

Pseudomyxoma peritonei refers to a condition with gelatinous material present in the abdomen and pelvis with mucinous implants in the peritoneum. The patient most often presents with increasing abdominal girth. Inguinal hernia is the second most common symptom. On workup, liver scalloping may be seen on CT scan. It can be caused by rupture of a benign cystadenoma. One of the most effective therapies available for pseudomyxoma peritonei involves aggressive cytoreduction and intraperitoneal chemotherapy, often referred to as HIPEC.

Pyoderma gangrenosum

Pyoderma gangrenosum is an extra-intestinal manifestation of inflammatory bowel disease. The most common site affected is the pretibial region of the lower extremities. It is thought to arise due to an impaired immune function and can be managed with steroids. There is improvement in the disease process once the inflammatory bowel disease is treated (such as in patients with ulcerative colitis who undergo colectomy). Pyoderma gangrenosum is not due to an inflammation of the adipose tissue.The condition is often chronic and starts with a small papule that increases in size.

Radiation colitis

Radiation therapy is a frequent and successful modality used in the primary and adjuvant treatment of many malignancies. Radiation therapy, however, can cause damage to the treated areas. Chronic radiation injury appears most often at 6-12 months following radiation and is secondary to a progressive fibrosis of the microvasculature (i.e., obliterative arteritis). This endothelial thickening in the small blood vessels eventually leads to thrombosis or reduced flow leading to ischemia. Clinically, it may manifest as nonhealing ulcerations and telangiectasias of the bowel wall, fistulas, and sepsis, as well as more common findings of thickening, stricture, and obstruction. Additionally, there are reported higher rates of secondary colorectal cancer. Answer A: Radiation therapy causes changes via two major mechanisms: direct damage to DNA and production of oxygen-free radicals. Answer B: Rapidly dividing crypt cells are most sensitive to radiation damage, and patients will experience atrophy of the villi causing degeneration of the mucosal lining. Answer C: Chronic radiation injury appears most often at 6-12 months following radiation and is secondary to a progressive fibrosis of the microvasculature (i.e., obliterative arteritis). Answer E: External beam radiation therapy is the method of choice for delivery in patients with rectal cancer. Brachytherapy is generally used for patients with prostate cancer. Bottom Line: Chronic radiation injury is secondary to a progressive fibrosis of the microvasculature (i.e., obliterative arteritis). This endothelial thickening in the small blood vessels eventually leads to a wide spectrum of findings including nonhealing ulcerations and telangiectasias of the bowel wall, fistulas, and sepsis.

Hemophilia A

Recombinant human factor VIII, fresh frozen plasma, desmopressin, or cryoprecipitate are all acceptable methods of replenishing factor VIII. Purified factor products should be used whenever possible to avoid potential transfusion reactions and transfusion-transmitted infection.

Rectal cancer

Rectal cancer is staged with the same tumor, nodes, metastases (TNM) classification system as colon cancer, which takes into account the tumor depth, nodal status, and evidence of metastatic lesions. Rectal cancer is more responsive to radiation therapy than colon cancer and, therefore, neoadjuvant therapy is often employed for stage II and above rectal cancers. Some rectal cancers can achieve a complete response to neoadjuvant therapy, but most will still require a surgical resection for cure. Answer A: A synchronous lesion may represent a second primary and is not necessarily metastatic disease. Answer B: Stage II rectal cancers are candidates for neoadjuvant radiation therapy, whereas surgical resection alone is the treatment for equal stage colon cancer. Answer C: If the tumor migrates through the muscle and into the perirectal tissue it is a T3 lesion. Answer E: Stage I rectal cancers can be treated with surgical resection alone. Bottom Line: Colon and rectal cancers are both staged with the same tumor, nodes, metastases (TNM) classification system. TrueLearn Insight : Tumor, nodes, metastases (TNM) classifications are not commonly tested on the exam, with the exception of colon and rectal cancer. Be sure to know the TNM classes and the stages for these cancers.

Rectal injuries

Rectal injuries are typically divided into intra or extra peritoneal. Intraperitoneal injuries present in similar fashion as colonic injuries, with radiological findings or peritonitis. They can also be diagnosed intra-operatively. Extraperitoneal injuries are more difficult to diagnose due to the lack of peritoneal signs. Diagnosis is based on high index of suspicion, digital rectal exam, rigid proctosigmoidoscopy, and CT scan (with or without rectal contrast). The diagnostic accuracy of the digital rectal exam and rigid proctosigmoidoscopy ranges from 80% to 95% but false negative rates can be as high as 31%. Another study that can be used is the gastrografin enema. Answer B: CT scan should be done but an initial evaluation should include a rectal examination to evaluate for the presence of blood Answer C: Rigid proctosigmoidoscopy can be done but an initial evaluation should include a rectal examination to evaluate for the presence of blood Answer D: A gastrografin enema study could be done but an initial evaluation should include a rectal examination to evaluate for the presence of blood Answer E: Rectal ultrasound maybe obscured by edema and inflammation in an acute rectal injury. The utility of rectal ultrasound in traumatic injuries is unclear. Bottom Line: Diagnosis of rectal injuries can be made using multiple modalities including digital rectal examination, CT scan, rigid proctosigmoidoscopy, and gastrografin enema study. A high index of suspicion must be maintained.

Rectus sheath hematoma

Rectus sheath hematomas are a rare entity. They are divided into three subtypes. Type I rectus sheath hematomas are small and confined within the rectus muscle. It does not cross the midline or dissect fascial planes. Type II rectus sheath hematomas are also confined within the rectus muscle but can dissect along the transversalis fascial plane or cross the midline. Type III rectus sheath hematomas are large, usually below the arcuate line, and often presents with evidence of hemoperitoneum and/or blood within the prevesical space of Retzius. Type I rectus sheath hematomas are treated conservatively, which includes bedrest, analgesia, compression of the hematoma, and reversal of anticoagulation when appropriate. Type II and III rectus sheath hematomas can be treated conservatively if the patient is hemodynamically stable. However, if the patient is not hemodynamically stable, Intervention is warranted, which includes angioembolization or surgical management (a midline incision with entrance into the posterior rectus sheath, facilitating identification of the bleeding vessel). In this case, the patient was only transiently responsive to resuscitation and so warrants intervention.

Inguinal hernia repair

Recurrence has always been a problem for the hernia surgeon. Historically recurrence was extremely high but with modern techniques of tension free with mesh that number has dwindled down to 1-4%. That is markedly improved but still leaves a small portion of the population with recurrence and mesh in place. If the reason for the recurrence is infection the answer is to remove the mesh. But in most cases the mesh should be left in place and a new mesh placed. The repair should be undertaken in a new clean plane to avoid nerve injury from previous scarring. Answer A: Although smoking increases the risks of wound infection and chronic cough increases the risk of another recurrence, this should not preclude the patient from treatment of a recurrent hernia. While it is not unreasonable to watch asymptomatic hernias, this patient reports increasing pain at the hernia site. Answer B: Current guidelines provide Level IA evidence that for recurrent hernias after conventional open repair, endoscopic inguinal hernia techniques result in less postoperative pain, faster reconvalescence and less chronic pain than the Lichtenstein technique. Answer C: This is also an option; however, recent meta-analyses have demonstrated that patients who underwent an open repair, after recurrence, experience greater post-operative pain and have a higher incidence of chronic pain. Level IA evidence now supports endoscopic approach to recurrent inguinal hernia that was performed in an open fashion. Answer D: There is no reason to remove a previous mesh if it is not infected. Answer E: The repair can be approached with mesh as this type of repair has the lowest recurrence rates. Bottom Line: For recurrent hernias after conventional open repair, endoscopic inguinal hernia techniques result in less postoperative pain, faster reconvalescence and less chronic pain than the Lichtenstein technique.

Liver transplant rejection

Rejection of a liver transplantation may occur in 20-50% of patients. Mild rejection is often characterized by failure to normalize or elevation of serum transaminases and/or bilirubin levels. Histologic examination shows a mixed portal cellular infiltrate with bile duct epithelial injury and endotheliitis of the central vein. The target for most cases of rejection is the biliary system. If left untreated the bile ducts can be destroyed and may disappear (vanishing bile duct syndrome). Early diagnosis is beneficial for therapeutic success. Treatment of mild rejection involves increasing immunosuppressant levels or pulse dose corticosteroids. If unsuccessful, administration of antilymphocyte preparation is the next step. Answer A: Increasing calcineurin inhibitor dosing beyond normal therapeutic levels is not the next step in managing rejection. The symptoms of shaking and diarrhea could be side effects of the calcineurin inhibitors, so increasing the dosage would not be recommended. Answer C: ERCP and biliary stent placement would be appropriate management for a bile duct stenosis. Answer D: CMV disease can present with a variety of symptoms, the most common of which are fever, leukopenia, thrombocytopenia, malaise, and arthralgias. Infection typically occurs one to six months post transplant. Answer E: Thymoglobulin (anti-thymocyte globulin therapy) is not the next step in treating rejection. If he fails corticosteroids, or develops severe rejection, it can be considered. Bottom Line: Initial management of mild hepatic rejection is elevation of immunosuppressant levels or pulse dose corticosteroid administration.

Gallstone ileus management

Relieving the obstruction by removing the gallstone through a proximal enterotomy is the initial management of gallstone ileus; the stone is "milked" proximally and then removed from a healthy portion of bowel. Care should be given to examine the area of the impacted stone. If there has been ischemic compromise to that section of bowel, this portion should be resected to prevent postimpaction wall necrosis and leak. Thorough evaluation for other gallstones should be performed because 10% of patients will have recurrent obstruction. Takedown of the biliary-enteric fistula and cholecystectomy during the same procedure is warranted because recurrent cholecystitis and cholangitis are common; however, in patients with a significant inflammatory process in the right upper quadrant or who are unstable to withstand a prolonged operative procedure, the fistula can be addressed at a second laparotomy.

Sunitinib for RCC

Renal cell carcinoma (RCC) most often originates in the lining of the tubules of the kidneys and is a very vascular tumor. RCC makes up 90% of all kidney cancers and 3% of all adult cancers. Advanced renal cell carcinoma is the term used to describe either locally advanced tumors or those that have spread to the lymph nodes. These are stage II tumors. Metastatic renal cell carcinoma refers to spread beyond the lymph nodes to other regions of the body. These tumors are stage IV. There is no treatment available to offer a cure for advanced or metastatic RCC. Thus, the primary treatment objectives are symptomatic relief and maintenance of patient function and performance status. Renal cell carcinoma is mostly resistant to chemotherapy, radiation therapy, and hormonal therapy. Treatment options thus include systemic therapy with either targeted therapy agents or immunotherapy agents. Targeted therapy is a broad class of drugs which in the case of RCC treatment includes tyrosine kinase inhibitors (sunitinib, pazopanib, axitinib, sorafenib, cabozantinib, and lenvatinib), vascular endothelial growth factor inhibitor (bevacizumab), mTOR inhibitors (everolimus, temsirolimbus). Immunotherapy agents include high-dose IL-2, interferon alpha-2a (INF-a) and anti-PD1 drugs (such as nivolumab). Patients receiving immunotherapy for advanced RCC have a median survival of 11.4 months (vs. 7.6 months in control group). Sunitinib is the first-line treatment for metastatic RCC in patients who are suitable for therapy and have an ECOG performance status of 0 or 1. Answer A: Targeted treatment agents, such as vascular endothelial growth factor receptor tyrosine kinase inhibitors and mTOR inhibitors have replaced nonspecific immunotherapy with cytokines, such as high-dose IL-2. Targeted therapy has been shown to improve progression-free survival, tumor response, and overall survival compared to immunotherapy. Answer B: Targeted treatment agents, such as vascular endothelial growth factor (VEGF) receptor tyrosine kinase inhibitors and mTOR inhibitors have replaced nonspecific immunotherapy. Targeted therapy has been shown to improve progression-free survival, tumor response, and overall survival compared to immunotherapy. Nivolumab is a programmed cell death 1 (PD1) checkpoint inhibitor. It has been shown to improve overall survival benefit compared to everolimus following failure of either one or two VEGF receptor-targeted therapies. Thus, anti-PD1 agents, such as nivolumab, are useful for patients who previously have been treated with a VEGF receptor-inhibiting agent. Answer C: Everolimus is useful for patients who have failed therapy with a tyrosine kinase inhibitor. Answer E: mTOR inhibitors are used as second-line therapy. Bottom Line: Targeted therapy has replaced cytokine therapy as the gold standard treatment for metastatic renal cell carcinoma. First-line treatment options are sunitinib and panzopanib. Second-line treatment options are TKIs or mTOR inhibitors. Third-line options include everolimus and sorafenib. TrueLearn Insight : It is important to know the treatment options for renal cell carcinoma, including surgical options in early stage cancers and systemic treatment options for advanced or metastatic cancers.

Renal cell carcinoma

Renal cell carcinoma is optimally treated with a radical nephrectomy. If the tumor grows into the inferior vena cava (IVC), it is recommended to pull the mass out of the IVC and continue with the radical nephrectomy. Renal cell carcinoma is poorly responsive to chemoradiation therapy.

Uremia platelet dysfunction

Renal disease and uremia cause a reversible bleeding disorder related to platelet dysfunction. There is a decrease in aggregation and adhesiveness of platelets and levels of platelet factor II, which results in a prolonged bleeding time. Treatments for uremic bleeding target the various factors that seem to have a role in platelet dysfunction. Therefore, platelet transfusion is not considered useful because transfused platelets are also affected by the uremia. Answer A: Estrogen is commonly used for hormone replacement therapy, but it also has a unique place in the treatment of uremic bleeding. Conjugated estrogen can shorten the bleeding time in uremic patients. It has been postulated that the hormones decrease production of l-arginine, a precursor of nitric oxide, which then aids in forming a platelet plug. Conjugated testosterone does not have the same effect as estrogen. Answer B: The first factor contributing to uremic bleeding is dysfunctional von Willebrand factor (vWF). Cryoprecipitate is a blood product rich in factor VIII, vWF, and fibrinogen. Fresh frozen plasma does not contain vWF and therefore is not useful in this patient. Answer D: Patients with chronic renal failure commonly experience anemia resulting from decreased erythropoietin (EPO) production and reduced longevity of red blood cells. The deficiency of circulating red blood cells causes platelets to travel in a more midstream position, further away from the subendothelium, making it less likely that platelets will react when damage to the vasculature occurs. EPO could be considered for prevention as well as for treatment. Erythrocytes carry oxygen and do not affect the clotting cascade. Answer E: Platelet transfusion will not aid in the qualitative platelet dysfunction seen in uremic patients. Bottom Line: Abnormal bleeding times and coagulopathy in patients with renal failure may be reversed with desmopressin, cryoprecipitate, conjugated estrogens, erythropoietin, and blood transfusions.

Hepatic adenoma

Resection of these masses to prevent possible malignant degeneration is true for hepatic adenoma.

Resuscitation with normal saline

Resuscitation with normal saline is associated with non-anion gap hyperchloremic metabolic acidosis due to accumulation of excess chloride in relation to sodium with a decrease in bicarbonate. Anion gap = Na-(Cl + HCO3). In this condition, there is excess Cl, decrease in HCO3, and normal anion gap.

Central Line Placement

Retrospective review has shown that there are fundamental differences between line placement locations. The risk of infection with any line is present, however, subclavian lines have been shown to be the least likely to developed a CRBSI. Femoral vein access has been shown to predispose to DVT formation, and are higher risk for infection in obese patients. Answer A: Subclavian vein access has been shown to decrease the risk of CRBSI when compared to internal jugular access. Answer B: Femoral catheters should be avoided when feasible as they are associated not only with a higher colonization and infection rate, but also of a higher risk for development of deep venous thrombosis. When an emergency demands such a line, it should be changed to an upper extremity line as soon as realistically feasible.

Rocuronium

Rocuronium has a rapid onset of action and intermediate duration of action, which make it useful for short procedures, and it is eliminated by the liver.

Potassium replacement

Roughly each 10 meq of potassium will raise serum potassium by 0.1 meq/L. To calculate the amount of potassium needed for replacement, we use the equation ((target potassium level-current potassium level)/serum creatinine)*100. In this scenario, we need ((3.5-2.9)/1)*100=60 meq of potassium. The deficit is preferably replaced orally. If the patient cannot take per oral or has serum potassium < 3.0, potassium should be replaced intravenously (IV). Maximum rate of IV replacement is 20-40 meq/hour. Cardiac monitoring is advised with infusion at such high rates.

What is the rule of 6's for av fistula formation

Rule of 6's: For a fistula to be considered successful, it must be usable. In general, a working fistula must have all the following characteristics: 1. Blood flow adequate to support dialysis (greater than 600 mL/min) 2. Diameter greater than 0.6 cm in a location accessible for cannulation and discernible margins to allow for repetitive cannulation 3. Depth of approximately 0.6 cm (between 0.5-1.0 cm from skin surface)

Breast screening

Screening for breast cancer is rapidly evolving. Recommendations vary based upon which guidelines are followed. Based upon the National Comprehensive Cancer Network guidelines, as of January 2015, average risk women (no history of breast cancer, low Gail score, no history of thoracic radiation, overall low lifetime risk) should start screening mammography annually starting at age 40. Clinical breast exams should be performed annually. Breast awareness should also be emphasized. For average risk women aged 25-40, clinical breast exam should be performed every 1-3 years.

Gastrinoma

Secretin is released by acid in the duodenum and stimulates pancreatic fluid and bicarbonate secretion, leading to neutralization of acidic chyme in the intestine. Secretin also inhibits gastric acid secretion and intestinal motility. In physiologic concentrations, secretin normally inhibits gastrin release, gastric acid secretion, and gastric motility. The most common clinical application of secretin is in the diagnosis of gastrin-secreting tumors. The secretin stimulation test is the most sensitive and specific provocative test for gastrinoma and aids in the differentiation between gastrinomas and other causes of ulcerogenic hypergastrinemia. An increase in serum gastrin of greater than 200 pg/mL above basal levels is specific for gastrinoma

Semi-lunar hernia (Spigelian)

Semilunar hernias, also known as Spigelian hernias, occur at the lateral edge of the rectus abdominis muscle at the semilunar line. Most semilunar hernias occur at the intersection of the arcuate line and the semilunar line. The arcuate line is the inferior edge of the posterior rectus sheath. Most semilunar hernias are spontaneous, but the placement of a laparoscopic trocar or drain through the semilunar line can increase the risk of developing a hernia. Many have small orifices and pose a significant risk of strangulation. A mass may not be palpable because the hernia sac may not penetrate all layers of the anterior abdominal wall. All semilunar line hernias should be repaired. Open and laparoscopic repairs have been described. Answer A: A Richter's hernia is characterized by its contents rather than by its location. A Richter's hernia contains the anti-mesenteric side of the bowel wall and comes with a high risk of strangulation. Answer C: Indirect inguinal hernias pass through the internal inguinal ring into the inguinal canal then through the external inguinal ring. Answer D: Ventral hernia is a general term for any hernia of the anterior abdominal wall. Though a semilunar hernia is a type of ventral hernia, choice B is more specific and therefore a better answer. Answer E: Obturator hernias pass along the course of the obturator nerve through the obturator canal. Most occur in elderly women. Pain may be present on internal rotation of the thigh. Repair is indicated for all obturator hernias. Bottom Line: Semilunar or Spigelian hernias occur at the lateral edge of the rectus abdominis and most occur at the level of the arcuate line. TrueLearn Insight : All semilunar line hernias should be repaired

SLNB melanoma staging

Sentinel lymph node biopsy (SLNB) is important in the staging and management of malignant melanoma. Suspected lesions should undergo an excisional biopsy with a generous amount of tissue to accurately stage the depth of the lesion. If the lesion is between 0-0.75 mm no lymph node biopsy is indicated. If it is between 0.76 cm and 1 mm, has no ulceration and no mitoses, a sentinel lymph node biopsy could be preformed if the patient is a good surgical candidate and after a lengthy discussion of the risks and benefits with the patient. This lesion is considered Stage 1A. If there is depth 0.76-1 mm with mitosis, angiolymphatic invasion, or ulceration, a SLNB is recommended as this is a stage 1B lesion. Malignant melanoma with a depth of invasion greater than 0.75 mm should be considered for SLNBx when there are high risk features such as with mitosis, angiolymphatic invasion, or ulceration.

SLNB for melanoma

Sentinel lymph node biopsy should be discussed and offered to patients with melanomas Stage 1B and above.

Hemodynamic readings due to sepsis

Sepsis produces high output cardiac failure, which is demonstrated by an elevated cardiac index. The SVR is decreased due to toxins that produce vasodilation. This is reflected in a low systemic blood pressure. Central venous pressures should be low due to the loss of intravascular volume due to third spacing. SvO2 should be high because the tissues are unable to extract oxygen from the blood effectively. Please note norms: CVP: 2-6 mmHg SVR: (800-1200 dyn s/cm5 CI: 2.5-4 L/min/m2 SV02: 65-70% PCWP: 4-12 mmHg

Septic pelvic thrombophlebitis

Septic pelvic thrombophlebitis is a rare disorder that may be caused by venous seeding of microorganisms that may trigger thrombosis. It may occur in the first 2-4 days postpartum and present with signs and symptoms of fever, tachycardia, abdominal pain that radiates to the groin or upper abdomen, nausea, and vomiting. Physical exam may reveal a rope-like mass on the affected side. CT and MRI may show a thrombus in the large pelvic vessels. Response to a trial of systemic heparin therapy is the best diagnosis. Patients should be treated for a 7-10 day course with systemic heparin to double the partial thromboplastin time as well as administration of broad spectrum antibiotics for the duration of heparin therapy. Long-term anticoagulation should be considered if the patient develops extensive pelvic clot formation or pulmonary emboli. Failure to respond to this treatment may require surgical intervention. Ligation of the affected vessel may be curative. Extensive clot formation may require embolectomy. Abscess formation may require removal of the affected vessel and ipsilateral adnexa

Septic thrombophlebitis

Septic thrombophlebitis usually affects the upper extremities after insertion of IV needles/catheters or with IV drug abuse. However, venous thrombi can become secondarily infected during bacteremia or if contiguous with areas of infection. Examples of contiguous infection include pelvic septic thrombophlebitis developing after obstetric infections, internal jugular thrombophlebitis associated with severe pharyngitis and peritonsillar abscesses, and visceral thrombophlebitis stemming from appendicitis. S. aureus is the most common bacteria associated with this, but gram-negative and polymicrobial infections are also common when a contiguous infection is the cause. Symptoms include fevers and chills. The clinical diagnosis is apparent when peripheral veins in the extremities are involved. In addition to pain, swelling, and redness along the course of the vein, pus may be expressed at the site of catheter or with needle insertion. Answer A: S. aureus is the most common organism cultured from patients with peripheral septic thrombophlebitis. Answer B: Gram-negative and polymicrobial infections are common when a contiguous infection is the cause. Answer C: Venous thrombi can become secondarily infected during bacteremia or contiguity with areas of infection; however, this is not the most common source of infection in patients with septic thrombophlebitis. Answer D: Septic thrombophlebitis most commonly results from the insertion of intravenous catheters and needles or intravenous drug abuse. Bottom Line: Septic thrombophlebitis most commonly results from the insertion of intravenous catheters and needles or intravenous drug abuse. However, venous thrombi can become secondarily infected during bacteremia or if contiguous to areas of infection.

Seroma after hernia repair laparoscopically

Seroma formation after laparoscopic hernia repair may occur into the dead space left from the hernia and dissection. Drains may be placed in this cavity at the time of initial operation, but seroma formation may still occur when the drain is removed. This condition is typically self-limiting and resolves as the mesh incorporates into the defect. Aspiration should not be done unless the seroma is symptomatic or lasts for longer than 6-8 weeks due to the risk of mesh infection.

Intra-operative hypothermia protects against cerebral ischemia but has other negative effects. what are they?

Several randomized trials have demonstrated that preservation of normothermia by infusion of fluids, heated to body temperature and using an upper-body forced-air heating cover, reduces wound infections, cardiac complications and bleeding, and transfusion requirements. Prevention of intraoperative hypothermia reduces the severity of the endocrine-metabolic response and sympathetic reflexes, and changes the fibrinolytic-coagulatory balance, resulting in reduced bleeding. Answer A: Intraoperative hypothermia is implicated in increased wound infections. Answer B: A decrease in metabolism and renal excretion causes slower clearance of anesthetic agents, resulting in a slower emergence from anesthesia. Answer D: Shivering during the recovery period will cause an increase in metabolic stress and worsen postoperative pain. Answer E: A decrease of 2-3 degrees in core body temperature may protect against cerebral ischemia. Bottom Line: Intraoperative hypothermia may result in coagulopathy and bleeding, increased postoperative pain, increased wound infections, slower clearance of anesthetic agents, and protection against cerebral ischemia.

Hypokalemia

Severe hypokalemia, with serum potassium concentrations of 2.5-3 meq/l (Nl: 3.5-5.0 meq/l), may cause muscle weakness, myalgia, tremor, and muscle cramps (owing to disturbed function of skeletal muscle), and constipation.

Shock

Shock is a general term used to characterize end organ malperfusion. There are multiple causes of shock, one of which is hypovolemia. The major physiologic derangement in hypovolemic shock (i.e., shock secondary to loss of blood volume) is a left ventricular end-diastolic volume (LVEDV) that is so small that the heart cannot produce adequate cardiac output. Causes include bleeding, protracted vomiting or diarrhea, fluid sequestration in obstructed gut or injured tissue, excessive use of diuretics, adrenal insufficiency, diabetes insipidus, and dehydration. In all cases of hypovolemic shock, the cardiac output will be low. In mild cases, the pressure may be normal, depending on the degree of compensatory arteriolar constriction, but the product of the cardiac output and the pressure will be low. In severe cases, the mean arterial pressure (MAP) will be low, and the product of the output and the pressure will be very low. Answer A: Chronic renal failure can be associated with pericardial effusion and cardiac tamponade. Beck's triad (Jugular venous distension, muffled heart sounds and pulsus paradoxus) does not exist in all cases. Right ventricular diastolic collapse is a sign of tamponade that can be seen on echo. Shock due to cardiac tamponade is known as obstructive shock. Answer B: Acute cholangitis can be associated with septic shock due to bacterial translocation from the biliary tree into the blood stream. Answer D: Neurogenic shock secondary to a spinal cord injury can result from disruption of sympathetic discharge that results in hypotension (due to peripheral vasodilation) and bradycardia (due to interruption of sympathetic discharge to the heart). Answer E: Pulmonary embolism can be associated with obstructive shock. Bottom Line: Hypovolemic shock can be associated with bleeding (hemorrhagic shock) or other causes of decreased intravascular volume including recurrent vomiting and diarrhea, fluid sequestration, adrenal insufficiency, diabetes insipidus, and dehydration.

Energy source for colonocytes?

Short chain fatty acids

Short bowel syndrome

Short-bowel syndrome results in patients with less than 200 cm of small intestine after resection. A competent ileocecal valve reduces the speed of small bowel transit, and aids in reabsorption of water, electrolytes and nutrients. The described lengths serve as general guidelines, and not as a strict anatomic definition of short-bowel syndrome. Gambee suture is an interrupted single layer suture that causes inversion of mucosa during small bowel anastomosis.

Hyponatremia

Signs and symptoms of hyponatremia include nausea and vomiting, headache, short-term memory loss, confusion, lethargy, fatigue, loss of appetite, irritability, muscle weakness, spasms or cramps, seizures, and decreased consciousness or coma.

Which burn therapy causes transient neutropenia?

Silver sulfadiazine

Post Transplant Tumors

Skin cancers are the most common malignancies following organ transplantation. They are usually located on sun-exposed areas. Squamous and basal cell carcinomas are the most frequent types. These malignancies have a higher likelihood to metastasize in this patient population. Treatment is the same as for non-transplant patients. Other malignancies are found to occur at increased rates in transplant recipients compared to the general population.

Skin grafts

Skin grafts survive in the recipient site by the process of imbibition, inosculation, and then angiogenesis. The four main mechanisms of failure are a poor wound bed vascularity, sheer forces, hematoma or seroma, and infection.

Skin infections from human bites

Skin infections from human bites are generally polymicrobial containing anaerobes and aerobes. Anaerobic organisms are similar to that of animal bites except that Bacteroides is more common in human bites. These organisms more commonly produce beta-lactamases in human carriers. Common aerobic organisms include staph and strep species, corynebacteria, and E. corrodens. Corynebacterium is a pleomorphic (club-shaped) gram positive rod commonly found in human bites. Treatment of a human bite includes proper irrigation with iodine solution, debridement if necessary, antimicrobial coverage, and tetanus booster. Vaccination for corynebacteria is included in the tetanus vaccination. Amoxicillin/clavulanate is the most appropriate antibiotic choice for non-complicated human bites. If the patient demonstrates signs of severe cellulitis, systemic infection, diabetes, infection refractory to oral antibiotic regimen, or significant damage to the hand including bone, joint, or tendon involvement, the patient should be admitted for further management and IV antibiotics. IV antibiotic therapy includes cefoxitin, cefotetan, or piperacillin-tazobactam. Doxycycline plus clindamycin or flagyl is the antibiotic regimen of choice for human bites in patients with a penicillin allergy.

Sleeve gastrectomy

Sleeve gastrectomy is quickly becoming one of the most popular surgeries for morbid obesity. Its relative ease and large amount of excessive weight loss has increased its popularity dramatically. The procedure is done by establishing pneumoperitoneum, entering the lesser sac 5-6 cm from the pylorus, removing the greater omentum, passing a bougie, and using a linear stapler to remove the excessive fundus. It you make the antrum too small, the patient will experience nausea and PO intolerance.

Laparoscopic roux-en-y gastric bypass

Small bowel obstruction in a patient that has undergone a LRYGBP must be managed very differently than a SBO in a general surgery patient. In the general surgery patients the most common cause is remembered by the mnemonic ABC (adhesion, bulge [hernia], Cancer). After LRYGBP the most common cause is internal hernia from inadequate closure of the mesenteric defect. This should be managed with emergent surgery. Even though the patient appears relatively stable the condition can rapidly deteriorate and cause infarction of the entire small bowel necessitating small bowel transplant if the patient survives.

Small bowel adenomas

Small-bowel adenomas are histologically classified as tubular, tubulovillous, or villous. Adenomas are found predominantly in the duodenum, with the majority found in the periampullary region. Approximately 25% of these villous and tubulovillous adenomas harbor malignancy; therefore, it is critical to identify and resect these tumors when they are identified. Adenomas in the duodenum present with obstructive jaundice or small-bowel obstruction in about one-third of the patients. Adenomas typically appear as small intraluminal filling defects. When patients undergo contrast series prior to endoscopy, a "soap bubble" or "paint brush" sign is seen. This is considered pathognomonic for villous adenoma.

Strongest risk factor for development of abdominal aortic aneurysm?

Smoking

Ulcerative colitis

Smoking is protective ironically

Solitary rectal ulcer syndrome

Solitary rectal ulcer syndrome (SRUS) is characterized by rectal bleeding, copious mucus discharge, anorectal pain, and difficulty with passing stool. Reducing symptoms and prevention are focused on etiological mechanisms. Conservative therapy (high-fiber diet, lifestyle changes, and biofeedback) will reduce symptoms in most patients and should be tried first. Patients without rectal intussusception are offered biofeedback to retrain their bowel function. Pharmacologic therapy (anti-inflammatory enemas or suppositories) has had limited success but is reasonable to try before embarking on surgery. Localized resection can be considered in some select patients. Patients with symptoms of prolapse are candidates for perineal procedures (mucosal or perineal proctectomy) and abdominal procedures (fixation or resection and rectopexy). Patients without prolapse may be offered excision, which varies from a transanal excision to a major resection with coloanal pull-through. Surgeons have been understandably hesitant to offer surgical therapy for this benign condition, and the results are often unsatisfactory. Answer A: Despite its name, with this condition, patients can have single, multiple, or no rectal ulcers. Answer B: The differential diagnosis of SRUS includes polyps, endometriosis, inflammatory granulomas, infectious disorders, drug-induced colitides, or mucus-producing adenocarcinoma. Differentiation among these entities is possible with an adequate biopsy. Answer C: When present, solitary rectal ulcers usually occur on the anterior rectal wall just above the anorectal ring. Less commonly, they may occur from just above to 15 cm above the dentate line. Answer D: The etiology of this condition remains unclear, but a common feature is chronic inflammation and/or trauma (internal intussusception or prolapse of the rectum, direct digital trauma, or the forces associated with evacuating a hard stool). Bottom Line: The initial treatment of solitary rectal ulcer syndrome should be directed toward controlling symptoms and treating the underlying cause. Surgery may be indicated in selected patients with persistent symptoms.

Somatostatin

Somatostatin is a 14 amino acid polypeptide weighing approximately 1.6 kDa. It is secreted by D cells in the stomach, duodenum, and pancreatic islets. It is known as the "great inhibitor" because its actions are largely inhibitory. Exogenous somatostatin has been demonstrated to inhibit the release of insulin, glucagon, and PP resulting in decreased gastric, pancreatic, and biliary secretion. Answer A: These actions are generated by paracrine effects on parietal and G cells. Answers B & C: Its actions include inhibition of gastrin and bicarbonate secretion. Answer E: Somatostatin is released from D cells located in the antrum in response to an acidic environment.

Somatostatin

Somatostatin is an inhibitory peptide found both in the hypothalamus as well as the pancreatic delta cells. It acts to suppress both endocrine release as well as exocrine secretions.

Pancreatic enzymes

Some of the pancreatic digestive enzymes are synthesized and secreted in their active forms without the need for an activation step (e.g., amylase, lipase, ribonuclease). Lipase does require colipase to function properly. Answers B & C & D: The endopeptidases, which include trypsin, chymotrypsin, and elastase act on peptide bonds at the interior of the protein molecule, producing peptides that are substrates for the exopeptidases (carboxypeptidases), which serially remove a single amino acid from the carboxyl end of the peptide. Answer E: Most of the digestive enzymes are synthesized and secreted by acinar cells as inactive proenzymes or zymogens that, in health, are activated only after they reach the duodenum where enterokinase activates trypsinogen and the trypsin catalyses the activation of the other zymogens.

Sorafenib

Sorafenib is the only known effective systemic medical therapy for patients with HCC. It is a multikinase inhibitor with activity against the vascular endothelial growth factor (VEGF) receptor II, c-kit receptor, platelet-derived growth factor (PDGF) receptor, and kinases in the b-Raf/Ras/mitogen-activated protein kinase kinase (MAPKK) pathway. Sorafenib has been studied in the phase III randomized controlled (SHARP) trial. Sorafenib has been shown to improve survival among patients with HCC who are not eligible for other therapies when compared to placebo

Rectus sheath hematoma

Specific treatment of a rectus sheath hematoma depends on the hemorrhage severity. Stable, unilateral hematomas may be observed without intervention. If small, the hematoma can even be observed without hospitalization. If there is concern for hematoma enlargement, active bleeding, or clinical deterioration, then first line therapy includes resuscitation and angiographic embolization. Hemoglobin and hematocrit should be monitored with transfusion as needed. Coagulation factor replacements should be considered with patients taking vitamin K antagonists. Surgical intervention and wound exploration is rare.

Preferred med for bp in patients with aldosteronoma?

Spirinolactone

Splenic abscess

Splenic abscess is most commonly the result of hematogenous spread from another active infection in the body, including endocarditis or osteomyelitis. It may also be secondary to an immunocompromised state, such as in AIDS. The implicating organisms can range from Gram-positive cocci to Gram-negative rods. The treatment of a splenic abscess is immediate drainage. If the collection is unilocular, this treatment can be achieved through a CT-guided drainage procedure. However, if the collection is complex and multi-loculated, splenectomy is the recommended treatment.

Meshing skin grafts

Split-thickness skin grafts can be enlarged up to six times the original size. Enlargement can be made with few manually made perforations with a 11 blade to systematic enlargement with hand-powered meshing device that makes slits at regular intervals. The ratio can vary from 1: 1.5 to 1:9. Meshing increases the surface area that can be covered, adapts to areas with irregular contours and increases the edges of reepithelialization. Meshing heals with secondary intention between the gaps leading to uneven appearance of the graft and poor aesthetic outcome. Answer B: Meshed grafts heal by secondary intention between the interstices, and the cosmetic outcome is inferior to full-thickness skin grafts. Answer C: Meshing allows the fluid to escape between the interstices and decreases the hematoma formation. Answer D: Meshing is used in large burns when the wound area exceeds the availability of healthy donor sites. Most commonly used mesh ratio is 1:1.5 in smaller wounds while 1:3 and 1:6 are used in larger wounds. Answer E: Aesthetically unpleasant growth of hypergranulation tissue is seen between the mesh gaps . It can be unsightly and cause contracture with poor functional outcomes in large mesh ratios. Bottom Line: Meshing allows drainage of fluid thus reducing the risk of seromas. In the first few days before the graft revascularized, the graft is nourished by oxygen and nutrients diffusing from plasma between the graft and wound bed.

SBP

Spontaneous bacterial peritonitis (SBP) is a potentially lethal complication of portal hypertension with ascites. The cause of spontaneous bacterial peritonitis is unknown. The vast majority of SBP are caused by a single organism, most commonly Gram-negative enteric bacteria, including Escherichia coli. Hematogenous spread may lead to infection with Streptococcus pneumoniae. If multiple organisms are present, the diagnosis of spontaneous bacterial peritonitis must be questioned, and a search for secondary peritonitis, such as a perforated viscus or diverticulitis, should be performed.

Swallowing center

Spontaneous contractions of the esophagus usually do not occur. Esophageal peristalsis occurs with swallowing. It is believed that striated esophageal musculature is stimulated by the vagus nerve. The swallowing center coordinates movements of both the striated and smooth esophageal musculature. The swallowing center is located in the medulla.

Actinic keratosis

Squamous cell carcinoma is the second most common non-melanoma skin cancer. It often arises from precursor skin lesions such as actinic keratosis. The latter usually arises in sun-exposed areas and presents as multiple, irregular, scaly lesions which range in color from dark-brown to flesh colored. The presence of erythema, pain, induration, ulceration or change in size may suggest malignant transformation of actinic keratosis. On histopathology, actinic keratosis is characterized by dysplastic changes of the basal layer of the epidermis, associated with parakeratosis (retention of nuclei in the stratum corneum), and dermal solar elastosis (accumulation of elastin in the dermal tissues).

Thoracic outlet syndrome

Surgery is indicated if conservative therapy fails, or if arterial complications or venous thrombosis develops. Answer A: The treatment for thoracic outlet syndrome is physical therapy, and most patients improve with non-operative management. Answer C: The most common symptoms are neurologic, often in the ulnar nerve distribution. Answer D: Thoracic outlet syndrome is caused by compression of the subclavian artery or vein and brachial plexus by a cervical rib or the scalene muscles. Answer E: A diagnosis can be made by performing clinical tests to try and demonstrate the disease. An Adson test would show a loss of the radial pulse with turning the head toward the affected side.

Mesenteric cysts

Structural abnormalities of the mesenteric lymphatic system are thought to be the etiology of mesenteric cysts. These abnormal lymphatics fail to communicate with the remainder of the lymphatic system. If aspirated, these cysts contain fluid that is straw-colored and at times milky, following fat ingestion. The fluid is proteinaceous in nature. Approximately 60% of these cysts involve the small bowel mesentery, and another 25% involve the large bowel mesentery. The treatment of choice for mesenteric cysts is total cystectomy in order to decrease the risk of recurrence and possible malignant transformation. The high triglyceride content of lymph within a mesenteric cyst is characterized by a straw-colored or milky proteinaceous fluid

Anti-reflux therapy

Successful outcome after antireflux surgery is defined by two objectives: the achievement of long-term relief of reflux symptoms and the absence of complications or complaints after the operation. Factors that predict successful outcome following antireflux surgery are (i) the presence of typical symptoms of GERD, (ii) an abnormal score on 24-hour esophageal pH monitoring; and (iii.) symptomatic improvement in response to acid suppression therapy prior to surgery. Studies have shown that patients with typical symptoms, in comparison to those with atypical symptoms, have a better response to fundoplication. A 10-year follow-up study reported 85% of patients with typical symptoms had a successful outcome after Nissen fundoplication, compared to only 41% with atypical symptoms. Patients who experience exaggerated symptoms when supine rather than standing tend to have better outcomes after fundoplication as well. In the supine position, transient lower esophageal relaxation (TLSR) periods increase. Studies have shown that fundoplication reduces TLSR frequency by 50% and thus decrease reflux events

Succinylcholine

Succinylcholine (given as part of rapid sequence intubation) leads to hyperkalemia in spinal cord injured patients, as well as burn patients, and should be avoided. This leads to a wide complex tachycardia, torsade de pointes, and cardiac arrest. Succinylcholine is a ultrashort acting neuromuscular blocking agent (NMBA) with the most rapid onset of action. It is the only depolarizing NMBA in clinical use. The most lethal adverse effect of succinylcholine is hyperkalemia. Although a modest increase in the serum potassium level (0.5 mEq/L) occurs in most patients following the administration of succinylcholine, more profound hyperkalemia may occur in various comorbid features, including patients with burns, massive tissue trauma, upper and lower motor neuron lesions, and extensive skeletal muscle denervation. Extrajunctional receptors are synthesized in these states, and plasma levels of potassium can increase significantly following the administration of succinylcholine as activation of these receptors results in the opening of ion channels and the release of intracellular potassium.

Reversal agent for rocuronium and vecuronium?

Sugammadex is a reversal agent for the neuromuscular blockers rocuronium and vecuronium.

Suppurative thrombophlebitis

Suppurative thrombophlebitis is associated with use of Intravenous (IV) cannula. The clinical features include fever, leukocytosis, pain and pus at the IV cannula site. Aerobic, anaerobic and mixed infections are seen . Organisms associated include Staphylococcus aureus, Pseudomonas species, Klebsiella species, Enterococcus species, Fusobacterium species and also Candida. Treatment includes removal of the foreign body, administration of antibiotics and rarely excision of veins is needed. Mondor's disease is defined as the thrombophlebitis of a thoracoepigastric vein of the breast and chest wall. It is associated with breast cancer or a hypercoagulable state or sometimes with no identifiable cause. Answer A: This patient is likely to have Suppurative thrombophlebitis due to the presence of intravenous catheter. Answer C: Migratory thrombosis is characterized by repeated thrombosis in superficial veins at various sites but commonly in lower extremities. It is associated with cancer (Trousseau's syndrome) and sometimes precede the diagnosis by many years. Answer D: Superficial thrombophlebitis is characterized by erythema and tenderness along the superficial veins with thrombosis suspected by the palpable cord. Answer E: Buerger's disease or Thromboangiitis obliterans is a chronic, inflammatory, non atherosclerotic obliterative vasculitis involving the infrapopliteal and medium-sized and small arteries distal to the elbow. Its usually seen in young male smokers and cause claudication leading to gangrene. Migratory superficial thrombophlebitis is also reported during the course of the disease. Bottom Line: Suppurative thrombophlebitis is characterized by fever, leukocytosis, intense pain and pus draining from the intravenous(IV) cannula site.

Supracondylar humerus fractures

Supracondylar distal humeral fractures can be associated with brachial arterial injuries and should prompt evaluation for compartment syndrome.

Surgery for Crohn's patients

Surgery for Crohn's patients should only be considered for intractable disease or complications. Indications for surgical intervention include perforation, fistula, abscess, GI bleeding, and obstruction. Bowel preserving surgery is optimal to avoid complications of short gut syndrome, but strictureplasty procedures are difficult in the duodenum and are only anatomically achievable in select patients. Therefore, for duodenal obstructions, the indicated surgical treatment is bypass with a gastrojejunostomy, especially given the multiple strictures within a short segment. The contraindications for strictureplasty include excessive tension, perforation, fistula/abscess, hemorrhagic stricture, multiple strictures within a short segment, malnutrition, and malignancy. A stricturoplasty is an excellent intervention of jejunal and ileal strictures and helps save bowel length. The duodenal anatomy makes stricturoplasty difficult due to its lack of mobility and the close anatomic relationship with the pancreas and common bile duct. Furthermore, it is contraindicated in patients with multiple strictures within a short segment. Answer C: Both Finney and Heineke-Mikulicz strictureplasty are usually performed in jejunal and ileal strictures.

Puestow procedure

Surgical operations for patients with chronic pancreatitis are generally divided into 4 basic types: resection, drainage, combined resection and drainage, and denervation. Choosing the correct operation is based on the different underlying anatomic and morphologic features. The longitudinal pancreaticojejunostomy (Puestow procedure) is classically used to treat patients with large duct pancreatitis by decompressing the entire length of the pancreatic duct (head to tail) into a defunctionalized (Roux-en-Y) limb of jejunum. Answer A: Patients with no other surgical targets (i.e., minimal change pancreatitis) can be treated using total pancreatectomy. In those with high surgical risk, bilateral thoracoscopic splanchnicectomy can offer a good alternative for pain relief. Answer B: Pancreatic cancer is not an indication for pancreaticojejunostomy. Answer D: Patients with small duct and a large, hypertrophic pancreatic head are best treated with the Beger operation. However, due to technical difficulties associated with this procedure, most surgeons would perform pancreaticodoudenectomy instead. Answer E: Obstructive pancreatitis is characterized by the presence of a significant stricture of the main pancreatic duct with upstream (toward the spleen) changes of pancreatic duct dilatation. Patients with obstructive pancreatitis are best treated with distal pancreatectomy. Bottom Line: The Puestow procedure (longitudinal pancreaticojejunostomy) is typically indicated to treat patients with large duct pancreatitis.

Zenkers diverticulum

Surgical or endoscopic repair of a Zenker diverticulum is the gold standard of treatment. Traditionally, an open repair through the left neck was advocated; however, in recent years, endoscopic exclusion has gained popularity. Two types of open repair are performed: resection and pexy of the diverticulum. Both the diverticulectomy and the diverticulopexy are performed through an incision in the left neck. Answer B: The endoscopic Dohlman procedure method divides the distal cricopharyngeus muscle while obliterating the sac. The esophagus and diverticulum form a common channel. This technique requires maximal extension of the neck and can be difficult to perform in elderly patients with cervical stenosis. This technique has gained favor and is advocated in patients with diverticula between 2 and 5 cm. Answer C: For any diverticulum greater than 3 cm, the results are the same. Answer D: In all cases, a myotomy is performed of the proximal and distal thyropharyngeus and cricopharyngeus muscles. In cases of a small diverticulum (<2 cm), a myotomy alone is often sufficient. Answer E: The results of open repair vs. endoscopic repair have been well studied. For diverticula 3 cm or less in size, surgical repair is superior to endoscopic repair in eliminating symptoms. The risk for an incomplete myotomy increases with smaller diverticula less than 3 cm in size. Bottom Line: The treatment for a Zenker diverticulum is cricopharyngeal myotomy independent of size and diverticulectomy in those >2 cm.

aldosteronoma

Surgical removal of an aldosterone-secreting adenoma results in durable improvement of hypertension and hypokalemia in 70-90% of patients. Preoperative spironolactone or eplerenone and potassium are given to replenish potassium stores and correct alkalosis before anesthesia. Preoperatively, a significant fall in blood pressure with aldosterone receptor antagonists predicts a successful outcome after adrenalectomy. Response to adrenalectomy is also influenced by the duration and severity of hypertension and the presence of histologic changes in the kidney. Age greater than 50 years, male sex, and the presence of multiple nodules within the adrenal are also associated with a poor response to surgery. Answer A: Laparoscopic adrenalectomy is the preferred approach to remove these tumors. Morbidity and mortality following these procedures are almost negligible. Answers C & D: Medical treatment is only appropriate for patients with bilateral adrenal hyperplasia because fewer than 20-30% of patients with this disease are cured by adrenalectomy. Idiopathic adrenal hyperplasia is treated with spironolactone or with the newer aldosterone antagonist eplerenone. Other potassium-sparing diuretics may be used, including triamterene and amiloride. Treatment of glucocorticoid-suppressible hyperaldosteronism includes dexamethasone 0.5-1.0 mg daily. Glucocorticoids are used in small doses to avoid Cushing syndrome.

Adrenocortical carcinoma

Surgical resection is the standard of care treatment for all stages of adrenocortical carcinoma. Complete resection of locally confined tumor is the only chance for cure from adrenocortical carcinoma; however, distant or local spread is evident in 65% of cases at presentation, and a few patients are resectable with curative intent. Recurrence rates after surgery range from 38 to 85%, depending on the stage at presentation. The prognosis of adrenocortical carcinoma is poor. Median survival following diagnosis for all patients is approximately 18 months. Overall survival following resection for all stages of adrenocortical cancer is 15-47% at 5 years. Stage-specific 5-year survival is 30-45% for stage I, 12-57% for stage II, 5-18% for stage III, and 0% for stage IV disease. According to the TNM staging system (tumor size/node status/metastasis) of adrenocortical carcinoma, this patient's cancer is stage II (T2N0M0) as the tumor size is >5 cm and there is no fat invasion, lymph node involvement, or distant metastasis. Answer A: Stage I is T1N0M0. According to the TNM staging, T1 is a tumor <5 cm with no invasion. Answers C & D: Stage III is either T1-2, N1, M0, or T3N0M0. According to the TNM staging, N1 should have lymph node involvement, and T3 is a tumor with invasion to the fat. This patient has neither lymph node involvement nor periadrenal fat invasion. Answer E: Stage IV is either any T, any N, M1, or T3-4N1M0. According to the TNM staging, T4 is a tumor with organ invasion, and M1 should have distant metastasis. This patient has neither organ invasion nor distant metastasis. Bottom Line: Localized adrenocortical carcinoma (stages I and II) has no local invasion, lymph node involvement, organ involvement, or distant metastasis. The distinction between stage I and stage II is based on tumor size (<5 cm or >5 cm, respectively).

SIADH

Syndrome of inappropriate antidiuretic hormone (SIADH) is a condition in which there is an inability to suppress ADH excretion that results in impaired water excretion and thus a decrease in urine output and water retention. If water intake exceeds the decreased urine output, the water retained leads to the development of hyponatremia. The hypoosmolality seen in SIADH results in a shift of water from extracellular space to intracellular space, resulting in overhydration of the intracellular compartment. Common features seen in patients with SIADH include: Hyponatremia (the most common fluid and electrolyte abnormality) Hypoosmolality (SIADH is one of the most common causes of hypoosmolality) Increased total body water Decreased serum hematocrit Decreased serum protein concentration Elevated urine osmolarity (typically >100 mosmol/kg) Elevated urine sodium concentration (typically >40) In the setting of normal serum potassium, there is lack of an acid-base disturbance Decreased serum uric acid concentration Decreased serum urea levels (typically <30 mg/dL; seen mostly in young patients with SIADH as a result of high renal clearance; less specific in elderly patients who often have a lower clearance of urea) Lower anion gap with nearly normal CO2 and serum potassium (compared to salt-depleted patients) Elevated urine sodium (usually >30 mEq/L) Elevated fractional excretion of sodium (70% of patients with SIADH have fractional excretion of sodium >0.5%) Decreased serum creatinine concentration (can be seen in young patients with SIADH-induced hyponatremia)

Colon cancer staging

T = Tumor grade for the primary tumor. T1: Tumor invades submucosa, T2: Tumor invades muscularis propria, T3: Tumor invades through the muscularis propria into subserosa, T4a: Tumor penetrates to the surface of the visceral peritoneum, T4b: Tumor directly invades or is adherent to other organs or structures. N= Regional Lymph Node. N1: Metastasis in 1-3 regional lymph nodes (Subdivided in groups: N1a: Metastasis in one regional lymph node, N1b: Metastasis in 2-3 regional lymph nodes, N1c: Tumor deposits in the subserosa, mesentery, or non-peritonealized pericolic or perirectal tissues without regional nodal metastasis) N2: Metastasis in four or more regional lymph nodes (N2a: Metastasis in 4-6 regional lymph nodes, N2b: Metastasis in > 7 regional lymph nodes). M = Distant metastasis. M1: Distant metastasis (M1a: Metastasis confined to one organ or site (liver, lung, ovary, non-regional node), M1b: Metastases in more than on organ/site or the peritoneum

T-Tube s/p cholecystectomy

T-tubes are commonly inserted whenever there is a concern for retained gallstones after a CBD exploration to decompress the biliary tree. Usually a T-tube cholangiography is done 2 weeks post-operatively to evaluate any retained stones and for possible removal of the T-tube. A minimum of 4-6 weeks should pass before any instrumentation is performed to allow for the tract to mature.

Breslow staging

T1: ≤1mm T2: 1.01-2.0 mm T3: 2.01-4.0 mm T4: >4.0 mm

Breast cancer in males

Tamoxifen confers a survival benefit for males with hormone receptor positive breast cancer. Orchiectomy is a second line hormonal manipulation in males with estrogen receptor positive, metastatic breast cancer. In the above case orchiectomy is not indicated because the patient does not have evidence of metastatic disease

TIPS

TIPS has been compared to chronic endoscopic therapy in 11 randomized controlled trials. Fewer patients rebled after TIPS (19%) than after endoscopic treatment (47%), but encephalopathy was significantly more common in TIPS patients (34%). Angiographic and doppler ultrasound studies suggest that TIPS, when it effectively decompresses varices, is a nonselective shunt and completely diverts portal flow. Clinical evidence of the non selectivity of TIPS is its effectiveness in resolving medically intractable ascites, but with a fairly high frequency of post-TIPS encephalopathy (∼30%). Answer A: Shunt stenosis, which is usually secondary to neointimal hyperplasia, is more common than thrombosis and can often be resolved by balloon dilation of the TIPS or, in some cases, by placement of a second shunt. Both shunt stenosis and shunt thrombosis are often followed by recurrent portal hypertensive bleeding. Answer C: Because it functions as a side-to-side portosystemic shunt, TIPS is also effective in the treatment of medically intractable ascites. Answer D: Transjugular intrahepatic portosystemic shunt (TIPS) is being increasingly used as a definitive treatment for patients who bleed from portal hypertension. A major limitation of TIPS, however, is a high incidence (up to 50%) of shunt stenosis or shunt thrombosis within the first year. Answer E: The rate of post-TIPS encephalopathy is about 30% Bottom Line: Transjugular intrahepatic portosystemic shunt (TIPS) is ideally used for short-term portal decompression. It can be used as a bridge to liver transplant in appropriate patients. It does have a 30% encephalopathy rate associated with it.

TIPS

TIPS is indicated in the refractory bleeding patient. It allows decompression of the portal system by directing portal blood into the systemic system. This bypasses the liver and can place patients at higher risk for hepatic encephalopathy. In more elective cases, selective shunts isolate and decompress only a portion of the portal venous system: only the gastroesophageal junction, proximal stomach, and spleen. The most widely used selective shunt is the distal splenorenal shunt or Warren shunt. Portal hypertensive blood flow is maintained to the liver through the uninterrupted superior mesenteric and portal veins, and hepatic sinusoidal pressure remains elevated. It is for this reason that selective shunts are ineffective in treating ascites. The distal splenorenal shunt is constructed by anastomosing the distal end of the splenic vein from the preserved spleen to the side of the left renal vein. The right and left gastric and right epiploic veins and greater curve perforating branches to the gastroepiploic vein are ligated, but the short gastric veins are preserved to decompress the gastroesophageal junction through the shunt. Additionally, all pancreatic branches to the splenic vein from the splenic hilus to the portal vein are divided to prevent the late loss of selectivity of this shunt. Experienced surgeons who perform the distal splenorenal shunt can prevent variceal bleeding in more than 90% of patients. Answer A: This is not indicated in the bleeding patient. Answer B: This can be used in a more elective setting. Answer D: A partial portosystemic shunt is a type of side-to-side portocaval shunt. The shunt is inserted between the portal vein and the inferior vena cava. Shunt diameters of 8 mm usually reduce the portal pressure to less than 12 mm Hg and maintain most antegrade portal blood flow to the liver. Construction of a partial shunt is technically simpler than that of a distal splenorenal shunt. Although partial shunts are nonselective because they do not selectively decompress an isolated portion of the portal system, their efficacy in terms of reduction of variceal bleeding and the rate of encephalopathy is similar to that seen with a distal splenorenal shunt. Answer E: These shunts are very effective in controlling bleeding and ascites because the hepatic sinusoids are decompressed, but hepatic encephalopathy occurs in about 40% of patients followed long term. Moreover, these shunts may be associated with increased morbidity and intraoperative transfusion requirements in those patients who undergo liver transplantation. Bottom Line: TIPS is indicated in the refractory bleeding patient. It allows decompression of the portal system by directing portal blood into the systemic system.

MOA of tacrolimus

Tacrolimus (FK506) was released in 1994 for use in organ transplant immunosuppressant regimes. It has a similar mechanism of action to cyclosporine but is 100 times more potent. Tacrolimus specifically binds to the FK506-binding proteins which inhibit the enzyme calcineurin. Calcineurin is necessary for transcription factor activation in response to increased intracellular calcium after T-cell receptor stimulation. The effect of calcineurin inhibition is decreased IL-2 and other cytokine production which inhibits T-cell function.

Tetralogy of Fallot

Tetralogy of Fallot is associated with pulmonary stenosis, ventricular septal defect, overriding aorta with deviation of the origin to the right side and concentric right ventricular hypertrophy. The path of pulmonary blood flow may be impeded at multiple levels, including the infundibulum, the pulmonary valve and annulus, and the main and branch pulmonary arteries. Answer A: The murmur in patients with Tetralogy of Fallot is due to the right ventricular outflow obstruction, not the ventricular septal defect. Thus, the murmur is typically a crescendo-decrescendo with a harsh systolic ejection quality. It may be appreciated best over the left mid-upper sternal border. It radiates posteriorly. Answer B: Tetralogy of Fallot is associated with right atrial enlargement and right ventricular hypertrophy on EKG. Additional findings may include right axis deviation, prominent anterior R waves and posterior S waves, upright T wave in V1, and a qR pattern in the right chest leads. Answer D: Tetralogy of Fallot is associated with a ventricular septal defect, not atrial septal defect. Answer E: Pulmonary stenosis, rather than aortic stenosis, is seen with Tetralogy of Fallot. Bottom Line: Tetralogy of Fallot is characterized by an overriding aorta, ventricular septal defect, right ventricular outflow tract obstruction, and right ventricular hypertrophy.

Appendicitis

The "classic" presentation of acute appendicitis is uncommon in clinical practice; however, several physiologic explanations make it testable by ABSITE. The initial presentation is heralded by vague abdominal pain, often described as epigastric, due to visceral inflammation, which then progresses to nausea. The onset of nausea prior to pain is more consistent with gastroenteritis. As the inflammation increases the pain becomes localized to the location of the appendix due to somatic innervation. Symptomatology in this stage may help locate the appendix. A pelvic appendix is more likely to cause rectal irritation, resulting in loose stools and urinary symptoms from bladder irritation. Physical exam may also help determine the location of the appendix. The iliopsoas test (pain with extension of right thigh when lying on side) indicates a retrocecal appendix. The obturator test (pain with internal rotation of the thigh) indicates a pelvic appendix. Additional maneuvers are also classic for appendicitis, such as Rovsing's sign (pain in the right lower quadrant with deep palpation of the left lower quadrant). In many patients, history and physical exam alone are likely sufficient for diagnosis of appendicitis. In some, such as young women, imaging with ultrasound or CT scan may be helpful to narrow the differential diagnosis. Historically, appendicitis was considered a surgical emergency. New evidence suggests, however, that appendectomy may be safely delayed up to 24 hours without increased risk. Nausea usually comes first in gastroenteritis, whereas pain comes first in appendicitis. Answer D: The iliopsoas test (pain with extension of the right thigh when lying on the side) indicates a retrocecal appendix. The obturator test (pain with internal rotation of the thigh) indicates a pelvic appendix.

Adson's test

The Adson test (scalene test) is performed by having the patient maximally extend their neck and turn their head toward the effected extremity. The ipsilateral radial pulse is palpated, and the test is considered positive if the pulse decreases or disappears. Treatment involves physical therapy, neck stretching, avoidance of repetitive trauma and movements, and surgical decompression as a last resort.

Hepatocellular carcinoma staging

The American Joint Committee on Cancer classifies hepatocellular carcinoma based on size, vascular invasion, lymph node status, and metastasis. T1 lesions are any solitary tumor regardless of size with no vascular invasion. T2 lesions are solitary tumors with vascular invasion or multiple tumors if all are < 5 cm. T3 represents multiple lesions >5 cm or tumor invasion of a major branch of the portal or hepatic veins. T4 lesions directly invade adjacent organs other than the gallbladder or perforate the visceral peritoneum.

Berger procedure

The Berger procedure entails resection of the pancreatic head with preservation of the duodenum, requiring a sleeve of pancreatic head tissue to remain behind in order to preserve the vasculature. It is technically more difficult than a Whipple procedure, and is uncommonly performed.

Uncomplicated UTI

The CT scan demonstrates thickening of the sigmoid colon with surrounding fat stranding. Multiple diverticuli are present. There is no evidence of perforation or free air. There is also no abscess or free fluid to drain. Based on this CT and his history and physical, the patient has uncomplicated diverticulitis. He should be treated with antibiotics. Repeated episodes of uncomplicated diverticulitis or certain episodes of complicated diverticulitis may warrant surgery or drainage by interventional radiology.

frey procedure

The Frey procedure involves subtotal resection of the head of the pancreas and drainage via pancreaticojejunostomy. It is best used in patients with ductal dilatation localized to the head and body of the

Pancreatic surgeries

The Frey procedure is generally indicated in patients with severe chronic pancreatitis who also have a) multiple pancreatic duct strictures, b) a dilated obstructed main pancreatic duct, and/or c) an enlarged fibrotic pancreatic head with multiple impacted ductal calculi. The Frey procedure is a local resection of the head of the pancreas with lateral pancreaticojejunostomy. The procedure involves coring out the pancreatic head using electrocautery followed by bringing a standard Roux-en-Y jejunal limb through a defect in the transverse mesocolon to create a pancreaticojejunostomy. The Beger procedure is a duodenum-preserving pancreatic head resection (Beger procedure). Although the subtotal resection of the pancreatic head in the Frey procedure is similar to that performed during the Beger procedure, the pancreatic neck is not transected. Answer A: Distal pancreatectomy involves resecting the pancreatic parenchyma to the left of the confluence of the splenic vein with the portal vein (body and tail of the pancreas) with or without splenectomy. It would not be the appropriate procedure for a patient with severe chronic pancreatitis. Answer B: Pancreaticodoudenectomy is known as Whipple procedure. It is not the procedure of choice in a patient with severe chronic pancreatitis, especially if it is not isolated to the head of the pancreas. Answer C: Longitudinal Pancreaticojejunostomy is known as Puestow procedure. Answer E: Resection of the pancreatic head with a Roux en-Y jejunal loop as side-to-end and side-to-side pancreatico-jejunostomy is known as Beger procedure. Bottom Line: The coring of the pancreatic head is more limited in the Frey procedure compared with the Beger procedure and is created in continuity with a longitudinal ductotomy of the main duct in the body and tail of pancreas.

Gail model for invasive breast cancer

The Gail model for assessment of invasive breast cancer risk includes: Age Age at first period Age at first live birth first-degree relatives with breast cancer Previous breast biopsy Race

GCS

The Glasgow Coma Scale (GCS) is one of the most widely used methods for assessing neurologic status in trauma victims. A score is assigned based on the best performance in three categories including: eye-opening (1 - 4) , verbal response (1 - 5), and motor activity (1 - 6) with a score of 15 being the highest obtainable and that of 3 being assigned to a moribund unresponsive patient. Unfortunately, the score is heavily weighted towards higher cognitive function and there are a number of confounding variables including: acute drug and alcohol intoxication, hypothermia, hypotension, and hypoxemia that can affect the overall score. The eye-opening and verbal categories are most readily affected by such influences. The motor component of the exam is the most predictive of serious anatomic injury to the brain. Attempts to withdraw would receive a motor score of 4 where as the extensor position indicates a significant cerebral event and lowers the score of this component to 2.

Haggit levels

The Haggitt classification model describes the level of invasion of malignant polyps. Level 1 polyps invade the head. Level 2 polyps invade the neck. Level 3 polyps invade the stalk. Level 4 polyps either invade the base or are involved in a sessile polyp. Generally, Haggitt level 4 polyps require a formal segmentectomy. Haggitt levels describe the depth of invasion of malignant polyps.

Howship-Romberg sign?

The Howship-Romberg sign is a physical exam test that is used to confirm the diagnosis of an obturator hernia. A positive result occurs when pain extends down the medial aspect of the thigh with abduction, extension, or internal rotation of the knee. The sign is positive in about 50% of patients with an obturator hernia.

Milan Criteria

The Milan criteria were developed to identify patients with hepatocellular cancer who would be good candidates for a liver transplant. The criteria includes: - A single lesion less than 5 cm - Up to three lesions, each less than 3 cm - No extra-hepatic metastatic disease or major vessel involvement Liver transplant is the gold standard for patients with Child-Pugh B/C cirrhosis and limited hepatic reserve

Rives-Stoppa-Wantz repair

The Rives-Stoppa-Wantz repair starts by dissecting off the subcutaneous layer from the anterior rectus sheath. The edge of the rectus sheath is then opened and the rectus muscle is dissected off of the posterior rectus sheath all the way to the lateral edge of the muscle. The posterior sheath is then closed to keep the mesh away from the bowel. Absorbable mesh is then placed anterior to the posterior rectus sheath in a retrorectus fashion. Finally, the anterior rectus sheath is closed over the mesh using several U-shaped stitches. Answer A: Intraperitoneal Mesh Repair, as the name suggests, involves placing the mesh intraperitoneally in a sub-lay fashion. Answer C: Chevrel Onlay method of open hernia repair involves placing the mesh over the fascial repair. Answer D: Preperitoneal repair is similar to the intraperitoneal repair in terms of placing the mesh in a sublay fashion. However, in the preperitoneal repair the mesh is separated from the bowel by the peritoneum. Answer E: In Stoppa's repair, the mesh is placed between the rectus muscle and the posterior rectus sheath. Bottom Line: In Rives-Stoppa-Wantz repair, the mesh is placed anterior to the posterior sheath in the retrorectus position (Sublay).

Advantages of robotic surgery

The Robotic surgical system provides seven degrees of motion in comparison to four degrees of motion with the laparoscopic surgical system. The degrees of motion in both laparoscopic and robotic systems are 1) Up and down 2) forward and back 3) Right and left 4) Grip. The degrees of motion that are exclusive to robotic systems are 1) Wrist Yaw 2) Wrist pitch 3) Wrist roll. The elimination of hand tremors is a benefit of both laparoscopy and robotics. Robotic surgery also provides a high definition 3-D view.

HIV

The Society for Healthcare Epidemiology of America (SHEA) has published guidelines for the management of healthcare workers who are infected with HIV and other bloodborne pathogens, such as hepatitis B and C viruses. In general, SHEA states that HIV-infected healthcare providers should not be prohibited from participating in patient-care activities solely based on HIV status. Specific recommendations are made based upon the healthcare worker's viral burden and category of clinical activity. The three categories of procedures include: Category 1 — Those with minimal risk of blood transmission (e.g., endoscopy, or routine rectal or vaginal examinations) Category 2 — Those with possible risk of blood transmission, but unlikely (e.g., endodontic dental procedures, abscess drainage, biopsies under local anesthesia) Category 3 — Those with definite risk of blood transmission (e.g., general surgery) The guidelines recommend that HIV-infected healthcare providers should not be restricted from Category 1 or 2 procedures if they have a viral load assay showing "no detectable HIV" (<50 copies/mL). Those with HIV RNA >102 copies/mL should refrain from Category 3 procedures. Those healthcare workers who have less than 102 copies/mL may be allowed to engage in Category 3 procedures if the following criteria have been met: There is no evidence of prior transmission from the healthcare provider to a patient. The infected healthcare worker continues to have twice-yearly followup with the occupational medicine division regarding maintenance of viral suppression or low-level viremia. The healthcare provider gets HIV care from an expert in the field, who communicates with the expert panel regarding the provider's clinical status. The healthcare provider strictly complies with all infection control procedures.

Stanford system for aortic dissection

The Stanford system groups together as type A all dissections that involve the ascending aorta. Stanford type B includes dissections that originate in the descending thoracic and thoracoabdominal aorta. In acute dissections, the following are indications for operative management: Type A: Presence The goals of surgery in acute type A dissection are to prevent or treat an aortic catastrophe while restoring blood flow to the true lumen of the aorta. Aortic catastrophe includes aortic rupture into the pericardium or pleural space, dissection and occlusion of the coronary ostia, and progression to aortic valvular incompetence. The presence of ascending aortic involvement is therefore an indication for operative management in all but the highest-risk patients. Type B: Failure of medical management (persistent or recurrent pain or medically uncontrolled hypertension) Expanding aortic diameter Progressive dissection Impending or actual rupture Malperfusion Most patients with acute type B aortic dissections survive the acute and subacute phases with medical management alone; however, approximately 20-30% of patients present with complicated type B dissection, which require urgent operative (surgical or endovascular) intervention. Complicated dissection can be defined as imminent or actual aortic rupture, aortic expansion, hemodynamic instability, persistent pain despite medical management, drug-resistant hypertension, and malperfusion syndrome. The most frequent causes of death in acute type B dissection are aortic rupture and visceral malperfusion.

Stanford system operative approaches

The Stanford system groups together as type A all dissections that involve the ascending aorta. The exposure for procedures performed on the ascending aorta and proximal arch is through a median sternotomy. This can be modified with supraclavicular, cervical, or trapdoor incisions to gain exposure to the brachiocephalic vessels or descending thoracic aorta. When dissecting the distal arch, it is important to identify and protect both the left vagus nerve with its recurrent branch and the left phrenic nerve. Stanford type B includes dissections that originate in the descending thoracic and thoracoabdominal aorta. The patient is positioned in right lateral decubitus. The pelvis is canted posteriorly to allow access to both femoral vessels. A posterolateral thoracotomy in the fourth intercostal space provides sufficient access to the aorta. A thoracoabdominal incision may be required to access the abdominal aorta in the case of visceral malperfusion. This may be performed through either a transperitoneal or a retroperitoneal approach. Answer A: The exposure for procedures performed on the ascending aorta and proximal arch is through a median sternotomy. Answer C: Stanford type B, not type A, includes dissections that originate in the descending thoracic and thoracoabdominal aorta. The patient is positioned in right lateral decubitus. Answer D: Type B, not type A, aortic dissections may be performed through a retroperitoneal approach. Answer E: The exposure for procedures performed on the ascending aorta and proximal arch is through a median sternotomy. This can be modified with supraclavicular, cervical, or trapdoor incisions to gain exposure to the brachiocephalic vessels or descending thoracic aorta. Bottom Line: Type A aortic dissections involve the ascending aorta and are best approached through a median sternotomy. Type B aortic dissections involving the descending thoracic and thoracoabdominal aorta are best approached through a posterolateral thoracotomy.

Prophylactic antibiotics for surgery

The Surgical Care Improvement Project (SCIP) began in 2003 to reduce complications associated with surgical care. Postoperative complications increase patient mortality, length of stay, and hospital costs. Recommendations of SCIP include the administration of preoperative antibiotics to decrease the risk of surgical site infection. They recommend administration of a full dose of preoperative antibiotics within 1 hour of skin incision. Preoperative vancomycin is an exception to this rule. Vancomycin should be administered as an infusion over a minimum of 1 hour to prevent infusion-related complications such as Red Man syndrome.

Surgical care improvement project guidelines

The Surgical Care Improvement Project (SCIP) is a strategy for improving surgical quality by stimulating the use of evidence-based processes by hospitals across the United States. SCIP links Medicare hospital reimbursement to adherence to processes which may reduce the incidence of surgical site infection, postoperative cardiac events, venous thromboembolism, and ventilator-associated pneumonia according to best available scientific evidence. Current core measures include: 1.) Surgical patients should have prophylactic antibiotics initiated within one hour prior to surgical incision. The goal of antibiotic prophylaxis is to obtain bactericidal tissues and/or serum levels at the time of skin incision. This rationale is based on previous data which demonstrated a common reason for failure of antibiotic prophylaxis was that there was a delay in administration until after the operation occurred. The lowest incidence of post-operative infection is found in patients in whom antibiotic administration occurs within one hour prior to making incision to start the case and the risk of infection increases progressively as this time interval increases. In the event that vancomycin is the drug of choice, infusion may begin up to 2 hours prior to beginning the case due to a prolonged infusion time. 2.) Surgical patients should receive prophylactic antibiotics consistent with current guidelines and specific to each type of surgical procedure. The goal of antibiotic prophylaxis is to select an agent that not only covers the most probable bacterial flora that will be encountered but that is also safe and cost-effective. First- and second-generation cephalosporins suffice for most surgical procedures but may need supplemented by an agent with anaerobic coverage for colonic surgery. Due to the risk of developing resistance, Vancomycin is not recommended as a routine agent but may be appropriate for a patient who is allergic to beta-lactams. 3.) Prophylactic antibiotics are to be discontinued within 24 hours after the surgical end time (or 48 hours for coronary artery bypass grafting or other cardiac surgery). The goal of antibiotic prophylaxis is provide the benefit of reduced infection with as little risk as possible. The risk of infection with Clostridium difficile or resistant organisms clearly increases as antibiotic duration is prolonged. 4.) Cardiac patients should have controlled 6 am blood glucose (less than or equal to 200 mg/dL) on postoperative day one (POD 1) and POD 2 with anesthesia end date being considered POD 0. Risk of infection is increased in the face of hyperglycemia in patients undergoing CABG regardless of diabetic status. Evidence demonstrates that intensive insulin therapy is associated with a reduction in overall in-hospital mortality, blood stream infections, acute renal failure, transfusion requirements, ventilator support, and intensive care. 5.) Surgery patients requiring hair removal should have it done with clippers or depilatory. Shaving is inappropriate. Shaving causes multiple skin abrasions which actually increases the risk for postoperative infections. Hair clippers, on the other hand, do not increase the risk of infection and are appropriate when indicated. 6.) Urinary catheters should be removed on POD 1 or 2 with day of surgery being day zero. It has been well established that the longer a catheter remains in place the higher the risk for a catheter-associated urinary tract infection (UTI). Bacteriuria develops in 26% of patients after 2 to 10 days of catheterization, of whom 24% will develop a symptomatic UTI and 3.6% will develop bacteremia. 7.) Surgery patients should have normothermia maintained intraoperatively using active warming or body temperature should be equal to or greater than 36 degrees Celsius within the 30 minutes immediately prior to or 15 minutes immediately after anesthesia end time. Hypothermia is associated with increased risk of surgical site infection, impaired wound healing, adverse cardiac events, altered drug metabolism, and coagulopathy. 8.) Surgery patients on beta-blocker therapy prior to arrival should receive a beta-blocker during the perioperative period (1 day prior to surgery through POD 2). There is an increased risk of perioperative mortality in those patients in whom beta-blockade is discontinued postoperatively. 9.) Surgery patients should have venous thromboembolism (VTE) prophylaxis ordered within 24 hours following anesthesia end time. Surgery significantly increases the risk of VTE and thromboprophylaxis has a positive risk/benefit ratio and is cost-effective. Answer A: Prophylactic antibiotics should be received within one hour of making surgical incision. Answer B: Excessive bodily hair in the surgical field should be cleared with the use of clippers prior to making incision. Answer C: Cardiac patients should have blood glucose levels of less than 200 mg/dL by 6 am on postoperative day (POD) 1 and POD 2. Answer E: Surgery patients should receive appropriate venous thromboembolism prophylaxis within 24 hours of anesthesia end time. Bottom Line: Surgical quality may be improved by instituting evidence-based processes such as timely administration of antibiotic and VTE prophylaxis as well as beta-blockade when appropriate. Additionally, postoperative infections may be reduced with the use of clippers rather than a razor for hair removal, prompt removal of urinary catheters, and maintenance of normothermia, as well as appropriate glucose control.

Treating an anastomotic leak GI antibiotics

The Surgical Infection Society guidelines for treatment of intraabdominal infections include the following regimens: Single Agent Regimens: cefoxitin, cefotetan, ampicillin/sulbactam, ticarcillin/ clavulanic acid, meropenem, piperacillin/tazobactam, and imipenem/cilastatin. Combination Regimens: cefuroxime plus metronidazole, 3rd or 4th generation cephalosporin plus an antianaerobe, aztreonam plus clindamycin, ciprofloxacin plus metronidazole, aminoglycoside plus clindamycin or metronidazole.

Steroids potency

The actions of the synthetic steroids are similar to that of cortisol. They bind to specific intracellular receptor proteins and produce the same effects but have different ratios of glucocorticoid to mineralocorticoid potency. Short- to medium-acting glucocorticoids are hydrocortisone, prednisone, prednisolone, methylprednisolone, and meprednisone. Intermediate-acting glucocorticoids are triamcinolone, paramethasone, and fluprednisolone. Long-acting glucocorticoids are betamethasone and dexamethasone. Synthetic mineralocorticoids include fludrocortisone and desoxycorticosterone acetate. The relative potencies of exogenous glucocorticoids are listed below: Hydrocortisone (relative potency: 1) Prednisone (relative potency: 4) Methylprednisone (relative potency: 5) Dexamethasone (relative potency: 30)

Atherosclerosis

The atherosclerotic plaque is comprised of a fibrous cap made up of smooth muscle cells and connective tissue, and lipid core which contains proteins and inflammatory cells. Rupture of the cap exposes the thrombogenic lipid core to the blood which can precipitate clot formation. The lipid core is primarily made from LDL and is a major stimulator of inflammatory cells. Plaques most commonly form at branch points and bifurcations due to alterations in laminar blood flow. Arterial lesions are compensated for by a dilation of the vessel diameter for plaques that cause up to 40% stenosis. Answer B: The vessel lumen is not compromised until over 40% stenosis occurs. Answer C: Rupture of the cap exposes the thrombogenic lipid core to the blood which can precipitate clot formation. Answer D: The lipid core is primarily made from LDL. Answer E: Platelet derived growth factor is a major contributor to smooth muscle cell proliferation. Bottom Line: The lipid core is comprised of lipids and inflammatory cells and is thrombogenic after fibrous cap rupture.

Compartment syndrome

The best confirmatory test for a suspected diagnosis of extremity compartment syndrome is checking compartment pressures with a transducer that is inserted into the affected extremity. The patient's compartment pressure is subtracted from the patient's diastolic blood pressure, and if the resulting difference (Delta P) is less than 30 mmHg, a diagnosis of compartment syndrome is supported.

Hepatic venous wedge pressure

The best measurement of portal pressures in patients with sinusoidal disease, such as in cirrhotics, is by the hepatic venous wedge pressure. Answer A: Patients with presinusoidal disease will have normal hepatic wedge pressures, thereby giving falsely low measurements. Answer B: Portal hypertension is evident with portal pressures that exceed 5 mmHg. Once pressures reach 10-12 mmHg, there is increased risk for esophageal variceal bleeding. Answer D: Patients with presinusoidal disease need direct cannulation of transhepatic veins for an accurate portal pressure measurement. Answer E: Portal pressure is calculated by the difference in IVC and portal vein pressures. Bottom Line: The best measurement of portal hypertension in cirrhotics is with the hepatic venous wedge pressure.

Best test to diagnose hepatic artery thrombosis

The best test to confirm hepatic artery thrombosis is the use of Doppler ultrasound . This can be used to examine flow with the vessels in real time and can be done at bedside if needed. Doppler ultrasonography is usually used to confirm flow within the extrahepatic and intrahepatic arteries. If inappropriate waveforms or no flow is identifed, the diagnosis can be confirmed by angiography or reexploration. At exploration, flow can be restored within the hepatic artery approximately 80% of the time if the diagnosis is made early. However, despite restoring arterial flow, many of these recipients will still develop biliary complications. CT of the abdomen requires transport and is not the first line diagnostic study in this case. Hepatobiliary scan would only allow assessment of liver function and potential diagnosis of a bile leak if the liver is currently producing bile. KUB would not help diagnose what is going on with the liver, but may show free air if the biliary-enteric anastomosis has broken down. ERCP would not be advocated because accessing a fresh biliary-enteric anastomosis carries a high risk of disrupting it (if it hasn't broken down already) and would only potentially diagnose a bile leak but not its cause. Hepatic artery thrombosis carries with it a host of potentially disastrous complications. Potential complications related to hepatic artery thrombosis, as seen in this patient, include graft dysfunction and biliary anastomosis breakdown. In addition to that, it can lead to graft loss, intrahepatic abscess, and ultimately biliary stricture

Effects of aging on cardiovascular system

The cardiac index decreases 1%/year and systemic vascular resistance increases 1%/year. Both the response to adrenergic stimulation and the maximal heart rate are reduced with age. As a result, trauma patients over age 65 have a significantly lower cardiac index, oxygen delivery, and oxygen consumption than younger patients.

SIADH

The patient has syndrome of inappropriate antidiuretic hormone secretion (SIADH) as a result of the trauma to his head. The symptoms of SIADH include hyponatremia, low serum osmolality, elevated urine osmolality, and normal renal function. The initial treatment for SIADH is fluid restriction. If this fails to correct the hyponatremia, other options include starting normal saline and adding diuretics.

Inflammatory breast cancer

The characteristic clinical findings which define the syndrome of inflammatory breast cancer are the result of infiltration and obstruction of dermal lymphatics by tumor cells. The skin involvement of inflammatory breast cancer makes this a T4 lesion, and therefore Stage IIIa, a locally advanced cancer. Skin biopsy may reveal tumor cells and lymphocytes in the dermal lymphatics, but absence of these findings are not sufficient evidence to exclude the diagnosis of inflammatory breast cancer

Right thoracotomy

The choice of incision is very important and the choice of the incision depends on several factors including the indication for operation, the mechanism of injury, the presence of associated injuries, and the results of preoperative studies. When the site of injury has been already determined preoperatively, the choice of incision should follow a simplified approach. Injuries to azygos vein, right pulmonary artery, right lung, right hilum, right internal mammary, proximal esophagus, carina, and right main stem are best approached through a right thoracotomy

Malignant hyperthermia

The clinical manifestations of malignant hyperthermia are not uniform and vary in onset and severity. Some patients manifest the abortive form of malignant hyperthermia (tachycardia, arrhythmia, raised temperature, and acidosis). Others, after intubation with succinylcholine, demonstrate loss of twitches on neuromuscular stimulation, and muscle rigidity develops. An inability to open the mouth as a result of masseter muscle spasm is a pathognomonic early sign and indicates susceptibility to malignant hyperthermia. Other manifestations include tachypnea, hypercapnia, skin flushing, hypoxemia, hypotension, electrolyte abnormalities, rhabdomyolysis, and hyperthermia. Management of malignant hyperthermia is as follows: Discontinue the triggering anesthetic. Hyperventilate the patient with 100% oxygen. Terminate surgery. Administer dantrolene, 2.5 mg/kg as a bolus, and repeat every 5 minutes, then 1-2 mg/kg/hr until normalization or disappearance of symptoms. Administer glucose. Administer alternative anesthesia. Administer HCO3. Use cooling blankets. Check and monitor arterial blood gas, creatine kinase, electrolytes, lactate, and myoglobin. Monitor the electrocardiogram, vital signs, and urine output.

BCC

The clinical presentation and histological examination in this patient is classic for basal cell carcinoma (BCC). A skin biopsy is essential for the diagnosis of any skin cancer. Margins for low-risk BCC range from 0.3 to 0.5 cm when performing surgical excision. Margins of at least 4 mm are needed, even with the least aggressive BCCs, to achieve 95% cure rates and as much at least 8 mm for more aggressive tumors. Mohs surgery should be considered for basal cell and squamous cell carcinomas that are primary BCCs occurring at sites known to have a high initial treatment failure rate, tumors with poorly defined clinical borders, diameters greater than 1 cm located anywhere on the face, diameters greater than 2 cm, and infiltrative or morpheaform/sclerotic histopathologic patterns.

Pancreatic lyphoma

The clinical presentation of patients with pancreatic lymphoma is often nonspecific, consisting of weight loss, nausea, vomiting, and abdominal pain. B-type lymphoma symptoms such as fever and night sweats may be present. CT images can suggest the diagnosis of pancreatic lymphoma by the presence of a bulky pancreatic mass with surrounding lymphadenopathy. CT evidence of a bulky pancreatic mass in the absence of weight loss, back pain, and extrahepatic biliary obstruction (normal bilirubin associated with an elevated lactate dehydrogenase) should cause one to consider lymphoma in the differential diagnosis. Treatment of pancreatic lymphoma traditionally has involved cytotoxic chemotherapy. The most common chemotherapy regimens include cyclophosphamide, doxorubicin (adriamycin), vincristine, and prednisone.Endoscopic stenting to relieve jaundice followed by chemotherapy is the standard treatment, and long-term remission is often achieved. Bottom Line: Pancreatic lymphoma may present similar to pancreatic adenocarcinoma but is treated with chemotherapy alone. Relief of biliary obstruction is accomplished with endoscopic stenting.

CBD in anterior to portal vein and lateral to hepatic artery

The common bile duct (CBD) is formed by the union of the cystic and common hepatic ducts. The CBD is approximately 8 cm in length, but it varies in length according to the point of union of the cystic duct and the common hepatic duct. The normal diameter of the CBD ranges from 4 to 9 mm. The common bile duct is considered enlarged if the duct diameter exceeds 10 mm. The upper third, or supraduodenal portion, of the CBD courses downward within the hepatoduodenal ligament along the free edge of the lesser omentum, anterior to the portal vein and to the right and therefore lateral to the proper hepatic artery. The middle third, or retroduodenal portion, of the CBD passes behind the first portion of the duodenum, lateral to the portal vein and anterior to the inferior vena cava. The lower third, or intrapancreatic portion, of the CBD traverses the posterior aspect of the pancreas in a tunnel or groove to enter the second portion of the duodenum, where it is usually joined by the pancreatic duct. The intramural or intraduodenal portion of the CBD passes obliquely through the duodenal wall to enter the duodenum at the papilla of Vater.

Ureteral stents

The correct answer is: B There have been many debates, especially with the advent of lighted ureteral stents, on their overall benefit. Preoperative stenting has the main benefit of helping to identify an injury once it occurs. Ureteral stents have not been definitively shown to prevent injuries, but this could change as more studies are done with new lighted stents. Stents are never used as the sole treatment for ureteral injuries. Answer A: Ureteral stents do not prevent ureteral injuries but help to identify them when they happen. Answer C: There are a few different ways to repair the ureter over a stent. The stent is NOT the only modality.

TTP

The correct answer is: B Thrombotic thrombocytopenia purpura (TTP) is a poorly understood disease that affects many body systems. The primary pathology results from formation of hyaline membranes composed of platelets and fibrinogen. These membranes cause arteriole and capillary occlusion throughout the body. It is believed to be caused by an autoimmune reaction to small vessel endothelial cell antigen. However, the exact pathophysiology has yet to be determined. The TTP pentad of symptoms include: fever, change in mental status, hemolytic anemia, renal failure or hematuria, and thrombocytopenic purpura. Initial treatment of this disease is with medical optimization. Medications include antiplatelet agents, such as aspirin and dipyridamole, which protect against platelet aggregation. However, given that the exact mechanism is still unknown, some treatments, such as fresh frozen plasma and plasmapheresis, are used in an empiric way given their proven efficacy but unknown physiologic effect. Corticosteroids are used with some efficacy, speaking to the believed autoimmune nature of the disease. Splenectomy is reserved for those patients who have failed nonoperative management. It has shown long-term mortality benefits for this disease. Elective splenectomy patients should always receive preoperative vaccinations for encapsulated organisms

Pancreas transplant with bladder drainage

The patient has undergone pancreas transplant with bladder drainage which places him at risk for developing metabolic acidosis secondary to increase bicarbonate losses from pancreatic secretions in the urine. At two months postoperatively, one would suspect adequate respiratory compensation for the metabolic acidosis.

Lung Anatomy

The correct answer is: C The right lung has 3 lobes that are separated by two fissures. The major, or oblique, fissure separates the lower lobe from the middle and upper lobes. The minor fissure separates the upper lobe from the middle lobe. Answer A: The lung is divided into bronchopulmonary segments that each have their own blood supply and bronchiole. Answer B: The lingula is a part of the upper lobe of the left lung. Answer D: The right lung has 3 lobes that are separated by two fissures. The major, or oblique, fissure separates the lower lobe from the middle and upper lobes. The minor fissure separates the upper lobe from the middle lobe. Answer E: There are ten segments on the right and eight on the left.

Trans-anal rectal mass excision

The criteria include a well-differentiated T1 lesion, less than 3 cm in size, involving less than 30% of the circumference, at a distance less than 8 cm from the anal verge, with no lymphovascular involvement or mucin production. Transanal excision is better tolerated by the patient but comes with a higher risk of recurrence. T2 lesions have an even higher risk of recurrence and should only be excised locally if the patient is a poor surgical candidates

Severe mitral stenosis

The cross-sectional area of a normal mitral valve is between 4 and 6 cm2. When this is narrowed to an area of <1 cm2 and the transvalvular gradient rises above 10 mm Hg, the patient is considered to have severe mitral stenosis. If the elevation is severe enough, additional signs of right-sided heart failure can be found, including jugular venous distension and peripheral edema. Answer A: Syncope is an early and frequent finding in patients with aortic stenosis. Syncope results from a lack of forward flow from the heart and a transient decrease in cerebral perfusion.

Cystic artery

The cystic artery has a high incidence of anatomic variability. It normally passes posterior to the common hepatic duct. Another anomaly that has been identified is double cystic arteries, which are found in up to 10% of patients. Answer A: The cystic artery is found in the triangle of Calot. Answer B: The cystic artery divides into superficial and deep branches prior to entering the gallbladder. Answer C: The cystic artery has a high incidence of anatomic variability. It normally passes posterior to the common hepatic duct. Answer E: The cystic artery most frequently comes off the right hepatic artery but has been seen originating from the common hepatic, gastroduodenal, and superior mesenteric arteries. Bottom Line: The cystic artery usually comes off the right hepatic artery and courses through the triangle of Calot.

Vitamin D levels

The daily maintenance dose of vitamin D varies by age, but most children and adults generally require 600-2000 IU of vitamin D daily. Children less than 1 year old may require 400 IU of vitamin D. For vitamin D-deficient children and adults, higher doses of vitamin D given either daily or weekly are recommended, followed by an increase in the daily dose of vitamin D. Screening for vitamin D deficiency is recommended only for those individuals who are at high risk for vitamin D deficiency, including patients with osteoporosis or a malabsorption syndrome, as well as black and Hispanic individuals, obese persons (BMI >30 kg/m2), and those with several other medical conditions. Answer A: 25 hydroxycholecalciferol is the best test to determine vitamin D status. Answer C: Ultraviolet light type B (UVB) is more efficacious than ingested vitamin D in raising 25 (OH) Vit D. Answer D: Most dietary sources of vitamin D contain insufficient amounts of vitamin D to satisfy daily requirements. Answer E: Very high levels of vitamin D (>36ng) and low levels of vit D (< 20ng) both increase the risk of acute coronary syndrome and mortality.

Rectal prolapse

The definitive treatment for prolapse is surgical. Conservative measures have little impact. Two surgical approaches exist for rectal prolapse, abdominal and perineal. Due to a higher rate of recurrence by perineal approach, abdominal surgical approach is recommended for younger patients. Answer A: Prolapse will appear as concentric rings of tissue, whereas hemorrhoids will have a radial pattern. Answer B: Management of rectal prolapse is surgery, conservation measures have not been shown to be beneficial. Answer D: The perineal approach has increased rates of recurrence, but is better tolerated by an older, less stable patient population. Answer E: The patient is describing symptoms of rectal prolapse. Rectal prolapse is more often seen in females with multiple vaginal deliveries and chronic constipation. It can be diagnosed by observing the prolapse during abdominal strain. Other studies, such as defecography or anal manometry may help in discerning the etiology of the prolapse. Bottom Line: Rectal prolapse is surgically treated with an abdominal approach for healthy patients, and a perineal approach for the elder

Hepatic Artery Thrombosis s/p liver transplant

The degree of preoperative debilitation and the complexity of the operative procedure make complications following liver transplantation very common. Prompt recognition and treatment are essential. However, the usual signs or symptoms that would be expected in the general population are often absent or present to a lesser degree, and therefore, a high level of suspicion must be maintained. Early warnings, such as fever, leukocytosis, or pain, may be suppressed as a result of immunosuppression. This clinical picture (rising liver enzymes, declining protein levels, and signs of biliary leak) is most concerning for hepatic artery thrombosis.. Hepatic artery thrombosis is suspected in the setting of unexpectedly high liver enzymes, an elevation in liver enzymes rather than a gradual decline, or poor synthetic function during the first week to 10 days posttransplant. Other signs of biliary ischemia from hepatic arterial thrombosis include the development of biliary leaks, strictures, or intrahepatic abscesses. Portal vein thrombosis is less common. Symptoms of portal vein thrombosis include increasing ascites and signs of portal hypertension. Hepatic vein thrombosis and inferior vena cava thrombosis share similar symptoms which include ascites and renal dysfunction.

Esophageal leiomyoma

The description of the mass strongly suggests a leiomyoma of the esophagus. Leiomyomas are benign, slow-growing lesions of mesenchymal origin that may cause dysphagia even at small sizes. Answer A: Observation is an option for an asymptomatic mass that is less than 2 cm in size, but all symptomatic leiomyomas require resection. Answer B: Imatinib may have some clinical benefit for leiomyomas, but it is not the standard of care. Answer C: A biopsy is contraindicated for a suspected leiomyoma because the resulting scar tissue makes the resection more difficult and increases the risk of a perforation. Answer E: Proximal and mid-esophageal lesions require a right thoracotomy approach, whereas a distal lesion would necessitate a left thoracotomy incision. Bottom Line: All symptomatic esophageal leiomyomas are treated with enucleation.

Mastitis

The differential diagnosis in a breastfeeding patient includes mastitis, abscess, or inflammatory breast cancer. If the patient fails antibiotic treatment, a biopsy be warranted. Staphylococcus aureus and Streptococcus species are common causes of an infected breast. Breast abscesses present with tenderness, erythema, and hyperthermia. These abscesses are related to lactation and occur within the first few weeks of breastfeeding. Preoperative ultrasonography is effective in delineating the required extent of the drainage procedure, which is best accomplished via circumareolar incisions or incisions paralleling Langer's lines. Superficial infections should be treated with local wound care, including application of warm compresses, and the administration of antibiotics (penicillins or cephalosporins). Given that the findings are consistent with superficial infectious mastitis during breast-feeding without fluctuation and low risk for breast cancer, it is ideal to try antibiotics.

Small bowel lymphoma

The distribution of lymphoma in the small intestine parallels the lymphoid tissue distribution. Lymphoid follicules are most prominent in the ileum compared to any other location in the small intestine; therefore, the ileum represents the most common location of small intestinal lymphomas. They are characteristically bulky and over 50% of these tumors are larger than 5 cm in diameter at presentation. Of note, gastrointestinal lymphomas are the most frequently occurring extra-nodal lymphoma. Answer A: The duodenum may serve as a metastatic site of a melanoma or primarily of an adenoma. Small intestine adenomas are characterized as villous, simple, tubular, and Brunner's gland adenomas. The villous subtype has the largest potential for malignant degeneration. Answer B: A small intestine adenoma is a more likely tumor found in the duodenum than a lymphoma. If an ampullary adenoma is present, then patients present with painless jaundice or other symptoms of biliary obstruction such as pruritus. Answer C: Jejunum has less lymphoid tissue than the ileum and therefore is less likely to harbor a lymphoma. However, a consideration should be made towards patient's with Crohn's disease whose incidence of small bowel cancer tends to decrease from proximal to distal small bowel.

Peri-operative diabetes control

The doses of insulin and other hypoglycemic drugs need to be titrated during the perioperative period. The oral hypoglycemic agents are withheld on the day of surgery and resumed after the surgery except Metformin. If there is altered renal action, then Metformin has to be withheld until the renal function stabilizes due to the risk of lactic acidosis. During this period hyperglycemia is managed with a short-acting insulin preparation based on blood glucose monitoring. Rapid and short-acting insulin is withheld when the patient is NPO and covered based on blood glucose monitoring. Intermediate and long-acting insulin is given at the normal dose the night before surgery. If there is a morning dose, then half the normal dose on the morning is given. Some patients use insulin pumps for glucose management. The pumps have variable delivery rate based on endogenous insulin production. On the day of surgery, the patient is managed with basal insulin infusion and the pump is used to correct the glucose level as it is measured.

Bile Salts

The entero-hepatic circulation is the secretion of bile salts by the liver into the intestine, followed by absorption by the ileum, and return of the bile salts to the liver via the portal vein. The entero-hepatic circulation is not a primary source of bilirubin. Cholic acid and chenodeoxycholic acid are the primary bile acids. They are conjugated with either taurine or glycine and function to digest fats and oils. Conjugated bile acids are not a source of bilirubin.

Steps for gastric sleeve

The essential steps for completion of sleeve gastrectomy include: 1. Devascularization of the greater curve of the stomach 2. Insertion of a bougie (34 French - 40 French) or endoscope to size the gastric sleeve 3. Transection of the stomach starting at a location 2 cm to 6 cm proximal to the pylorus 4. Specimen extraction.

GI tract losses fluid

The estimated amount of daily fluid passing through the gastointestinal tract is about 9 L/day. The breakdown is as follows: Exogenous PO intake: about 2000 mL Saliva: 1500 mL Stomach: 1000-2000 mL Biliary: 500 mL Pancreas: 1500 mL Small bowel: 1500 mL Most of those 9 L are absorbed in the small and large bowels as follows: Small bowel: 8500 mL Large bowel: 400 mL The net loss of fluid with stool is about 100-200 mL/day. Answers B & C & D & E: Stomach secretion is about 1-2 L/day. The salivary glands, pancreas, and the small intestine are estimated to secrete about 1500 mL/day each. Bottom Line: Biliary secretion is about 500-1000 mL/day.

Pancreatic secretions

The exocrine pancreatic response to a meal traditionally is considered to involve three phases: 1. The cephalic phase: triggered by the sight or smell of food, accounts for 10% to 20% of the pancreatic secretion during a meal. 2. The gastric phase: triggered by food entering the stomach and gastric distention, accounts for 15% to 20% of meal-stimulated pancreatic secretion. 3. The intestinal phase: triggered by food and acid entering the duodenum and proximal jejunum, accounts for 60% to 70% of meal-stimulated pancreatic secretion. Acidification (i.e., to a pH < 4.5) of the duodenum causes specialized cells (s cells) within the mucosa to release secretin into the circulation. Secretin potently stimulates pancreatic duct cells to secrete pancreatic fluid and bicarbonate. Elevated CCK levels cause acinar cells to secrete digestive enzymes and other proteins into the pancreatic juice. Somatostatin exerts its effects on the endocrine function of the pancreas. Gastrin is involved with gastric acid secretion. CCK is the most potent stimulator of pancreatic enzyme secretion, while secretin stimulates fluid and bicarbonate secretion.

Expected weight loss after sleeve gastrectomy

The expected excess weight loss after 2 years following sleeve gastrectomy is approximately 60%. Percentage excess weight loss (EWL) is calculated using the formula: %EWL = weight loss (kg)/excess weight (kg), with excess weight being the difference between actual weight and ideal weight. Since this patient weighs 150 kg and his ideal weight is 75 kg, his excess weight is 75 kg. He is expected to lose approximately 60% of this excess weight or approximately 45 kg.

Blunt cardiac injury

The expression cardiac or myocardial contusion should be exchanged for the more updated term "blunt cardiac injury". This is an ill-defined entity that should be considered in the face of minor ECG abnormalities, the presence of complex arrhythmias, or cardiac failure following blunt chest trauma. The Eastern Association for the Surgery of Trauma (EAST) practice management guidelines have tried to delineate the appropriate evaluation for BCI because there is no true diagnostic "gold standard". Recommendations are as follows: 1.) An admission ECG and another at 8 hours should be performed in any patient suspected of having BCI. If the admission ECG results are normal then pursuit of the diagnosis can be terminated. However, if they are abnormal then the patient should be admitted for cardiac monitoring for 24-48 hours. If the patient is hemodynamically UNstable then an imaging study such as an echocardiogram should be obtained. The presence of a sternal fracture does NOT predict the presence of BCI and thus does not mandate continuous monitoring in the face of a normal ECG. Cardiac enzymes have not been shown to be superior to serial EKGs and do not need to be routinely obtained on every patient. Answer B: Only sternal fracture patients with EKG changes need observation with continuous EKG monitoring. Most sternal fractures are not associated with BCI. Answer C: Echocardiogram should be obtained in any patient with blunt thoracic trauma and unexplained, persistent shock out of proportion to apparent injuries or despite aggressive resuscitation, and in any patient with signs consistent with significant BCI. These may include signs of heart failure (eg, pulmonary edema, distended jugular veins) or abnormal heart sounds (eg, muffled sounds, holosystolic or diastolic murmur). Answer D: Sternal fracture has not been shown to be a predictor of mortality. Most sternal fractures are occult and only seen on CT scan and therefore not predictive of BCI and mortality. Answer E: Cardiac biomarkers should be obtained in patients with hemodynamic instability, signs of severe injury, or an ECG with significant abnormalities (arrhythmias other than mild sinus tachycardia, a new conduction block, and ST or T wave abnormalities suggestive of ischemia). The use of biomarkers has not been shown to be more accurate than serial EKGs. Bottom Line: There is no single entity that can be used to diagnose blunt cardiac injury. It's presence can not be predicted by the presence of sternal fracture or elevation of cardiac enzymes. If admission ECG is normal then no further work-up is required. Echocardiogram is not mandated unless the patient is hemodynamically unstable.

External branch of the superior laryngeal - cricothyroid muscle

The external branch of the superior laryngeal nerve provides motor innervation to the cricothyroid muscle, which tilts the larynx during speaking to affect pitch. Injury to one or both recurrent laryngeal nerves, which innervate the posterior cricoarytenoid muscles, can result in airway compromise if the cords remain in an abducted position, narrowing the airway. Answer A: This patient has an injury to the external branch of the superior laryngeal nerve. It is more common than injury to the recurrent laryngeal nerve. Answer B: Unfortunately, there is no treatment. Answers C & E: The nerve injured in this patient usually crosses the superior thyroid vessels more than 1 cm above the level of the superior thyroid pole. Bottom Line: Injury to the external branch of the superior laryngeal nerve affects voice pitch. Injury to the recurrent laryngeal nerve can result in airway compromise.

Carcinoid

The findings of a small bowel obstruction along with typical "carcinoid syndrome" symptoms of flushing and bronchospasm suggests the presence of a small bowel carcinoid tumor. Carcinoid is the most common malignancy of the small bowel and is found most commonly in the ileum. Carcinoids arise from enterochromaffin-like cells, also known as Kulchitsky cells, and often present as a small bowel obstruction or intussusception. Carcinoids should be treated similarly to an adenocarcinoma and will require a surgical resection. The tumor can release serotonin and other neuroendocrine metabolites into the bloodstream that cause symptoms such as flushing, diarrhea, bronchospasm, and right-sided heart failure. These metabolites are usually cleared by the liver, and therefore patients who exhibit these symptoms will likely have liver metastasis.

Surgical Management of Intussusception

The intussusceptum is delivered through a transverse incision in the right side of the abdomen and reduced by squeezing the mass retrograde from distal to proximal until completely reduced. Warm lap pads may be placed over the bowel and a period of observation may be warranted in cases of questionable bowel viability. Adhesive bands around the ileocecal junction are divided, and an appendectomy is then performed. The recurrence rates are very low after surgical reduction. Invariably, the lymphoid tissue within the ileocecal valve region is thickened, edematous and may be mistaken for a tumor within the small bowel.

Free water deficit

The formulas to calculate the free water deficit in a patient with hypernatremia: Free water deficit (men)= Weight x 0.6 x [(Current serum Na - 140)/140] Free water deficit (women)= Weight x 0.5 x [(Current serum Na - 140)/140] It is important not to drop serum sodium at a rate faster than 0.5-1 meq/L/hr. The rapid drop in serum sodium can be associated with brain edema and permanent neurological deficit. In this patient, free water deficit will be 70 kg * 0.6* (160-140)/140=6 liters.

Damage control surgery

The goal of damage control surgery is to expedite the transfer of the patient out of the operating room and to the intensive care unit for resuscitation. Before the patient is returned to the ICU, the abdomen must be temporarily closed. Originally, penetrating towel clips were used to approximate the skin. However, the ensuing bowel edema often produced a delayed abdominal compartment syndrome. Currently, temporary closure of the abdomen is accomplished using an antimicrobial surgical incise drape (Ioban). In this technique, the bowel is covered with a fenestrated subfascial sterile drape, and two Jackson-Pratt drains are placed along the fascial edges; this is then covered using an Ioban drape, which allows closed suction to control reperfusion-related ascitic fluid egress while providing adequate space for bowel expansion to prevent abdominal compartment syndrome. During the initial postoperative stage, the subfascial sterile drape should not covered by a blue towel so that the status of the bowel and hemorrhage control can be assessed. Answer A: The primary objectives of damage control laparotomy include control of surgical bleeding while preventing ischemia and limiting GI spillage. Answer B: Return to the OR in 12 to 24 hours is planned once the goals of resuscitation are achieved which include core temperature of >35°C (95°F), correction of acidosis (base deficit of <6 mmol/L), and normal coagulation indices. Answer C: The most common indication for damage control surgery is the lethal triad: coagulopathy, hypothermia, and metabolic acidosis. Answer D: The operative techniques used are temporary measures, with definitive repair of injuries delayed until the patient is physiologically replete. Answer E: Placing an abdominal vac is the current standard of care. Bogota bags were used previously. Bottom Line: The primary objectives of damage control laparotomy include control of surgical bleeding while preventing ischemia and limiting GI spillage.

Endovascular therapy for aortic dissection

The goal of endovascular therapy is to seal any dissection entry points, promote thrombosis of the false lumen, induce aortic remodeling, and prevent further aneurysmal dilatation, rupture, or malperfusion. Answer A: Endovascular therapy has been recently used for management of chronic type B aortic dissections. The endovascular stenting procedure can be performed either in the angiography suite or an operating room with advanced imaging capabilities. Answer B: The goal of endovascular therapy is to seal all dissection entry points. Answer C: Endovascular stents promote thrombosis of the false lumen, induce aortic remodeling, and prevent further aneurysmal dilatation, rupture, or malperfusion. Answer E: Endovascular interventions sometimes require carotid subclavian bypass if the left subclavian is covered and there is inadequate collateral circulation. This is not the case every time. Bottom Line: Endovascular therapy has been recently used for management of chronic type B aortic dissections.

Organs affected by hypothermia

The heart is one of the most sensitive organs to hypothermia. Ventricular fibrillation and asystole can occur when body temperature reaches 21 - 24º C. Characteristic EKG changes include a prolonged PR interval, interventricular conduction delay, and presence of J waves. Patients with cardiopulmonary arrest due to hypothermia can potentially recover. Therefore, these patients should not be considered dead until they are warmed to normal body temperature. Survival rates have been quoted near 50% for patients with core temperature < 32º C. Comorbidities can increase death rates to > 75% when present. Hepatic, renal, adrenal, pulmonary, and pancreatic dysfunction can occur with hypothermia and be exacerbated during rewarming due to increased capillary permeability

LES tone

The hormones gastrin and motilin have been shown to increase LES pressure.

Nitroprusside toxicity

The patient is exhibiting symptoms of cyanide toxicity secondary to the nitroprusside medication. These patients can have weakness and confusion and develop pulmonary edema. Thiocyanate levels can be checked to ensure the patient does not have a toxic level of cyanide. Antidotes for cyanide toxicity include amyl nitrite, sodium nitrite, and sodium thiosulfate.

Chronic pancreatitis

The ideal treatment for patients with a painful dilated duct in the setting of chronic pancreatitis involves creating an anastomotic connection between the dilated duct and the intestinal lumen. Longitudinal pancreaticojejunostomy, a modification of the Puestow procedure, has been reported to result in immediate pain relief in more than 80% of patients and long-term pain relief in roughly 60% of patients. Pain from the pancreas is carried in sympathetic fibers that traverse the celiac ganglia, reach the sympathetic chain through the splanchnic nerves, and then ascend to the cortex. Celiac plexus nerve blocks performed either percutaneously or endoscopically have been employed to abolish this pain with inconsistent results. In this patient a surgical approach is warranted. Patients with small (< 4-6 mm) pancreatic ducts, but in whom pancreatitis is caused by obstruction at the ampullary level, may benefit from transduodenal sphincteroplasty of the common bile duct with division of the septum that lies between the pancreatic duct and bile duct (pancreatic septotomy) The loss of pancreatic exocrine function in chronic pancreatitis affects the output of all pancreatic digestive enzymes, but it is mostly fat absorption that is abnormal, and it is the delivery of lipolytic enzyme activity to the small intestine that determines the success of treatment. Treatment involves acid suppression with a proton pump inhibitor, a low-fat diet, and lipase doses of 90-150,000 IU per meal, although control of steatorrhea is often incomplete even with this treatment. Complete abstinence from ethanol is advised for patients with alcohol-induced pancreatitis, but symptoms may persist even after complete abstinence. Answer B: This may help the pain but will not address the underlying problem. A surgical correction is warranted. Answer C: This procedure can be used for a patient with a small duct and obstruction at the ampulla. Answer D: Pancreatic enzymes and a proton pump inhibitor may help with some symptomatic relief of the steatorrhea but not affect his pain. Answer E: Complete abstinence from alcohol is recommended. Bottom Line: The pain associated with chronic pancreatitis is difficult to control and various techniques have been described. The Puestow procedure is best used with patients that have intractable pain and a dilated pancreatic duct.

Coronary artery anomalies

The incidence of coronary artery anomalies is about 1% and many may not be significant. The most common is origin of the circumflex (normally arising from the left coronary artery) from the RCA or right coronary sinus.

Splenic echinococcal cysts

The incidence of splenic cysts have increased with the advent of CT and Ultrasound. Splenic cysts are classified as parasitic cysts and Non -parasitic cysts. Pseudocysts, commonly seen in those with a history of trauma, constitute 70-80% of the non-parasitic cysts. Asymptomatic and small (< 4cm) cysts do not require treatment and involute with time. Primary true cysts of the spleen account for approximately 10% of the cysts of the spleen. They are lined with squamous epithelium and are congenital. These epithelial cells stain for CA 19-9 and CEA. Patients with these cysts may have elevated levels of one or two of these tumor markers. These are benign and have the same malignant potential as the surrounding tissue. Non-parasitic cysts do not need surveillance. Rarely hemangiomas and cavernous hemangiomas present as cysts of the spleen. Parasitic cysts are treated with total or partial splenectomy. Most of the true cysts of the spleen are parasitic cysts (Echinococcus spp) and occurs primarily in endemic areas. The Hydatid disease is uncommon in North America but should be excluded before any invasive procedures that might result in spillage of cyst contents into the abdomen. Rupture and spillage may result in anaphylactic shock and intra-peritoneal spillage of the contents. As with Hydatid disease of the liver, the cysts can be sterilized by injection of 3% sodium chloride, alcohol or 0.5% silver nitrate. Splenectomy is the treatment of choice. Laparoscopic cyst unroofing or decapsulation is one of the methods of treating non - parasitic cysts. It is important to exclude the parasitic infection before unroofing as the spillage may result in anaphylactic shock or intraperitoneal spread.

hemobilia

The initial study when hemobilia is expected should be upper endoscopy to rule out other sources of upper GI bleed. Angiography is the best test to diagnose and treat hemobilia with a 80-100% success rate after embolization. Surgery is reserved as a last resort if embolization has failed.

Innominate artery

The innominate artery, or brachiocephalic artery, is the first branch of the aortic arch and soon bifurcates into the right subclavian and right common carotid arteries. It gives blood supply to the right arm, head, and neck. There are typically no branch arteries from the innominate artery, but the thyroid ima artery, which supplies the medial aspects of both lobes of thyroid gland, may originate from here. There is no innominate artery on the left side of the body. However, there are two brachiocephalic veins. The left common carotid and the left subclavian artery come directly off the aortic arch

Zone 3 Hematoma

The internal iliac arteries supply blood to the pelvic organs, bones, and soft tissues. The largest branch is the superior gluteal artery, also the most common artery to be injured. However, the most common bleeding source is the sacral venous plexus which tends to tear and shred after pelvic fracture. Zone III bleeding occurs in the pelvic retroperitoneum.

Adrenal incidentalomas

The key factors in evaluating an adrenal incidentaloma are to characterize its size, functionality, and risk of malignancy. Resection is indicated for all functioning adrenal incidentalomas regardless of size and those suspected of harboring primary adrenal cancer. Size cutoff for resecting adrenal incidentalomas has drifted to include smaller and smaller lesions. The prevalence of primary adrenal carcinoma in adrenal incidentalomas is related to mass size. The risk of primary adrenal carcinoma is less than 2% in lesions smaller than 4 cm whereas the incidence rises to 25% for lesions larger than 6 cm. Lesions larger than 6 cm and smaller lesions with suspicious criteria on imaging should be resected. Lesions smaller than 4 cm with benign imaging characteristics should be followed. For lesions between 4 and 6 cm, either resection or observation is acceptable. Decisions should not be based on size alone but also on imaging characteristics, including CT attenuation values.

LES tone

The lower esophageal sphincter (LES) is the most critical antireflux defense mechanism and has intrinsic myogenic tone modulated by neural and hormonal mechanisms. Alpha-adrenergic neurotransmitters or beta-blockers stimulate the LES, and alpha blockers and beta stimulants decrease its pressure. The hormones gastrin and motilin have been shown to increase LES pressure; and cholecystokinin, estrogen, glucagon, progesterone, somatostatin, and secretin decrease LES pressure.

lymphoid tissue

The lymphoid system consists of primary and secondary organs. The primary lymphoid organs act to generate lymphocytes from immature cells. The primary lymphoid organs are the liver, bone, and thymus. The secondary lymphoid organs act to maintain current lymphocytes and act to initiate the adaptive immune response. The secondary lymphoid organs are the lymph nodes, spleen, Peyer's patches, tonsils, adenoids (mucosa associated lymphoid tissue - MALT).

Esophagogastrectomy

The main steps of an esophagogastrectomy, regardless of technique, are proper oncologic resection of the tumor, creation of an esophageal conduit (either from the stomach or intestine), and the creation of a tension-free anastomosis. Mobilization of the duodenum via medial rotation (Kocher maneuver) is commonly needed when the stomach is used as a conduit. This will allow the distal stomach and proximal duodenum to travel more superior toward the chest and eventual anastomosis. Answer A: The next step would not be to divide the right gastric artery. During creation of the gastric conduit the left gastric artery is taken, leaving the right gastroepiploic artery as the only blood supply to the conduit. Answer B: The next step would not be to mobilize more of the esophagus. Given the segmental blood supply of the esophagus, more mobilization would decrease the blood supply and put the subsequent anastomosis at risk for ischemia. Answer C: The next step would not be to divide the splenic artery. During creation of the gastric conduit the left gastric artery is taken. This can be done, given the vast collateral blood supply of the stomach and the right gastroepiploic artery is preserved during an esophagogastrectomy. Answer E: The next step would not be to divide the azygous vein. This would not allow any greater length on the gastric conduit or the proximal esophagus. The azygous also may have already been divided during the mobilization of the tumor. Bottom Line: The Kocher maneuver, or medial rotation of the duodenum, may be required during an esophagogastrectomy to help the gastric conduit meet the proximal esophagus for a tension-free anastomosis

Chronic Crohn's disease

The patient has developed chronic Crohn's disease. Although he is responding to the treatments for his acute episodes, he should be started on medications to achieve remission. Long-term steroids have many side effects and are generally avoided when possible. Immune modulators such as Azathioprine and Infliximab are better utilized for long-term remission but take several weeks to become effective. Over 50% of all Crohn's patients will require surgery at some point during their disease course. Patients with chronic inflammatory bowel disease will need screening colonoscopies 8-10 years after diagnosis and then annually to ensure no dysplasia has developed.

Esophageal stricture

The mainstay of treatment for esophageal stricture is dilation for symptomatic relief. Either bougie or balloon dilation may be used. Balloon dilation has the theoretical advantage of applying a radial force across the stricture, whereas bougie dilation applies an additional longitudinal (shearing) force. While many prefer balloon dilation due to this theoretical risk, no clear difference has been demonstrated. Strictures may be divided into simple and complex (>2 cm, tortuous, or significantly narrowed). Many simple strictures resolve after 1-3 dilations, while 25-35% require repeat dilations. Complex strictures are often a result of caustic injury or severe underlying disease. These often require repeat dilations and are associated with high recurrence rates. Complications of gastroesophageal reflux disease (GERD), such as esophageal stricture or development of Barrett esophagus, are an indication for an antireflux procedure. While there is no unanimous consensus on how to define a shortened esophagus, the endoscopic finding of an esophageal stricture is often predictive. Esophagram may also be beneficial in the workup. During fundoplication, at least 2.5-3 cm of tension-free distal esophagus must be mobilized within the abdomen. Failure to do so increases axial tension and recurrence rates after repair. Collis gastroplasty has been shown in several series to decrease recurrence rates after antireflux operations with a shortened esophagus. Answer A: The mainstay of treatment for benign esophageal stricture is dilation; however, recurrence rates are high for more complex strictures. Answer B: Strictures often require repeat dilations. This is especially true of complex strictures (narrow, long segment, and tortuous). Answer C: While balloon dilation has less shearing as it exerts force radially rather than longitudinally, no clear advantage has been demonstrated. Answer D: Antireflux indications include failed medical management, noncompliance, severe esophagitis, extraesophageal symptoms, or complications of GERD (stricture, Barrett esophagus). Bottom Line: Dilation of esophageal strictures provides symptomatic relief, but repeat dilations are often necessary. Development of stricture is an indication for an antireflux operation. Strictures may indicate esophageal shortening, which would require a Collis gastroplasty.

SSI

The major determinants of surgical site infections (SSI) are bacterial factors, local wound factors, and patient factors. Wound classification is a bacterial factor and is only one component of the overall risk of an SSI. Commonly quoted rates of SSI associated with wound classification vary among textbooks, but in general are clean wounds: 1-5% clean-contaminated wounds: 3-11% contaminated wounds: 10-17% dirty wounds: >27%.

Lymphedema

The management of lymphedema can be challenging. Comprehensive (combined) decongestive therapy (CDT) is one of the best initial therapies. Intermittent pneumatic compression is an alternative. CDT is a 2-phase program that is backed by longstanding experience. The initial phase of decompression involves skin care, manual lymph drainage (light massage that is different from the public massage), range of motion exercise, and compression using bandage wrap. The second phase is a maintenance phase to conserve the results achieved in first phase. Answer A: Diuretic therapy is not useful for lymphedema but can be useful for associated co-morbidities such as congestive heart failure. Answer B: Patient counseling about skin care is important regardless of the method of treatment because patients with lymphedema have higher risk of skin infection. Answer C: Blood pressure measurement and venipunctures on the affected side should be avoided as they may impede lymph flow and worsen the condition. Answer E: Operative intervention is a second-line therapy that should be considered only when the conservative management fails. Bottom Line: Lymphedema can be managed by comprehensive decongestive therapy, which is a 2-phase program that includes initiation phase to reduce the swelling followed by a maintenance phase to conserve the result.

Budd Chiari

The management of patients with acute onset Budd-Chiari syndrome (BCS) is with anticoagulation, which addresses the concern for majority of the patients. Mechanical decompression with a side-to-side portosystemic shunt can be performed if the patients do not respond to anticoagulation or have chronic BCS. These radical procedures have been largely replaced by percutaneous portocaval shunting (transjugular intrahepatic portosystemic shunt [TIPS]). The procedure is performed by placing a stent from hepatic vein to portal vein. Stents can be placed out to IVC if occlusion of hepatic veins is noted. The rates of encephalopathy associated with this procedure are usually not increased in patients with BCS. Answer B: Major surgery, including mesoatrial shunting and even liver transplantation, has been traditionally used in the past as the mainstay therapy for this syndrome; however, anticoagulation followed by TIPS is the preferred treatment at this time. Answer C: Liver transplant is for decompensated BCS. Answer D: TIPS is employed once anticoagulation has been tried if the patient presents with chronic BCS. Answer E: Open portosystemic shunts can be used if the patient is expected to have a transplant; however, anticoagulation in this case would still be the first-line treatment. Bottom Line: The mainstay of therapy for Budd-Chiari syndrome is anticoagulation followed by transjugular intrahepatic portosystemic shunt procedure for severe cases.

Chronic pancreatitis

The medical treatment of chronic or recurrent pain in chronic pancreatitis requires the use of analgesics, a cessation of alcohol use, oral enzyme therapy, and the selective use of antisecretory therapy. Antisecretory therapy includes somatostatin or somatostatin analogue (octreotide acetate) administration, which has been shown to inhibit pancreatic exocrine secretion and CCK release. Although different forms of antacids was not effective in controlling the patient's pain as stated in the previous question, PPI may assist in preventing inactivation of pancreatic enzyme when used in conjunction.

Mediastinitis

The patient has mediastinitis as a complication from his previous cardiac surgery. The risk factors for developing mediastinitis include the use of bilateral internal mammary arteries, diabetes, and obesity. The definitive treatment for mediastinitis is debridement and coverage with pectoralis muscle flaps.

diastasis recti

The midline epigastrum (linea alba) is a physiologic area of weakness in the abdomen. Two main processes can occur in this area. First, is diastasis recti. The second are abdominal wall hernias. Rectus abdominus diastasis occurs when the rectus muscles have separated and when the muscles contract, as in a sit-up, the bulge appears. This is classic in postpartum women or patients after weight loss. Obesity is a risk factor. This is not a true hernia, though it has the appearance of one. Treatment is with abdominal wall strengthening and weight loss. Operative repair, though typically not required, can be performed and typically involves a vertical plication

Benign biliary stricture

The most common cause of a benign biliary stricture is an injury after a previous operation, such as laparoscopic cholecystectomy. Treatment depends on the level of the stricture. Transhepatic or endoscopic dilation and stent placement are successful in about 50% of patients. Surgical correction may involve Roux-en-Y choledocojejunostomy or hepaticojejunostomy. These surgical treatments are successful in >80% of patients.

Early postoperative SBO

The most common cause of a small bowel obstruction is due to adhesive disease and the majority of these can be treated with nonoperative therapy. Early postoperative small bowel obstruction is less likely than other causes of small bowel obstruction to cause strangulation, and is likely to resolve with nonoperative management. Nonoperative management includes nasogastric tube decompression, bowel rest, electrolyte replacement, and ambulation. A water-soluble contrast study can be considered in this setting as well, and can be both diagnostic and therapeutic. The decision to operate on an early postoperative small bowel obstruction should be individualized to the patient, but nonoperative management can be safely continued for a week or more provided there are no signs of bowel compromise or worsening clinical status. Answer A: Metoclopramide (Reglan) is a prokinetic agent which can be helpful for delayed gastric emptying, but has no effect on early postoperative small bowel obstruction. Answer B: Surgery should be used selectively in early postoperative bowel obstruction. A laparoscopic approach can be used safely in some cases of early postoperative small bowel obstruction, including for patients whose original surgery was laparoscopic. The patient in this scenario has no signs of bowel compromise or high grade bowel obstruction that would prompt an immediate return to the operating room. Answer C: The patient in this scenario has an early postoperative partial small bowel obstruction without signs of intestinal compromise or high grade obstruction. Early postoperative small bowel obstruction is likely due to adhesions and is likely to resolve with nonoperative management. Indications for operation include failure of the obstruction to resolve after a trial of nonoperative management (which can be safely continued for up to a week or more, depending on the clinical scenario), or signs of peritonitis, closed loop obstruction or high grade obstruction should prompt operative intervention. Answer E: Further imaging is indicated if there is a significant change in clinical status, but serial CT scans should not be used routinely to monitor the progress of a small bowel obstruction. Bottom Line: Early adhesive post-operative small bowel obstruction without signs of bowel compromise can be managed nonoperatively and is likely to resolve without operation, in comparison to other causes of small bowel obstruction. The decision to operate should be individualized to the patient's situation.

#1 cause of immunodeficiency

The most common cause of immune deficiency in the general surgical population is malnutrition. There can be multiple factors leading to a small or significant degree of malnutrition, but the end result is a malnourished individual with a blunted immune response. Furthermore, malnutrition leading to poor wound healing and post-operative recovery leads to worsening malnutrition

Appendiceal cancer

The most common tumor of the appendix is carcinoid, but not all appendiceal carcinoids are malignant. Malignant features include size over 1-2 cm, presence in lymph nodes, or invasion of tissues adjacent to the appendix. Adenocarcinomas are more common primary malignancies of the appendix than carcinoids. Even so, adenocarcinoma originating in the appendix is rare, and usually resembles colonic adenocarcinoma microscopically. The signet-ring cell carcinoma form carries a worse prognosis. Most mucinous tumors of the appendix are benign, mucinous cystadenomas. The rare malignant form, mucinous cystadenocarcinoma, is identified by invasion through the wall of the appendix or epithelial cells in mucus found in the peritoneum. Appendiceal tumors can present with peritoneal seeding, resulting in peritoneal carcinomatosis or pseudomyxoma peritonei. Gastrointestinal stromal tumors are rare tumors with malignant potential. Primary lymphomas can occur in the appendix. Breast cancer, colon cancer, and tumors of the female genital tract may metastasize to the appendix.

Lung Abscess

The patient has developed a lung abscess secondary to an episode of aspiration. Lung abscesses are more common on the right and present with an air-fluid level on imaging. Fungal infections and tuberculosis will not have an air-fluid level. The majority of patients can be treated with antibiotics alone. However, if medical management does not clear the infection after 8 weeks, surgery is indicated. Other surgical indications include large cavities over 4-6 cm, hemoptysis, and ruling out malignancy. Answer A: The left main stem bronchus is more angulated than the right. Therefore, aspiration contents prefer the right lung. Answer B: Bronchoscopy can help to identify the organism but will not be able to treat the abscess. Answer C: Lung abscesses rarely rupture. Answer E: This is the case if the patient is confirmed positive for TB. However, there is no information to suggest that in this question.

FNH

The natural history of focal nodular hyperplasia (FNH) is not fully understood, but in general, FNH is a benign and indolent tumor. Asymptomatic patients mostly remain so over long periods of time. Rupture, bleeding, and infarction are exceedingly rare, and malignant degeneration of FNH has never been reported. The treatment of FNH, therefore, depends on diagnostic certainty and symptoms. Asymptomatic patients with typical radiologic features do not require treatment. If diagnostic uncertainty exists, resection may be necessary for histologic confirmation. Fine-needle aspiration is not recommended for FNH. Core needle biopsy is not indicated as the diagnosis is made based on radiographic characteristics. Careful observation of symptomatic FNH with serial imaging is reasonable because symptoms may resolve in a significant number of cases; however, if the patient is persistently symptomatic, surgery is indicated. Patients with persistent symptomatic FNH or an enlarging mass need to be considered for resection. Because FNH is a benign diagnosis, resection must be performed with minimal morbidity and mortality.

Cyanosis in the newborn

The newborn has a transposition of the great vessels which often has an associated ventricular septal defect and aortic arch hypoplasia. This is a cyanotic congenital heart defect that will progress to death if left untreated. Transposition of the great vessels can be treated with prostaglandin E1 which helps to keep the ductus arteriosus open to improve pulmonary blood flow and decrease the degree of cyanosis. A balloon septostomy can also be performed to improve the mixing of blood and oxygenation. Answer A: Truncus arteriosus has a single outflow track. If it was atretic, the condition would not be survivable. Answer C: Tetralogy of Fallot has the following characteristics: overriding aorta, right ventricular hypertrophy, ventricular septal defect, and right ventricular outflow tract obstruction. Answer D: Univentricular heart is not one of the congenital cyanotic heart conditions in newborns. Answer E: The 5 Ts of cyanosis in the newborn are Tetralogy of Fallot, transposition of the great arteries, truncus arteriosus, tricuspid atresia, and total anomalous pulmonary venous connection.

Methotrexate for ectopic pregnancy

The optimal candidates for methotrexate treatment of ectopic pregnancy are hemodynamically stable, willing and able to comply with post-treatment followup, have a human chorionic gonadotropin (hCG) concentration ≤5000 mIU/mL, and no fetal cardiac activity. Ectopic mass size less than 3-4 cm is also commonly used as a patient selection criterion; however, this has not been confirmed as a predictor of successful treatment. Another contraindication to methotrexate use is a concomitant intrauterine pregnancy. Methotrexate is toxic to the kidneys and liver. Creatinine >1.5 or liver enzymes >2 times normal are considered contraindications to its use.

Rectus sheath hematoma

The patient developed a rectus sheath hematoma from forceful contraction of his anterior abdominal wall while coughing. The hematoma is caused by hemorrhage from the network of collateralizing vessels within the rectus sheath and muscles. History and physical examination alone can be diagnostic. A palpable mass that remains unchanged with contraction of the rectus muscles is associated with rectus sheath hematoma and is called Fothergill's sign. Pain is usually increased with contraction of the rectus muscles. A hemoglobin and/or hematocrit level and coagulation studies should be obtained, especially with patients taking anticoagulation. Both ultrasonography and CT can provide confirmation and exclude other disorders. Answer A: Diastasis recti is a protrusion of a weakened anterior abdominal wall without a fascial defect. It is often misdiagnosed as a hernia. Contraction of the anterior abdominal wall muscles would visualize diastasis recti as opposed to a relaxed position.

Primary sclerosing cholangitis

The patient has Primary Sclerosing Cholangitis (PSC) which is an immunologic disease that causes fibrosis and strictures in the biliary tree. These strictures lead to cholestatic liver disease and may cause cirrhosis and biliary sepsis. Stenting can help to relieve the obstructions, but the definitive treatment for PSC is liver transplantation. PSC does not improve after resolution of Ulcerative colitis

Breast conservation therapy

The patient has a Stage II breast tumor which is amenable to breast conserving therapy. If the nodes were matted or fixed this would make the patient an N2 and upstage her to a stage III tumor. Stage III undergo neoadjuvant chemotherapy. Patients are candidates for a partial mastectomy if the tumor size is less than 5 cm and margins of 2-3 mm can be achieved while maintaining good cosmesis. Modified radical mastectomy is also a reasonable choice if they opt to not have breast conservation. The patient must also have no contraindications to radiation. Any contraindications to radiation, including connective tissue disease or pregnancy, will mandate a mastectomy. An axillary dissection must be performed with clinically palpable nodes.

Paget-von-Schroetter syndrome

The patient has a classic presentation of Paget-von Schroetter syndrome otherwise known as venous thoracic outlet syndrome (TOS) or effort thrombosis. It results in thrombosis or severe narrowing of the subclavian-axillary vein secondary to chronic extrinsic mechanical compression within the thoracic outlet. The venous pathology is a direct result of repetitive injury to the subclavian vein at the level of the costoclavicular space, the most medial aspect of the thoracic outlet. The main structures causing compression of the vein are the first rib, the clavicle with its associated subclavius muscle and fibrous costocoracoid ligament, and the anterior scalene muscle and tubercle. Embolic events are arterial and the patient has palpable wrist pulses, which go against this as a source of his arm swelling. Occlusion of the SVC can cause significant arm swelling but usually with associated neck swelling. A mass can produce swelling but usually in a gradual manner and not with an acute onset.

Femoral hernia

The patient has an incarcerated femoral hernia, which usually occur below the inguinal ligament. If the hernia is difficult to reduce, the inguinal ligament can be divided for better exposure of the femoral canal, which is posterior to the inguinal ligament. Answer A: All femoral hernias require surgical repair due to the high likelihood of incarceration. For incarcerated femoral hernias, an inguinal approach is recommended. Femoral hernias occur through the femoral canal. The boundaries of the femoral canal include the iliopubic tract superiorly, Cooper ligament inferiorly, femoral vein laterally, lacunar ligament medially (junction of the iliopubic tract and Cooper ligament). A femoral hernia results in a bulge below the inguinal ligament on exam. All femoral hernias require surgical repair due to high incidence of strangulation. Repair is performed using the standard Cooper ligament repair. The basic steps include dissection and reduction of the hernia sac and obliteration of the defect in the femoral canal.

histoplasmosis

The patient is presenting with a Histoplasmosis infection. These fungal infections are commonly seen in the Ohio and Mississippi river valleys and are identified on chest x-rays as calcified lesions. Histoplasmosis infections are self limiting and do not require any further intervention if the patient's condition is uncomplicated. These patients can have bulky mediastinal nodes that can cause compressive symptoms. Treatment of symptomatic lesions typically involve the use of antifungal agents. Amphotericin B, with its significant side effects, is reserved for the initial treatment of patients who have moderately severe or severe infection.

Subacute thyroiditis

The patient is presenting with a subacute thyroiditis, likely related to her recent upper respiratory infection. Subacute thyroiditis presents with fevers, fatigue, and tenderness over the thyroid. The disease is usually self-limiting but can be treated with NSAIDs or steroids. Subacute thyroiditis is associated with mild alterations to thyroid function. Answer A: Subacute thyroiditis is associated with mild alterations to thyroid function. Answer C: Antibiotics are not necessary as the underlying cause is typically viral and the course is self-limiting. Answer D: Propylthiouracil is unnecessary as the hyperthyroidism is self-limiting. Answer E: In patients with subacute thyroiditis and hyperthyroidism, a radioiodine or technetium imaging study will show low uptake. The test is unnecessary as the diagnosis is typically clinical. Bottom Line: Subacute thyroiditis can be treated with NSAIDs or steroids.

Hemobilia

The patient is presenting with hemobilia secondary to his previous liver injury. Trauma is one of the most common causes of hemobilia, which is due to a fistula formation between the biliary tree and a blood vessel. In stable patients, the first step is an upper endoscopy to rule out other sources of GI bleeding. However, with an unstable patient and a high index of suspicion, the best test is angiography. In these situations, angiography is not only diagnostic, but also therapeutic. Angiography is preferred over open surgery as first line therapy due to lower morbidity.

Hereditary spherocytosis

The patient is presenting with hereditary spherocytosis. This is most commonly an autosomal dominant disease which leads to a deficiency of spectrin in the erythrocytes, making them more fragile as they pass through the spleen. Ultimately, the cells are trapped and destroyed which may lead to anemia, reticulocytosis, splenomegaly, pigmented gallstones, jaundice. Splenectomy is effective in preventing hemolysis and correcting anemia in the setting of severe hemolysis, however spherocytes will still remain. If a patient is being considered for splenectomy an ultrasound should be performed preoperatively to rule out gallstones so that a cholecystectomy can be performed during the same operation. Children are advised delay a splenectomy until after the age of 5, to allow proper development of their immune systems.

Treatment of Hypervolemic hyponatremia

The patient now presents with acute, symptomatic, hypervolemic, hyponatremia due to excessive water intake. The appropriate treatment is correction of the sodium deficit with hypertonic saline. Sodium deficit is calculated based upon actual and desired sodium level and total body water. The appropriate rate of sodium correction in this setting is 0.5 mEq/L/hour.

Mondor disease

The patient presents with Mondor disease. This condition is characterized by a thrombophlebitis of the subcutaneous veins of the anterior chest wall. The sudden appearance of a subcutaneous cord, which is initially red and tender, is characteristic. Frequently involved veins include the thoracoepigastric, lateral thoracic, and superficial epigastric veins. This is a benign process that resolves in 4-6 weeks with supportive therapy that includes nonsteroidal anti-inflammatory drugs and warm compresses. Answer A: If the thrombophlebitis involves the greater veins and progresses to the deep venous system it can lead to a pulmonary embolism. Answer B: Mondor disease is a clinical diagnosis. An MRI is not needed for diagnosis of Mondor disease, and the patient does not have suspicious lesions that would benefit from further analysis by MRI. Answer C: Infection or abscess is not found in this patient. If her condition progressed to suppurative thrombophlebitis, excision of the vein would be indicated. Answer D: No suspicious breast mass was found, so biopsy would not be necessary. Bottom Line: Mondor disease is a benign process that typically resolves in 4-6 weeks with supportive therapy that includes nonsteroidal anti-inflammatory drugs and warm compresses. TrueLearn Insight : Not all breast lesions are malignant. A course of treatment is appropriate for benign conditions prior to further workup for malignancy.

Intussusception in adults

The patient presents with a bowel obstruction secondary to intussusception. In children, this can be treated with an air contrast enema that usually resolves the obstruction. In adults, the presence of intussusception should raise concern for a possible malignancy as the lead point of the obstruction. With this likely etiology in mind, it is recommended to perform a segmental resection of the obstructed portion of bowel and rule out a malignant lesion

Cecal volvulus

The patient presents with a cecal volvulus, which is more common in middle-aged females. Endoscopic decompression is generally avoided as a treatment option because of low success rate. The gold standard is surgical resection with an ileocolonic anastomosis. The exception is if there is gangrenous bowel or a perforation, where it would be safer to avoid an anastomosis and place an end ileostomy. In this patient, with normal vital signs and no peritoneal signs, it is unlikely that a compromised bowel would be encountered. Answer A: Without signs of perforation or ischemia, a primary anastomosis can be performed. Answer B: Endoscopic decompression should not be attempted for cecal volvulus. Answer D: Colonic pexy has a high incidence of recurrence. A resection is the preferred procedure. Answer E: The patient demographic and description of the abdominal film favor a cecal volvulus. Bottom Line: The treatment for a cecal volvulus is with ileocolic resection and primary anastomosis.

Diverticulitis

The patient presents with acute diverticulitis, which is more common in the older population on low-fiber diets. Diverticulae become inflamed secondary to obstruction of the tic, which leads to venous congestion. The tic can then perforate, which is often contained within the pericolic mesentery. More severe cases cause a focal or distant abscess and may also lead to frank perforation of the colon with fecal peritonitis. Answer A: More aggressive courses of diverticulitis are seen in younger males. Answer B: Western diets low in fiber have an increased prevalence of diverticulitis. Answer D: Colonoscopy should be performed 6 weeks after the inflammation resolves to rule out malignancy. Answer E: Diverticulitis is most common at the sigmoid colon. Bottom Line: Diverticulitis is secondary to obstruction of the diverticulae and venous congestion.

Ischemic colitis

The patient presents with ischemic colitis secondary to vasopressor use and intermittent hypotension. Generally, ischemic colitis is unlikely to be caused by large vessel occlusion. It is more commonly due to a low flow state or small vessel occlusion. Signs and symptoms of ischemic colitis reflect the extent of bowel ischemia. In mild cases, patients may have diarrhea (usually bloody) without abdominal pain. With more severe ischemia, intense abdominal pain (often out of proportion to the clinical examination), tenderness, fever, and leukocytosis are present. Peritonitis and/or systemic toxicity are signs of full-thickness necrosis and perforation. Treatment is supportive if there is no evidence of transmural ischemia or perforation. 80% of patients will recover with nonoperative management. While there is minimal evidence for use, antibiotics are generally given to prevent theoretical bacterial translocation. Anticoagulation is usually not needed since most of the time the cause is nonocclusive ischemia. Operation is required with clinical deterioration and may require fecal diversion or avoidance of primary anastomosis if ischemia is still present. Answers A & D & E: Surgery is indicated if there are signs and symptoms of infarcted bowel, but oftentimes mucosal ischemia can be treated with resuscitation and antibiotics alone. If surgery is performed, creating a stoma and delaying a primary anastomosis is the safest approach.

biliary stricture

The patient presents with signs of a biliary stricture that has led to acute cholangitis. Biliary strictures are most commonly due to obstructing stones, but they may also be due to chronic pancreatitis, Mirizzi syndrome, primary sclerosing cholangitis, or malignant strictures. The treatment for these patients is IV antibiotics and urgent biliary decompression with endoscopic retrograde cholangiopancreatography (ERCP) to relieve the obstruction. Once the obstruction is relieved, the patient is not at risk for sepsis, and definitive management of the cholangitis can be initiated. Answer A: Delaying biliary decompression can be life threatening in ascending cholangitis. Answer B: The diagnosis has been made. A CT may only delay treatment. Answer D: Magnetic resonance cholangiopancreatography would add little and is not curative like an ERCP would be. Answer E: An ERCP should not be delayed in this patient. Bottom Line: The treatment for acute cholangitis is IV antibiotics and endoscopic retrograde cholangiopancreatography for biliary decompression.

Duodenal diverticula

The patient with abdominal pain is dealing with an acute episode of pancreatitis caused by a duodenal diverticula. The most common location of a duodenal diverticulum is in the 2nd portion of the duodenum (62%), and most are located near the ampulla (88%). The majority of duodenal diverticula are asymptomatic and do not warrant any treatment if discovered incidentally; however, symptomatic diverticula that cause pancreatitis or cholangitis do require surgical intervention. The most common procedure performed for uncomplicated duodenal diverticula is an open diverticulectomy by performing a wide Kocher maneuver and then closing the duodenal defect primarily. These operations should be delayed until after the acute bout of pancreatitis is resolved. In this case, the patient should be treated with NPO, IV fluids, and bowel rest until the pancreatitis is resolved. Subsequently, the diverticula should be addressed with diverticulectomy.

Sliding hernia

The patient's findings suggest a hiatus hernia as the cause for her symptoms of reflux disorder. This is classified as a Type I hernia, which is also known as a sliding hernia. Sliding hernias are treated with medical management and should be started on a proton pump inhibitor. Surgical correction is necessary for Type II, III, and IV hernias due to the risk of a volvulus.

Hyperparathyroidism

The patient's laboratory workup is consistent with primary hyperparathyroidism. The presence of kidney stones is an indication for surgery. The majority of hyperparathyroidism is caused by an adenoma, and less commonly by parathyroid hyperplasia. Current recommendations are to perform a preoperative localization study to minimize the extent of the neck dissection. Sestamibi

Carcinoid syndrome

The patient's symptoms and findings suggest she has a carcinoid syndrome secondary to metastatic disease. Carcinoid tumors can occur anywhere along the GI tract and are common in the small bowel. The tumor releases serotonin in the bloodstream which causes the typical carcinoid symptoms of diarrhea, facial flushing, and asthma. Carcinoid tumors produce both serotonin and kallikrein. Kallikrein is an enzyme that converts kininogen to lysyl-bradykinin and subsequently bradykinin.

PA catheter

The placement of a pulmonary artery catheter can yield valuable information in the critically ill patient. The indications for a PA catheter include treatment of myocardial infarction, assessment of fluid status, evaluation of pulmonary edema or hypertension, evaluation of valvular disease or cardiac tamponade, and evaluation of postoperative heart surgery patients. There is no indication for a PA catheter to evaluate cardiac arrhythmias , and the placement of the catheter can be, in fact, arrhythmogenic

Pleural space

The pleural space is constantly turning fluid over due to oncotic and hydrostatic pressures within the pleural space. Upwards of 10 liters of fluid may turnover in the course of the day. Answer B: There is always some fluid present in the pleural space, but about 300 mL is needed to be detected on chest x-ray. Answer C: The parietal pleural space contains most of the lymphatic tissue and is the site for most of the fluid resorption. Answer D: Increased hydrostatic pressures and decreased plasma oncotic pressures can both contribute to the development of an effusion. Answer E: Effusions are classified as transudate or exudates based on their protein and LDH concentrations, where transudates will have a low pleural to serum ratio. Bottom Line: The pleural space turns over 5-10 liters of fluid a day.

C diff in pregnant patients

The preferred medical treatment of C. difficile infections in pregnant females is with oral vancomycin.IV vancomycin is not used to treat C. difficile. Answers C & D: Flagyl (metronidazole) treatment should be avoided in pregnancy and while breast-feeding (both PO and IV forms). First trimester exposure to metronidazole is not recommended in FDA guidelines because of concern regarding ready placental transmission and case reports describing facial anomalies following exposure. Metronidazole and its active metabolites are readily detected in breast milk and in the plasma of infants.

Bilirubin

The primary source of bilirubin in the body is from breakdown of red blood cells. Whether due to aged cells or active hemolysis, hemoglobin is broken down into bilirubin and ultimately secreted into bile.

Anal Squamous cell carcinoma

The primary treatment for localized squamous cell carcinoma is chemoradiation, also known as the Nigro protocol, named after Norman Nigro in 1974. The protocol consists of 5-fluorouracil (5-FU), mitomycin-C, and preoperative radiation. Up to 30% of patients will have recurrence or have residual disease, and these patients are the ones who benefit from surgery. The surgical procedure for excision of a lesion in the anal canal should be an abdominoperineal resection (APR). Although there is some new evidence that recurrent disease can be treated with a 5-FU-based regimen, National Comprehensive Cancer Network guidelines still recommend APR for recurrent disease.

wound healing

The process of wound healing occurs in three phases: inflammatory, proliferative, and remodeling. The two main goals of the inflammatory phase is to achieve hemostasis and start the process of tissue repair. After tissue injury, the blood vessels in the area of injury constrict immediately, and the exposed tissue collagen primarily from the endothelium of the lacerated vessels produce thromboplastic product to stimulate platelet aggregation. The aggregated platelets form the initial hemostatic plug. Next, the intrinsic and the extrinsic coagulation cascades are initiated. This will lead to the formation of fibrin, which is subsequently transformed into a stable clot and the hemostasis in the wound is achieved. Answer A: Collagen production, degradation, and remodeling occurs in the third phase of wound healing, the remodeling phase. Answer B: Fibroblast proliferation occurs in the second phase of wound healing, the proliferative phase. Answers D & E: Although all these events do occur in the first phase of wound healing, the order of them is incorrect. Bottom Line: After tissue injury, vasoconstriction, platelet aggregation, and activation of coagulation cascade results in hemostasis.

Neck Zones

The question asks about evaluation of Zone I neck injuries. Penetrating injuries to the neck are divided into two triangles: the anterior and posterior triangle. The posterior triangle is bound by the sternocleidomastoid anteriorly, the trapezius posteriorly and the clavicle inferiorly. The anterior triangle is located anterior to the anterior border of the sternocleidomastoid and is divided into three zones: Zone I is located from the clavicle to the cricoid. Injuries to this zone may involve lung apex, trachea, brachiocephalic or subclavian artery and veins, nerve roots and esophagus. Zone II is located from the cricoid to the angle of the mandible. Injury in this location may involve the carotid or vertebral arteries, the jugular veins, esophagus or trachea. Zone III is located from the angle of the mandible to the skull base. Injury to this zone may result in external or internal carotid injury, jugular injury, cranial nerve injury or hypopharyngeal injury. Those patients that are hemodynamically unstable require operative intervention, regardless of zone. Other indications for operative intervention include hard signs of vascular injury (bruit, thrill, expanding or pulsatile hematoma) or tracheal injury (subcutaneous air or bubbling from the wound). A chest x-ray is performed to evaluate for hemo or pneumothorax, retropharyngeal air or apical capping. If non-operative intervention is chosen, evaluation for vascular, esophageal and tracheal injuries should take place. This can include four vessel angiography, CTA, or even color flow Doppler. Additionally, esophagography with barium or esophagoscopy should be performed. Laryngotracheobronchoscopy may also be done at the same time. Choice of modalities is often institution dependent.

Retrorectal tumors

The retrorectal space is bounded by the rectum anteriorly, the sacrum and investing sacral fascia posteriorly, and the ureters, internal iliac arteries, and rectal stalks laterally. The space extends cranially to the peritoneal reflection. Classification of retrorectal tumors varies widely. These tumors are broadly categorized as inflammatory, congenital, neurogenic, osseous, and miscellaneous. In addition, they may also be distinguished as benign, malignant, congenital, and acquired. Improved preoperative imaging and the use of colonoscopy help exclude metastatic and/or locally advanced colorectal and genitourinary carcinomas from the category of retrorectal tumors. There are inflammatory conditions that mimic primary tumors and are usually (but not always) used to diagnose prior to surgical exploration. Even if asymptomatic, appropriate surgical candidates diagnosed with this tumor should proceed with resection. Multidisciplinary approaches involving colorectal surgeons, neurosurgeons, and possibly orthopedic surgeons are imperative in preoperative planning. In general, preservation of at least unilateral S3 nerve roots typically allows normal bowel and bladder function following sacral resection. The surgical approach is based on preoperative imaging and expected extent of resection. Answer A: Retrorectal tumors are frequently asymptomatic, and they are often found incidentally during evaluation for unrelated physical complaints. Answer B: Transrectal ultrasound is very sensitive for retrorectal tumors. Normal transrectal ultrasonography performed by an experienced surgeon effectively rules out a retrorectal tumor. Answer C: CT or MRI is the standard for preoperative evaluation of retrorectal tumors. Either modality will demonstrate whether the lesion is cystic or solid and the extent of the lesion, but radiologists have difficulty differentiating benign from malignant lesions. Answer D: Biopsy of a retrorectal tumor prior to surgical resection should be avoided. Transrectal biopsy can lead to superinfection of cystic lesions and produce rapidly fatal meningitis in patients with anterior sacral meningocele and seed needle tracts. Perhaps the only indication for biopsy is to make a diagnosis in a patient who is not a candidate for surgery but may benefit from palliative radiation or chemotherapy. Bottom Line: All retrorectal tumors should be resected, even if asymptomatic.

Right coronary artery

The right coronary artery (RCA) emerges from the right coronary sinus and courses over the anterior surface of the heart in the atrioventricular groove where it gives off the blood supply to the right ventricle. The RCA bifurcates into the posterior descending artery (PDA) and the right posterolateral artery. About 90% of patients will have their PDA arise from the RCA and are termed as having a "right dominant" system. The RCA supplies multiple right ventricular branches, as well as branches to the AV node (90% of patients). The sinoatrial nodal branches are less consistent and arise from the RCA in about 50% of patients.

Replaced right hepatic artery

The right hepatic artery passes posterior to the common hepatic duct as it ascends to the liver in 85% of individuals and anterior to the common hepatic duct in the remaining 15%. In approximately 15% of individuals, a replaced or aberrant right hepatic artery originates from the superior mesenteric artery and courses through the medial aspect of Calot's triangle, posterior to the cystic duct. Answers A & B & E: The left gastric artery is contained within the hepatogastric ligament along with the left gastric vein and hepatic division of the left vagal trunk. The hepatogastric ligament will contain an aberrant left hepatic artery arising from the left gastric artery in as many as 25% of subjects. The hepatogastric ligament is medial to the portal triad and unlikely to be entered during a routine cholecystectomy.

Gastrinoma

The secretin stimulation test is the test of choice because it has a sensitivity of 85% or greater. With this test, a 2-U/kg bolus of secretin is administered intravenously, and fasting serum levels of gastrin are measured before and 2, 5, 10, and 15 minutes after administration. Answer A: An increase of 200 pg/mL in the gastrin level after secretin administration is consistent with a diagnosis of Zollinger-Ellison syndrome (ZES). Answer B: Measurement of the fasting serum level of gastrin is the initial study to diagnose ZES. Of patients with ZES, virtually 100% will have a fasting serum gastrin level greater than 1000 pg/mL; however, fasting serum gastrin levels in individuals with renal failure, pernicious anemia, or atrophic gastritis may exceed 300 pg/mL, and therefore concomitant measurement of basal acid output is necessary to confirm the diagnosis of ZES. Answer C: In most cases, there is a delay in diagnosis of ZES, with a mean period of 6 years from initial symptoms to diagnosis. Answer D: The secretin test is positive in only 80% of ZES patients, so it is not absolutely required for diagnosis. Its false negative rate is 6-20%. Bottom Line: Gastrinoma can be diagnosed with fasting serum gastrin levels >1000 pg/mL and basal acid output >15 mEq/hr. Secretin stimulation test will show an increase in gastrin of >200 pg/mL, but it is not necessary to make the diagnosis. TrueLearn Insight : Gastrinomas will show a paradoxical increase in gastrin level after a secretin stimulation test.

Bile secretion

The secretion of bile is dependent on a chloride channel that is stimulated mostly by secretin. Has high concentration of bicarbonate ions

Sensory innervation of Larynx and epiglottis

The sensory nerve supply of the upper epiglottis is the glossopharyngeal nerve, which provides the sensory pathway of the gag reflex. The lower epiglottis and the laryngeal mucosa above the vocal cords is supplied by the internal branch of the superior laryngeal nerve, which contributes to the cough reflex. The sensory supply of the larynx below the vocal cords is via the recurrent laryngeal nerve. The recurrent laryngeal nerve also provides motor innervation to the intrinsic muscles of the larynx, except the cricothyroid muscles.

Most common site of perforation during colonoscopy?

The sigmoid colon is the most common site of iatrogenic perforation. Perforations follow three principal mechanisms: mechanical perforation by the endoscope's tip, barotrauma from overinsufflation, and therapeutic procedures. The sigmoid colon accounts for roughly 70% of all noted perforations. The majority of perforations diagnosed during the procedure are surgically repaired. Watchful waiting with antibiotic therapy is a valid alternative in patients with sealed perforations who do not demonstrate signs of peritonitis. The descending, transverse, and ascending colon are injured in decreasing order of frequency.

Bladder tumors

The stage and grade of the tumor are important for determining treatment. These two factors are independently correlated with prognosis. Bladder tumors that involve the mucosa (Ta) and the lamina propria (T1) are referred to as superficial disease, whereas tumors that extend beyond the lamina propria and invade muscle (T2a) are referred to as invasive disease. Disease that extends through the muscle into the fat (T3), node-positive disease, or metastatic disease requires further treatment with neoadjuvant or adjuvant chemotherapy. Answer A: Endoscopic resection is optimal treatment for patients with low risk, superficial disease. Answer B: High-grade T1 lesions recur in more than 80% of cases and progress to muscle-invasive disease in 50% of patients within 3 years. Patients with high-risk superficial disease—defined as carcinoma in situ, stage T1 lesions, or large, high-grade, recurrent, or multifocal Ta lesions—should receive further treatment with intravesical bacillus Calmette-Guérin (BCG). Answer C: Radical cystectomy alone is inadequate surgery given the invasive disease. Answer D: Radical cystectomy alone or with lymph nodes dissection is inadequate treatment for this patient as he has invasive disease that should be treated with either neoadjuvant therapy or radical resection, lymph nodes dissection followed by adjuvant chemotherapy. Bottom Line: Disease that extends through the muscle into the fat (T3), node-positive disease, or metastatic disease requires further treatment with neoadjuvant or adjuvant chemotherapy.

Rapid sequence intubation

The steps in rapid sequence intubation are: Preoxygenation—Administration of 100% oxygen for 3 minutes of normal, tidal volume breathing in a normal, healthy adult establishes an adequate oxygen reservoir to permit 8 minutes of apnea before oxygen desaturation to less than 90% occurs. Pretreatment—Patients with significant cardiovascular disease (e.g., ischemic coronary disease) who are being intubated in the emergency department may benefit from the administration of the synthetic opioid, fentanyl, in a dose of 3 µg/kg to mitigate the release of catecholamines in response to airway manipulation. Paralysis with induction—In this phase, a potent sedative agent (etomidate) is administered by rapid IV push in a dose capable of rapidly producing unconsciousness. This is immediately followed by rapid administration of an intubating dose of a neuromuscular blocking agent, usually succinylcholine. It is usual to wait 45 seconds from the time the succinylcholine is given to allow sufficient paralysis to occur. Positioning—The patient should be positioned for intubation as consciousness is lost. Usually, positioning involves head extension, often with flexion of the neck on the body, but there is evidence that simple extension of the head alone, or extension of both the head and neck (the extension-extension position) are equivalent or superior. The Sellick maneuver (application of firm backward-directed pressure over the cricoid cartilage) has long been recommended to minimize the risk of passive regurgitation and, hence, aspiration. With a trauma patient, head extension should be avoided until the c-spine is cleared. Until then, inline stabilization should be maintained or a rigid cervical collar should be kept on. Tube placement and confirmation of placement—End-tidal CO2 detector.

Heller Myotomy

The technical aspects of the transthoracic heller myotomy are as follows: Place the patient in the right lateral decubitus position. Enter the pleural space in the 7th intercostal space. Incise the inferior pulmonary ligament. Retract the lung medially and cephalad. Incise the mediastinal pleura. Encircle the esophagus with a penrose drain. Identify both vagus nerves. Perform the esophagomyotomy. This procedure is often used for diffuse esophageal spasm as the length required for myotomy is longer than what can be accessed via an abdominal approach. Answer A: Retracting the lung medially and cephalad cannot be done unless the isolated perfused lung is mobilized. Answer B: To afford better visualization of this space the lung should be mobilized first. Answer D: Encircling the esophagus with a penrose drain is not possible unless you have incised the mediastinal pleura first. Answer E: The esophagomyotomy should not be performed without adequate visualization. Bottom Line: Mobilization of the inferior pulmonary ligament is the first step once inside the chest. TrueLearn Insight : Knowing operative steps is important for the ABSITE.

UTI

The term urinary tract infection (UTI) is a generalized term used to cover multiple clinical entities such as bacteriuria, cystitis, and pyelonephritis. UTIs are characterized by urothelial inflammation secondary to microbial invasion. Although they occasionally result from viral or fungal infections, the majority are caused by bacteria, most commonly Escherichia coli. Women are at a higher risk for UTI than men because their urethra is shorter. Catheter associated UTI are a form of health care associated infection responsible for 20,000 infections each year. The overwhelming majority are caused my E. coli (27%). Following E. coli is Enterococcus responsible for 15% Answer A: Candida is third most common at 13% of catheter related UTI's. Klebsiella is the second most commonly isolated organism in UTIs, but not those associated with urinary catheters. Klebsiella are nonmotile, rod-shaped, gram-negative bacteria with a prominent polysaccharide capsule. This capsule encases the entire cell surface, accounts for the large appearance of the organism on gram stain, and provides resistance against many host defense mechanisms. They are increasingly common causes of nosocomial infections at 11%.

Thoracic duct

The thoracic duct is the main lymphatic channel for the abdomen and chest. Chylomicrons and long-chained fatty acids enter into this lymphatic system, whereas short and medium-chained fatty acids are transported in the portal system. It originates at the cisterna chyli at L1-L2 and courses superiorly through the aortic hiatus. It runs along right of midline until crossing to the left at T4-T5. The duct then empties in to the left subclavian vein at the junction with the internal jugular vein.

Inhalation injury

The three primary mechanisms that lead to injury in smoke inhalation are thermal damage, asphyxiation, and pulmonary irritation. Airway damage is caused by inhaled toxins that travel into the distal airways. Heat is dissipated along the upper airways. Immediately after inhalational injury, pulmonary blood flow increases. Increased capillary permeability contributes to pulmonary edema. Proteins found in this fluid create fibrin casts, which may be difficult to clear. Patients should be intubated early secondary to increased airway edema after burn injury. These patients may have sustained injuries in closed spaces, exhibit hoarseness, singed facial hair, and have carbonaceous sputum, but each of these findings has poor sensitivity and specificity. Bronchoscopic diagnosis is often necessary.

Respiratory quotient

The total amount of CO2 produced by systemic metabolism is roughly equivalent to the amount of O2 consumed. The ratio between CO2 produced and O2 consumed is referred to as the respiratory quotient (RQ) and varies slightly depending on whether carbohydrate, protein, or fat is being metabolized. An RQ of 1 or greater suggests that the principal nutrient being metabolized is carbohydrate, whereas an RQ of 0.7 indicates that primarily lipids are being utilized. VCO2/VO2

Treatment of DIC

The treatment for disseminated intravascular coagulation (DIC) is to address the underlying disorder. This patient is septic and requires empiric antibiotics and resuscitation. Blood products can transiently improve the abnormal laboratory values, but the definitive treatment of DIC is to correct its cause. Even with optimal therapeutic intervention, the outcomes in patients with DIC are poor.

Septic thromophlebitis

The treatment of septic thrombophlebitis is dictated by its location. Antibiotics and excision of a superficial vein (the entire length) are curative. This becomes problematic when deep veins or central veins of the chest or pelvis are involved, because it is difficult and may result in large losses of blood. The mainstays of therapy for septic deep or central vein thrombophlebitis are intensive antibiotic therapy and heparin anticoagulation to halt the thrombotic process and prevent embolization. This may last 2 to 3 weeks but is usually successful; surgery is performed (venous thrombectomy and vein excision) if other treatments are not successful.

Microstaging for melanoma

The tumor thickness measured by Breslow depth of invasion is the single most important variable in the treatment and prognosis of patients with localized melanoma. An excisional full-thickness biopsy with 1- to 2-mm margins, to the adipose tissue, is the preferred biopsy method for any lesion highly suspect for melanoma and to provide enough specimen for histologic interpretation and accurate microstaging.

hepatic adenoma

The two major risks of hepatic adenomas are rupture (with potentially life-threatening intraperitoneal hemorrhage) and malignant transformation. Quantifying the risk for rupture is difficult, but it has been estimated to be as high as 30-50% and may be related to size. Although there are numerous reports of transformation of a hepatic adenoma into hepatocellular carcinoma, the true risk for transformation is probably low.

Umbilical hernias

The umbilicus is anatomically divided into an upper and a lower part. The lower part contains the obliterated vessels and the urachus and is partially reinforced and protected from the underlying preperitoneal fat and abdominal viscera. The upper part consists of thin transversalis fascia only and pieces of preperitoneal fat. Therefore, preperitoneal fat and viscera can be extruded forward to initiate hernia formation. Umbilical hernias present as a reducible bulge at the umbilicus. Regardless of the size, incarceration and strangulation are always a possibility; therefore, umbilical hernias should be repaired unless there is a strong contraindication.

DVT acute vs. chronic

The veins with acute deep venous thrombosis are distended with thin and smooth walls that are compressible or partially compressible. They do not have collateral veins .The acute clots are echolucent in venous duplex scanning. The veins with chronic deep venous thrombosis are contracted secondary to inflammatory response from thrombus and have irregular, thick walls. The collateral veins develop but absence of them does not exclude the presence of deep venous thrombosis. The chronic clots appear echogenic in the venous duplex scanning. Answer A: Presence of dilated collateral veins is a sign of chronic deep venous thrombosis. Answer C: Partial recanalization of the thrombosed veins is seen in chronic deep venous thrombosis. Answer D: The walls of veins with acute thrombosis are thin and smooth while those with chronic deep venous thrombosis are thick and contracted secondary to the inflammatory response from the thrombus. Answer E: The deep veins with acute thrombosis are dilated with thin walls while those with chronic thrombosis are contracted with thick and irregular walls. Bottom Line: Veins with acute thrombosis are echolucent while chronic thrombosis appear echogenic.

Adrenal masses

This patient has a cortisol-producing adrenal adenoma greater than 6 cm in size. The treatment for this lesion is an open adrenal resection. Depending on the surgeon and author the approach varies, from a purely retroperitoneal or transperitoneal approach. Classically, it was recommended that tumors greater than 6 cm should be approached in an open fashion as they have a higher rate of adrenal cancer; however, the limits are being pushed each and every day. Tumors with concerning findings on CT such as lymphadenopathy or loss of local fat planes should be approached in an open fashion as not to spread the tumor within the peritoneal cavity. Tumors less than 6 cm with clear fat planes should be approached laparoscopically.

Bariatric surgery

There are many options for weight loss surgery. Some are more historical and not performed much today. The biliopancreatic diversion is both a restrictive and absorptive operation where a smaller pouch limits intake and a large portion of small bowel is bypassed leading to decreased absorption. This comes with a high rate of nutritional deficiency and the nutritional morbidity is between 40-77%. But the operation comes with the highest rate of excessive weight lost: 70.1%. Gastric banding is a restrictive only procedure and has a 47% excessive weight loss and nutritional morbidity of 0-10%. Roux-en-Y gastric bypass is a combination procedure and has a 60% excessive weight loss and 15-25% nutritional morbidity

CBD exploration

There are multiple different extraction methods to allow for laparoscopic clearance of common bile duct (CBD) stones which include basket extraction using fluoroscopy, choledochoscopic basket extraction, and laparoscopic choledochochotomy with balloon sweep extraction. Laparoscopic choledochotomy is considered an advanced laparoscopic technique and should only be attempted if one is familiar with this technique. After this is completed, a T-tube should be left in place in a similar manner to an open procedure. Choledochoscopy is limited by the necessity for the cystic duct to be dilated up to 8 mm. Basket extraction using fluoroscopy is a skill that many surgeons are quite comfortable with given its similarities to angiography. However, if a surgeon is not familiar with these techniques and there are no other options available (either surgically or radiographically) to extract the stone, then converting to an open CBD exploration should be performed. Transcystic choledochotomy and stone retrieval usually entails making a transverse incision about halfway of the diameter of the cystic duct and inserting an endoscope to explore. In a pinch, a ureteroscope can be used if a choledochoscope is unavailable.

Pneumocytes

There are three major cell types in the alveolar wall (pneumocytes): Type I (Squamous Alveolar) cells that form the structure of an alveolar wall Type II (Great Alveolar) cells that secrete pulmonary surfactant to lower the surface tension of water and allows the membrane to separate, therefore increasing its capability to exchange gases. Surfactant forms an underlying aqueous protein-containing hypophase and an overlying phospholipid film. Macrophages that destroy foreign material, such as bacteria.

Succinylcholine

There are two major categories of neuromuscular blockers (NMBs): depolarizing and nondepolarizing. Depolarizing NMBs mimic acetylcholine (ACh) by binding ACh receptors of the motor end plate and causing depolarization of the muscle, which is seen clinically as muscle fasciculations. Succinylcholine, the only depolarizing NMB available for use, is characterized by a rapid onset and a short half-life. Therefore, it is the paralytic of choice for rapid sequence intubation and may be useful for short invasive procedures. Succinylcholine is degraded by plasma pseudocholinesterase and has a very short half-life, but in patients with a deficiency of this enzyme, prolonged effects can occur. Side effects of succinylcholine include muscle pain, rhabdomyolysis, ocular hypertension, malignant hyperthermia, and hyperkalemia. Patients with spinal cord injuries, large burns, upper and lower motor neuron disease, renal failure, or prolonged immobility are at particular risk for hyperkalemia and resultant cardiac dysrhythmias. Cisatracurium undergoes Hoffman degradation and can be used in liver and renal failure. Pancuronium undergoes renal excretion. Answer E: Rocuronium undergoes hepatic metabolism. As succinylcholine is the only depolarizing neuromuscular blocker, it is commonly tested and shows up every year on the boards. It is important to know its mechanism of elimination (by pseudocholinesterase), uses (rapid onset, short half-life are ideal for rapid sequence intubation), side effects (malignant hyperthermia, hyperkalemia), and contraindications (spinal cord injuries, large burns, renal failure).

Esophageal diverticulum

There are two types of diverticulum that form in the esophagus, traction and pulsion. Traction diverticula are true diverticula and typically occur in the mid-esophagus. They occur as a result of inflammatory processes in the nearby lymph nodes that pull on the esophageal wall. They contain all layers of the esophageal wall. Answers A & C & D & E: Pulsion diverticula can be a Zenker diverticulum, which occurs above the cricopharyngeus muscle, or epiphrenic diverticula, which occur in the distal esophagus. These are false diverticula that do not contain all layers of the esophageal wall. They only contain the mucosa and submucosa that herniated through the muscular wall. Bottom Line: Traction diverticula contain all layers of the esophageal wall and usually occur in the mid-esophagus. Pulsion diverticula only contain the mucosa and submucosa and usually occur in the proximal or distal esophagus.

Lung Resection critical value

There is an array of pulmonary function tests that are used to determine if a patient can tolerate a lung resection. The most important of these values is the FEV1. If the predicted postoperative FEV1 is greater than 0.8 L, then the patient will likely tolerate the procedure. The recommended predicted postoperative FVC is greater than 1.5 L. The DLCO is a measure of the diffusion capacity of the lung. A procedure is contraindicated if the predicted postoperative value is less than 40%.

Thiamine deficiency

Thiamine deficiency results in beri-beri, which is characterized by anion gap metabolic acidosis, altered mental status, diabetes insipidus, hyperbilirubinemia, and thrombocytopenia. Thiamine is needed for glucose entry into the Krebs cycle. Thiamine is a cofactor for pyruvate dehydrogenase. Pyruvate levels build up when patients are deficient in thiamine. The pyruvate is then shifted to lactate which results in metabolic acidosis. Therefore, a deficiency in this vitamin causes a refractory lactic acidosis. Alcoholics are at an extremely high risk of developing a thiamine deficiency due to the poor diets they consume, especially when the diet is low in vegetables. Thiamine deficiency is treated by administering 100 mg of thiamine daily.

Neuroblastoma

This child is presenting with a neuroblastoma, which is the most common type of solid organ tumor in the pediatric population. Children often present with a palpable flank mass, and may have symptoms such as diarrhea, hypertension, or ataxia. Work-up will reveal an elevation in catecholamines and metanephrines, and is easily diagnosed by CT scan or MRI. A poor prognosis is associated with age greater than 1-year-old, and elevations in neuron specific enolase (NSE), LDH, and N-myc amplification. Tumors that are hyperdiploid will have more DNA within the tumor cells which make them more susceptible to chemotherapy and portends a favorable prognosis. Answer A: Age greater than 1 is associated with poor prognosis. Answer B: Elevated neuron specific enolase is associated with poor prognosis. Answer C: An elevated LDH level is associated with poor prognosis. Answer E: Tumors that are hyperdiploid will have more DNA within the tumor cells which make them more susceptible to chemotherapy and portends a favorable prognosis.

Li-Fraumeni syndrome

This family is presenting with a familial cancer known as Li-Fraumeni syndrome. This syndrome is characterized by the diagnosis of a soft tissue sarcoma before the age of 45, along with the diagnosis of any cancer in a first and second degree relative before 45 years old. The most common type of alternate cancers described are breast and brain neoplasms. The mutation associated with Li-Fraumeni syndrome is with the p53 gene and is inherited in an autosomal dominant pattern.

Peutz-Jeghers syndrome

This patient has Peutz-Jeghers syndrome, an autosomal dominant disease that has several identifying characteristics, such as intestinal hamartomas and hyperpigmented lesions of the oral mucosa. Patients with Peutz-Jeghers syndrome have been shown to have an increased risk of extracolonic cancers, such as breast, cervical, thyroid, and lung. Periodic screening for these cancers should begin at age 25. Patients with Peutz-Jeghers syndrome have an increased risk for a GI malignancy. A screening colonoscopy should be performed every 2 years. AUTOSOMAL DOMINANT

Surgical correction of a recto-vaginal fistula

This patient has adequate anal sphincter tone and good performance status. Therefore, resecting the diseased rectum and restoring continuity is reasonable. A good vascular pedicle should be placed between the vagina and the anastomosis to help decrease reformation of the fistula. In addition, some authors also advocate temporary fecal diversion. Answers A & D: For patients that had poor anal sphincter tone, restoring continuity would most likely result in fecal incontinence. If that were the case, an end colostomy with or without resection would then be appropriate. Answer B: Patient performance factors heavily impact the correct procedure. For a frail patient with poor performance factors that could not tolerate a major colonic resection, a loop sigmoid colostomy would be appropriate. Answer E: The fact that the patient's rectal mucosa is friable and fibrotic heavily impacts the potential surgical options. A mucosal advancement flap would most likely fail as the rectal tissue is not healthy and most likely has compromised blood flow due to the radiation damage. This repair would likely break down. Bottom Line: Radiation induced rectovaginal fistula patients with adequate anal sphincter tone and performance status should be offered resection and reanastomosis with a vascularized buttress.

Blind Loop Syndrome

This patient has blind loop syndrome from his enteroenteric fistula. Strictures, diverticula, fistulas, or blind (poorly emptying) segments of intestine are anatomic lesions that cause stagnation and permit bacterial proliferation. The continual flow of luminal contents by peristalsis, the interdigestive migrating myoelectric complex, gastric acidity, local effects of immunoglobulins, and the prevention of reflux of colonic contents by the ileocecal valve are all mechanisms that limit bacterial populations. Disturbance of any of these mechanisms can lead to bacterial overgrowth and blind loop (contaminated small bowel, intestinal bacterial overgrowth) syndrome. Steatorrhea, diarrhea, megaloblastic anemia, and malnutrition are the hallmarks of blind loop syndrome. Hypocalcemia occurs because calcium is bound to unabsorbed fatty acids in the intestinal lumen.

.VAP treatment guidelines

This patient is concerning for development of ventilator-associated pneumonia (VAP). Emphasis on treatment of VAP is placed on rapid broad-spectrum escalation and de-escalation of antibiotics due to growing multi-drug resistance (MDR) pathogens in health care setting. Recent 2016 guidelines from Infectious Disease Society suggest the following empirical antibiotics regimen for VAP: Any of the following present, which increases the likelihood of MDR-VAP, should be treated with combination of 3 antibiotics (Zosyn/Cefepime/Ceftazidime/Imipinem/Meropenem/Aztreonam + Levofloxacin/Ciprofloxacin/Amikacin/Gentamicin/Tobramycin + Vancomycin/Linzeolid): - IV antibiotic use within previous 90 days - Septic shock at time of VAP - ARDS prior to VAP - More than 5 days of hospitalization prior to VAP - Acute renal replacement therapy prior to VAP If none of the above risk factors are present, then two drug therapy is sufficient (Zosyn/Cefepime/Ceftazidime/Levofloxacin/Ciprofloxacin/Imipenem/Meropenem/Aztreonam + Vanco/Linezolid), if risk factors for MRSA are present: - Treatment in a unit in which 10-20% S. aureus are methicillin-resistant - Treatment in a unit in which MRSA rate is unknown If none of the above risk factors for MDR-VAP are present AND no risk factors for MRSA are present then single monotherapy with Zosyn/Cefepime/Levofloxacin/Imipenem/Meropenem is sufficient.

Gastric banding patient who presents with erythema and induration around port?

This patient is presenting with band erosion into the stomach. These patients may present with vague abdominal pain and normal vital signs. Diagnosis may be confirmed with upper endoscopy which can visualize the band. Treatment should consist of complete removal of the band and repair of the gastric perforation. If the gastric perforation is walled off by omentum, no further attempts to visualize the defect and repair it are necessary. Band erosion most commonly presents as port site infection

Aldosteronoma

This patient is suffering from aldosteronoma. He has consistent laboratory findings with the disease and a CT scan demonstrating a right adrenal mass. Patients with aldosteronoma have hypokalemia (which can result in muscle weakness and cramping), elevated aldosterone levels and suppressed renin levels. Diagnosis is usually confirmed with aldosterone:renin ration of 30, although this is not seen in all the patients. Serum or urine metanephrines should be sent to rule out a pheochromocytoma but once a functional mass is identified, the treatment is surgical excision. Adrenal vein sampling is generally reserved for patients with no lesions on imaging, bilateral lesions on imagine or bilateral hypertrophy of the adrenal glands. Once the adrenal gland causing the disease is identified, resection is the treatment of choice. Patient's fluid balance and hypertension should be corrected prior to the operation. The prefered medication for blood pressure control in this population is spironolactone. Laparoscopic adrenalectomy is the least invasive method for management of this solitary adrenal mass.

Damage control surgery

This patient is suffering from persistent hypotension in the setting of coagulopathic bleeding which is often accompanied by worsening acidosis and hypothermia. The best course of treatment is to stabilize any major hemorrhage and resuscitate the patient prior to any definitive intervention. Initiation of a damage control operation is performed to minimize hemorrhage and operative time in the setting of physiologic derangement. Some of the major indications for damage control are: Core temperature 35 °C (95 °F) SBP <80 mmHg pH <7.2 Base deficit >14 mmol/L INR or PTT >50% normal Blood loss >4L Blood transfusion >10 units Fluid replacement >0 L Persistent non-surgical bleeding The goals of damage control are rapid evaluation and management of life-threatening injuries with definitive repair to be performed at a later time once physiologic capture has take place. Hollow viscus injuries are rapidly sutured to prevent contamination. Complex injuries to the duodenum may be treated with debridement and primary repair or pyloric exclusion. Pancreatic injuries are treated with drainage alone. Liver and splenic injuries are treated based on the severity of the injury with a primary goal of hemostasis. A second look laparotomy is then planned if bowel viability is questioned. After the initial operation, the resuscitative phase is begun. In addition to using active rewarming to bring core temperature to 37 °C (98.6 °F), coagulopathy must be corrected. Packed red blood cells, fresh frozen plasma, and platelets should be administered in a 1:1:1 ratio for maximum benefit. Goals of resuscitation should be PT <15 or INR <1.2 using fresh frozen plasma, vitamin K, calcium, and limited use of crystalloids Fibrinogen >100 mg/dL using cryoprecipitate or fibrinogen concentrate Platelets >100,000/mm3 using packed platelets Answer A: Damage control is indicated when core temperature is below 35 °C (95 °F). Answer C: While acidosis is an indication, pH must be <7.2. Answer D: Blood transfusion >10 units is considered an indication for damage control. Answer E: Blood loss >4 L is considered an indication for damage control. Bottom Line: Persistent hypotension and coagulopathy may be a sign of proofed physiologic derangement in trauma. A damage control operation may be indicated.

Post-transplant lympho-proliferative disorder

This patient most likely has post-transplant lymphoproliferative disorder, or PTLD. While the symptoms can be vague and nondescript, they most commonly include fever, lymphadenopathy, and bowel obstruction. Syndromes can range from mononucleosis-like illness to widespread lymphoma. A high clinical index of suspicion must be maintained in the transplant population. PTLD results from the immortalization of Epstein-Barr virus-positive B-cells. Treatment initially involves reduction in immunosuppression. In the lung transplant population, PTLD typically occurs within the first year.

Complications following AAA repair

This patient most likely is having a myocardial infarction (MI). This typically develops secondary to blood loss, resuscitation, and aortic clamping and declamping in patients with coronary disease. MI in patients with a ruptured AAA develops in 14-24% of patients and carries with it a mortality of 19%. Pneumothorax is unlikely as the CXR was clear and there is no decrease in oxygenation. Bleeding could be a possibility, but with a stable hemoglobin and normal abdominal exam, bleeding is further down on the differential. Answer B: With a normal chest x-ray, a pneumothorax causing hypotension is highly unlikely. Answer C: Bleeding is unlikely with a hemoglobin level of 13. In addition, the hemoglobin is stable from six hours ago. Answer D: Pulmonary embolism is unlikely with normal oxygenation, lack of tachycardia, and normal ventilator settings. Answer E: Septic shock would be unlikely this soon post operatively and would manifest with multiple SIRS criteria. Bottom Line: Myocardial infarction (MI) is the main complication after major vascular procedures.

Fistula-in-ano

This patient presents with a fistula-in-ano (FIA) secondary to previous abscess formation. The treatment for FIA depends on the location and complexity of the fistula. Simple, intersphincteric fistulas can be treated with a fistulotomy as long as there is minimal involvement of the external anal sphincter. Setons are recommended for trans-sphincteric fistulas and can help in obliterating the tract over a period of time. More complex or deep fistulas are difficult to manage because aggressive therapy can compromise sphincter function and lead to incontinence. Answer A: Setons are commonly used for fistulas transversing the external anal sphincter. Answer B: Setons are commonly used for fistulas transversing the external anal sphincter to preserve continence. Answer D: Lateral internal sphincterotomy is the procedure of choice for most anal fissures. Answer E: A sitz bath may be helpful for anal fissures but is not first-line treatment for fistulas. Bottom Line: Fistulotomy is the treatment of choice for an intersphincteric fistula-in-ano that does not involve the external anal sphincter.

Toxic megacolon

This patient presents with toxic megacolon as a complication from his ulcerative colitis. He presents with fevers, abdominal pain, distension, and leukocytosis. Initial management requires bowel rest, nasogastric decompression, and supportive care. All antimotility agents, opiates, and anticholinergics should be discontinued. Ulcerative colitis flares can be treated safely with steroids. Toxic megacolon requires surgery with evidence of perforation, massive hemorrhage, or peritonitis with worsening sepsis. In those cases, the procedure of choice is a subtotal colectomy with end ileostomy. A completion proctectomy with ileal anal pouch anastomosis can be performed at a later date when the patient has stabilized. Sigmoidoscopies are contraindicated due to increased risk of perforation of the diseased bowel.

Liver injuries

This question asks about surgical technique in operative management of liver lacerations. Indications for operative management in a liver injury are: hemodynamic instability, peritonitis or continued transfusion requirement. The approach is through either a midline incision or a subcostal incision (often bilateral). Large liver lacerations may easily produce exsanguinating hemorrhage. Therefore, after entering the abdomen, the liver should be quickly packed for temporary tamponade and allow the anesthesiologist to resuscitate the patient. Packs should not be placed in the liver laceration as this will exacerbate bleeding and potentially worsen the laceration by distraction. A posterior directed pressure will help tamponade any posterior injuries such as IVC injury. The liver must be then be fully mobilized by dividing the triangular and falciform ligaments. If able, this may be done initially to help facilitate adequate packing of the liver. For low-grade injuries, packing and simple techniques such as electrocautery alone may be enough to gain hemostasis. If bleeding persists or for high-grade injuries, the Pringle maneuver may be performed by clamping the hepatoduodenal ligament with an atraumatic clamp. This occludes blood flow to the hepatic artery and portal vein. If this slows the bleeding, the injury may simply be oversewn or laceration reapproximated. The clamp should be removed intermittently during the repair to prevent ischemia. Hepatic artery ligation may result in abscesses or biloma and should only be performed selectively. If there is little effect from the Pringle maneuver, the injury is likely from the retrohepatic IVC or hepatic vein. Individual hepatic vessels may be transected by finger fracture of the liver tissue, electrocautery, or even stapling of liver tissue then individually tied. Devitalized liver should be resected, but anatomic resection is usually not necessary. Closed suction drainage for Grade III or greater injuries should be considered and for biliary injuries. Answer A: Packing alone will control bleeding in low-grade injuries, but will be insufficient in high-grade injuries. Answer B: Placing packs directly into the laceration will worsen bleeding by distraction. Answer D: Hepatic artery ligation is associated with liver abscess and should only be performed if necessary. Answer E: Devitalized segments of liver should be resected, but formal resection of liver segments is usually unnecessary.

Biloma

This question asks about the management of a fluid collection in the right upper quadrant after trauma. Biloma is the most likely diagnosis given the high-grade of injury and elevated bilirubin and lack of white blood cell count elevation or fever (which would indicate abscess). In trauma, biliary injury in hepatic trauma is relatively common. The majority of injuries are to small intrahepatic bile ducts. These are often treated intraoperatively with electrocautery and tying off large visualized vessels, debridement of devitalized tissues and placement of closed suction drains in high-grade liver injuries. Extrahepatic biliary tract injury is uncommon, with the gallbladder being the most common site. If the gallbladder is injured, a cholecystectomy is performed. If further injury is suspected, the porta hepatis should be inspected closely. Simple injury < 50% in diameter may be primarily repaired. Complex injury may require Roux-en-Y reconstruction as stricture rate approaches 50% in this setting. Those that develop and abscess or biloma are usually treatable with percutaneous drainage alone. Further treatment is based upon drain output. Those that drain less than 300 mL / day will usually close spontaneously. The more than 300 mL drains daily, the injury should be localized with fistulogram, ERCP, radionucleotide scan or transhepatic cholangiogram. Sphincterotomy may help close biliary leaks. Major ductal injuries may be stented or require operative repair. Persistence of drainage more than 50 mL/ day beyond 2 weeks indicates development of a biliary fistula. These often resolve without further intervention

Liver lacerations

This question asks about the role of angiography in blunt liver injuries. While approach to liver laceration is highly dependent upon the specific institution, it is generally agreed upon that hemodynamically stable patients with low grade liver lacerations (I-III) may undergo non-operative management (NOM). Those with active extravasation or with a continued transfusion requirement should undergo angiography and possible embolization. For those considering treatment of liver injury with NOM, the patient should be hemodynamically stable, lack continued transfusion requirements (though an exact number of transfusions is not established), and have no other injuries that mandate laparotomy, such as peritonitis. There is no evidence that supports frequent hemoglobin monitoring, bed rest or reimaging. Many will repeat a CT in 8 weeks after the injury in order to document healing. Other indications for imaging are suspicion of complications from liver injury, including: biloma, bilious ascites or hemoperitoneum. Bilious drainage may necessitate ERCP and sphincterotomy. Liver grading determines success of NOM. Most blunt hepatic injuries Grade I-III are successfully treated with NOM, while higher grade injuries (IV or V) will fail. However, high grade injury does not necessarily mandate operative intervention in hemodynamically stable patients. Other indications of failure are left lobe injuries as they are less contained than right lobe injuries or those that involve major vessels. Answer B: Indications for angiography are active extravasation on CT or continued transfusion requirements. Answer C: Hemodynamically unstable patients should receive an exploratory laparotomy. Answer D: Up to 85% of patients are successfully treated with non-operative management. Stable patients with a blush can go to angiography. Answer E: An enlarging lesion is considered a failure on non operative management; however, repeat imaging is not necessary. Bottom Line: Angiography should be performed in all blunt liver injuries who are hemodynamically stable with blush on CT. Other indications include a continued transfusion requirement or enlarging liver lesion.

Aortic dissection

Thoracic aortic dissection occurs when an intimal tear allows redirection of blood flow from the aorta (true lumen) through the intimal defect into the media of the aortic wall (false lumen). A dissection plane that separates the intima from the overlying adventitia forms within the media. Aortic dissection should always be considered in the setting of severe, unrelenting chest pain, which is present in most patients. The character of the pain is often described as "ripping" or "tearing." The pain is constant with greatest intensity at the onset. Patients usually have no previous episodes of similar pain, which often causes anxiety. Pain is usually located in the midsternum for ascending aortic dissection, while in the interscapular region for descending thoracic aortic dissection. Risk factors include primary hypertension, presence of aneurysmal disease of the aorta, or familial connective tissue disorders. The differential diagnosis of chest pain as a result of aortic dissection includes diagnoses such as myocardial ischemia, aortic aneurysm, acute aortic regurgitation, pericarditis, musculoskeletal pain, and pulmonary embolus. It is essential to consider aortic dissection in each case, as specific therapy (eg, thrombolytic therapy for acute myocardial infarction) may negatively impact the survivability of acute dissection.

Thoracic outlet normal anatomy

Thoracic outlet syndrome is a condition that involves pain, weakness, and paresthesias in the back, neck, and/or arm that is worse with palpation and/or manipulation. This is the most common condition producing upper extremity vascular complications in young adults. Understanding the anatomy of the thoracic outlet will help in making a differential diagnosis for thoracic outlet syndrome. The normal anatomy from anterior to posterior is subclavian vein, phrenic nerve, anterior scalene muscle, subclavian artery, and then brachial plexu

Thoracic outlet syndrome

Thoracic outlet syndrome occurs in about 5% of the population. It is characterized by compression of the brachial plexus, subclavian artery, or subclavian vein where these structures pass between the first rib and scalene muscles. Symptoms include intermittent pain, paresthesia, and weakness in the brachial plexus. Upper extremity edema, venous distention, or effort thrombosis (Paget-Schroetter syndrome) will occur if the subclavian vessels are involved. These symptoms may be worsened by turning and tilting the head or repetitive use of the ipsilateral upper extremity. Diagnosis is made clinically. The Adson test (scalene test) is performed by having the patient maximally extend their neck and turn their head toward the effected extremity. The ipsilateral radial pulse is palpated, and the test is considered positive if the pulse decreases or disappears. Treatment involves physical therapy, neck stretching, avoidance of repetitive trauma and movements, and surgical decompression as a last resort.

angioplasty

Though there are many endovascular treatments for stenosis and occlusions, angioplasty has shown to be very favorable for short segments. Adjuncts to angioplasty include stents and atherectomy devices. Patency of angioplasty is determined not only by the percentage of stenosis but also the length of the occlusion. If considered by location alone, patency after angioplasty is very high in the iliac system with 5-year patency of up to 75%. As you move more distally in the extremity the patency of angioplasty worsens.

H. pylori

Three major patterns of inflammatory response are associated with H. pylori infection, which are related to the pathogenesis of peptic ulcer. The most common pattern is associated with mild to moderate inflammation of all regions of the stomach. Most individuals with this pattern do not develop peptic ulcer disease. About 15% of patients with H. pylori infection have antral gastritis which is associated with high acid output and development of ulcers in the duodenum and prepyloric areas. The third pattern of inflammation is predominantly affecting the body of the stomach. It is associated with gastric atrophy, hypergastrinemia, achlorhydria and it is considered a precursor for gastric cancer. Answer A: About 15% of peptic ulcers are attributed to NSAIDs. NSAID-induced mucosal ulceration is more common in the stomach than in the duodenum. Answer B: H. pylori infection is present in almost all patients with duodenal ulcer and in about 70% of patients with gastric ulcer; however, only 20% of patients with H. Pylori infection develop peptic ulcer disease. Answer D: Patients with duodenal ulcer have a lower basal and stimulatory duodenal bicarbonate secretion. Impairment of duodenal bicarbonate secretion that represents a mucosal defense mechanism is a contributing factor to the pathogenesis of duodenal ulcer. Answer E: Patients with duodenal ulcer have higher basal and peak acid output compared to healthy individuals. They also have hypergastrinemia, higher parietal cell mass and faster gastric emptying time. All these changes are attributed to H. pylori infection. Bottom Line: There are three different patterns of inflammation associated with H. pylori infection: diffuse, antral and stomach-body related. The first is most common, but it is not associated with the development of peptic ulcer.

TTP

Thrombotic thrombocytopenia purpura (or Moschcowitz syndrome) is characterized by widespread thrombosis of arterioles and capillaries by hyalin membranes composed of platelets and fibrinogen. The classic pentad of symptoms includes thrombocytopenic purpura, neurologic manifestations due to microvascular disease in the brain, kidney injury or hematuria due to microvascular disease in the kidney, hemolytic anemia due to destruction of red cells, and fever. The precise etiology is unknown.

Gustilo Fractures

Timely antibiotic administration (within one hour) is one of the most important determinants of outcomes following open lower extremity fractures. The choice and duration of antibiotics is determined by the fracture grade. The most commonly used system is the Gustilo classification: Gustilo I fractures have < 1cm clean wounds within minimal contamination, Gustilo II fractures have >1cm skin lacerations but preserved soft tissue viability, Gustilo III fractures have extensive soft tissue damage and contamination. Grade I and II fractures should receive gram positive coverage while those with grade III injuries should receive both gram positive and gram negative coverage. Answer B: Gram positive coverage alone is adequate for grade I fractures. Gram negative coverage should be added for grade III injuries. Answer C: Debridement is an important aspect in the care of open lower extremity fractures however it is imperative that patients are thoroughly evaluated for additional injuries prior to proceeding to the OR. Time to debridement is not a predictor of infection as long as it is performed within the first 24 hours Answer D: The associated soft tissue injuries have a signficant impact on outcomes following tibial shaft fractures given the limited soft tissue coverage on the medial tibia. Answer E: Compartment syndrome is a major concern following tibial fractures and may occur in both open and closed fractures. Bottom Line: Timely and correct antibiotic administration following open lower extremity fractures is an important predictor of outcomes.

Torsades de pointes

Torsades often occurs in the setting of an existing prolonged QT interval during sinus rhythm, a reflection of abnormal ventricular repolarization. A QT interval of 500 msec or longer indicates an increased risk of torsades. Acquired QT prolongation and torsades are often multifactorial, including interactions between drug therapy, myocardial ischemia, and electrolyte disturbances (hypokalemia and hypomagnesemia). Treatment of intermittent torsades in stable patients is based on correcting any underlying metabolic or electrolyte abnormalities and increasing the heart rate to shorten ventricular repolarization. Intravenous magnesium sulfate is also effective in treating paroxysmal torsades. Answer A: Torsades is caused by a prolonged QT interval. Beta blockade may exacerbate the problem.

Toxic epidermal necrolysis

Toxic epidermal necrolysis (TEN) is a life-threatening condition marked by dermoepidermal separation. It results in full thickness epidermis loss. Antibiotics are the culprit 40% of the time, anticonvulsants 11% of the time, and analgesics 5-23% of the time. Mortality from TEN is not changed with different causative drug categories. Symptoms usually begin with fever and upper respiratory tract infection complaints about 1 week before onset of rash. Stomatitis, conjunctivitis, and pruritis may occur 24-48 hours before rash. Painful erythema covers the epidermis which will slide and separate from the dermis with light pressure (Nikolsky sign). Mucous membranes of the mouth, eyes, respiratory tract, vagina, and urethra may become denuded, crusted, and ulcerated. These allow for entry of microbes and infection which may progress to overwhelming septicemia. The most common cause of death with TEN is overwhelming sepsis. Treatment may include cyclosporine, cyclophosamide, plasma exchange, and immuglobulin administration. The use of steroids is controversial. Patients may benefit from admission to a burn unit in severe cases. If the dermis can be protected and infections managed, re-epithelialization will occur from the intact dermal appendages without scar formation.

hemorrhoidectomy

Transanal hemorrhoidal dearterialization (THD) is the newest addition to surgical armamentarium of hemorrhoidectomy is Doppler-guided arterial ligation with hemorrhoidopexy. THD is a non-excisional operative technique that relies upon identification and ligation of the superior rectal arteries along with suture rectopexy. Suturing should be performed proximal to the dentate line in order to avoid the highly sensitive anoderm. The rational behind suture hemorrhoidopexy is to addresses the redundant and prolapsing mucosal and internal hemorrhoid. Answer A: The closed (Ferguson) hemorrhoidectomy is the most common operation for hemorrhoids. Answer B: The procedure is performed in the operating room and requires anesthesia. Answer D: Suture hemorrhoidopexy is usually repeated with each artery ligation; therefore, the procedure is performed 2-4 times. Answer E: A specialized anoscope and Doppler ultrasound probe are used during THD. Those instrument are disposable which add a significant cost when compared with excisional hemorrhoidectomy. Bottom Line: THD is a non-excisional operative technique that relies upon identification and ligation of the superior rectal arteries along with suture rectopexy.

Transfusion requirements in trauma

Transfusion in trauma has undergone several revisions. Primarily, there has been a reduction in-goal Hb levels. Now, patients that are hemodynamically normal with Hb levels >7 mg/dL are not transfused. Patients with active myocardial infarction have higher transfusion levels, with a Hb <10 g/dL often tolerated. In the setting of hemorrhagic shock and active ongoing bleeding, patients should be transfused regardless of their current Hb level. One must also remember not to neglect other blood components as well, especially in the setting of multiple blood transfusions. Current evidence favors a 1:1:1 transfusion ratio for packed red blood cells (pRBCs), fresh frozen plasma, and platelets. A typical unit of pRBCs contains about 300 mL, hematocrit of 70% and will raise the Hb by 1 g/dL. Blood typing can take at least 15 minutes to complete. Crossmatching blood will take even longer. In the setting of hemodynamic instability, uncrossed blood should be used (type O negative for women of childbearing age and positive for everyone else). The maximum number of units of uncrossed blood that can be transfused is uncertain. It is recommended, however, that after 8-10 units, the crossmatch should be repeated.

Transhiatal esophagectomy

Transhiatal esophagectomy (THE) is best performed for patients with a high probability of advanced disease. It is best suited for distal esophageal tumors and is performed through an upper midline incision and a left cervical incision. Starting with a laparotomy, the liver is mobilized and retracted laterally. The phrenoesophageal ligament is then taken down and the esophagus mobilized at the hiatus. The short gastric vessels are ligated. Care must be taken to preserve the right gastroepiploic artery as this will serve as the blood supply for the conduit. The pars flaccida is then taken down and the left gastric artery ligated. The vagus nerves are then ligated. The esophagus is then mobilized. The distal third of the dissection may be done under direct visualization, while the proximal two-thirds must be done bluntly. Only a limited and blind thoracic lymphadenectomy is able to be performed. A left cervical incision is made along the border of the sternocleidomastoid. The carotid sheath is retracted laterally. The cervical esophagus is then isolated. After completion of mobilization of the entire esophagus, it is ligated in the neck and brought through the laparotomy incision. The stomach is divided, usually taking a portion of the lesser curvature with it. The gastric conduit is then passed through the chest, and a cervical anastomosis is performed. Answer A: Thoracotomy is a step in Ivor Lewis and three-hole esophagectomies. Answer C: The right (not the left) gastroepiploic artery is preserved in a THE as this serves as the primary blood supply of the conduit. Answer D: In a THE, a cervical anastomosis is performed. Thoracic anastomosis is performed in an Ivor Lewis esophagectomy. Answer E: Ligation of the thoracic duct is often performed in an Ivor Lewis esophagectomy due to the possibility of a chyle leak after extensive thoracic lymphadenectomy. Bottom Line: Transhiatal esophagectomy involves a midline laparotomy and a left cervical incision. The esophagus is bluntly dissected through the hiatus and a cervical anastomosis is performed.

Stone extraction

Transoral endoscopic retrograde cholangiography (ERCP) is not recommended since the patient's gastric pouch is a considerable distance from the patient's duodenum due to her Roux-en-Y bypass. However, it is possible to laparoscopically assist a successful ERCP by bringing the remnant stomach to the abdominal wall and allowing access by creating a gastrostomy. Percutaneous transhepatic cholangiography (PTC) with stone extraction is possible. Stone extraction through a previously placed T-tube remains the best and easiest option in this patient. If all else fails, an open common duct exploration is always a valid option.

Hyponatremia

Treatment of asymptomatic, euvolemic or hypervolemic hyponatremia is free water restriction and observation of sodium levels. Hypovolemic hyponatremia is treated with volume resuscitation with either lactated ringer's solution or normal saline solution. If the patient were symptomatic, treatment would be correction with hypertonic saline solution.

Which burn wound therapy causes hyponatremia?

Treatment of burn wounds is centered around multiple topical therapies. The primary goal of these treatments is to prevent overwhelming burn wound infection and sepsis. Silver nitrate is among the most commonly used topical agents. Silver nitrate has been found to have a broad-spectrum antimicrobial activity in the treatment of burn wounds. Due to the need to dilute the solution before application, its topical application can lead to extravasation of sodium, resulting in subsequent hyponatremia. Methemoglobinemia is another, but rarer, complication of silver nitrate use. One main benefit of this therapy is its relatively low cost.

Malignant melanoma

Treatment of malignant melanoma depends on tumor depth. A superficial shave biopsy is not accurate in defining the thickness of the skin lesion. Therefore, before deciding on the margin of excision, a full thickness incisional or excisional biopsy with 1-2 mm margins should be performed. All other options for margins are indicated once a diagnosis is obtained. When the tumor depth is determined, the recommended excisional margins are as show below: in situ: margin size 0.5-1.0 cm smaller than 1 mm: margin size 1 cm 1-2 mm: margin size 1-2 cm 2-4 mm: margin size 2 cm greater than 4 mm: margin size at least 2 cm

tumor markers

Tumor markers used in ovarian cancer include: CA-125 has been used to help in diagnosis, assessment of response to treatment, and monitoring for recurrence. Alpha-fetoprotein (AFP) is used in ovarian germ cell tumors. It can also be used to assess stage, prognosis, and response to treatment. Beta-hCG (Beta Human Chorionic Gonadotropin) is used in ovarian germ cell tumors to assess stage, prognosis, and response to treatment. Inhibin A & B are hormones that are normally produced by ovarian tissue, but may be elevated in certain types of ovarian cancer (mucinous epithelial carcinoma, granulosa cell tumors). It can be used to assess response to treatment and monitor for recurrence. CEA (Carcinoembryonic antigen) can be elevated in ovarian tumors and may be used to evaluate response to treatment. Answer A: Alpha-fetoprotein (AFP) is associated with liver cancer. Answer B: Beta-human chorionic gonadotropin (β-hCG) is elevated in some testicular cancers. Answer D: CA 125 is a tumor marker for ovarian cancer Answer E: Carcinoembryonic antigen (CEA) is used for colon cancer. Bottom Line: CA 125 is the tumor marker associated with ovarian cancer.

Cholangiocarcinoma

Type I tumors involve the common hepatic duct. Type II tumors are at the hepatic bifurcation. Type III tumors involve the secondary hepatic ducts on one side. Type IV tumors involve the secondary hepatic ducts on both sides. This is the Bismuth-Corlette classification for perihilar cholangiocarcinomas.

Undifferentiated spindle cell tumor (malignant fibrous histiocytoma of bone)

Undifferentiated spindle cell tumor (malignant fibrous histiocytoma of bone) is an uncommon tumor. The most common site of occurrence is the proximal tibia and distal metaphyses of the femur. It may also be found in the pelvis, humerus, and scapula. Radiographic features include loss of normal trabeculation and cortical destruction. Adjacent soft tissue invasion and mass formation may occur. MRI with contrast is the imaging study of choice. Pathology commonly shows high-grade lesions (> 90%) and a tumor showing fibroblasts in a whirling pattern with multinucleated giant cells, inflammatory cells, and foamy mononuclear giant cells. Neoadjuvant chemotherapy can be administered in which it can relieve pain and decrease local edema, contracture, and the size of the soft tissue tumor. Urgent surgical resection is indicated if at high risk for pathologic fracture. Surgical excision with wide margin is advised whether or not the patient receives neoadjuvant therapy. Amputation can normally be avoided.

Urethral injuries

Urethral injuries are an uncommon but significantly morbid sequella of pelvic fracture. Long term consequences can include stricture, incontinence, sexual dysfunction and chronic pain. Injury should be suspected if the patient complains of inability to void, or physical exam reveals a high riding prostate, blood at the urethral meatus, a palpable bladder, or butterfly perineal hematoma. Other indications are pubic symphysis fractures, associated bladder injury, and pelvic hematomas. Pubic diastasis and inferior pubic rami fractures are termed "straddle fracture" and correlate highly with urethral injuries. Diagnosis of urethral injuries is made by retrograde urethrogram. In those with blood at the urethral meatus, some will perform a single attempt at Foley catheterization. If the catheterization fails, a retrograde urethrogram may then be performed. There is no strong evidence that a single attempt will convert a partial tear to a complete disruption. Diagnosis may be more difficult in female patients and bladder neck injuries. While cystoscopy may provide additional information, it usually is not performed as a primary diagnostic modality. Urethral injuries are graded I-V by American Association of for the Surgery of Trauma guidelines: Grade I injuries are contusions. Grade II are urethral stretch injuries. Grade III are partial disruptions. Grades IV and V are complete disruptions with or without extensive separation. Low grade injuries need only catheter drainage. Grade III injuries may need surgical alignment. Grade IV and V may need suprapubic cystostomy and endoscopic realignment. The optimal management of these injuries is largely dependent on concomitant injuries, and diversion with delayed repair is an option. Bladder neck injuries or severe bladder displacement may require additional surgical repair. After realignment, repeat retrograde urethrogram is performed 6 weeks after repair to document healing. Retrograde urethrogram is the best modality for suspected urethral injuries. CT cystography is best for bladder injuries. Answer D: A single attempt may be made at Foley insertion, but repeated attempts are inadvisable.

Anti-coagulation agents

Used to degrade thrombi, fibrinolytic drugs can be administered systemically, or they can be delivered via catheters directly into the substance of the thrombus. Systemic delivery is used for treatment of acute myocardial infarction, acute ischemic stroke, and most cases of massive pulmonary embolism. The goal of therapy is to produce rapid thrombus dissolution, thereby restoring antegrade blood flow. Currently approved fibrinolytic agents include streptokinase, acylated plasminogen streptokinase activator complex (anistreplase), urokinase, recombinant tissue-type plasminogen activator (rt-PA) (alteplase; activase), and two recombinant derivatives of rt-PA, tenecteplase (TNKase) and reteplase (retaplase). All of these agents act by converting the proenzyme, plasminogen, to plasmin, the active enzyme. Answer A: Heparin acts as an anticoagulant by activating antithrombin III and accelerating the rate at which antithrombin inhibits enzymes involved in blood coagulation, particularly thrombin and factor Xa. Answer C: In contrast, direct thrombin inhibitors do not require a plasma cofactor; instead, these agents bind directly to thrombin and block its interaction with its substrates. Approved parenteral direct thrombin inhibitors include lepirudin (refludan), argatroban (novastan), and bivalirudin (angiomax). Answer D: Fondaparinux binds only to antithrombin, catalyzing factor Xa inhibition. Answer E: Heparin and low-molecular-weight heparin are indirect inhibitors of thrombin because their activity is mediated by antithrombin. Bottom Line: Urokinase directly converts plasminogen to plasmin.

Pancreatic pseudocyst

Usually located within pancreas or adjacent in lesser sac, CT scan is diagnostic test of choice. For cysts greater than 5 cm, treatment is usually recommended over expectant management, because most cysts can be promptly eliminated by percutaneous catheter drainage or surgical drainage into the stomach or intestine. Either duct may be dilated and in need of surgical drainage in conjunction with drainage of the pseudocyst.

VAP

VAP is defined as pneumonia (defined as new lung infiltrate plus clinical evidence of infectious origin of infiltrate including fever, purulent sputum or increased secretions, leukocytosis, and increasing oxygenation requirement) occurring >48 hours after endotracheal intubation. Patients with risk factors for multi-drug resistant pathogens should receive empiric antibiotic therapy consisting of drugs against gram-positive, B-lactam anti-pseudomonal, and non-B-lactam anti-pseudomonal.

Ectopic pregnancy

Vaginal bleeding, lower abdominal +/- pelvic pain, and presence of an adnexal mass are most likely suggestive of an ectopic pregnancy. An ectopic pregnancy is a pregnancy that occurs outside of the uterus (ie, extrauterine), typically in the fallopian tube. The most common presenting symptoms include vaginal bleeding and abdominal/pelvic pain. Diagnosis is usually made with serial serum human chorionic gonadotropin (hCG) testing and transvaginal ultrasound. Pelvic inflammatory disease (PID) remains a significant risk factor for ectopic pregnancy.Although a ruptured ovarian cyst can present in a similar manner, the constellation of findings in a reproductive-aged woman most likely suggests an ectopic pregnancy. Additionally, patients with a ruptured ovarian cyst will typically present with a fever, unlike the patient in this vignette. Precipitating factors for the abdominal pain in these patients typically results from strenuous physical activity (eg, exercise, sexual intercourse).

Bipolar electrocautery

Vessel sealing devices are low voltage, high amplitude bipolar electrosurgery instruments that, in combination with optimal pressure delivery by the instrument, fuse vessel walls and create a permanent seal. The output is feedback-controlled in a continuous manner so that a seal is achieved in minimal time, independent of the type or amount of tissue in the jaws. Best results can be achieved by sealing of vessels ≤ 7 mm in diameter with minimal thermal spread. There are many benefits of bipolar electrosurgery. They include: 1. Hemostasis and division of unsupported vascular tissues ≤ 7 mm in diameter using coagulating mode. 2. Versatility as a laparoscopic tool that grasps, dissects, cuts, and coagulates tissue. This versatility may result in fewer instrument exchanges during procedures. 3. Less thermal injury occurs compared to monopolar electrosurgery devices. 4. No capacitive coupling occurs and inadvertent direct coupling is unlikely. 5. Fewer accessories are required such as grounding electrodes, eliminating the possibility of alternate site burns. 6. Safety of the technology for use in laparoscopy has been established.

Vital capacity

Vital capacity is the total amount of air that can be expelled after a maximal inhalation. A maximal inhalation is the tidal volume plus the inspiratory reserve volume, which is known as the inspiratory capacity. The expiratory reserve volume is the amount of air that is expelled after a normal exhalation. The vital capacity includes the inspiratory capacity plus the expiratory reserve volume. The vital capacity plus the residual volume make up the total lung capacity.

VHL

Von Hippel Lindau syndrome results from a germline mutation in the tumor suppressor gene VHL. The gene product of VHL, pVHL, regulates the transcription factor hypoxia induced factor by targeting it for ubiquitin-mediated proteasome degradation. Loss of pVHL function results in intracellular accumulation of HIF and upregulation of multiple gene products including vascular endothelial growth factor (VEGF). The upregulation of VEGF is believed to be the cause for the hypervascular tumors that characterize vHLs: central nervous system and retinal hemangioblastomas and clear cell renal cell carcinomas, pheochromocytoma, pancreatic cystadenomas and neuroendocrine tumors, endolymphatic sac tumors, and papillary cystadenomas of the epididymis and broad ligament. Many of the tumors of vHLs tend to be multifocal and bilateral.

vWF disease

Von Willebrand disease is associated with decreased production of Von Willebrand factor (vWF) by the endothelial cells. vWF is essential for platelet aggregation. Von Willebrand disease is suspected in those with prolonged bleeding time but normal platelet count and clot retraction. This is diagnosed with Ristocetin test and by direct measurement of vWF levels.

Warthin's tumor

Warthin's Tumor or Papillary cystadenoma lymphomatosum, is the 2nd most common salivary tumor and is rarely seen outside of the parotid gland. It has a strong association with middle-aged men who smoke. 10% are bilateral and 10% are multicentric. This FNA finding is not uncommon given the cystic nature of the lesion. Frequently, a definitive diagnosis of Warthin's tumor is likely to occur after a surgical specimen is obtained. Few, if any, salivary gland neoplasms have bilateral presentations. Answer A: Pleomorphic adenoma's are the most common salivary gland tumor. They are more common in women in their 5th decade of life, and are slow-growing. They are also T2 "bright" on MRI. They have a low malignant potential but are very aggressive.Warthin's tumors are common tumors arising from salivary glands, and are strongly associated with smoking. 10% are bilateral and 10% are multicentric. They have rare malignant potential. If you hear, smoking, parotid, bilateral, think Warthin's!

Acute lymphangitis

When an infection spreads along lymphatic channels it is referred to as acute lymphangitis. This commonly occurs in extremities. Signs and symptoms may include red linear streaking proximal to the initial site of infection that travels toward regional lymph nodes. Nodes my become tender and enlarged. Chronic lymphangitis is usually caused by S. schenckii.

emphysematous cholecystitis

When bubbles of gas form in the gallbladder lumen, wall and pericholecystic space a diagnosis of emphysematous cholecystitis is likely. The most commonly causative organisms are Clostridia species (anaerobic, gram-positive rods) (choice A). Other causative organisms include E. coli and streptococci. The gallbladder frequently is not found to contain stones indicating this occurs from a primary bacterial cause. Signs and symptoms include rapidly worsening right upper quadrant abdominal pain, fever and leukocytosis. Abdominal radiographs can show tissue emphysema in an outline of the gallbladder. It may also show air-fluid levels inside. IV antibiotic treatment should begin when the diagnosis is suspected to cover clostridial species, E. coli and streptococci. Emergent cholecystectomy should be done after initial resuscitation. Critically ill patients may be better served with cholecystostomy tube placement but these must be used with caution in the setting of emphysematous cholecystitis as they will not remove the infected tissue.

intussusception

When the clinical index of suspicion for intussusception is high, hydrostatic reduction by enema using contrast or air is the diagnostic and therapeutic procedure of choice. Contraindications to this study include the presence of peritonitis or hemodynamic instability.

Pancreatitis

When the mass is located in the epigastrium or the left upper quadrant, a pancreatic source is most likely. Quick diagnosis is crucial because severe pancreatic infection is fatal if left untreated. The key to successful treatment is early diagnosis of infected pancreatic necrosis, infected pseudocyst, and pancreatic abscess, which is done by understanding their pathophysiology. Pancreatic abscesses form by liquefaction of infected necrosis. They usually occur after week 5 of pancreatitis, when the acute phase has subsided. Answer A: Clinical evaluation alone is generally insufficient to diagnose pancreatic infection as signs and symptoms are vague and nonspecific. Answer B: A clearly defined upper abdominal mass is palpable in only 50-75% of cases. Answer C: Infected pancreatic necrosis usually occurs in week 2 or 3 after development of the acute pancreatitis. Answer E: Pancreatic abscesses are associated with a lower mortality than that for infected pancreatic necrosis. Bottom Line: Pancreatic abscesses form by liquefaction of infected necrosis. They usually occur after week 5 of pancreatitis, when the acute phase of the disease has subsided.

Silvadene

When using Silvadene for burn wounds avoid application to the face, in pregnant patients, and newborns. Can cause neutropenia

Injuries during laparoscopic access

While relatively rare, less than 1% of patients, injuries from abdominal access for laparoscopy do occur. According to FDA studies, up to 25% of such injuries involve the small bowel (the majority). Open entry techniques such as Hasson port placement are not associated with lower rates of injury. Other potential intra-abdominal injuries from abdominal access include injury to major vasculature, bladder, solid organs, or nerves. Those such injuries are less common. Almost half of injuries to small bowel and colon are unrecognized for 24 hours.

Mineral deficiencies

Zinc (Zn) is a trace mineral that has a large number of physiologic roles, in particular being required for growth, functioning of the immune system, and wound healing. Zinc deficiency can occur in cases of reduced absorption or increased gastrointestinal losses such as in patients with gastric bypass, Crohn disease, chronic liver and renal disease, or prolonged TPN use without trace mineral supplementation. Deficiency consequences include failure to thrive, skin rash, and impaired wound healing. Answer A: Selenium is associated with cardiomyopathy, hypothyroidism, and neurological changes. Answer B: Chromium deficiency is associated with hyperglycemia, confusion, and peripheral neuropathy. Answer C: Copper deficiency is associated with pancytopenia and myelopathy (neuropathy with ataxia). Answer E: Vitamin B12 deficiency is associated with megaloblastic anemia and peripheral neuropathy.

Neck Zones

Zone I is located from the clavicle to the cricoid. Injuries to this zone may involve lung apex, trachea, brachiocephalic or subclavian artery and veins, nerve roots and esophagus. Zone II is located from the cricoid to the angle of the mandible. Injury in this location may involve the carotid or vertebral arteries, the jugular veins, esophagus or trachea. Zone III is located from the angle of the mandible to the skull base. Injury to this zone may result in external or internal carotid injury, jugular injury, cranial nerve injury or hypopharyngeal injury. Those patients that are hemodynamically unstable require operative intervention, regardless of zone. Other indications for operative intervention include hard signs of vascular injury (bruit, thrill, expanding or pulsatile hematoma) or tracheal injury (subcutaneous air or bubbling from the wound).

Best surgery for duodenal atresia?

best (and safest procedure) is an end-to-side or side-to-side duodenoduodenostomy

Type 3 VWF disease

causes the most severe bleeding. It is characterized by complete absence of production of vWF. Since the vWF protects coagulation factor VIII from proteolytic degradation, total absence of vWF leads to extremely low factor VIII levels, equivalent to that seen in severe hemophilia A with its clinical manifestations of life threatening external and internal hemorrhages. The inheritance pattern of vWD type III is autosomal recessive, while the inheritance pattern of hemophilia A is x-linked recessive. All 3 types can be treated with cryoprecipitate

Selenium deficiency?

characterized by cardiomyopathy, growth retardation, pain, and weakness.

Organ has higher secretion of potassium?

colon

Chronic pancreatitis secretin test?

decreased bicarbonate only

Malnutrition secretin test?

decreased enzyme

Pancreatic cancer secretin test?

decreased total volume only

End stage pancreatitis secretin test?

decreased total volume, bicarb and enzymes

Type 1 VWF disease

due to low circulating vWF and may be treated with DDAVP or conjugated estrogens, which can stimulate the release of vWF from the endothelium. Type I (partial deficit) and Type III (complete deficiency) are due to the quantity of vWF, and Type II is a qualitative defect of vWF

Type 2 VWF disease

due to qualitative defects of the vWF and requires cryoprecipitate for effective treatment

most frequently used test to diagnose acute aortic dissection?

helical ct. Not aortography

Kasabach-Merritt syndrome

hemangioma, thrombocytopenia, fibrinogenopenia, consumptive coagulopathy. Usually seen in infants

Type A

include ascending aorta and anything else

Zollinger ellison secretin test?

increased total volume

what is transition point for type a vs type b aortic dissection

left subclavian artery

Parotidectomy complications

njury to the facial nerve is a common complication during parotid surgery. Fortunately, most of these injuries resolve over a period of time. Permanent facial nerve dysfunction only occurs in 0% to 4% of patients. To minimize injury, meticulous dissection should be carried out while separating the nerve branches from the gland. Answer A: Facial nerve dysfuction occurs in about 20% to 40% of patients. Answer C: Frey syndrome is caused by inappropriate regrowth of parasympathetic nerve fibers associated with the auriculotemporal nerve into the sweat glands of the skin overlying the parotid gland. Answer D: Salivary fistula is an uncommon complication after parotid surgery. Answer E: The greater auricular nerve is often transected during parotid surgery to allow mobilization of the tail of the parotid from the sternocleidomastoid muscle. Injury to this nerve results in sensory loss in the lower third of the external ear and surrounding pre-and post-auricular skin. Bottom Line: Facial nerve dysfunction, Frey's syndrome, sensory loss, and salivary fistula are the most common complications of parotid surgery.

Cattell Braasch

right colon mobilization

Treatment of type 3 choledochal cyst?

transduodenal approach with either marsupialization or excision of the cyst


Kaugnay na mga set ng pag-aaral

Foundations of Nursing Chapter 14 & 15

View Set

Chapter 4: Specific Factors and Income Distribution

View Set

Base Conversion - Binary,Octal, Decimal, and Hexadecimal

View Set

Assignment 6 - EF, Energy Finance Exam 2, Energy Finance exam 2, FIN 4363 Energy FIN (FINAL), Energy Finance Final Exam, Energy Ratios exam 2, Energy Finance Exam 2, Energy Finance Exam 2 Part 1, Energy Finance exam 2, Energy Finance Exam 1 Joe Byers...

View Set